Sei sulla pagina 1di 160

CORPORATION LAW CASES MAYORDO, M.

A
LLB 3

INTRODUCTION/ CLASSIFICATION OF PRIVATE amount of P59,000,000.00. Part of the assets transferred to


CORPORATIONS the DBP was the glass plant in Rosario, Cavite, which DBP 2. Holding that the assets of the Pioneer Glass
leased and subsequently sold to herein petitioner Union taken over by DBP and part of which was delivered
G.R. No. L-64013 November 28, 1983 Glass and Container Corporation, hereinafter referred to as to Union Glass particularly the glass plant to be
UNION GLASS & CONTAINER CORPORATION and CARLOS Union Glass. returned accordingly.
PALANCA, JR., in his capacity as President of Union Glass
& Container Corporation, petitioners, 
 On April 1, 1981, Carolina Hofileña filed a complaint before 3. That the DBP be ordered to accept and recognize
vs. THE SECURITIES AND EXCHANGE COMMISSION and the respondent Securities and Exchange Commission against the appraisal conducted by the Asian Appraisal Inc.
CAROLINA HOFILEÑA, respondents. the DBP, Union Glass and Pioneer Glass, docketed as SEC in 1975 and again in t978 of the asset of Pioneer
Case No. 2035. Of the five causes of action pleaded Glass. 1
ESCOLIN, J.: therein, only the first cause of action concerned petitioner
This petition for certiorari and prohibition seeks to annul Union Glass as transferee and possessor of the glass plant. In her common prayer, Hofileña asked that DBP be
and set aside the Order of the Securities and Exchange Said first cause of action was based on the alleged illegality sentenced to pay Pioneer Glass actual, consequential,
Commission, dated September 25, 1981, upholding its of the aforesaid dacion en pago resulting from: [1] the moral and exemplary damages, for its alleged illegal acts
jurisdiction in SEC Case No. 2035, entitled "Carolina supposed unilateral and unsupported undervaluation of the and gross bad faith; and for DBP and Union Glass to pay her
Hofileña, Complainant, versus Development Bank of the assets of Pioneer Glass covered by the agreement; [2] the a reasonable amount as attorney's fees. 2
Philippines, et al., Respondents." self-dealing indulged in by DBP, having acted both as
stockholder/director and secured creditor of Pioneer Glass; On April 21, 1981, Pioneer Glass filed its answer. On May 8,
Private respondent Carolina Hofileña, complainant in SEC and [3] the wrongful inclusion by DBP in its statement of 1981, petitioners moved for dismissal of the case on the
Case No. 2035, is a stockholder of Pioneer Glass account of P26M as due from Pioneer Glass when the same ground that the SEC had no jurisdiction over the subject
Manufacturing Corporation, Pioneer Glass for short, a had already been converted into equity. matter or nature of the suit. Respondent Hofileña filed her
domestic corporation engaged in the operation of silica opposition to said motion, to which herein petitioners filed
mines and the manufacture of glass and glassware. Since Thus, with respect to said first cause of action, respondent a rejoinder.
1967, Pioneer Glass had obtained various loan Hofileña prayed that the SEC issue an order:
accommodations from the Development Bank of the On July 23, 1981, SEC Hearing Officer Eugenio E. Reyes, to
Philippines [DBP], and also from other local and foreign 1. Holding that the so called dacion en pago whom the case was assigned, granted the motion to dismiss
sources which DBP guaranteed. conveying all the assets of Pioneer Glass and the for lack of jurisdiction. However, on September 25, 1981,
Hofileña personal properties to Union Glass be upon motion for reconsideration filed by respondent
As security for said loan accommodations, Pioneer Glass declared null and void on the ground that the said Hofileña, Hearing Officer Reyes reversed his original order
mortgaged and/or assigned its assets, real and personal, to conveyance was tainted with. by upholding the SEC's jurisdiction over the subject matter
the DBP, in addition to the mortgages executed by some of and over the persons of petitioners. Unable to secure a
its corporate officers over their personal assets. The A. Self-dealing on the part of DBP which was reconsideration of the Order as well as to have the same
proceeds of said financial exposure of the DBP were used in acting both as a controlling stockholder/ reviewed by the Commission En Banc, petitioners filed the
the construction of a glass plant in Rosario, Cavite, and the director and as secured creditor of the Pioneer instant petition for certiorari and prohibition to set aside
operation of seven silica mining claims owned by the Glass, all to its advantage and to that of Union the order of September 25, 1981, and to prevent
corporation. Glass, and to the gross prejudice of the Pioneer respondent SEC from taking cognizance of SEC Case No.
Glass, 2035.
It appears that through the conversion into equity of the
accumulated unpaid interests on the various loans B. That the dacion en pago is void because The issue raised in the petition may be propounded thus: Is
amounting to P5.4 million as of January 1975, and there was gross undervaluation of the assets it the regular court or the SEC that has jurisdiction over the
subsequently increased by another P2.2 million in 1976, the included in the so-called dacion en pago by case?
DBP was able to gain control of the outstanding shares of more than 100% to the prejudice of Pioneer
common stocks of Pioneer Glass, and to get two, later Glass and to the undue advantage of DBP and In upholding the SEC's jurisdiction over the case Hearing
three, regular seats in the corporation's board of directors. Union Glass; Officer Reyes rationalized his conclusion thus:

Sometime in March, 1978, when Pioneer Glass suffered C. That the DBP unduly favored Union Glass As correctly pointed out by the complainant, the
serious liquidity problems such that it could no longer meet over another buyer, San Miguel Corporation, present action is in the form of a derivative suit
its financial obligations with DBP, it entered into a dacion notwithstanding the clearly advantageous terms instituted by a stockholder for the benefit of the
en pago agreement with the latter, whereby all its assets offered by the latter to the prejudice of c o r p o r a t i o n , r e s p o n d e n t Pi o n e e r G l a s s a n d
mortgaged to DBP were ceded to the latter in full Pioneer Glass, its other creditors and so-called Manufacturing Corporation, principally against another
satisfaction of the corporation's obligations in the total 'Minority stockholders.' stockholder, respondent Development Bank of the

Page 1 of 160
CORPORATION LAW CASES MAYORDO, M.A
LLB 3

Philippines, for alleged illegal acts and gross bad faith partners, amounting to fraud and misrepresentation The fact that the controversy at bar involves the rights of
which resulted in the dacion en pago arrangement now which may be detrimental to the interest of the public petitioner Union Glass who has no intra-corporate relation
being questioned by complainant. These alleged illegal and/or the stockholders, partners, members of either with complainant or the DBP, places the suit beyond
acts and gross bad faith came about precisely by virtue associations or organizations registered with the the jurisdiction of the respondent SEC. The case should be
of respondent Development Bank of the Philippine's Commission tried and decided by the court of general jurisdiction, the
status as a stockholder of co-respondent Pioneer Glass Regional Trial Court. This view is in accord with the
Manufacturing Corporation although its status as such b] Controversies arising out of intra-corporate or rudimentary principle that administrative agencies, like the
stockholder, was gained as a result of its being a partnership relations, between and among SEC, are tribunals of limited jurisdiction 6 and, as such,
creditor of the latter. The derivative nature of this stockholders, members or associates; between any or could wield only such powers as are specifically granted to
instant action can also be gleaned from the common all of them and the corporation, partnership, or them by their enabling statutes. 7 As We held in Sunset
prayer of the complainant which seeks for an order association of which they are stockholders, members or View Condominium Corp. vs. Campos, Jr.: 8
directing respondent Development Bank of the associates, respectively; and between such corporation,
Philippines to pay co-respondent Pioneer Glass partnership or association and the state insofar as it Inasmuch as the private respondents are not
Manufacturing Corporation damages for the alleged concerns their individual franchise or right to exist as shareholders of the petitioner condominium
illegal acts and gross bad faith as above-mentioned. such entity; corporation, the instant cases for collection cannot be
a 'controversy arising out of intra-corporate or
As far as respondent Union Glass and Container c] Controversies in the election or appointments of partnership relations between and among stockholders,
Corporation is concerned, its inclusion as a party- directors, trustees, officers or managers of such members or associates; between any or all of them and
respondent by virtue of its being an indispensable party corporations, partnerships or associations. the corporation, partnership or association of which
to the present action, it being in possession of the they are stockholders, members or associates,
assets subject of the dacion en pago and, therefore, This grant of jurisdiction must be viewed in the light of the respectively,' which controversies are under the original
situated in such a way that it will be affected by any nature and function of the SEC under the law. Section 3 of and exclusive jurisdiction of the Securities & Exchange
judgment thereon, 3 PD No. 902-A confers upon the latter "absolute jurisdiction, Commission, pursuant to Section 5 [b] of P.D. No. 902-
supervision, and control over all corporations, partnerships A. ...
In the ordinary course of things, petitioner Union Glass, as or associations, who are grantees of primary franchise and/
transferee and possessor of the glass plant covered by the or license or permit issued by the government to operate in As heretofore pointed out, petitioner Union Glass is
dacion en pago agreement, should be joined as party- the Philippines ... " The principal function of the SEC is the involved only in the first cause of action of Hofileñas
defendant under the general rule which requires the supervision and control over corporations, partnerships and complaint in SEC Case No, 2035. While the Rules of Court,
joinder of every party who has an interest in or lien on the associations with the end in view that investment in these which applies suppletorily to proceedings before the SEC,
property subject matter of the dispute. 4 Such joinder of entities may be encouraged and protected, and their allows the joinder of causes of action in one complaint,
parties avoids multiplicity of suits as well as ensures the activities pursued for the promotion of economic such procedure however is subject to the rules regarding
convenient, speedy and orderly administration of justice. development. 5 jurisdiction, venue and joinder of parties. 9 Since petitioner
has no intra-corporate relationship with the complainant, it
But since petitioner Union Glass has no intra-corporate It is in aid of this office that the adjudicative power of the cannot be joined as party-defendant in said case as to do so
relation with either the complainant or the DBP, its joinder SEC must be exercised. Thus the law explicitly specified would violate the rule or jurisdiction. Hofileñas complaint
as party-defendant in SEC Case No. 2035 brings the cause of and delimited its jurisdiction to matters intrinsically against petitioner for cancellation of the sale of the glass
action asserted against it outside the jurisdiction of the connected with the regulation of corporations, partnerships plant should therefore be brought separately before the
respondent SEC. and associations and those dealing with the internal affairs regular court But such action, if instituted, shall be
The jurisdiction of the SEC is delineated by Section 5 of PD of such corporations, partnerships or associations. suspended to await the final outcome of SEC Case No.
No. 902-A as follows: 2035, for the issue of the validity of the dacion en pago
Otherwise stated, in order that the SEC can take posed in the last mentioned case is a prejudicial question,
Sec. 5. In addition to the regulatory and adjudicative cognizance of a case, the controversy must pertain to any the resolution of which is a logical antecedent of the issue
function of the Securities and Exchange Commission of the following relationships: [a] between the corporation, involved in the action against petitioner Union Glass. Thus,
over corporations, partnerships and other forms of partnership or association and the public; [b] between the Hofileñas complaint against the latter can only prosper if
associations registered with it as expressly granted corporation, partnership or association and its final judgment is rendered in SEC Case No. 2035, annulling
under existing laws and devices, it shall have original stockholders, partners, members, or officers; [c] between the dacion en pago executed in favor of the DBP.
and exclusive jurisdiction to hear and decide cases the corporation, partnership or association and the state in
involving: so far as its franchise, permit or license to operate is WHEREFORE, the instant petition is hereby granted, and
concerned; and [d] among the stockholders, partners or the questioned Orders of respondent SEC, dated September
a] Devices and schemes employed by or any acts, of the associates themselves. 25, 1981, March 25, 1982 and May 28, 1982, are hereby set
board of directors, business associates, its officers or aside. Respondent Commission is ordered to drop petitioner

Page 2 of 160
CORPORATION LAW CASES MAYORDO, M.A
LLB 3

Union Glass from SEC Case No. 2035, without prejudice to issued an order granting Braga's motion for reconsideration
the filing of a separate suit before the regular court of Norberto Braga, the corporate secretary and son of the and dismissed SEC Case No. 02379.
justice. No pronouncement as to costs. Bragas, refused to register the aforesaid transfer of shares
in t e corporate oo s, asserting that the Bragas claim 6. On February 11, 1983, the Bragas filed their Motion to
SO ORDERED. preemptive rights over the 133,000 Abejo shares and that Dismiss the injunction case, SEC Case No. 02395. On April 8,
Virginia Braga never transferred her 63,000 shares to 1985, the SEC Director, Eugenio Reyes, acting upon the
G.R. No. L-63558 May 19, 1987 Telectronics but had lost the five stock certificates Abejos'ex-parte motion, created a three-man committee
SPOUSES JOSE ABEJO AND AURORA ABEJO, TELEC. representing those shares. composed of Atty. Emmanuel Sison as Chairman and Attys.
TRONIC SYSTEMS, INC., petitioners, vs. HON. RAFAEL DE Alfredo Oca and Joaquin Garaygay as members, to hear and
LA CRUZ, JUDGE OF THE REGIONAL TRIAL COURT This triggered off the series of intertwined actions between decide the two SEC cases (Nos. 02379 and 02395).
(NATIONAL CAPITAL JUDICIAL REGION, BRANCH CLX- the protagonists, all centered on the question of
PASIG), SPOUSES AGAPITO BRAGA AND VIRGINIA BRAGA, jurisdiction over the dispute, which were to culminate in 7. On April 13, 1983, the SEC three-man committee issued
VIRGILIO BRAGA AND NORBERTO BRAGA, respondents. the filing of the two cases at bar. an order reconsidering the aforesaid order of March 21,
1983 of the SEC Hearing Officer Garaygay (dismissing the
No. L-68450-51 May 19, 1987 The Bragas assert that the regular civil court has original mandamus petition SEC Case No. 02379) and directing
POCKET BELL PHILIPPINES, INC., AGAPITO T. BRAGA, and exclusive jurisdiction as against the Securities and corporate secretary Norberto Braga to file his answer to the
VIRGILIO T. BRAGA, NORBERTO BRAGA, and VIRGINIA Exchange Commission, while the Abejos claim the contrary. petitioner therein.
BRAGA, petitioners, vs. THE HONORABLE SECURITIES AND A summary of the actions resorted to by the parties follows:
EXCHANGE COMMISSION, TELECTRONIC SYSTEMS, INC., B. BRAGAS' ACTION IN SEC
JOSE ABEJO, JOSE LUIS SANTIAGO, SIMEON A. MIRAVITE, A. ABEJOS ACTIONS IN SEC
SR., ANDRES T. VELARDE AND L. QUIDATO BANDOLINO, 1. The Abejos and Telectronics and the latter's nominees, as 8. On December 12, 1983, the Bragas filed a petition for
respondents. new majority shareholders, filed SEC Cases Nos. 02379 and certiorari, prohibition and mandamus with the SEC en banc,
02395 against the Bragas on December 17, 1982 and SEC Case No. EB #049, seeking the dismissal of SEC Cases
TEEHANKEE, C.J.: February 14, 1983, respectively. Nos.' 02379 and 02395 for lack of jurisdiction of the Comn-
These two cases, jointly heard, are jointly herein decided. iission and the setting aside of the various orders issued by
They involve the question of who, between the Regional 2. In SEC Case No. 02379, they prayed for mandamus from the SEC three-man committee in the course of the
Trial Court and the Securities and Exchange Commission the SEC ordering Norberto Braga, as corporate secretary of proceedings in the two SEC cases.
(SEC), has original and exclusive jurisdiction over the Pocket Bell to register in their names the transfer and sale
dispute between the principal stockholders of the of the aforesaid 196,000 Pocket Bell shares (of the Abejos 1 9. On May 15, 1984, the SEC en banc issued an order
corporation Pocket Bell Philippines, Inc. (Pocket Bell), a and Virginia Braga 2, cancel the surrendered certificates as dismissing the Bragas' petition in SEC Case No. EB#049 for
"tone and voice paging corporation," namely, the spouses duly endorsed and to issue new certificates in their names. lack of merit and at the same time ordering the SEC
Jose Abejo and Aurora Abejo (hereinafter referred to as the Hearing Committee to continue with the hearings of the
Abejos) and the purchaser, Telectronic Systems, Inc. 3. In SEC Case No.02395, they prayed for injunction and a Abejos and Telectronics SEC Cases Nos. 02379 and 02395,
(hereinafter referred to as Telectronics) of their 133,000 temporary restraining order that the SEC enjoin the Bragas ruhng that the "issue is not the ownership of shares but
minority shareholdings (for P5 million) and of 63,000 shares from disbursing or disposing funds and assets of Pocket Bell rather the nonperformance by the Corporate Secretary of
registered in the name of Virginia Braga and covered by and from performing such other acts pertaining to the the ministerial duty of recording transfers of shares of
five stock certificates endorsed in blank by her (for functions of corporate officers. stock of the corporation of which he is secretary."
P1,674,450.00), and the spouses Agapito Braga and Virginia
Braga (hereinafter referred to as the Bragas), erstwhile 4. Pocket Bell's corporate secretary, Norberto Braga, filed a 10. On May 15, 1984 the Bragas filed a motion for
majority stockholders. With the said purchases, Motion to Dismiss the mandamus case (SEC Case No. 02379) reconsideration but the SEC en banc denied the same on
Telectronics would become the majority stockholder, contending that the SEC has no jurisdiction over the nature August 9, 1984.
holding 56% of the outstanding stock and voting power of of the action since it does not involve an intracorporate
the corporation Pocket Bell. controversy between stockholders, the principal petitioners C. BRAGAS' ACTION IN CFI (NOWRTC)
therein, Telectronics, not being a stockholder of record of
With the said purchases in 1982, Telectronics requested the Pocket Bell. 11. On November 25, 1982, following the corporate
corporate secretary of the corporation, Norberto Braga, to secretary's refusal to register the transfer of the shares in
register and transfer to its name, and those of its nominees 5. On January 8, 1983, SEC Hearing Officer Joaquin question, the Bragas filed a complaint against the Abejos
the total 196,000 Pocket Bell shares in the corporation's Garaygay denied the motion. On January 14, 1983, the and Telectronics in the Court of First Instance of Pasig,
transfer book, cancel the surrendered certificates of stock corporate secretary filed a Motion for Reconsideration. On Branch 21 (now the Regional Trial Court, Branch 160)
and issue the corresponding new certificates of stock in its March 21, 1983, SEC Hearing Officer Joaquin Garaygay docketed as Civil Case No. 48746 for: (a) rescission and
name and those of its nominees. annulment of the sale of the shares of stock in Pocket Bell

Page 3 of 160
CORPORATION LAW CASES MAYORDO, M.A
LLB 3

made by the Abejos in favor of Telectronics on the ground performed without jurisdiction and with grave abuse of 1. The SEC ruling upholding its primary and exclusive
that it violated the Bragas' alleged pre-emptive right over discretion, filed their herein Petition for certiorari and jurisdiction over the dispute is correctly premised on, and
the Abej os' shareholdings and an alleged perfected Prohibition with Preliminary Injunction. They prayed that fully supported by, the applicable provisions of P.D. No.
contract with the Abejos to sell the same shares in their the challenged orders of respondent Judge dated February 902-A which reorganized the SEC with additional powers "in
(Bragas) favor, (Ist cause of action); plus damages for bad 14, 1983 and March 11, 1983 be set aside for lack of line with the government's policy of encouraging
faith; and (b) declaration ofnullity of any transfer, jurisdiction and that he be ordered to permanently desist investments, both domestic and foreign, and more active
assignment or endorsement of Virginia Bragas' stock from further proceedings in Civil Case No. 48746. publicParticipation in the affairs of private corporations
certificates for 63,000 shares in Pocket Ben to Telectronics Respondent judge desisted from further proceedings in the and enterprises through which desirable activities may be
for want of consent and consideration, alleging that said case, dispensing with the need of issuing any restraining pursued for the promotion of economic development; and,
stock certificates, which were intended as security for a order. to promote a wider and more meaningful equitable
loan application and were thus endorsed by her in blank, distribution of wealth," and accordingly provided that:
had been lost (2nd cause of action). E. BRAGAS' PETITION AT BAR
SEC. 3. The Commission shall have absolute
12. On January 4, 1983, the Abejos filed a Motion to Dismiss 17. On August 29, 1984, the Bragas, alleging in turn that jurisdiction, supervision and control ouer all
the complaint on the ground that it is the SEC that is the SEC has no jurisdiction over SEC Cases Nos. 02379 and corporations, partnerships or associations, who are
vested under PD 902-A with original and exclusive 02395 and that it acted arbitrarily, whimsically and the grantees of primary franchise and/or a license
jurisdiction to hear and decide cases involving, among capriciously in dismissing their petition (in SEC Case No. EB or permit issued by the government to operate in
others, controversies "between and among stockholders" #049) for dismissal of the said cases, filed their herein the Philippines; ...
and that the Bragas' suit is such a controversy as the issues Petition for certiorari and Prohibition with Preliminary
involved therein are the stockholders' alleged pre-emptive Injunction or TRO. The petitioner seeks the reversal and/or SEC. 5. In addition to the regulatory and
rights, the validity of the transfer and endorsement of setting aside of the SEC Order dated May 15, 1984 adjudicative functions of the Securities and
certificates of stock, the election of corporate officers and dismissing their petition in said SEC Case No. EB #049 and Exchange Commission over corporations,
the management and control of the corporation's sustaining its jurisdiction over SEC Cases Nos. 02379 and partnerships and other forms of associations
operations. The dismissal motion was granted by Presiding 02395, filed by the Abejos. On September 24, 1984, this registered with it as expressly granted under
Judge G. Pineda on January 14, 1983. Court issued a temporary restraining order to maintain the existing laws and decrees, it shall have original and
status quo and restrained the SEC and/or any of its officers exclusive jurisdiction to hear and decide cases
13. On January 24, 1983, the Bragas filed a motion for or hearing committees from further proceeding with the involving:
reconsideration. The Abejos opposed. Meanwhile, hearings in SEC Cases Nos. 02379 and 02395 and from
respondent Judge Rafael de la Cruz was appointed enforcing any and all orders and/or resolutions issued in a) Devices or schemes employed by or any acts,
presiding judge of the court (renamed Regional Trial Court) connection with the said cases. of the board of directors, business associations,
in place of Judge G. Pineda. its officers or partners, amounting to fruud and
14. On February 14, 1983, respondent Judge de la Cruz The cases, having been given due course, were jointly misrepresentation which may be detrimental to
issued an order rescinding the January 14, 1983 order and heard by the Court on March 27, 1985 and the parties the interest of the public andlor of the
reviving the temporary restraining order previously issued thereafter filed on April 16, 1985 their respective stockholder, partners, members of associations
on December 23, 1982 restraining Telectronics' agents or memoranda in amplification of oral argument on the points or organizations registered with the
representatives from enforcing their resolution constituting of law that were crystalled during the hearing, Commission.
themselves as the new set of officers of Pocket Bell and
from assuming control of the corporation and discharging The Court rules that the SEC has original and exclusive b) Controversies arising out of intracorporate
their functions. jurisdiction over the dispute between the principal or partnership relations, between and among
stockholders of the corporation Pocket Bell, namely, the stockholders, members, or associates; between
15. On March 2, 1983, the Abejos filed a motion for Abejos and Telectronics, the purchasers of the 56% majority any andlor all of them and the corporation,
reconsideration, which motion was duly opposed by the stock (supra, at page 2) on the one hand, and the Bragas, partnership or association of which they are
Bragas. On March 11, 1983, respondent Judge denied the erstwhile majority stockholders, on the other, and that the stockholders, members or assmiates,
motion for reconsideration. SEC, through its en banc Resolution of May 15, 1984 respectively; and between such corporation,
co"ectly ruled in dismissing the Bragas' Petition questioning partnership or assmiation and the state insofar
D. ABEJOS' PETITION AT BAR its jurisdiction, that "the issue is not the ownership of as it concems their individual franchise or right
shares but rather the nonperformance by the Corporate to exist as such entity;
16. On March 26, 1983, the Abejos, alleging that the acts of Secretary of the ministerial duty of recording transfers of
respondent Judge in refusing to dismiss the complaint shares of stock of the Corporation of which he is secretary." c) Controversies in the election or
despite clear lack of jurisdiction over the action and in appointments of directors, trustees, officers or
refusing to reconsider his erroneous position were

Page 4 of 160
CORPORATION LAW CASES MAYORDO, M.A
LLB 3

managers of such corporations, partnerships or the case of Virginia Braga's shares. Such dispute c learly elect the corporate directors and officers and manage its
associations. 3 involve's controversies "between and among stockholders, " business and operations, which falls under paragraph (c).
as to the Abej os' right to sell and dispose of their shares to
Section 6 further grants the SEC "in order to effectively Telectronics, the validity of the latter's acquisition of 5. Most of the cases that have come to this Court involve
exercise such jurisdiction," the power, inter alia, "to issue Virginia Braga's shares, who between the Bragas and the those under paragraph (b), i.e. whether the controversy is
preliminary or permanent injunctions, whether prohibitory Abejos' transferee should be recognized as the controlling an intra-corporate one, arising "between and among
or mandatory, in all cases in which it has jurisdiction, and shareholders of the corporation, with the right to elect the stockholders" or "between any or allof them and the
in which cases the pertinent provisions of the Rules of corporate officers and the management and control of its corporation." The parties have focused their arguments on
Court shall apply." operations. Such a dispute and case clearly fag within the this question. The Bragas' contention in his field must
original and exclusive jurisdiction of the SEC to decide, likewise fail. In Philex Mining Corp. v. Reyes, 8 the Court
2. Basically and indubitably, the dispute at bar, as held by under Section 5 of P.D. 902-A, above-quoted. The spelled out that"'an intra-corporate controversy is one
the SEC, is an intracorporate dispute that has arisen restraining order issued by the Regional Trial Court which arises between a stockholder and the corporation.
between and among the principal stockholders of the restraining Telectronics agents and representatives from There is no distinction, qualification, nor any exemption
corporation Pocket Bell due to the refusal of the corporate enforcing their resolution constituting themselves as the whatsoever. The provision is broad and covers all kinds of
secretary, backed up by his parents as erstwhile majority new set of officers of Pocket Bell and from assuming controversies between stockholders and corporations. The
shareholders, to perform his "ministerial duty" to record the control of the corporation and discharging their functions issue of whether or not a corporation is bound to replace a
transfers of the corporation's controlling (56%) shares of patently encroached upon the SEC's exclusive jurisdiction stockholder's lost certificate of stock is a matter purely
stock, covered by duly endorsed certificates of stock, in over such specialized corporate controversies calling for its between a stockholder and the corporation. It is a typical
favor of Telectronics as the purchaser thereof. mandamus special competence. As stressed by the Solicitor General on intra-corporate dispute. The quqsjion of damage's raised is
in the SEC to compel the corporate secretary to register behalf of the SEC, the Court has held that "Nowhere does merely incidental to that main issue. The Court rejected
the transfers and issue new certificates in favor of the law [PD 902-A] empower any Court of First Instance the stockholders' theory of excluding his complaint (for
Telectronics and its nominees was properly resorted to [now Regional Trial Court] to interfere with the orders of replacement of a lost stock [dividend] certificate which he
under Rule XXI, Section 1 of the SEC's New Rules of the Commission," 5 and consequently "any ruling by the trial claimed to have never received) from the classification of
Procedure, 4 which provides for the filing of such petitions court on the issue of ownership of the shares of stock is not intra-corporate controversies as one that "does not square
with the SEC. Section 3 of said Rules further authorizes the binding on the Commission 6 for want of jurisdiction. with the intent of the law, which is to segregate from the
SEC to "issue orders expediting the proceedings ... and also general jurisdiction of regular Courts controversies
[to] grant a preliminary injunction for the preservation of 4. The dispute therefore clearly falls within the general involving corporations and their stockholders and to bring
the rights of the parties pending such proceedings, " classification of cases within the SEC's original and them to the SEC for exclusive resolution, in much the same
exclusive jurisdiction to hear and decide, under the way that labor disputes are now brought to the Ministry-of
The claims of the Bragas, which they assert in their aforequoted governing section 5 of the law. Insofar as the Labor and Employment (MOLE) and the National Labor
complaint in the Regional Trial Court, praying for rescission Bragas and their corporate secretary's refusal on behalf of Relations Commission (NLRC), and not to the Courts."
and annulment of the sale made by the Abejos in favor of the corporation Pocket Bell to record the transfer of the
Telectronics on the ground that they had an alleged 56% majority shares to Telectronics may be deemed a (a) The Bragas contend that Telectronics, as
perfected preemptive right over the Abejos' shares as well device or scheme amounting to fraud and buyertransferee of the 56% majority shares is not a
as for annulment of sale to Telectronics of Virginia Braga's misrepresentation emplolyed by them to keep themselves registered stockholder, because they, through their
shares covered by street certificates duly endorsed by her in control of the corporation to the detriment of son the corporate secretary, appear to have refused
in blank, may in no way deprive the SEC of its primary and Telectronics (as buyer and substantial investor in the to perform "the ministerial duty of recording
exclusive jurisdiction to grant or not the writ of mandamus corporate stock) and the Abejos (as substantial transfers of shares of stock of the corporation of
ordering the registration of the shares so transferred. The stockholders-sellers), the case falls under paragraph (a). which he is the secretary," and that the dispute is
Bragas' contention that the question of ordering the The dispute is likewise an intra-corporate controversy therefore, not an intracorporate one. This
recording of the transfers ultimately hinges on the question between and among the majority and minority stockholders contention begs the question which must properly
of ownership or right thereto over the shares as to the transfer and disposition of the controlling shares be resolved by the SEC, but which they would
notwithstanding, the jurisdiction over the dispute is clearly of the corporation, failing under paragraph (b). As stressed prevent by their own act, through their son, of
vested in the SEC. by the Court in DMRC Enterprises v. Este del Sol Mountain blocking the due recording of the transfer and
Reserve, Inc, 7 Considering the announced policy of PD 902- cannot be sanctioned. It can be seen from their
3. The very complaint of the Bragas for annulment of the A, the expanded jurisdiction of the respondent Securities very complaint in the regular courts that they with
sales and transfers as filed by them in the regular court and Exchange Commission under said decree extends their two sons constituting the plaintiffs are all
questions the validity of the transfer and endorsement of exclusively to matters arising from contracts involving stockholders while the defendants are the Abejos
the certificates of stock, claiming alleged pre-emptive investments in private corporations, partnerships and who are also stockholders whose sale of the shares
rights in the case of the Abejos' shares and alleged loss of associations." The dispute also concerns the fundamental to Telectronics they would annul.
thio certificates and lack of consent and consideration in issue ofwhether the Bragas or Telectronics have the right to

Page 5 of 160
CORPORATION LAW CASES MAYORDO, M.A
LLB 3

(b) There can be no question that the dispute certificates for 63,000 shares equivalent to 18% of special knowledge, experience, and seruices of the
between the Abejos and the Bragas as to the sale the corporation's outstanding stock and received administratiue tribunal to determine technical and
and transfer of the former's shares to Telectronics the cash price thereof. 13 But as to the sale and intricate matters of fact, and a uniformity of ruling is
for P5 million is an intracorporate one under transfer of the Abejos' shares, the Bragas cannot essential to comply uith the purposes of the regulatory
section 5 (b), prescinding from the applicability of oust the SEC of its original and exclusive statute administered " 16
section 5 (a) and (c), (supra, par. 4) lt is the SEC jurisdiction to hear and decide the case, by
which must resolve the Bragas' claim in their own blocking through the corporate secretary, their son, In this era of clogged court dockets, the need for
complaint in the court case filed by them of an the due recording of the transfer and sale of the specialized administrative boards or commissions with the
alleged pre-emptive right to buy the Abejos' shares shares in question and claiming that Telectronics is special knowledge, experience and capability to hear and
by virtue of "on-going negotiations," which they not a stockholder of the corporation – which is the determine promptly disputes on technical matters or
may submit as their defense to the mandamus very issue that the SEC is called upon to resolve. As essentially factual matters, subject to judicial review in
petition to register the sale of the shares to the SEC maintains, "There is no requirement that a case of grave abuse of discretion, has become well nigh
Telectronics. But asserting such preemptive rights stockholder of a corporation must be a registered indispensable. Thus, in 1984, the Court noted that
and asking that the same be enforced is a far cry one in order that,the Securities and Exchange "between the power lodged in an administrative body and a
from the Bragas' claim that "the case relates to Commission may take cognizance of a suit seeking court, the unmistakable trend has been to refer it to the
questions of ownership" over the shares in question. to enforce his rights as such stockholder." 14 This is former. 'Increasingly, this Court has been committed to the
9 (Not to mention, as pointed out by the Abejos, because the SEC by express mandate has "absolute view that unless the law speaks clearly and unequivocably,
that the corporation is not a close corporation, and jurisdiction, supervision and control over all the choice should fall on [an administrative agency.]' " 17
no restriction over the free transferability of the corporations" and is called upon to enforce the The Court in the earlier case of Ebon vs. De Guzman 18
shares appears in the Articles of Incorporation, as provisions of the Corporation Code, among which is noted that the lawmaking authority, in restoring to the
well as in the by-laws 10 and the certificates of the stock purchaser's right to secure the labor arbiters and the NLRC their jurisdiction to award all
stock themselves, as required by law for the corresponding certificate in his name under the kinds of damages in labor cases, as against the previous
enforcement of such restriction. See Go Soc & Sons, provisions of Section 63 of the Code. Needless to P.D. amendment splitting their jurisdiction with the regular
etc. v. IAC, G.R. No. 72342, Resolution of February say, any problem encountered in securing the courts, "evidently ... had second thoughts about depriving
19, 1987.) certificates of stock representing the investment the Labor Arbiters and the NLRC of the jurisdiction to
made by the buyer must be expeditiously dealt with award damages in labor cases because that setup would
(c) The dispute between the Bragas and through administrative mandamus proceedings with mean duplicity of suits, splitting the cause of action and
Telectronics as to the sale and transfer for the SEC, rather than through the usual tedious possible conflicting findings and conclusions by two
P1,674,450.00 of Virginia Braga's 63.000 shares regular court procedure. Furthermore, as stated in tribunals on one and the same claim."
covered by Street certificates duly endorsed in the SEC order of April 13, 1983, notice given to the
blank by her is within the special competence and corporation of the sale of the shares and 7. Thus, the Corporation Code (B.P. No. 178) enacted on
jurisdiction of the SEC, dealing as it does with the presentation of the certificates for transfer May 1, 1980 specifically vests the SEC with the Rule-making
free transferability of corporate shares, particularly is ,equivalent to registration: "Whether the refusal power in the discharge of its task of implementing the
street certificates," as guaranteed by the of the (corporation) to effect the same is ivalid or provisions of the Code and particularly charges it with the
Corporation Code and its proclaimed policy of not is still subject to the outcome of the hearing on duty of preventing fraud and abuses on the part of
encouraging foreign and domestic investments in the merits of the case. 15 controlling stockholders, directors and officers, as follows:
Philippine private corpora. tions and more active
public participation therein for the Promotion of 6. In the fifties, the Court taking cognizance of the move to SEC. 143. Rule-making power of the Securities and
economic development. Here again, Virginia Braga's vest jurisdiction in administrative commissions and boards Exchange Commission. — The Securities and
claim of loss of her street certificates 11 or theft the power to resolve specialized disputes in the field of Exchange Commission shall have the power and
thereof (denounced by Telectronics as 11 labor (as in corporations, public transportation and public authority to implement the provisions of this Code,
perjurious" 12 ) must be pleaded by her as a utilities) ruled that Congress in requiring the Industrial and to promulgate rules and regulations reasonably
defense against Telectronics'petition for mandamus Court's intervention in the resolution of labor-management necessary to enable it to perform its duties
and recognition now as the controlling stockholder controversies likely to cause strikes or lockouts meant such hereunder, particularly in the prevention of fraud
of the corporation in the light of the joint affidavit jurisdiction to be exclusive, although it did not so expressly and abuses on the part of the controlling
of Geneml Cerefino S. Carreon of the National state in the law. The Court held that under the "sense- stockholders, members, directors, trustees or
Telecommunications Commission and private making and expeditious doctrine of primary jurisdiction ... officers. (Emphasis supplied)
respondent Jose Luis Santiago of Telectronics the courts cannot or will n6t determine a controversy
narrating the facts and circumstances of how the involving a question which is within the jurisdiction of an The dispute between the contending parties for control of
former sold and delivered to Telectronics on behalf administrative tribunal, where the question demands the thecorporation manifestly fans within the primary and
of his compadres, the Bragas, Virginia Braga's street exercise of sound administrative discretion requiring the exclusive jurisdiction of the SEC in whom the law has

Page 6 of 160
CORPORATION LAW CASES MAYORDO, M.A
LLB 3

reserved such jurisdiction as an administrative agency of Union Glass' defenses as a third party buyer in good faith. restraining order or injunction filed by the Bragas seeking
special competence to deal promptly and expeditiously Similarly, in the DMRC case, therein petitioner's,tomplaint the disbandment of the Hearing Committee and the setting
therewith. for collection of the amounts due to it as payment of aside of its Orders, and its Resolution of August 9, 1984,
rentals for the lease of its heavy equipment in the form denying reconsideration (Annex "X", Petition), due to the
As the Court stressed in Union Glass & Container Corp. v. mainly of cash and part in shares of stock of the debtor- Bragas' filing of the petition at bar.
SEC, 19 "This grant of jurisdiction [in Section 51 must be defendant corporation was held to be not covered by the
viewed in the light of the nature and functions of the SEC SEC's exclusive jurisdiction over intracorporate disputes, Prescinding from the great concern of damage and
under the law. Section 3 of PD No. 902-A confers upon the since "to pass upon a money claim under a lease contract prejudice expressed by Telectronics due to the Bragas
latter 'absolute jurisdiction, supervision, and control over would be beyond the competence Of the Securities and having remained in control of the corporation and having
all corporations, partnerships or associations, who are Exchange Commission and to separate the claim for money allegedly committed acts of gross mismanagement and
grantees of primary franchise and/or license or permit from the claim for shares of stock would be splitting a misapplication of funds, the Court finds that under the
issued by the government to operate in the Philippines ... single cause of action resulting in a multiplicity of suitS." 21 facts and circumstances of record, it is but fair and just
The principal function of the SEC is the supervision and Such an action for collection of a debt does not involve that the SEC's order creating a receivership committee be
control over corporations, partnerships and associations enforcement Of rights and obligations under the implemented forthwith, in accordance with its terms, as
with the end in view that investment in these entities may Corporation Code nor the in. temal or intracorporate affairs follows:
be encouraged and protected, and their activities pursued of the debtor corporation. But in aR disputes affecting and
for the promotion of economic development. dealing With the interests of the corporation and its The three-man receivership committee shall be
stockholders, following the trend and clear legislative composed of a representative from the commission,
"It is in aid of this office that the adjudicative power of the intent of entmsting all disputes of a specialized nature to in the person of the Director, Examiners and
SEC must be exercised. Thus the law explicitly specified administrative agencies possessing. the requisite Appraisers Department or his designated
and delin-dted its jurisdiction to matters intrinsically competence, special knowledge, experience and services representative, and a representative from the
connected with the regulation of corporations, partnerships and facilities to expeditiously resolve them and determine petitioners and a representative of the respondent.
and associations and those dealing with the internal affairs the essential facts including technical and intricate
of such corporations, partnerships or associations. matters, as in labor and public utilities rates disputes, the The petitioners and respondent are therefore
SEC has been given "the original and exclusive jurisdiction directed to sub. mit to the Commission the name of
"Otherwise stated, in order that the SEC can take to hear anddecide" them (under section 5 of P.D. 902-A) "in their designated representative within three (3)
cognizance of a case, the controversy must pertain to any addition to [its] regulatory and adjudicative days from receipt of this order. The Conunission
of the following relationships: [al between the corporation, functions" (under Section 3, vesting in it "absolute shall appoint the other representatives if either or
partnership or association and the public; [b] between the jurisdiction, supervision and control over all corporations" both parties fafl to comply with the requirement
corporation, partnership or association and its and the Rule-making power granted it in Section 143 of the within the stated time.
stockholders, partners, members, or officers; [c] between Corporation Code, supra). As stressed by the Court in the
the corporation, partnership or association and the state in Philex case, supra, "(T)here is no distinction, qualification, ACCORDINGLY, judgment is hereby rendered:
so far as its franchise, permit or license to operate is nor any exemption whatsoever. The provision is broad and
concerned; and Id] among the stockholders, partners or covers all kinds of controversies between stockholders and (a) Granting the petition in G.R. No. 63558,
associates themselves." 20 corporations." annulling the challenged Orders of respondent
Judge clated February 14, 1983 and March i 1, 1983
Parenthetically, the cited case of Union Glass illustrates by It only remains now to deal with the Order dated April 15, (Annexes "L" and "P" of the Abejos' petition) and
way of contrast what disputes do not fall within the special 1983 (Annex H, Petition) 22 of the SEC's three-member prohibiting respondent Judge from further
jurisdiction of the SEC. In this case, the SEC had properly Hearing Conunittee granting Telectronics' motion for proceeding in Civil Case No. 48746 filed in his Court
assumed jurisdiction over the dissenting stockholders' com. creation of a receivership or management committee with other than to dismiss the same for lack or
Plaint against the corporation Pioneer Glass questioning its the ample powers therein enumerated for the preservation jurisdiction over the subject-matter;
dacion en pago of its glass plant and all its assets in favor pendente lite of the corporation's assets and in discharge of
of the DBP which was clearly an intra-corporate controversy its "power and duty to preserve the rights of the parties, (b) Dismissing the petition in G.R. Nos. 68450-51
dealing with its internal affairs. But the Court held that the the stockholders, the public availing of the corporation's and lifting the temporary restraining order issued
SEC had no jurisdiction over petitioner Union Glass Corp., services and the rights of creditors," as well as "for reasons on September 24, 1984, effective immediately upon
imPle,aded as third party purchaser of the plant from DBP of equity and justice ... (and) to prevent possible promulgation hereof,
in the action to annul the dacion en pago. The Court held paralization of corporate business." The said Order has not
that such action for recovery of the glass plant could be been implemented notwithstanding its having been upheld (c) Directing the SEC through its Hearing Committee
brought by the dissenting stockholder to the regular courts per the SEC en banc's Order of May 15, 1984 (Annex "V", to proceed immediately with hearing and resolving
only if and when the SE C rendered final judgment Petition) dismissing for lack of merit the petition for the pending mandamus petition for recording in the
annulling the dacion en pago and furthermore subject to certiorari, prohibition and mandamus with prayer for corporate books the transfer to Telectronics and its

Page 7 of 160
CORPORATION LAW CASES MAYORDO, M.A
LLB 3

nominees of the majority (56%) shares of stock of (a) P50,000.00, the principal obligation, plus
the corporation Pocket Bell pertaining to the Abejos On September 12, 1983 with expired permit to issue interest at the legal rate from September 12, 1983
and Virginia Braga and all related issues, taking into commercial papers (Ibid., p. 8) and with intention not to until the full amount is paid;
consideration, without need of resubmittal to it, pay or defraud its creditors, Premiere induced and misled (b) P10,000.00 both for moral and exemplary
the pleadings, annexes and exhibits filed by the Magalad into making a money market placement of damages;
contending parties in the cases at bar; and P50,000.00 at 22% interest per annum for which it issued a (c) P5,000.00 for and as attorney's fees; and
receipt (Ibid., Exh. "B", p. 8). Aside from the receipt, (d) the costs of suit.
(d) Likewise directing the SEC through its Hearing Premier likewise issued two (2) post-dated checks in the SO ORDERED. (Ibid., p. 30)
Committee to proceed immediately with the total sum of P51,079.00 (Ibid., Exh. "C", p. 9) and assigned
implementation of its receivership or management to Magalad its receivable from a certain David Saman for Premiere filed a motion for reconsideration of the foregoing
committee Order of April 15, 1983 in SEC Case No. the same amount (Ibid., Exh. "C", p. 10). decision, based principally on a question of law alleging
2379 and for the purpose, the contending parties that the Securities and Exchange Commission (SEC) has
are ordered to submit to said Hearing Committee When the said checks were presented for payment on their exclusive and original jurisdiction over a corporation under
the name of their designated representatives in the due dates, the drawee bank dishonored the checks for lack a state of suspension of payments (Ibid., pp. 32-41).
receivership/management committee within three of sufficient funds to cover the amount (Ibid., Exhs. "D-1",
(3) days from receipt of this decision, on pain of "E-1", pp. 11-12). Despite demands by Magalad for the Magalad filed an opposition to the motion for
forfeiture of such right in case of failure to comply replacement of said checks with cash, Premiere, for no reconsideration on January 8, 1985 alleging among others
herewith, as provided in the said Order; and valid reason, failed and refused to honor such demands and that the regular court has jurisdiction over the case to the
ordering the Bragas to perform only caretaker acts due to fraudulent acts of Premiere, Magalad suffered exclusion of the SEC. (Ibid., pp. 51-53).
in the corporation pending the organization of such sleepless nights, mental anguish, fright, serious anxiety,
receivership/management committee and considering the fact that the money she invested is blood On May 28, 1986 the lower court issued an order denying
assumption of its functions. money and is the only source of support for her family the motion for reconsideration (Ibid., p. 61).
(Ibid., p. 4).
This decision shall be immediately executory upon its Magalad in order to seek redress and retrieve her blood On June 11, 1986 Premiere filed his notice of appeal which
promulgation. money, availed of the service of counsel for which she led to the issuance of the order of the lower court dated
SO ORDERED. agreed to pay twenty percent (20%) of the amount due as July 29, 1986 elevating the case to the Court of Appeals
and for attorney's fees (Ibid.) (CA) (Ibid., pp. 62-63).
G.R. No. 87135 May 22, 1992 The Court of Appeals in its resolution dated September 8,
ALMA MAGALAD, petitioner, vs. PREMIERE FINANCING On January 10, 1984, Magalad filed a complaint for 1987 dismissed the case for failure of Premiere to file its
CORP., respondent. damages with prayer for writ of preliminary attachment brief despite the ninety-day extension granted to it, which
with the RTC, Branch LXXXV, Quezon City, docketed as Civil expired on June 10, 1987 (Rollo, p. 16).
PARAS, J.: Case No. Q-40392 against herein Premiere (Ibid., p. 3-6).
An omnibus motion for reconsideration and admission of
This is an appeal originally filed with the Court of Appeals Premiere having failed to file an answer and acting on late filing of Premiere's brief was filed on September 22,
but certified to this court for disposition since it involves Magalad's motion, the lower court declared Premiere in 1987 (Rollo, pp. 17-19; 32).
purely questions of law from the decision of the Regional default by virtue of an order dated April 5, 1984 allowing
Trial Court (RTC), Branch LXXXV, Quezon City, dated May Magalad to present evidence ex-parte (Ibid., pp. 21; 22) On September 30, 1987 the Court of Appeals issued a
22, 1984, in Civil Case No. Q-40392, ordering the resolution which reconsidered its previous resolution dated
defendant-appellant Premiere Financing Corporation On May 22, 1984 the lower court rendered a default September 5, 1987 and admitted the Premiere's brief
(Premiere for short) to pay to the plaintiff-appellee Alma judgment against Premiere, the dispositive portion of which (Rollo, p. 26).
Magalad (Magalad for short) the sum of: 
 reads:
(a) P50,000.00, the principal obligation, plus interest at the On January 31, 1989 the Court of Appeals issued a
legal rate from September 12, 1983, until the full amount is From the foregoing evidence, the court finds that resolution certifying the instant case to this Court on the
paid; (b) P10,000.00, both for moral and exemplary plaintiff has fully established her claim that ground that the case involves a question of law, the
damages; (c) P5,000.00, for and as attorney's fees and (d) defendant had indeed acted fraudulently in dispositive part of which stating:
the costs of suit. incurring the obligation and considering that no
The antecedent facts of the case are as follows: evidence has been adduced by the defendant to ACCORDINGLY, pursuant to Rule 50, Sec. 3, in
contradict the same, judgment is hereby rendered relation to the Judiciary Act of 1948, Sec. 17, par.
Premiere is a financing company engaged in soliciting and ordering the defendant to pay plaintiff as follows: 4(3) (4), the Appeal in this case is hereby certified
accepting money market placements or deposits (Original to the Supreme Court on the ground that the only
Record, p. 29).

Page 8 of 160
CORPORATION LAW CASES MAYORDO, M.A
LLB 3

issue raised concerns the jurisdiction of the trial the public and/or to the stockholders, partners, cases, nowhere in the complaints therein is found any
court and only a question of law. (Rollo, p. 33) members of associations or organizations averment of fraud or misrepresentation committed by the
registered with the Commission. (Emphasis respective corporations involved. The causes of action,
Hence, this appeal. supplied) therefore, were nothing more than simple money claims.

The pivotal issue in this case is whether or not the court a Considering that Magalad's complaint sufficiently alleges Further bolstering the jurisdiction of the SEC in this case is
quo had jurisdiction to try the instant case. acts amounting to fraud and misrepresentation committed the fact that said agency had already appointed a
by Premiere, the SEC must be held to retain its original and Rehabilitation Receiver for Premiere and has directed all
At the very core of this appeal assailing the aforesaid exclusive jurisdiction over the case, despite the fact that proceedings or claims against it be suspended. This,
pronouncement of the lower court, and around which the suit involves collection of sums of money paid to said pursuant to Sec. 6(c) of Pres. Decree No. 902-A providing
revolve the arguments of the parties, is the applicability of corporation, the recovery of which would ordinarily fall that "upon appointment of a . . . rehabilitation
Presidential Decree No. 902-A (Reorganization of the SEC within the jurisdiction of regular courts. The fraud receiver . . . all actions for claims against corporations . . .
with Additional Powers), as amended by Presidential committed is detrimental to the interest of the public and, under receivership pending before any court, tribunal,
Decrees Nos. 1653, 1758 and 1799. Magalad submits that therefore, encompasses a category of relationship within board or body shall be suspended accordingly."
the legal suit which she has brought against Premiere is an the SEC jurisdiction.
ordinary action for damages with the preliminary By so doing, SEC has exercised its original and exclusive
attachment cognizable solely by the RTC. Premiere, on the Otherwise stated, in order that the SEC can take jurisdiction to hear and decide cases involving:
other hand, espouses the original and exclusive jurisdiction cognizance of a case, the controversy must pertain to any
of the Securities and Exchange Commission. of the following relationships: (a) between the corporation, a) Petitions of corporations, partnerships or
partnership or association and the public; (b) between the associations to be declared in the state of
Presidential Decree No. 902-A, Section 3, provides: corporation, partnership or association and its suspension of payments in cases where the
stockholders, partners, members or officers; (c) between corporation, partnership or association possesses
Sec. 3. The Commission shall have absolute the corporation, partnership or association and the state so sufficient property to cover all its debts but
jurisdiction, supervision and control over all far as its franchise, permit or license to operate is foresees the impossibility of meeting them when
corporations, partnerships or associations, who are concerned; and (d) among the stockholders, partners or they respectively fall due or in cases where the
the grantees of primary franchises and/or a license associates themselves (Union Glass & Container Corp. v. corporation, partnership or association has no
or permit issued by the government to operate in SEC, 126 SCRA 31; 38; 1983; Abejo v. De la Cruz, 149 SCRA sufficient assets to cover its liabilities but is under
the Philippines; and in the exercise of its authority, 654, 1987). the management of a Rehabilitation Receiver or
it shall have the power to enlist the aid and support In this case, the recitals of the complaint sufficiently allege Management of a Rehabilitation Receiver or
of and to deputize any and all enforcement that devices or schemes amounting to fraud and Management Committee created pursuant to this
agencies of the government, civil or military as well misrepresentation detrimental to the interest of the public Decree. (Section 5(d) of Pres. Decree No. 902-A as
as any private institution, corporation, firm, have been resorted to by Premiere Corporation. It can not added by Pres. Decree 1758).
association or person. (As amended by Presidential but be conceded, therefore, that the SEC may exercise its
Decree No. 1758). adjudicative powers pursuant to Sec. 5(a) of Pres. Decree In fine, the adjudicative powers of the SEC being clearly
No. 902-A (Supra). defined by law, its jurisdiction over this case has to be
Sec. 3 of Pres. Decree No. 902-A should also be read in upheld.
conjunction with Sec. 5 of the same law, providing: The fact that Premiere's authority to engage in financing
already expired will not have the effect of divesting the PREMISES CONSIDERED, the instant appeal is GRANTED, and
Sec. 5. In addition to the regulatory and SEC of its original and exclusive jurisdiction. The expanded the order of the Presiding Judge of the Regional Trial Court,
adjudicative functions of the Securities and jurisdiction of the SEC was conceived primarily to protect Quezon City, Branch LXXXV dated May 22, 1984, in Civil
Exchange Commission over corporations, the interest of the investing public. That Magalad's money Case No. Q-40392 is REVERSED and SET ASIDE, without
partnerships and other forms of associations placements were in the nature of investments in Premiere prejudice to the filing by Alma Magalad of the appropriate
registered with it as expressly granted under the can not be gainsaid. Magalad had reasonably expected to complaint against Premiere Financing Corporation with the
existing laws and decrees, it shall have original and receive returns from moneys she had paid to Premiere. Securities and Exchange Commission.
exclusive jurisdiction to hear and decide cases Unfortunately, however, she was the victim of alleged fraud SO ORDERED.
involving: and misrepresentation.
G.R. No. L-12719             May 31, 1962
a) Devises or schemes employed by or any acts Reliance by Magalad on the cases of DMRC v. Este del Sol, THE COLLECTOR OF INTERNAL REVENUE, petitioner, 

of the Board of Directors, business associates, (132 SCRA 293) and Union Glass & Container Corp. v. SEC vs. THE CLUB FILIPINO, INC. DE CEBU, respondent.
its officers or partners, amounting to fraud and (126 SCRA 31), where the jurisdiction of the ordinary Courts
misrepresentation which may be detrimental to was upheld, is misplaced for, as explicitly stated in those PAREDES, J.:

Page 9 of 160
CORPORATION LAW CASES MAYORDO, M.A
LLB 3

This is a petition to review the decision of the Court of Tax facts of which are similar to the ones at bar; Manila Polo
As fixed tax for the years 1946 to
Appeals, reversing the decision of the Collector of Internal 70.00 Club v. B. L. Meer, etc., No. L-10854, Jan. 27, 1960).
1952
Revenue, assessing against and demanding from the "Club
Filipino, Inc. de Cebu", the sum of P12,068.84 as fixed and Compromise penalty 500.00 Having found as a fact that the Club was organized to
percentage taxes, surcharge and compromise penalty, develop and cultivate sports of all class and denomination,
allegedly due from it as a keeper of bar and restaurant. The Club wrote the Collector, requesting for the for the healthful recreation and entertainment of its
cancellation of the assessment. The request having been stockholders and members; that upon its dissolution, its
As found by the Court of Tax Appeals, the "Club Filipino, denied, the Club filed the instant petition for review. remaining assets, after paying debts, shall be donated to a
Inc. de Cebu," (Club, for short), is a civic corporation charitable Philippine Institution in Cebu; that it is operated
organized under the laws of the Philippines with an original The dominant issues involved in this case are twofold: mainly with funds derived from membership fees and dues;
authorized capital stock of P22,000.00, which was that the Club's bar and restaurant catered only to its
subsequently increased to P200,000.00, among others, to it 1. Whether the respondent Club is liable for the payment of members and their guests; that there was in fact no cash
"proporcionar, operar, y mantener un campo de golf, tenis, the sum of 12,068.84, as fixed and percentage taxes and dividend distribution to its stockholders and that whatever
gimnesio (gymnasiums), juego de bolos (bowling alleys), surcharges prescribed in sections 182, 183 and 191 of the was derived on retail from its bar and restaurant was used
mesas de billar y pool, y toda clase de juegos no prohibidos Tax Code, under which the assessment was made, in to defray its overall overhead expenses and to improve its
por leyes generales y ordenanzas generales; y desarollar y connection with the operation of its bar and restaurant, golf-course (cost-plus-expenses-basis), it stands to reason
cultivar deportes de toda clase y denominacion cualquiera during the periods mentioned above; and that the Club is not engaged in the business of an operator
para el recreo y entrenamiento saludable de sus miembros of bar and restaurant (same authorities, cited above).
y accionistas" (sec. 2, Escritura de Incorporacion del Club 2. Whether it is liable for the payment of the sum of
Filipino, Inc. Exh. A). Neither in the articles or by-laws is P500.00 as compromise penalty. It is conceded that the Club derived profit from the
there a provision relative to dividends and their operation of its bar and restaurant, but such fact does not
distribution, although it is covenanted that upon its Section 182, of the Tax Code states, "Unless otherwise necessarily convert it into a profit-making enterprise. The
dissolution, the Club's remaining assets, after paying debts, provided, every person engaging in a business on which the bar and restaurant are necessary adjuncts of the Club to
shall be donated to a charitable Philippine Institution in percentage tax is imposed shall pay in full a fixed annual foster its purposes and the profits derived therefrom are
Cebu (Art. 27, Estatutos del Club, Exh. A-a.). tax of ten pesos for each calendar year or fraction thereof necessarily incidental to the primary object of developing
in which such person shall engage in said business." Section and cultivating sports for the healthful recreation and
The Club owns and operates a club house, a bowling alley, a 183 provides in general that "the percentage taxes on entertainment of the stockholders and members. That a
golf course (on a lot leased from the government), and a business shall be payable at the end of each calendar Club makes some profit, does not make it a profit-making
bar-restaurant where it sells wines and liquors, soft drinks, quarter in the amount lawfully due on the business Club. As has been remarked a club should always strive,
meals and short orders to its members and their guests. transacted during each quarter; etc." And section 191, whenever possible, to have surplus (Jesus Sacred Heart
The bar-restaurant was a necessary incident to the same Tax Code, provides "Percentage tax . . . Keepers of College v. Collector of Int. Rev., G.R. No. L-6807, May 24,
operation of the club and its golf-course. The club is restaurants, refreshment parlors and other eating places 1954; Collector of Int. Rev. v. Sinco Educational Corp., G.R.
operated mainly with funds derived from membership fees shall pay a tax three per centum, and keepers of bar and No. L-9276, Oct. 23, 1956).
and dues. Whatever profits it had, were used to defray its cafes where wines or liquors are served five per centum of
overhead expenses and to improve its golf-course. In 1951. their gross receipts . . .". It has been held that the liability It is claimed that unlike the two cases just cited (supra),
as a result of a capital surplus, arising from the re- for fixed and percentage taxes, as provided by these which are non-stock, the appellee Club is a stock
valuation of its real properties, the value or price of which sections, does not ipso facto attach by mere reason of the corporation. This is unmeritorious. The facts that the
increased, the Club declared stock dividends; but no actual operation of a bar and restaurant. For the liability to capital stock of the respondent Club is divided into shares,
cash dividends were distributed to the stockholders. In attach, the operator thereof must be engaged in the does not detract from the finding of the trial court that it is
1952, a BIR agent discovered that the Club has never paid business as a barkeeper and restaurateur. The plain and not engaged in the business of operator of bar and
percentage tax on the gross receipts of its bar and ordinary meaning of business is restricted to activities or restaurant. What is determinative of whether or not the
restaurant, although it secured B-4, B-9(a) and B-7 licenses. affairs where profit is the purpose or livelihood is the Club is engaged in such business is its object or purpose, as
In a letter dated December 22, 1852, the Collector of motive, and the term business when used without stated in its articles and by-laws. It is a familiar rule that
Internal Revenue assessed against and demanded from the qualification, should be construed in its plain and ordinary the actual purpose is not controlled by the corporate form
Club, the following sums: — meaning, restricted to activities for profit or livelihood or by the commercial aspect of the business prosecuted,
(The Coll. of Int. Rev. v. Manila Lodge No. 761 of the BPOE but may be shown by extrinsic evidence, including the by-
As percentage tax on its gross [Manila Elks Club] & Court of Tax Appeals, G.R. No. laws and the method of operation. From the extrinsic
receipts 
 P9,599.0 L-11176, June 29, 1959, giving full definitions of the word evidence adduced, the Tax Court concluded that the Club is
during the tax years 1946 to 1951 7 "business"; Coll. of Int. Rev. v. Sweeney, et al. [International not engaged in the business as a barkeeper and
Club of Iloilo, Inc.], G.R. No. L-12178, Aug. 21, 1959, the restaurateur.
Surcharge therein 2,399.77

Page 10 of 160
CORPORATION LAW CASES MAYORDO, M.A
LLB 3

Moreover, for a stock corporation to exist, two requisites "Edgardo D. Pabalan, et al., vs. Spouses Florentino Petitioner corporation through its president, Manuel Dulay,
must be complied with, to wit: (1) a capital stock divided Manalastas, et al.," is dismissed for lack of merits; obtained various loans for the construction of its hotel
into shares and (2) an authority to distribute to the holders project, Dulay Continental Hotel (now Frederick Hotel). It
of such shares, dividends or allotments of the surplus In Civil Case No. 8278-P, the complaint filed by even had to borrow money from petitioner Virgilio Dulay to
profits on the basis of the shares held (sec. 3, Act No. Manuel R. Dulay Enterprises, Inc. for cancellation of be able to continue the hotel project. As a result of said
1459). In the case at bar, nowhere in its articles of title of Manuel A. Torres, Jr. (TCT No. 24799 of the loan, petitioner Virgilio Dulay occupied one of the unit
incorporation or by-laws could be found an authority for Register of Deeds of Pasay City) and reconveyance, apartments of the subject property since property since
the distribution of its dividends or surplus profits. Strictly is dismissed for lack or merit, and, 1973 while at the same time managing the Dulay Apartment
speaking, it cannot, therefore, be considered a stock at his shareholdings in the corporation was subsequently
corporation, within the contemplation of the corporation In Civil Case No. 8198-P, defendants Manuel R. increased by his father. 5
law. Dulay Enterprises, Inc. and Virgilio E. Dulay are
ordered to surrender and deliver possession of the On December 23, 1976, Manuel Dulay by virtue of Board
A tax is a burden, and, as such, it should not be deemed parcel of land, together with all the improvements Resolution No 186 of petitioner corporation sold the subject
imposed upon fraternal, civic, non-profit, nonstock thereon, described in Transfer Certificate of Title property to private respondents spouses Maria Theresa and
organizations, unless the intent to the contrary is manifest No. 24799 of the Register of Deeds of Pasay City, in Castrense Veloso in the amount of P300,000.00 as
and patent" (Collector v. BPOE Elks Club, et al., supra), favor of therein plaintiffs Manuel A. Torres, Jr. as evidenced by the Deed of Absolute Sale.7 Thereafter, TCT
which is not the case in the present appeal. owner and Edgardo D. Pabalan as real estate No. 17880 was cancelled and TCT No. 23225 was issued to
administrator of said Manuel A. Torres, Jr.; to private respondent Maria Theresa Veloso. 8 Subsequently,
Having arrived at the conclusion that respondent Club is not account for and return to said plaintiffs the rentals Manuel Dulay and private respondents spouses Veloso
engaged in the business as an operator of a bar and from dwelling unit No. 8-A of the apartment executed a Memorandum to the Deed of Absolute Sale of
restaurant, and therefore, not liable for fixed and building (Dulay Apartment) from June 1980 up to December 23, 1976 9 dated December 9, 1977 giving Manuel
percentage taxes, it follows that it is not liable for any the present, to indemnify plaintiffs, jointly and Dulay within (2) years or until December 9, 1979 to
penalty, much less of a compromise penalty. severally, expenses of litigation in the amount of repurchase the subject property for P200,000.00 which
P4,000.00 and attorney's fees in the sum of was, however, not annotated either in TCT No. 17880 or
WHEREFORE, the decision appealed from is affirmed P6,000.00, for all the three (3) cases. Co-defendant TCT No. 23225.
without costs. Nepomuceno Redovan is ordered to pay the current
and subsequent rentals on the premises leased by On December 24, 1976, private respondent Maria Veloso,
G.R. No. 91889 August 27, 1993 him to plaintiffs. without the knowledge of Manuel Dulay, mortgaged the
MANUEL R. DULAY ENTERPRISES, INC., VIRGILIO E. DULAY subject property to private respondent Manuel A. Torres for
AND NEPOMUCENO REDOVAN, petitioners, 
 The counterclaim of defendants Virgilio E. Dulay a loan of P250,000.00 which was duly annotated as Entry
vs. THE HONORABLE COURT OF APPEALS, EDGARDO D. and Manuel R. Dulay Enterprises, Inc. and N. No. 68139 in TCT No. 23225. 10
PABALAN, MANUEL A. TORRES, JR., MARIA THERESA V. Redovan, dismissed for lack of merit. With costs Upon the failure of private respondent Maria Veloso to pay
VELOSO AND CASTRENSE C. VELOSO, respondents. against the three (3) aforenamed defendants. 3 private respondent Torres, the subject property was sold on
April 5, 1978 to private respondent Torres as the highest
NOCON, J.: The facts as found by the trial court are as follows: bidder in an extrajudicial foreclosure sale as evidenced by
This is a petition for review on certiorari to annul and set the Certificate of Sheriff's Sale 11 issued on April 20, 1978.
aside the decision 1 of the Court of Appeals affirming the Petitioner Manuel R. Dulay Enterprises, Inc, a domestic
decision2 of the Regional Trial Court of Pasay, Branch 114 corporation with the following as members of its Board of On July 20, 1978, private respondent Maria Veloso executed
Civil Cases Nos. 8198-P, and 2880-P, the dispositive portion Directors: Manuel R. Dulay with 19,960 shares and a Deed of Absolute Assignment of the Right to Redeem 12 in
of which reads, as follows: designated as president, treasurer and general manager, favor of Manuel Dulay assigning her right to repurchase the
Atty. Virgilio E. Dulay with 10 shares and designated as subject property from private respondent Torres as a result
Wherefore, in view of all the foregoing vice-president; Linda E. Dulay with 10 shares; Celia Dulay- of the extra sale held on April 25, 1978.
considerations, in this Court hereby renders Mendoza with 10 shares; and Atty. Plaridel C. Jose with 10
judgment, as follows: shares and designated as secretary, owned a property As neither private respondent Maria Veloso nor her assignee
covered by TCT No. 17880 4 and known as Dulay Apartment Manuel Dulay was able to redeem the subject property
In Civil Case No. 2880-P, the petition filed by consisting of sixteen (16) apartment units on a six hundred within the one year statutory period for redemption,
Manuel R. Dulay Enterprises, Inc. and Virgilio E. eighty-nine (689) square meters lot, more or less, located private respondent Torres filed an Affidavit of Consolidation
Dulay for annulment or declaration of nullity of the at Seventh Street (now Buendia Extension) and F.B. of Ownership 13 with the Registry of Deeds of Pasay City and
decision of the Metropolitan Trial Court, Branch 46, Harrison Street, Pasay City. TCT No. 24799 14 was subsequently issued to private
Pasay City, in its Civil Case No. 38-81 entitled respondent Manuel Torres on April 23, 1979.

Page 11 of 160
CORPORATION LAW CASES MAYORDO, M.A
LLB 3

On October 1, 1979, private respondent Torres filed a Thereafter or on May 17, 1985, petitioner corporation and 1. Before or after such action is taken, written
petition for the issuance of a writ of possession against Virgilio Dulay filed an action against the presiding judge of consent thereto is signed by all the directors, or
private respondents spouses Veloso and Manuel Dulay in the Metropolitan Trial Court of Pasay City, private
LRC Case No. 1742-P. However, when petitioner Virgilio respondents Pabalan and Torres for the annulment of said 2. All the stockholders have actual or implied
Dulay was never authorized by the petitioner corporation to decision with the Regional Trial Court of Pasay in Civil Case knowledge of the action and make no prompt
sell or mortgage the subject property, the trial court No. 2880-P. objection thereto in writing; or
ordered private respondent Torres to implead petitioner 3. The directors are accustomed to take informal
corporation as an indispensable party but the latter moved Thereafter, the three (3) cases were jointly tried and the action with the express or implied acquiese of all
for the dismissal of his petition which was granted in an trial court rendered a decision in favor of private the stockholders, or
Order dated April 8, 1980. respondents.
4. All the directors have express or implied
On June 20, 1980, private respondent Torres and Edgardo Not satisfied with said decision, petitioners appealed to the knowledge of the action in question and none of
Pabalan, real estate administrator of Torres, filed an action Court of Appeals which rendered a decision on October 23, them makes prompt objection thereto in writing.
against petitioner corporation, Virgilio Dulay and 1989, the dispositive portion of which reads, as follows:
Nepomuceno Redovan, a tenant of Dulay Apartment Unit If a directors' meeting is held without call or notice,
No. 8-A for the recovery of possession, sum of money and PREMISES CONSIDERED, the decision being appealed an action taken therein within the corporate
damages with preliminary injunction in Civil Case, No. should be as it is hereby AFFIRMED in full. 16 powers is deemed ratified by a director who failed
8198-P with the then Court of First Instance of Rizal. to attend, unless he promptly files his written
On November 8, 1989, petitioners filed a Motion for objection with the secretary of the corporation
On July 21, 1980, petitioner corporation filed an action Reconsideration which was denied on January 26, 1990. after having knowledge thereof.
against private respondents spouses Veloso and Torres for Hence, this petition.
the cancellation of the Certificate of Sheriff's Sale and TCT In the instant case, petitioner corporation is classified as a
No. 24799 in Civil Case No. 8278-P with the then Court of During the pendency of this petition, private respondent close corporation and consequently a board resolution
First Instance of Rizal. Torres died on April 3, 1991 as shown in his death authorizing the sale or mortgage of the subject property is
certificate 17 and named Torres-Pabalan Realty & not necessary to bind the corporation for the action of its
On January 29, 1981, private respondents Pabalan and Development Corporation as his heir in his holographic will president. At any rate, corporate action taken at a board
Torres filed an action against spouses Florentino and Elvira 18 dated October 31, 1986. meeting without proper call or notice in a close corporation
Manalastas, a tenant of Dulay Apartment Unit No. 7-B, with is deemed ratified by the absent director unless the latter
petitioner corporation as intervenor for ejectment in Civil Petitioners contend that the respondent court had acted promptly files his written objection with the secretary of
Case No. 38-81 with the Metropolitan Trial Court of Pasay with grave abuse of discretion when it applied the doctrine the corporation after having knowledge of the meeting
City which rendered a decision on April 25, 1985, of piercing the veil of corporate entity in the instant case which, in his case, petitioner Virgilio Dulay failed to do.
dispositive portion of which reads, as follows: considering that the sale of the subject property between
private respondents spouses Veloso and Manuel Dulay has It is relevant to note that although a corporation is an
Wherefore, judgment is hereby rendered in favor of no binding effect on petitioner corporation as Board entity which has a personality distinct and separate from
the plaintiff (herein private respondents) and Resolution No. 18 which authorized the sale of the subject its individual stockholders or members, 19 the veil of
against the defendants: property was resolved without the approval of all the corporate fiction may be pierced when it is used to defeat
members of the board of directors and said Board public convenience justify wrong, protect fraud or defend
1. Ordering the defendants and all persons claiming Resolution was prepared by a person not designated by the crime. 20 The privilege of being treated as an entity distinct
possession under them to vacate the premises. corporation to be its secretary. and separate from its stockholder or members is therefore
confined to its legitimate uses and is subject to certain
2. Ordering the defendants to pay the rents in the We do not agree. limitations to prevent the commission of fraud or other
sum of P500.000 a month from May, 1979 until they illegal or unfair act. When the corporation is used merely
shall have vacated the premises with interest at the Section 101 of the Corporation Code of the Philippines as an alter ego or business conduit of a person, the law will
legal rate; provides: regard the corporation as the act of that person. 21 The
Supreme Court had repeatedly disregarded the separate
3. Ordering the defendants to pay attorney's fees in Sec. 101. When board meeting is unnecessary or personality of the corporation where the corporate entity
the sum of P2,000.00 and P1,000.00 as other improperly held. Unless the by-laws provide was used to annul a valid contract executed by one of its
expenses of litigation and for them to pay the costs otherwise, any action by the directors of a close members.
of the suit.15 corporation without a meeting shall nevertheless be
deemed valid if: Petitioners' claim that the sale of the subject property by
its president, Manuel Dulay, to private respondents spouses

Page 12 of 160
CORPORATION LAW CASES MAYORDO, M.A
LLB 3

Veloso is null and void as the alleged Board Resolution No. who appeared on paper as the secretary. There is
18 was passed without the knowledge and consent of the no denying the fact, however, that Maria Socorro R. Therefore, prior physical delivery or possession is not
other members of the board of directors cannot be Dulay at times acted as secretary. . . ., the Court legally required since the execution of the Deed of Sale in
sustained. As correctly pointed out by the respondent Court can not lose sight of the fact that the Manuel R. deemed equivalent to delivery.
of Appeals: Dulay Enterprises, Inc. is a closed family
corporation where the incorporators and directors Finally, we hold that the respondent appellate court did not
Appellant Virgilio E. Dulay's protestations of belong to one single family. It cannot be concealed err in denying petitioner's motion for reconsideration
complete innocence to the effect that he never that Manuel R. Dulay as president, treasurer and despite the fact that private respondents failed to submit
participated nor was even aware of any meeting or general manager almost had absolute control over their comment to said motion as required by the
resolution authorizing the mortgage or sale of the the business and affairs of the corporation. 24 respondent appellate court from resolving petitioners'
subject premises (see par. 8, affidavit of Virgilio E. motion for reconsideration without the comment of the
Dulay, dated May 31, 1984, p. 14, Exh. "21") is Moreover, the appellate courts will not disturb the findings private respondent which was required merely to aid the
difficult to believe. On the contrary, he is very of the trial judge unless he has plainly overlooked certain court in the disposition of the motion. The courts are as
much privy to the transactions involved. To begin facts of substance and value that, if considered, might much interested as the parties in the early disposition of
with, he is a incorporator and one of the board of affect the result of the case, 25 which is not present in the cases before them. To require otherwise would
directors designated at the time of the organization instant case. unnecessarily clog the courts' dockets.
of Manuel R. Dulay Enterprise, Inc. In ordinary
parlance, the said entity is loosely referred to as a Petitioners' contention that private respondent Torres never WHEREFORE, the petition is DENIED and the decision
"family corporation". The nomenclature, if acquired ownership over the subject property since the appealed from is hereby AFFIRMED.
imprecise, however, fairly reflects the cohesiveness latter was never in actual possession of the subject SO ORDERED.
of a group and the parochial instincts of the property nor was the property ever delivered to him is also
individual members of such an aggrupation of which without merit. [G.R. Nos. 84132-33 :  December 10, 1990.]
Manuel R. Dulay Enterprises, Inc. is typical: four- 192 SCRA 257
fifths of its incorporators being close relatives Paragraph 1, Article 1498 of the New Civil Code provides: NATIONAL DEVELOPMENT COMPANY AND NEW AGRIX,
namely, three (3) children and their father whose INC., Petitioners, vs.  PHILIPPINE VETERANS BANK, THE
name identifies their corporation (Articles of When the sale is made through a public instrument, EX-OFFICIO SHERIFF and GODOFREDO QUILING, in his
Incorporation of Manuel R. Dulay Enterprises, Inc. the execution thereof shall be equivalent to the capacity as Deputy Sheriff of Calamba, Laguna,
Exh. "31-A"). 22 delivery of the thing which is the object of the Respondents.
contract, if from the deed the contrary do not
Besides, the fact that petitioner Virgilio Dulay on June 24, appear or cannot clearly be inferred. DECISION
1975 executed an affidavit 23 that he was a signatory CRUZ, J.:
witness to the execution of the post-dated Deed of Under the aforementioned article, the mere execution of  
Absolute Sale of the subject property in favor of private the deed of sale in a public document is equivalent to the This case involves the constitutionality of a presidential
respondent Torres indicates that he was aware of the delivery of the property. Likewise, this Court had held that: decree which, like all other issuances of President Marcos
transaction executed between his father and private during his regime, was at that time regarded as sacrosanct.
respondents and had, therefore, adequate knowledge about It is settled that the buyer in a foreclosure sale It is only now, in a freer atmosphere, that his acts are being
the sale of the subject property to private respondents. becomes the absolute owner of the property tested by the touchstone of the fundamental law that even
Consequently, petitioner corporation is liable for the act of purchased if it is not redeemed during the period of then was supposed to limit presidential action.
Manuel Dulay and the sale of the subject property to one year after the registration of the sale. As such,
private respondents by Manuel Dulay is valid and binding. he is entitled to the possession of the said property The particular enactment in question is Pres. Decree No.
As stated by the trial court: and can demand it at any time following the 1717, which ordered the rehabilitation of the Agrix Group
consolidation of ownership in his name and the of Companies to be administered mainly by the National
. . . the sale between Manuel R. Dulay Enterprises, issuance to him of a new transfer certificate of Development Company. The law outlined the procedure for
Inc. and the spouses Maria Theresa V. Veloso and title. The buyer can in fact demand possession of filing claims against the Agrix companies and created a
Castrense C. Veloso, was a corporate act of the the land even during the redemption period except Claims Committee to process these claims. Especially
former and not a personal transaction of Manuel R. that he has to post a bond in accordance with relevant to this case, and noted at the outset, is Sec. 4(1)
Dulay. This is so because Manuel R. Dulay was not Section 7 of Act No. 3133 as amended. No such thereof providing that "all mortgages and other liens
only president and treasurer but also the general bond is required after the redemption period if the presently attaching to any of the assets of the dissolved
manager of the corporation. The corporation was a property is not redeemed. Possession of the land corporations are hereby extinguished."
closed family corporation and the only non-relative then becomes an absolute right of the purchaser as
in the board of directors was Atty. Plaridel C. Jose confirmed owner. 26

Page 13 of 160
CORPORATION LAW CASES MAYORDO, M.A
LLB 3

Earlier, the Agrix Marketing, Inc. (AGRIX) had executed in but not resolved. The Court, after noting that the respondent. The petitioner limited itself to the argument
favor of private respondent Philippine Veterans Bank a real petitioners had already filed their claims with the AGRIX that the private respondent was estopped from questioning
estate mortgage dated July 7, 1978, over three (3) parcels Claims Committee created by the decree, had simply the decree because of its earlier compliance with its
of land situated in Los Baños, Laguna. During the existence dismissed the petition on the ground of estoppel. provisions.
of the mortgage, AGRIX went bankrupt. It was for the
expressed purpose of salvaging this and the other Agrix The petitioners stress that in the case at bar the private Independently of these observations, there is the
companies that the aforementioned decree was issued by respondent also invoked the provisions of Pres. Decree No. consideration that an affront to the Constitution cannot be
President Marcos. 1717 by filing a claim with the AGRIX Claims Committee. allowed to continue existing simply because of procedural
Failing to get results, it sought to foreclose the real estate inhibitions that exalt form over substance.
Pursuant thereto, the private respondent filed a claim with mortgage executed by AGRIX in its favor, which had been
the AGRIX Claims Committee for the payment of its loan extinguished by the decree. It was only when the The Court is especially disturbed by Section 4(1) of the
credit. In the meantime, the New Agrix, Inc. and the petitioners challenged the foreclosure on the basis of Sec. decree, quoted above, extinguishing all mortgages and
National Development Company, petitioners herein, 4 (1) of the decree, that the private respondent attacked other liens attaching to the assets of AGRIX. It also notes,
invoking Sec. 4 (1) of the decree, filed a petition with the the validity of the provision. At that stage, however, with equal concern, the restriction in Subsection (ii)
Regional Trial Court of Calamba, Laguna, for the consistent with Mendoza, the private respondent was thereof that all "unsecured obligations shall not bear
cancellation of the mortgage lien in favor of the private already estopped from questioning the constitutionality of interest" and in Subsection (iii) that "all accrued interests,
respondent. For its part, the private respondent took steps the decree. penalties or charges as of date hereof pertaining to the
to extrajudicially foreclose the mortgage, prompting the obligations, whether secured or unsecured, shall not be
petitioners to file a second case with the same court to The Court does not agree that the principle of estoppel is recognized."
stop the foreclosure. The two cases were consolidated. applicable.
These provisions must be read with the Bill of Rights, where
After the submission by the parties of their respective It is not denied that the private respondent did file a claim it is clearly provided in Section 1 that "no person shall be
pleadings, the trial court rendered the impugned decision. with the AGRIX Claims Committee pursuant to this decree. deprived of life, liberty or property without due course of
Judge Francisco Ma. Guerrero annulled not only the It must be noted, however, that this was done in 1980, law nor shall any person be denied the equal protection of
challenged provision, viz., Sec. 4 (1), but the entire Pres. when President Marcos was the absolute ruler of this the law" and in Section 10 that "no law impairing the
Decree No. 1717 on the grounds that: (1) the presidential country and his decrees were the absolute law. Any judicial obligation of contracts shall be passed."
exercise of legislative power was a violation of the challenge to them would have been futile, not to say
principle of separation of powers; (2) the law impaired the foolhardy. The private respondent, no less than the rest of In defending the decree, the petitioners argue that
obligation of contracts; and (3) the decree violated the the nation, was aware of that reality and knew it had no property rights, like all rights, are subject to regulation
equal protection clause. The motion for reconsideration of choice under the circumstances but to conform. under the police power for the promotion of the common
this decision having been denied, the present petition was welfare. The contention is that this inherent power of the
filed. It is true that there were a few venturesome souls who state may be exercised at any time for this purpose so long
dared to question the dictator's decisions before the courts as the taking of the property right, even if based on
The petition was originally assigned to the Third Division of of justice then. The record will show, however, that not a contract, is done with due process of law.
this Court but because of the constitutional questions single act or issuance of President Marcos was ever
involved it was transferred to the Court en banc. On August declared unconstitutional, not even by the highest court, as This argument is an over-simplification of the problem
30, 1988, the Court granted the petitioner's prayer for a long as he was in power. To rule now that the private before us. The police power is not a panacea for all
temporary restraining order and instructed the respondents respondent is estopped for having abided with the decree constitutional maladies. Neither does its mere invocation
to cease and desist from conducting a public auction sale of instead of boldly assailing it is to close our eyes to a cynical conjure an instant and automatic justification for every act
the lands in question. After the Solicitor General and the fact of life during that repressive time. of the government depriving a person of his life, liberty or
private respondent had filed their comments and the property.
petitioners their reply, the Court gave due course to the This case must be distinguished from Mendoza, where the
petition and ordered the parties to file simultaneous petitioners, after filing their claims with the AGRIX Claims A legislative act based on the police power requires the
memoranda. Upon compliance by the parties, the case was Committee, received in settlement thereof shares of stock concurrence of a lawful subject and a lawful method. In
deemed submitted. valued at P40,000.00 without protest or reservation. The more familiar words, a) the interests of the public
herein private respondent has not been paid a single generally, as distinguished from those of a particular class,
The petitioners contend that the private respondent is now centavo on its claim, which was kept pending for more than should justify the interference of the state; and b) the
estopped from contesting the validity of the decree. In seven years for alleged lack of supporting papers. means employed are reasonably necessary for the
support of this contention, it cites the recent case of Significantly, the validity of that claim was not questioned accomplishment of the purpose and not unduly oppressive
Mendoza v. Agrix Marketing, Inc., 1 where the by the petitioner when it sought to restrain the upon individuals. 2
constitutionality of Pres. Decree No. 1717 was also raised extrajudicial foreclosure of the mortgage by the private

Page 14 of 160
CORPORATION LAW CASES MAYORDO, M.A
LLB 3

Applying these criteria to the case at bar, the Court finds The only concession given to the secured creditors is that Agrix, Inc. is entirely private and so should have been
first of all that the interests of the public are not their loans are allowed to earn interest from the date of organized under the Corporation Law in accordance with
sufficiently involved to warrant the interference of the the decree, but that still does not justify the cancellation the above-cited constitutional provision.
government with the private contracts of AGRIX. The of the interests earned before that date. Such interests,
decree speaks vaguely of the "public, particularly the small whether due to the secured or the unsecured creditors, are The Court also feels that the decree impairs the obligation
investors," who would be prejudiced if the corporation all extinguished by the decree. Even assuming such of the contract between AGRIX and the private respondent
were not to be assisted. However, the record does not state cancellation to be valid, we still cannot see why all kinds of without justification. While it is true that the police power
how many there are of such investors, and who they are, creditors, regardless of security, are treated alike. is superior to the impairment clause, the principle will
and why they are being preferred to the private respondent apply only where the contract is so related to the public
and other creditors of AGRIX with vested property rights. Under the equal protection clause, all persons or things welfare that it will be considered congenitally susceptible
similarly situated must be treated alike, both in the to change by the legislature in the interest of the greater
The public interest supposedly involved is not identified or privileges conferred and the obligations imposed. number. 5 Most present-day contracts are of that nature.
explained. It has not been shown that by the creation of Conversely, all persons or things differently situated should But as already observed, the contracts of loan and
the New Agrix, Inc. and the extinction of the property be treated differently. In the case at bar, persons mortgage executed by AGRIX are purely private
rights of the creditors of AGRIX, the interests of the public differently situated are similarly treated, in disregard of transactions and have not been shown to be affected with
as a whole, as distinguished from those of a particular the principle that there should be equality only among public interest. There was therefore no warrant to amend
class, would be promoted or protected. The indispensable equals.- nad their provisions and deprive the private respondent of its
link to the welfare of the greater number has not been vested property rights.
established. On the contrary, it would appear that the One may also well wonder why AGRIX was singled out for It is worth noting that only recently in the case of the
decree was issued only to favor a special group of investors government help, among other corporations where the Development Bank of the Philippines v. NLRC, 6 we
who, for reasons not given, have been preferred to the stockholders or investors were also swindled. It is not clear sustained the preference in payment of a mortgage creditor
legitimate creditors of AGRIX. why other companies entitled to similar concern were not as against the argument that the claims of laborers should
similarly treated. And surely, the stockholders of the take precedence over all other claims, including those of
Assuming there is a valid public interest involved, the Court private respondent, whose mortgage lien had been the government. In arriving at this ruling, the Court
still finds that the means employed to rehabilitate AGRIX cancelled and legitimate claims to accrued interests recognized the mortgage lien as a property right protected
fall far short of the requirement that they shall not be rejected, were no less deserving of protection, which they by the due process and contract clauses notwithstanding
unduly oppressive. The oppressiveness is patent on the face did not get. The decree operated, to use the words of a the argument that the amendment in Section 110 of the
of the decree. The right to property in all mortgages, liens, celebrated case, 3 "with an evil eye and an uneven hand." Labor Code was a proper exercise of the police power.
interests, penalties and charges owing to the creditors of
AGRIX is arbitrarily destroyed. No consideration is paid for On top of all this, New Agrix, Inc. was created by special The Court reaffirms and applies that ruling in the case at
the extinction of the mortgage rights. The accrued decree notwithstanding the provision of Article XIV, Section bar.
interests and other charges are simply rejected by the 4 of the 1973 Constitution, then in force, that: Our finding, in sum, is that Pres. Decree No. 1717 is an
decree. The right to property is dissolved by legislative fiat invalid exercise of the police power, not being in
without regard to the private interest violated and, worse, SEC. 4. The Batasang Pambansa shall not, except by general conformity with the traditional requirements of a lawful
in favor of another private interest. law, provide for the formation, organization, or regulation subject and a lawful method. The extinction of the
of private corporations, unless such corporations are owned mortgage and other liens and of the interest and other
A mortgage lien is a property right derived from contract or controlled by the Government or any subdivision or charges pertaining to the legitimate creditors of AGRIX
and so comes under the protection of the Bill of Rights. So instrumentality thereof. 4 constitutes taking without due process of law, and this is
do interests on loans, as well as penalties and charges, compounded by the reduction of the secured creditors to
which are also vested rights once they accrue. Private The new corporation is neither owned nor controlled by the the category of unsecured creditors in violation of the
property cannot simply be taken by law from one person government. The National Development Corporation was equal protection clause. Moreover, the new corporation,
and given to another without compensation and any known merely required to extend a loan of not more than being neither owned nor controlled by the Government,
public purpose. This is plain arbitrariness and is not P10,000,000.00 to New Agrix, Inc. Pending payment should have been created only by general and not special
permitted under the Constitution. thereof, NDC would undertake the management of the law. And insofar as the decree also interferes with purely
corporation, but with the obligation of making periodic private agreements without any demonstrated connection
And not only is there arbitrary taking, there is reports to the Agrix board of directors. After payment of with the public interest, there is likewise an impairment of
discrimination as well. In extinguishing the mortgage and the loan, the said board can then appoint its own the obligation of the contract.
other liens, the decree lumps the secured creditors with management. The stocks of the new corporation are to be
the unsecured creditors and places them on the same level issued to the old investors and stockholders of AGRIX upon With the above pronouncements, we feel there is no more
in the prosecution of their respective claims. In this proof of their claims against the abolished corporation. need to rule on the authority of President Marcos to
respect, all of them are considered unsecured creditors. They shall then be the owners of the new corporation. New promulgate Pres. Decree No. 1717 under Amendment No. 6

Page 15 of 160
CORPORATION LAW CASES MAYORDO, M.A
LLB 3

of the 1973 Constitution. Even if he had such authority, the half, the amount of Pl84,878.74 with interest from
decree must fall just the same because of its violation of the filing of the cross-complaints until the amount In 1965, Jacob S. Lim (petitioner in G.R. No. 84157) was
the Bill of Rights. is fully paid; plus moral and exemplary damages in engaged in the airline business as owner-operator of
the amount of P184,878.84 with interest from the Southern Air Lines (SAL) a single proprietorship.
WHEREFORE, the petition is DISMISSED. Pres. Decree No. filing of the cross-complaints until the amount is
1717 is declared UNCONSTITUTIONAL. The temporary fully paid; plus moral and exemplary damages in On May 17, 1965, at Tokyo, Japan, Japan Domestic Airlines
restraining order dated August 30, 1988, is LIFTED. Costs the amount of P50,000.00 for each of the two (JDA) and Lim entered into and executed a sales contract
against the petitioners. Cervanteses. (Exhibit A) for the sale and purchase of two (2) DC-3A Type
SO ORDERED. aircrafts and one (1) set of necessary spare parts for the
Furthermore, he is required to pay P20,000.00 to total agreed price of US $109,000.00 to be paid in
G.R. No. 84197 July 28, 1989 Bormaheco and the Cervanteses, and another installments. One DC-3 Aircraft with Registry No. PIC-718,
PIONEER INSURANCE & SURETY CORPORATION, petitioner, 
 P20,000.00 to Constancio B. Maglana as attorney's arrived in Manila on June 7,1965 while the other aircraft,
vs. THE HON. COURT OF APPEALS, BORDER MACHINERY & fees. arrived in Manila on July 18,1965.
HEAVY EQUIPMENT, INC., (BORMAHECO), CONSTANCIO M. xxx xxx xxx
MAGLANA and JACOB S. LIM, respondents. On May 22, 1965, Pioneer Insurance and Surety Corporation
WHEREFORE, in view of all above, the complaint of (Pioneer, petitioner in G.R. No. 84197) as surety executed
G.R. No. 84157 July 28, 1989 plaintiff Pioneer against defendants Bormaheco, the and issued its Surety Bond No. 6639 (Exhibit C) in favor of
JACOB S. LIM, petitioner, 
 Cervanteses and Constancio B. Maglana, is JDA, in behalf of its principal, Lim, for the balance price of
vs. COURT OF APPEALS, PIONEER INSURANCE AND SURETY dismissed. Instead, plaintiff is required to the aircrafts and spare parts.
CORPORATION, BORDER MACHINERY and HEAVY indemnify the defendants Bormaheco and the
EQUIPMENT CO., INC,, FRANCISCO and MODESTO Cervanteses the amount of P20,000.00 as attorney's It appears that Border Machinery and Heavy Equipment
CERVANTES and CONSTANCIO MAGLANA, respondents. fees and the amount of P4,379.21, per year from Company, Inc. (Bormaheco), Francisco and Modesto
1966 with legal rate of interest up to the time it is Cervantes (Cervanteses) and Constancio Maglana
GUTIERREZ, JR., J.: paid. (respondents in both petitions) contributed some funds
The subject matter of these consolidated petitions is the used in the purchase of the above aircrafts and spare parts.
decision of the Court of Appeals in CA-G.R. CV No. 66195 Furthermore, the plaintiff is required to pay The funds were supposed to be their contributions to a new
which modified the decision of the then Court of First Constancio B. Maglana the amount of P20,000.00 as corporation proposed by Lim to expand his airline business.
Instance of Manila in Civil Case No. 66135. The plaintiffs attorney's fees and costs. They executed two (2) separate indemnity agreements
complaint (petitioner in G.R. No. 84197) against all (Exhibits D-1 and D-2) in favor of Pioneer, one signed by
defendants (respondents in G.R. No. 84197) was dismissed No moral or exemplary damages is awarded against Maglana and the other jointly signed by Lim for SAL,
but in all other respects the trial court's decision was plaintiff for this action was filed in good faith. The Bormaheco and the Cervanteses. The indemnity agreements
affirmed. fact that the properties of the Bormaheco and the stipulated that the indemnitors principally agree and bind
Cervanteses were attached and that they were themselves jointly and severally to indemnify and hold and
The dispositive portion of the trial court's decision reads as required to file a counterbond in order to dissolve save harmless Pioneer from and against any/all damages,
follows: the attachment, is not an act of bad faith. When a losses, costs, damages, taxes, penalties, charges and
man tries to protect his rights, he should not be expenses of whatever kind and nature which Pioneer may
WHEREFORE, judgment is rendered against saddled with moral or exemplary damages. incur in consequence of having become surety upon the
defendant Jacob S. Lim requiring Lim to pay Furthermore, the rights exercised were provided bond/note and to pay, reimburse and make good to Pioneer,
plaintiff the amount of P311,056.02, with interest for in the Rules of Court, and it was the court that its successors and assigns, all sums and amounts of money
at the rate of 12% per annum compounded monthly; ordered it, in the exercise of its discretion. which it or its representatives should or may pay or cause
plus 15% of the amount awarded to plaintiff as to be paid or become liable to pay on them of whatever
attorney's fees from July 2,1966, until full payment No damage is decided against Malayan Insurance kind and nature.
is made; plus P70,000.00 moral and exemplary Company, Inc., the third-party defendant, for it
damages. only secured the attachment prayed for by the On June 10, 1965, Lim doing business under the name and
plaintiff Pioneer. If an insurance company would be style of SAL executed in favor of Pioneer as deed of chattel
It is found in the records that the cross party liable for damages in performing an act which is mortgage as security for the latter's suretyship in favor of
plaintiffs incurred additional miscellaneous clearly within its power and which is the reason for the former. It was stipulated therein that Lim transfer and
expenses aside from Pl51,000.00,,making a total of its being, then nobody would engage in the convey to the surety the two aircrafts. The deed (Exhibit D)
P184,878.74. Defendant Jacob S. Lim is further insurance business. No further claim or counter- was duly registered with the Office of the Register of Deeds
required to pay cross party plaintiff, Bormaheco, claim for or against anybody is declared by this of the City of Manila and with the Civil Aeronautics
the Cervanteses one-half and Maglana the other Court. (Rollo - G.R. No. 24197, pp. 15-16) Administration pursuant to the Chattel Mortgage Law and

Page 16 of 160
CORPORATION LAW CASES MAYORDO, M.A
LLB 3

the Civil Aeronautics Law (Republic Act No. 776), We find no merit in plaintiffs appeal. It is P295,000.00, it is patent that plaintiff has been
respectively. undisputed that plaintiff Pioneer had reinsured its overpaid in the amount of P33,383.72 considering
risk of liability under the surety bond in favor of that the total amount it had paid to JDA totals to
Lim defaulted on his subsequent installment payments JDA and subsequently collected the proceeds of only P298,666.28. To allow plaintiff Pioneer to
prompting JDA to request payments from the surety. such reinsurance in the sum of P295,000.00. recover from defendants the amount in excess of
Pioneer paid a total sum of P298,626.12. Defendants' alleged obligation to Pioneer amounts P298,666.28 would be tantamount to unjust
to P295,000.00, hence, plaintiffs instant action for enrichment as it has already been paid by the
Pioneer then filed a petition for the extrajudicial the recovery of the amount of P298,666.28 from reinsurance company of the amount plaintiff has
foreclosure of the said chattel mortgage before the Sheriff defendants will no longer prosper. Plaintiff Pioneer paid to JDA as surety of defendant Lim vis-a-vis
of Davao City. The Cervanteses and Maglana, however, filed is not the real party in interest to institute the defendant Lim's liability to JDA. Well settled is the
a third party claim alleging that they are co-owners of the instant action as it does not stand to be benefited rule that no person should unjustly enrich himself
aircrafts, On July 19, 1966, Pioneer filed an action for or injured by the judgment. at the expense of another (Article 22, New Civil
judicial foreclosure with an application for a writ of Code). (Rollo-84197, pp. 24-25).
preliminary attachment against Lim and respondents, the Plaintiff Pioneer's contention that it is representing
Cervanteses, Bormaheco and Maglana. the reinsurer to recover the amount from The petitioner contends that-(1) it is at a loss where
defendants, hence, it instituted the action is respondent court based its finding that petitioner was paid
In their Answers, Maglana, Bormaheco and the Cervanteses utterly devoid of merit. Plaintiff did not even by its reinsurer in the aforesaid amount, as this matter has
filed cross-claims against Lim alleging that they were not present any evidence that it is the attorney-in-fact never been raised by any of the parties herein both in their
privies to the contracts signed by Lim and, by way of of the reinsurance company, authorized to institute answers in the court below and in their respective briefs
counterclaim, sought for damages for being exposed to an action for and in behalf of the latter. To qualify with respondent court; (Rollo, p. 11) (2) even assuming
litigation and for recovery of the sums of money they a person to be a real party in interest in whose hypothetically that it was paid by its reinsurer, still none of
advanced to Lim for the purchase of the aircrafts in name an action must be prosecuted, he must the respondents had any interest in the matter since the
question. appear to be the present real owner of the right reinsurance is strictly between the petitioner and the re-
sought to be enforced (Moran, Vol. I, Comments on insurer pursuant to section 91 of the Insurance Code; (3)
After trial on the merits, a decision was rendered holding the Rules of Court, 1979 ed., p. 155). It has been pursuant to the indemnity agreements, the petitioner is
Lim liable to pay Pioneer but dismissed Pioneer's complaint held that the real party in interest is the party who entitled to recover from respondents Bormaheco and
against all other defendants. would be benefited or injured by the judgment or Maglana; and (4) the principle of unjust enrichment is not
the party entitled to the avails of the suit (Salonga applicable considering that whatever amount he would
As stated earlier, the appellate court modified the trial v. Warner Barnes & Co., Ltd., 88 Phil. 125, 131). By recover from the co-indemnitor will be paid to the
court's decision in that the plaintiffs complaint against all real party in interest is meant a present substantial reinsurer.
the defendants was dismissed. In all other respects the trial interest as distinguished from a mere expectancy or The records belie the petitioner's contention that the issue
court's decision was affirmed. a future, contingent, subordinate or consequential on the reinsurance money was never raised by the parties.
interest (Garcia v. David, 67 Phil. 27; Oglleaby v.
We first resolve G.R. No. 84197. Springfield Marine Bank, 52 N.E. 2d 1600, 385 III, A cursory reading of the trial court's lengthy decision shows
414; Flowers v. Germans, 1 NW 2d 424; Weber v. that two of the issues threshed out were:
Petitioner Pioneer Insurance and Surety Corporation avers City of Cheye, 97 P. 2d 667, 669, quoting 47 C.V.
that: 35). xxx xxx xxx
1. Has Pioneer a cause of action against defendants
RESPONDENT COURT OF APPEALS GRIEVOUSLY Based on the foregoing premises, plaintiff Pioneer with respect to so much of its obligations to JDA as
ERRED WHEN IT DISMISSED THE APPEAL OF cannot be considered as the real party in interest has been paid with reinsurance money?
P E T I T I O N E R O N T H E S O L E G R O U N D T H AT as it has already been paid by the reinsurer the sum 2. If the answer to the preceding question is in the
PETITIONER HAD ALREADY COLLECTED THE of P295,000.00 — the bulk of defendants' alleged negative, has Pioneer still any claim against
PROCEEDS OF THE REINSURANCE ON ITS BOND IN obligation to Pioneer. defendants, considering the amount it has realized
FAVOR OF THE JDA AND THAT IT CANNOT REPRESENT from the sale of the mortgaged properties? (Record
A REINSURER TO RECOVER THE AMOUNT FROM In addition to the said proceeds of the reinsurance on Appeal, p. 359, Annex B of G.R. No. 84157).
HEREIN PRIVATE RESPONDENTS AS DEFENDANTS IN received by plaintiff Pioneer from its reinsurer, the
THE TRIAL COURT. (Rollo - G. R. No. 84197, p. 10) former was able to foreclose extra-judicially one of In resolving these issues, the trial court made the following
the subject airplanes and its spare engine, realizing findings:
The petitioner questions the following findings of the the total amount of P37,050.00 from the sale of the
appellate court: mortgaged chattels. Adding the sum of P37,050.00, It appearing that Pioneer reinsured its risk of
to the proceeds of the reinsurance amounting to liability under the surety bond it had executed in

Page 17 of 160
CORPORATION LAW CASES MAYORDO, M.A
LLB 3

favor of JDA, collected the proceeds of such portion of what it paid to JDA is the difference Manufacturing Corporation v. Court of Appeals (154 SCRA
reinsurance in the sum of P295,000, and paid with between the two amounts, or P3,666.28. This is the 650 [1987]):
the said amount the bulk of its alleged liability to amount for which Pioneer may sue defendants,
JDA under the said surety bond, it is plain that on assuming that the indemnity agreement is still valid Note that if a property is insured and the owner
this score it no longer has any right to collect to the and effective. But since the amount realized from receives the indemnity from the insurer, it is
extent of the said amount. the sale of the mortgaged chattels are P35,000.00 provided in said article that the insurer is deemed
for one of the airplanes and P2,050.00 for a spare subrogated to the rights of the insured against the
On the question of why it is Pioneer, instead of the engine, or a total of P37,050.00, Pioneer is still wrongdoer and if the amount paid by the insurer
reinsurance (sic), that is suing defendants for the overpaid by P33,383.72. Therefore, Pioneer has no does not fully cover the loss, then the aggrieved
amount paid to it by the reinsurers, more claim against defendants. (Record on Appeal, party is the one entitled to recover the deficiency.
notwithstanding that the cause of action pertains to pp. 360-363). Evidently, under this legal provision, the real party
the latter, Pioneer says: The reinsurers opted in interest with regard to the portion of the
instead that the Pioneer Insurance & Surety The payment to the petitioner made by the reinsurers was indemnity paid is the insurer and not the insured.
Corporation shall pursue alone the case.. . . . not disputed in the appellate court. Considering this (Emphasis supplied).
Pioneer Insurance & Surety Corporation is admitted payment, the only issue that cropped up was the
representing the reinsurers to recover the amount.' effect of payment made by the reinsurers to the petitioner. It is clear from the records that Pioneer sued in its own
In other words, insofar as the amount paid to it by Therefore, the petitioner's argument that the respondents name and not as an attorney-in-fact of the reinsurer.
the reinsurers Pioneer is suing defendants as their had no interest in the reinsurance contract as this is strictly
attorney-in-fact. between the petitioner as insured and the reinsuring Accordingly, the appellate court did not commit a
company pursuant to Section 91 (should be Section 98) of reversible error in dismissing the petitioner's complaint as
But in the first place, there is not the slightest the Insurance Code has no basis. against the respondents for the reason that the petitioner
indication in the complaint that Pioneer is suing as was not the real party in interest in the complaint and,
attorney-in- fact of the reinsurers for any amount. In general a reinsurer, on payment of a loss acquires therefore, has no cause of action against the respondents.
Lastly, and most important of all, Pioneer has no the same rights by subrogation as are acquired in
right to institute and maintain in its own name an similar cases where the original insurer pays a loss Nevertheless, the petitioner argues that the appeal as
action for the benefit of the reinsurers. It is well- (Universal Ins. Co. v. Old Time Molasses Co. C.C.A. regards the counter indemnitors should not have been
settled that an action brought by an attorney-in- La., 46 F 2nd 925). dismissed on the premise that the evidence on record
fact in his own name instead of that of the principal shows that it is entitled to recover from the counter
will not prosper, and this is so even where the name The rules of practice in actions on original indemnitors. It does not, however, cite any grounds except
of the principal is disclosed in the complaint. insurance policies are in general applicable to its allegation that respondent "Maglanas defense and
actions or contracts of reinsurance. (Delaware, Ins. evidence are certainly incredible" (p. 12, Rollo) to back up
Section 2 of Rule 3 of the Old Rules of Court Co. v. Pennsylvania Fire Ins. Co., 55 S.E. 330,126 its contention.
provides that 'Every action must be prosecuted GA. 380, 7 Ann. Con. 1134).
in the name of the real party in interest.' This On the other hand, we find the trial court's findings on the
provision is mandatory. The real party in Hence the applicable law is Article 2207 of the new Civil matter replete with evidence to substantiate its finding
interest is the party who would be benefitted Code, to wit: that the counter-indemnitors are not liable to the
or injured by the judgment or is the party Art. 2207. If the plaintiffs property has been petitioner. The trial court stated:
entitled to the avails of the suit. insured, and he has received indemnity from the
insurance company for the injury or loss arising out Apart from the foregoing proposition, the indemnity
This Court has held in various cases that an of the wrong or breach of contract complained of, agreement ceased to be valid and effective after
attorney-in-fact is not a real party in interest, the insurance company shall be subrogated to the the execution of the chattel mortgage.
that there is no law permitting an action to be rights of the insured against the wrongdoer or the
brought by an attorney-in-fact. Arroyo v. person who has violated the contract. If the amount Testimonies of defendants Francisco Cervantes and
Granada and Gentero, 18 Phil. Rep. 484; paid by the insurance company does not fully cover Modesto Cervantes.
Luchauco v. Limjuco and Gonzalo, 19 Phil. Rep. the injury or loss, the aggrieved party shall be
12; Filipinos Industrial Corporation v. San Diego entitled to recover the deficiency from the person Pioneer Insurance, knowing the value of the
G.R. No. L- 22347,1968, 23 SCRA 706, 710-714. causing the loss or injury. aircrafts and the spare parts involved, agreed to
issue the bond provided that the same would be
The total amount paid by Pioneer to JDA is Interpreting the aforesaid provision, we ruled in the case of mortgaged to it, but this was not possible because
P299,666.29. Since Pioneer has collected Phil. Air Lines, Inc. v. Heald Lumber Co. (101 Phil. 1031 the planes were still in Japan and could not be
P295,000.00 from the reinsurers, the uninsured [1957]) which we subsequently applied in Manila Mahogany mortgaged here in the Philippines. As soon as the

Page 18 of 160
CORPORATION LAW CASES MAYORDO, M.A
LLB 3

aircrafts were brought to the Philippines, they to recover any unpaid balance and any agreement three months, that of the last installment being
would be mortgaged to Pioneer Insurance to cover to the contrary is void.' Cruz, et al. v. Filipinas July 15, 1967.
the bond, and this indemnity agreement would be Investment & Finance Corp. No. L- 24772, May
cancelled. 27,1968, 23 SCRA 791, 795-6. These restructuring of the obligations with regard
to their maturity dates, effected twice, were done
The following is averred under oath by Pioneer in The operation of the foregoing provision cannot be without the knowledge, much less, would have it
the original complaint: escaped from through the contention that Pioneer believed that these defendants Maglana (sic).
is not the vendor but JDA. The reason is that Pioneer's official Numeriano Carbonel would have it
The various conflicting claims over the Pioneer is actually exercising the rights of JDA as believed that these defendants and defendant
mortgaged properties have impaired and vendor, having subrogated it in such rights. Nor may Maglana knew of and consented to the modification
rendered insufficient the security under the the application of the provision be validly opposed of the obligations. But if that were so, there would
chattel mortgage and there is thus no other on the ground that these defendants and defendant have been the corresponding documents in the form
sufficient security for the claim sought to be Maglana are not the vendee but indemnitors. of a written notice to as well as written conformity
enforced by this action. Pascual, et al. v. Universal Motors Corporation, G.R. of these defendants, and there are no such
No. L- 27862, Nov. 20,1974, 61 SCRA 124. document. The consequence of this was the
This is judicial admission and aside from the chattel extinguishment of the obligations and of the surety
mortgage there is no other security for the claim The restructuring of the obligations of SAL or Lim, bond secured by the indemnity agreement which
sought to be enforced by this action, which thru the change of their maturity dates discharged was thereby also extinguished. Applicable by
necessarily means that the indemnity agreement these defendants from any liability as alleged analogy are the rulings of the Supreme Court in the
had ceased to have any force and effect at the time indemnitors. The change of the maturity dates of case of Kabankalan Sugar Co. v. Pacheco, 55 Phil.
this action was instituted. Sec 2, Rule 129, Revised the obligations of Lim, or SAL extinguish the 553, 563, and the case of Asiatic Petroleum Co. v.
Rules of Court. original obligations thru novations thus discharging Hizon David, 45 Phil. 532, 538.
the indemnitors.
Prescinding from the foregoing, Pioneer, having Art. 2079. An extension granted to the debtor
foreclosed the chattel mortgage on the planes and The principal hereof shall be paid in eight equal by the creditor without the consent of the
spare parts, no longer has any further action successive three months interval installments, guarantor extinguishes the guaranty The mere
against the defendants as indemnitors to recover the first of which shall be due and payable 25 failure on the part of the creditor to demand
any unpaid balance of the price. The indemnity August 1965, the remainder of which ... shall payment after the debt has become due does
agreement was ipso jure extinguished upon the be due and payable on the 26th day x x x of not of itself constitute any extension time
foreclosure of the chattel mortgage. These each succeeding three months and the last of referred to herein, (New Civil Code).'
defendants, as indemnitors, would be entitled to be which shall be due and payable 26th May 1967.
subrogated to the right of Pioneer should they make Manresa, 4th ed., Vol. 12, pp. 316-317, Vol. VI, pp.
payments to the latter. Articles 2067 and 2080 of However, at the trial of this case, Pioneer produced 562-563, M.F. Stevenson & Co., Ltd., v. Climacom et
the New Civil Code of the Philippines. a memorandum executed by SAL or Lim and JDA, al. (C.A.) 36 O.G. 1571.
modifying the maturity dates of the obligations, as
Independently of the preceding proposition follows: Pioneer's liability as surety to JDA had already
Pioneer's election of the remedy of foreclosure p re s c ri b e d w h e n Pi on e e r p a i d t h e s a m e .
precludes any further action to recover any unpaid The principal hereof shall be paid in eight equal Consequently, Pioneer has no more cause of action
balance of the price. successive three month interval installments to recover from these defendants, as supposed
SAL or Lim, having failed to pay the second to the the first of which shall be due and payable 4 indemnitors, what it has paid to JDA. By virtue of
eight and last installments to JDA and Pioneer as September 1965, the remainder of which ... an express stipulation in the surety bond, the
surety having made of the payments to JDA, the shall be due and payable on the 4th day ... of failure of JDA to present its claim to Pioneer within
alternative remedies open to Pioneer were as each succeeding months and the last of which ten days from default of Lim or SAL on every
provided in Article 1484 of the New Civil Code, shall be due and payable 4th June 1967. installment, released Pioneer from liability from
known as the Recto Law. the claim.
Not only that, Pioneer also produced eight
Pioneer exercised the remedy of foreclosure of the purported promissory notes bearing maturity dates Therefore, Pioneer is not entitled to exact
chattel mortgage both by extrajudicial foreclosure different from that fixed in the aforesaid reimbursement from these defendants thru the
and the instant suit. Such being the case, as memorandum; the due date of the first installment indemnity.
provided by the aforementioned provisions, Pioneer appears as October 15, 1965, and those of the rest
shall have no further action against the purchaser of the installments, the 15th of each succeeding

Page 19 of 160
CORPORATION LAW CASES MAYORDO, M.A
LLB 3

Art. 1318. Payment by a solidary debtor shall Defendant Lim should pay one-half of the said purpose is that no partnership shall exist (London
not entitle him to reimbursement from his co- amount to Bormaheco and the Cervanteses and the Assur. Corp. v. Drennen, Minn., 6 S.Ct. 442, 116 U.S.
debtors if such payment is made after the other one-half to defendant Maglana. It is 461, 472, 29 L.Ed. 688), and it should be implied
obligation has prescribed or became illegal. established in the records that defendant Lim had only when necessary to do justice between the
duly received the amount of Pl51,000.00 from parties; thus, one who takes no part except to
These defendants are entitled to recover damages defendants Bormaheco and Maglana representing subscribe for stock in a proposed corporation which
and attorney's fees from Pioneer and its surety by the latter's participation in the ownership of the is never legally formed does not become a partner
reason of the filing of the instant case against them subject airplanes and spare parts (Exhibit 58). In with other subscribers who engage in business
and the attachment and garnishment of their addition, the cross-party plaintiffs incurred under the name of the pretended corporation, so
properties. The instant action is clearly unfounded additional expenses, hence, the total sum of P as to be liable as such in an action for settlement
insofar as plaintiff drags these defendants and 184,878.74. of the alleged partnership and contribution (Ward
defendant Maglana.' (Record on Appeal, pp. v. Brigham, 127 Mass. 24). A partnership relation
363-369, Rollo of G.R. No. 84157). We first state the principles. between certain stockholders and other
stockholders, who were also directors, will not be
We find no cogent reason to reverse or modify these While it has been held that as between themselves implied in the absence of an agreement, so as to
findings. the rights of the stockholders in a defectively make the former liable to contribute for payment
Hence, it is our conclusion that the petition in G.R. No. incorporated association should be governed by the of debts illegally contracted by the latter (Heald v.
84197 is not meritorious. supposed charter and the laws of the state relating Owen, 44 N.W. 210, 79 Iowa 23). (Corpus Juris
thereto and not by the rules governing partners Secundum, Vol. 68, p. 464). (Italics supplied).
We now discuss the merits of G.R. No. 84157. (Cannon v. Brush Electric Co., 54 A. 121, 96 Md.
Petitioner Jacob S. Lim poses the following issues: 446, 94 Am. S.R. 584), it is ordinarily held that In the instant case, it is to be noted that the petitioner was
persons who attempt, but fail, to form a declared non-suited for his failure to appear during the
l. What legal rules govern the relationship among corporation and who carry on business under the pretrial despite notification. In his answer, the petitioner
co-investors whose agreement was to do business corporate name occupy the position of partners denied having received any amount from respondents
through the corporate vehicle but who failed to inter se (Lynch v. Perryman, 119 P. 229, 29 Okl. 615, Bormaheco, the Cervanteses and Maglana. The trial court
incorporate the entity in which they had chosen to Ann. Cas. 1913A 1065). Thus, where persons and the appellate court, however, found through Exhibit
invest? How are the losses to be treated in associate themselves together under articles to 58, that the petitioner received the amount of P151,000.00
situations where their contributions to the intended purchase property to carry on a business, and their representing the participation of Bormaheco and Atty.
'corporation' were invested not through the organization is so defective as to come short of Constancio B. Maglana in the ownership of the subject
corporate form? This Petition presents these creating a corporation within the statute, they airplanes and spare parts. The record shows that defendant
fundamental questions which we believe were become in legal effect partners inter se, and their Maglana gave P75,000.00 to petitioner Jacob Lim thru the
resolved erroneously by the Court of Appeals ('CA'). rights as members of the company to the property Cervanteses.
(Rollo, p. 6). acquired by the company will be recognized (Smith
v. Schoodoc Pond Packing Co., 84 A. 268,109 Me. It is therefore clear that the petitioner never had the
These questions are premised on the petitioner's theory 555; Whipple v. Parker, 29 Mich. 369). So, where intention to form a corporation with the respondents
that as a result of the failure of respondents Bormaheco, certain persons associated themselves as a despite his representations to them. This gives credence to
Spouses Cervantes, Constancio Maglana and petitioner Lim corporation for the development of land for the cross-claims of the respondents to the effect that they
to incorporate, a de facto partnership among them was irrigation purposes, and each conveyed land to the were induced and lured by the petitioner to make
created, and that as a consequence of such relationship all corporation, and two of them contracted to pay a contributions to a proposed corporation which was never
must share in the losses and/or gains of the venture in third the difference in the proportionate value of formed because the petitioner reneged on their agreement.
proportion to their contribution. The petitioner, therefore, the land conveyed by him, and no stock was ever Maglana alleged in his cross-claim:
questions the appellate court's findings ordering him to issued in the corporation, it was treated as a
reimburse certain amounts given by the respondents to the trustee for the associates in an action between ... that sometime in early 1965, Jacob Lim
petitioner as their contributions to the intended them for an accounting, and its capital stock was proposed to Francisco Cervantes and Maglana to
corporation, to wit: treated as partnership assets, sold, and the expand his airline business. Lim was to procure two
proceeds distributed among them in proportion to DC-3's from Japan and secure the necessary
However, defendant Lim should be held liable to the value of the property contributed by each certificates of public convenience and necessity as
pay his co-defendants' cross-claims in the total (Shorb v. Beaudry, 56 Cal. 446). However, such a well as the required permits for the operation
amount of P184,878.74 as correctly found by the relation does not necessarily exist, for ordinarily thereof. Maglana sometime in May 1965, gave
trial court, with interest from the filing of the persons cannot be made to assume the relation of Cervantes his share of P75,000.00 for delivery to
cross-complaints until the amount is fully paid. partners, as between themselves, when their Lim which Cervantes did and Lim acknowledged

Page 20 of 160
CORPORATION LAW CASES MAYORDO, M.A
LLB 3

receipt thereof. Cervantes, likewise, delivered his WHEREFORE, the instant petitions are DISMISSED. The As a result of Private Respondent's refusal to amend its
share of the undertaking. Lim in an undertaking questioned decision of the Court of Appeals is AFFIRMED. Articles of Incorporation, Petitioners filed with the SEC, on
sometime on or about August 9,1965, promised to SO ORDERED. 6 February 1985, a Petition (SEC Case No. 2743) praying for
incorporate his airline in accordance with their the issuance of a Writ of Preliminary Injunction, alleging,
agreement and proceeded to acquire the planes on FORMATION AND ORGANIZATION OF CORPORATION among others, that Private Respondent's use of the word
his own account. Since then up to the filing of this PHILIPS amounts to an infringement and clear violation of
answer, Lim has refused, failed and still refuses to G.R. No. 96161 February 21, 1992 Petitioners' exclusive right to use the same considering that
set up the corporation or return the money of PHILIPS EXPORT B.V., PHILIPS ELECTRICAL LAMPS, INC. both parties engage in the same business.
Maglana. (Record on Appeal, pp. 337-338). and PHILIPS INDUSTRIAL DEVELOPMENT,
INC., petitioners, 
 In its Answer, dated 7 March 1985, Private Respondent
while respondents Bormaheco and the Cervanteses alleged vs. COURT OF APPEALS, SECURITIES & EXCHANGE countered that Petitioner PEBV has no legal capacity to
in their answer, counterclaim, cross-claim and third party COMMISSION and STANDARD PHILIPS sue; that its use of its corporate name is not at all similar
complaint: CORPORATION, respondents. to Petitioners' trademark PHILIPS when considered in its
entirety; and that its products consisting of chain rollers,
Sometime in April 1965, defendant Lim lured and MELENCIO-HERRERA, J.: belts, bearings and cutting saw are grossly different from
induced the answering defendants to purchase two Petitioners challenge the Decision of the Court of Appeals, Petitioners' electrical products.
airplanes and spare parts from Japan which the latter dated 31 July 1990, in CA-GR Sp. No. 20067, upholding the
considered as their lawful contribution and Order of the Securities and Exchange Commission, dated 2 After conducting hearings with respect to the prayer for
participation in the proposed corporation to be known January 1990, in SEC-AC No. 202, dismissing petitioners' Injunction; the SEC Hearing Officer, on 27 September 1985,
as SAL. Arrangements and negotiations were prayer for the cancellation or removal of the word "PHILIPS" ruled against the issuance of such Writ.
undertaken by defendant Lim. Down payments were from private respondent's corporate name.
advanced by defendants Bormaheco and the On 30 January 1987, the same Hearing Officer dismissed
Cervanteses and Constancio Maglana (Exh. E- 1). Petitioner Philips Export B.V. (PEBV), a  foreign corporation the Petition for lack of merit. In so ruling, the latter
Contrary to the agreement among the defendants, organized under the laws of the Netherlands, although not declared that inasmuch as the SEC found no sufficient
defendant Lim in connivance with the plaintiff, signed engaged in business here,  is the  registered owner of the ground for the granting of injunctive relief on the basis of
and executed the alleged chattel mortgage and surety trademarks PHILIPS and  PHILIPS SHIELD EMBLEM under the testimonial and documentary evidence presented, it
bond agreement in his personal capacity as the alleged Certificates of Registration Nos. R-1641 and R-1674, cannot order the removal or cancellation of the word
proprietor of the SAL. The answering defendants respectively issued by the Philippine Patents Office "PHILIPS" from Private Respondent's corporate name on the
learned for the first time of this trickery and (presently known as the Bureau of Patents, Trademarks and basis of the same evidence adopted in toto during trial on
misrepresentation of the other, Jacob Lim, when the Technology Transfer). Petitioners Philips Electrical Lamps, the merits. Besides, Section 18 of the Corporation Code
herein plaintiff chattel mortgage (sic) allegedly Inc. (Philips Electrical, for brevity) and Philips Industrial (infra) is applicable only when the corporate names in
executed by defendant Lim, thereby forcing them to Developments, Inc. (Philips Industrial, for short), question are identical. Here, there is no confusing
file an adverse claim in the form of third party claim. authorized users of the trademarks PHILIPS and PHILIPS similarity between Petitioners' and Private Respondent's
Notwithstanding repeated oral demands made by SHIELD EMBLEM, were incorporated on 29 August 1956 and corporate names as those of the Petitioners contain at least
defendants Bormaheco and Cervanteses, to defendant 25 May 1956, respectively. All petitioner corporations two words different from that of the Respondent.
Lim, to surrender the possession of the two planes and belong to the PHILIPS Group of Companies. Petitioners' Motion for Reconsideration was likewise denied
their accessories and or return the amount advanced by on 17 June 1987.
the former amounting to an aggregate sum of P Respondent Standard Philips Corporation (Standard Philips),
178,997.14 as evidenced by a statement of accounts, on the other hand, was issued a Certificate of Registration On appeal, the SEC  en banc  affirmed the dismissal
the latter ignored, omitted and refused to comply with by respondent Commission on 19 May 1982. declaring that the corporate names of Petitioners and
them. (Record on Appeal, pp. 341-342). Private Respondent hardly breed confusion inasmuch as
On 24 September 1984, Petitioners filed a letter complaint each contains at least two different words and, therefore,
Applying therefore the principles of law earlier cited to the with the Securities & Exchange Commission (SEC) asking for rules out any possibility of confusing one for the other.
facts of the case, necessarily, no de facto partnership was the cancellation of the word "PHILIPS" from Private
created among the parties which would entitle the Respondent's corporate name in view of the prior On 30 January 1990, Petitioners sought an extension of
petitioner to a reimbursement of the supposed losses of the registration with the Bureau of Patents of the trademark time to file a Petition for Review on Certiorari before this
proposed corporation. The record shows that the petitioner "PHILIPS" and the logo "PHILIPS SHIELD EMBLEM" in the name Court, which Petition was later referred to the Court of
was acting on his own and not in behalf of his other would- of Petitioner, PEBV, and the previous registration of Appeals in a Resolution dated 12 February 1990.
be incorporators in transacting the sale of the airplanes and Petitioners Philips Electrical and Philips Industrial with the
spare parts. SEC. In deciding to dismiss the petition on 31 July 1990, the
Court of Appeals1  swept aside Petitioners' claim that

Page 21 of 160
CORPORATION LAW CASES MAYORDO, M.A
LLB 3

following the ruling in  Converse Rubber Corporation v. of an individual designates the person (Cincinnati Oyster House v. Mihich, 75 Wash. 274, 134 Pac. 921). In this
Universal Converse Rubber Products, Inc., et al, (G. R. No. Cooperage Co. vs. Bate. 96 Ky 356, 26 SW 538; Newport regard, there is no doubt with respect to Petitioners' prior
L-27906, January 8, 1987, 147 SCRA 154), the word PHILIPS Mechanics Mfg. Co. vs. Starbird. 10 NH 123); and the right adoption of' the name ''PHILIPS" as part of its corporate
cannot be used as part of Private Respondent's corporate to use its corporate name is as much a part of the name. Petitioners Philips Electrical and Philips Industrial
name as the same constitutes a dominant part of corporate franchise as any other privilege granted (Federal were incorporated on 29 August 1956 and 25 May 1956,
Petitioners' corporate names. In so holding, the Appellate Secur. Co. vs. Federal Secur.Corp., 129 Or 375, 276 P 1100, respectively, while Respondent Standard Philips was issued
Court observed that the  Converse  case is not four-square 66 ALR 934; Paulino vs. Portuguese Beneficial Association, a Certificate of Registration on 12 April 1982, twenty-six
with the present case inasmuch as the contending parties 18 RI 165, 26 A 36). (26) years later (Rollo, p. 16). Petitioner PEBV has also used
in  Converse  are engaged in a similar business, that is, the the trademark "PHILIPS" on electrical lamps of all types and
manufacture of rubber shoes. Upholding the SEC, the A corporation acquires its name by choice and need not their accessories since 30 September 1922, as evidenced by
Appellate Court concluded that "private respondents' select a name identical with or similar to one already Certificate of Registration No. 1651.
products consisting of chain rollers, belts, bearings and appropriated by a senior corporation while an individual's
cutting saw are unrelated and non-competing with name is thrust upon him (See  Standard Oil Co. of New The second requisite no less exists in this case. In
petitioners' products  i.e.  electrical lamps such that Mexico, Inc. v. Standard Oil Co. of California, 56 F 2d 973, determining the existence of confusing similarity in
consumers would not in any probability mistake one as the 977). A corporation can no more use a corporate name in corporate names, the test is whether the similarity is such
source or origin of the product of the other." violation of the rights of others than an individual can use as to mislead a person, using ordinary care and
his name legally acquired so as to mislead the public and discrimination. In so doing, the Court must look to the
The Appellate Court denied Petitioners' Motion for injure another (Armington vs. Palmer, 21 RI 109. 42 A 308). record as well as the names themselves (Ohio Nat. Life Ins.
Reconsideration on 20 November 1990, hence, this Petition Co. v. Ohio Life Ins. Co., 210 NE 2d 298). While the
which was given due course on 22 April 1991, after which Our own Corporation Code, in its Section 18, expressly corporate names of Petitioners and Private Respondent are
the parties were required to submit their memoranda, the provides that: not identical, a reading of Petitioner's corporate names, to
latest of which was received on 2 July 1991. In December No corporate name may be allowed by the wit: PHILIPS EXPORT B.V., PHILIPS ELECTRICAL LAMPS, INC.
1991, the SEC was also required to elevate its records for Securities and Exchange Commission if the proposed and PHILIPS INDUSTRIAL DEVELOPMENT, INC., inevitably
the perusal of this Court, the same not having been name is  identical or deceptively or confusingly leads one to conclude that "PHILIPS" is, indeed, the
apparently before respondent Court of Appeals. similar to that of any existing corporation or to any dominant word in that all the companies affiliated or
other name already protected by law or is patently associated with the principal corporation, PEBV, are known
We find basis for petitioners' plea. deceptive, confusing or contrary to existing in the Philippines and abroad as the PHILIPS Group of
law.  Where a change in a corporate name is Companies.
As early as Western Equipment and Supply Co. v. Reyes, 51 approved, the commission shall issue an amended
Phil. 115 (1927), the Court declared that a corporation's certificate of incorporation under the amended Respondents maintain, however, that Petitioners did not
right to use its corporate and trade name is a property name. (Emphasis supplied) present an iota of proof of actual confusion or deception of
right, a right  in rem,  which it may assert and protect the public much less a single purchaser of their product
against the world in the same manner as it may protect its The statutory prohibition cannot be any clearer. To come who has been deceived or confused or showed any
tangible property, real or personal, against trespass or within its scope, two requisites must be proven, namely: likelihood of confusion. It is settled, however, that proof of
conversion. It is regarded, to a certain extent, as a actual confusion need not be shown. It suffices that
property right and one which cannot be impaired or (1) that the complainant corporation acquired a prior right confusion is probably or likely to occur (6 Fletcher [Perm
defeated by subsequent appropriation by another over the use of such corporate name; and Ed], pp. 107-108, enumerating a long line of cases).
corporation in the same field (Red Line Transportation Co.
vs. Rural Transit Co., September 8, 1934, 20 Phil 549). (2) the proposed name is either: It may be that Private Respondent's products also consist of
chain rollers, belts, bearing and the like, while petitioners
A name is peculiarly important as necessary to the very (a) identical; or deal principally with electrical products. It is significant to
existence of a corporation (American Steel Foundries vs. (b) deceptively or confusingly similar to that of any note, however, that even the Director of Patents had
Robertson, 269 US 372, 70 L ed 317, 46 S Ct 160; Lauman existing corporation or to any other name already denied Private Respondent's application for registration of
vs. Lebanon Valley R. Co., 30 Pa 42; First National Bank vs. protected by law; or the trademarks "Standard Philips & Device" for chain,
Huntington Distilling Co. 40 W Va 530, 23 SE 792). Its name (c) patently deceptive, confusing or contrary to rollers, belts, bearings and cutting saw. That office held
is one of its attributes, an element of its existence, and existing law. that PEBV, "had shipped to its subsidiaries in the Philippines
essential to its identity (6 Fletcher [Perm Ed], pp. 3-4). The equipment, machines and their parts which fall under
general rule as to corporations is that each corporation The right to the exclusive use of a corporate name with international class where "chains, rollers, belts, bearings
must have a name by which it is to sue and be sued and do freedom from infringement by similarity is determined by and cutting saw," the goods in connection with which
all legal acts. The name of a corporation in this respect priority of adoption (1 Thompson, p. 80  citing  Munn v. Respondent is seeking to register 'STANDARD PHILIPS' . . .
designates the corporation in the same manner as the name Americana Co., 82 N. Eq. 63, 88 Atl. 30; San Francisco

Page 22 of 160
CORPORATION LAW CASES MAYORDO, M.A
LLB 3

also belong" ( Inter Partes Case No. 2010, June 17, 1988, status of a well-known mark in the Philippines and
SEC Rollo). In allowing Private Respondent the continued use of its internationally as well (Bureau of Patents Decision No.
corporate name, the SEC maintains that the corporate 88-35 [TM], June 17, 1988, SEC Records).
Furthermore, the records show that among Private names of Petitioners PHILIPS ELECTRICAL LAMPS. INC. and
Respondent's primary purposes in its Articles of PHILIPS INDUSTRIAL DEVELOPMENT, INC. contain at least In support of its application for the registration of its
Incorporation (Annex D, Petition p. 37,  Rollo) are the two words different from that of the corporate name of Articles of Incorporation with the SEC, Private Respondent
following: respondent STANDARD PHILIPS CORPORATION, which words had submitted an undertaking "manifesting its willingness
will readily identify Private Respondent from Petitioners to change its corporate name in the event another person,
To  buy, sell, barter, trade, manufacture, import, and vice-versa. firm or entity has acquired a prior right to the use of the
export, or otherwise acquire, dispose of, and deal said firm name or one deceptively or confusingly similar to
in and deal with any kind  of goods, wares, and True, under the Guidelines in the Approval of Corporate and it." Private respondent must now be held to its undertaking.
merchandise such as but not limited to plastics, Partnership Names formulated by the SEC, the proposed
carbon products, office stationery and supplies, name "should not be similar to one already used by another As a general rule, parties organizing a corporation
hardware parts, electrical wiring devices, electrical corporation or partnership. If the proposed name contains a must choose a name at their peril; and the use of a
component parts, and/or complement of industrial, word already used as part of the firm name or style of a name similar to one adopted by another
agricultural or commercial machineries, registered company;  the proposed name must contain two corporation, whether a business or a nonbusiness or
constructive supplies,  electrical supplies  and other other words different from the company already non-profit organization if misleading and likely to
merchandise which are or may become articles of registered"  (Emphasis ours). It is then pointed out that injure it in the exercise in its corporate functions,
commerce except food, drugs and cosmetics and to Petitioners Philips Electrical and Philips Industrial have two regardless of intent, may be prevented by the
carry on such business as manufacturer, distributor, words different from that of Private Respondent's name. corporation having the prior right, by a suit for
d e a l e r, i n d e n t o r, f a c t o r, m a n u f a c t u r e r ' s injunction against the new corporation to prevent
representative capacity for domestic or foreign What is lost sight of, however, is that PHILIPS is a the use of the name (American Gold Star Mothers,
companies. (emphasis ours) trademark or trade name which was registered as far back Inc. v. National Gold Star Mothers, Inc., 89 App DC
as 1922. Petitioners, therefore, have the exclusive right to 269, 191 F 2d 488, 27 ALR 2d 948).
For its part, Philips Electrical also includes, among its its use which must be free from any infringement by
primary purposes, the following: similarity. A corporation has an exclusive right to the use of WHEREFORE, the Decision of the Court of Appeals dated 31
its name, which may be protected by injunction upon a July 1990, and its Resolution dated 20 November 1990, are
To develop manufacture and deal in electrical principle similar to that upon which persons are protected SET ASIDE and a new one entered ENJOINING private
products, including electronic, mechanical and in the use of trademarks and tradenames (18 C.J.S. 574). respondent from using "PHILIPS" as a feature of its
other similar products . . . (p. 30, Record of SEC Such principle proceeds upon the theory that it is a fraud corporate name, and ORDERING the Securities and
Case No. 2743) on the corporation which has acquired a right to that name Exchange Commission to amend private respondent's
and perhaps carried on its business thereunder, that Articles of Incorporation by deleting the word PHILIPS from
Given Private Respondent's aforesaid underlined primary another should attempt to use the same name, or the same the corporate name of private respondent.
purpose, nothing could prevent it from dealing in the same name with a slight variation in such a way as to induce No costs.
line of business of electrical devices, products or supplies persons to deal with it in the belief that they are dealing SO ORDERED.
which fall under its primary purposes. Besides, there is with the corporation which has given a reputation to the
showing that Private Respondent not only manufactured name (6 Fletcher [Perm Ed], pp. 39-40,  citing  Borden Ice G.R. No. 101897. March 5, 1993.
and sold ballasts for fluorescent lamps with their corporate Cream Co. v. Borden's Condensed Milk Co., 210 F 510). LYCEUM OF THE PHILIPPINES, INC., petitioner, vs. COURT
name printed thereon but also advertised the same as, Notably, too, Private Respondent's name actually contains OF APPEALS, LYCEUM OF APARRI, LYCEUM OF CABAGAN,
among others, Standard Philips (TSN, before the SEC, pp. only a single word, that is, "STANDARD", different from that LYCEUM OF CAMALANIUGAN, INC., LYCEUM OF LALLO,
14, 17, 25, 26, 37-42, June 14, 1985; pp. 16-19, July 25, of Petitioners inasmuch as the inclusion of the term INC., LYCEUM OF TUAO, INC., BUHI LYCEUM, CENTRAL
1985). As aptly pointed out by Petitioners, [p]rivate "Corporation" or "Corp." merely serves the Purpose of LYCEUM OF CATANDUANES, LYCEUM OF SOUTHERN
respondent's choice of "PHILIPS" as part of its corporate distinguishing the corporation from partnerships and other PHILIPPINES, LYCEUM OF EASTERN MINDANAO, INC. and
name [STANDARD PHILIPS CORPORATION] . . . tends to show business organizations. WESTERN PANGASINAN LYCEUM, INC., respondents.
said respondent's intention to ride on the popularity and
established goodwill of said petitioner's business throughout The fact that there are other companies engaged in other SYLLABUS
the world" (Rollo, p. 137). The subsequent appropriator of lines of business using the word "PHILIPS" as part of their 1. CORPORATION LAW; CORPORATE NAMES; REGISTRATION
the name or one confusingly similar thereto usually seeks corporate names is no defense and does not warrant the OF PROPOSED NAME WHICH IS IDENTICAL OR CONFUSINGLY
an unfair advantage, a free ride of another's goodwill use by Private Respondent of such word which constitutes SIMILAR TO THAT OF ANY EXISTING CORPORATION,
(American Gold Star Mothers, Inc. v. National Gold Star an essential feature of Petitioners' corporate name PROHIBITED; CONFUSION AND DECEPTION EFFECTIVELY
Mothers, Inc., et al, 89 App DC 269, 191 F 2d 488). previously adopted and registered and-having acquired the PRECLUDED BY THE APPENDING OF GEOGRAPHIC NAMES TO

Page 23 of 160
CORPORATION LAW CASES MAYORDO, M.A
LLB 3

THE WORD "LYCEUM". — The Articles of Incorporation of a geographically or otherwise descriptive, might nevertheless their names, but not registered with the SEC because they
corporation must, among other things, set out the name of have been used so long and so exclusively by one producer have not adopted the corporate form of organization.
the corporation. Section 18 of the Corporation Code with reference to his article that, in that trade and to that
establishes a restrictive rule insofar as corporate names are branch of the purchasing public, the word or phrase has 3. ID.; ID.; MUST BE EVALUATED IN THEIR ENTIRETY TO
concerned: "Section 18. Corporate name. — No corporate come to mean that the article was his product." The DETERMINE WHETHER THEY ARE CONFUSINGLY OR
name may be allowed by the Securities an Exchange question which arises, therefore, is whether or not the use DECEPTIVELY SIMILAR TO ANOTHER CORPORATE ENTITY'S
Commission if the proposed name is identical or by petitioner of "Lyceum" in its corporate name has been NAME. — petitioner institution is not entitled to a legally
deceptively or confusingly similar to that of any existing for such length of time and with such exclusivity as to have enforceable exclusive right to use the word "Lyceum" in its
corporation or to any other name already protected by law become associated or identified with the petitioner corporate name and that other institutions may use
or is patently deceptive, confusing or contrary to existing institution in the mind of the general public (or at least "Lyceum" as part of their corporate names. To determine
laws. When a change in the corporate name is approved, that portion of the general public which has to do with whether a given corporate name is "identical" or
the Commission shall issue an amended certificate of schools). The Court of Appeals recognized this issue and "confusingly or deceptively similar" with another entity's
incorporation under the amended name." The policy answered it in the negative: "Under the doctrine of corporate name, it is not enough to ascertain the presence
underlying the prohibition in Section 18 against the secondary meaning, a word or phrase originally incapable of "Lyceum" or "Liceo" in both names. One must evaluate
registration of a corporate name which is "identical or of exclusive appropriation with reference to an article in corporate names in their entirety and when the name of
deceptively or confusingly similar" to that of any existing the market, because geographical or otherwise descriptive petitioner is juxtaposed with the names of private
corporation or which is "patently deceptive" or "patently might nevertheless have been used so long and so respondents, they are not reasonably regarded as
confusing" or "contrary to existing laws," is the avoidance of exclusively by one producer with reference to this article "identical" or "confusingly or deceptively similar" with each
fraud upon the public which would have occasion to deal that, in that trade and to that group of the purchasing other.
with the entity concerned, the evasion of legal obligations public, the word or phrase has come to mean that the
and duties, and the reduction of difficulties of article was his produce (Ana Ang vs. Toribio Teodoro, 74 DECISION
administration and supervision over corporations. We do Phil. 56). This circumstance has been referred to as the FELICIANO, J p:
not consider that the corporate names of private distinctiveness into which the name or phrase has evolved Petitioner is an educational institution duly registered with
respondent institutions are "identical with, or deceptively through the substantial and exclusive use of the same for a the Securities and Exchange Commission ("SEC"). When it
or confusingly similar" to that of the petitioner institution. considerable period of time. . . . No evidence was ever first registered with the SEC on 21 September 1950, it used
True enough, the corporate names of private respondent presented in the hearing before the Commission which the corporate name Lyceum of the Philippines, Inc. and has
entities all carry the word "Lyceum" but confusion and sufficiently proved that the word 'Lyceum' has indeed used that name ever since.
deception are effectively precluded by the appending of acquired secondary meaning in favor of the appellant. If
geographic names to the word "Lyceum." Thus, we do not there was any of this kind, the same tend to prove only On 24 February 1984, petitioner instituted proceedings
believe that the "Lyceum of Aparri" can be mistaken by the that the appellant had been using the disputed word for a before the SEC to compel the private respondents, which
general public for the Lyceum of the Philippines, or that long period of time. . . . In other words, while the are also educational institutions, to delete the word
the "Lyceum of Camalaniugan" would be confused with the appellant may have proved that it had been using the word "Lyceum" from their corporate names and permanently to
Lyceum of the Philippines. 'Lyceum' for a long period of time, this fact alone did not enjoin them from using "Lyceum" as part of their respective
amount to mean that the said word had acquired secondary names.
2. ID.; ID.; DOCTRINE OF SECONDARY MEANING; USE OF meaning in its favor because the appellant failed to prove
WORD "LYCEUM," NOT ATTENDED WITH EXCLUSIVITY. — It is that it had been using the same word all by itself to the Some of the private respondents actively participated in
claimed, however, by petitioner that the word "Lyceum" has exclusion of others. More so, there was no evidence the proceedings before the SEC. These are the following,
acquired a secondary meaning in relation to petitioner with presented to prove that confusion will surely arise if the the dates of their original SEC registration being set out
the result that word, although originally a generic, has same word were to be used by other educational below opposite their respective names:
become appropriable by petitioner to the exclusion of institutions. Consequently, the allegations of the appellant
other institutions like private respondents herein. The in its first two assigned errors must necessarily fail." We Western Pangasinan Lyceum — 27 October 1950
doctrine of secondary meaning originated in the field of agree with the Court of Appeals. The number alone of the Lyceum of Cabagan — 31 October 1962
trademark law. Its application has, however, been extended private respondents in the case at bar suggests strongly Lyceum of Lallo, Inc. — 26 March 1972
to corporate names sine the right to use a corporate name that petitioner's use of the word "Lyceum" has not been Lyceum of Aparri — 28 March 1972
to the exclusion of others is based upon the same principle attended with the exclusivity essential for applicability of Lyceum of Tuao, Inc. — 28 March 1972
which underlies the right to use a particular trademark or the doctrine of secondary meaning. Petitioner's use of the Lyceum of Camalaniugan — 28 March 1972
tradename. In Philippine Nut Industry, Inc. v. Standard word "Lyceum" was not exclusive but was in truth shared
Brands, Inc., the doctrine of secondary meaning was with the Western Pangasinan Lyceum and a little later with The following private respondents were declared in default
elaborated in the following terms: " . . . a word or phrase other private respondent institutions which registered with for failure to file an answer despite service of summons:
originally incapable of exclusive appropriation with the SEC using "Lyceum" as part of their corporation names.
reference to an article on the market, because There may well be other schools using Lyceum or Liceo in Buhi Lyceum;

Page 24 of 160
CORPORATION LAW CASES MAYORDO, M.A
LLB 3

Central Lyceum of Catanduanes; ruling in the Lyceum of Baguio, Inc. case (SEC-Case No. re-examined Associate Commissioner Sulit's ruling in the
Lyceum of Eastern Mindanao, Inc.; and 1241) and held that the word "Lyceum" was capable of Lyceum of Baguio case. The Minute Resolution of the Court
Lyceum of Southern Philippines appropriation and that petitioner had acquired an in G.R. No. L-46595 was not a reasoned adoption of the
Petitioner's original complaint before the SEC had included enforceable exclusive right to the use of that word. Sulit ruling.
three (3) other entities: On appeal, however, by private respondents to the SEC En
1. The Lyceum of Malacanay; Banc, the decision of the hearing officer was reversed and The Articles of Incorporation of a corporation must, among
2. The Lyceum of Marbel; and set aside. The SEC En Banc did not consider the word other things, set out the name of the corporation. 6 Section
3. The Lyceum of Araullo "Lyceum" to have become so identified with petitioner as to 18 of the Corporation Code establishes a restrictive rule
render use thereof by other institutions as productive of insofar as corporate names are concerned:
The complaint was later withdrawn insofar as concerned confusion about the identity of the schools concerned in
the Lyceum of Malacanay and the Lyceum of Marbel, for the mind of the general public. Unlike its hearing officer, "SECTION 18.Corporate name. — No corporate name may be
failure to serve summons upon these two (2) entities. The the SEC En Banc held that the attaching of geographical allowed by the Securities an Exchange Commission if the
case against the Liceum of Araullo was dismissed when that names to the word "Lyceum" served sufficiently to proposed name is identical or deceptively or confusingly
school motu proprio change its corporate name to distinguish the schools from one another, especially in view similar to that of any existing corporation or to any other
"Pamantasan ng Araullo." of the fact that the campuses of petitioner and those of the name already protected by law or is patently deceptive,
private respondents were physically quite remote from confusing or contrary to existing laws. When a change in
The background of the case at bar needs some recounting. each other. 3 the corporate name is approved, the Commission shall issue
Petitioner had sometime before commenced in the SEC a Petitioner then went on appeal to the Court of Appeals. In an amended certificate of incorporation under the
proceeding (SEC-Case No. 1241) against the Lyceum of its Decision dated 28 June 1991, however, the Court of amended name." (Emphasis supplied)
Baguio, Inc. to require it to change its corporate name and Appeals affirmed the questioned Orders of the SEC En Banc.
to adopt another name not "similar [to] or identical" with 4 Petitioner filed a motion for reconsideration, without The policy underlying the prohibition in Section 18 against
that of petitioner. In an Order dated 20 April 1977, success. the registration of a corporate name which is "identical or
Associate Commissioner Julio Sulit held that the corporate deceptively or confusingly similar" to that of any existing
name of petitioner and that of the Lyceum of Baguio, Inc. Before this Court, petitioner asserts that the Court of corporation or which is "patently deceptive" or "patently
were substantially identical because of the presence of a Appeals committed the following errors: confusing" or "contrary to existing laws," is the avoidance of
"dominant" word, i.e., "Lyceum," the name of the fraud upon the public which would have occasion to deal
geographical location of the campus being the only word 1. The Court of Appeals erred in holding that the Resolution with the entity concerned, the evasion of legal obligations
which distinguished one from the other corporate name. of the Supreme Court in G.R. No. L-46595 did not and duties, and the reduction of difficulties of
The SEC also noted that petitioner had registered as a constitute stare decisis as to apply to this case and in not administration and supervision over corporations. 7
corporation ahead of the Lyceum of Baguio, Inc. in point of h o l d i n g t h a t s a i d Re s o l u t i o n b o u n d s u b s e q u e n t
time, 1 and ordered the latter to change its name to determinations on the right to exclusive use of the word We do not consider that the corporate names of private
another name "not similar or identical [with]" the names of Lyceum. respondent institutions are "identical with, or deceptively
previously registered entities. or confusingly similar" to that of the petitioner institution.
The Lyceum of Baguio, Inc. assailed the Order of the SEC 2. The Court of Appeals erred in holding that respondent True enough, the corporate names of private respondent
before the Supreme Court in a case docketed as G.R. No. Western Pangasinan Lyceum, Inc. was incorporated earlier entities all carry the word "Lyceum" but confusion and
L-46595. In a Minute Resolution dated 14 September 1977, than petitioner. deception are effectively precluded by the appending of
the Court denied the Petition for Review for lack of merit. geographic names to the word "Lyceum." Thus, we do not
Entry of judgment in that case was made on 21 October 3. The Court of Appeals erred in holding that the word believe that the "Lyceum of Aparri" can be mistaken by the
1977. 2 Lyceum has not acquired a secondary meaning in favor of general public for the Lyceum of the Philippines, or that
petitioner. the "Lyceum of Camalaniugan" would be confused with the
Armed with the Resolution of this Court in G.R. No. Lyceum of the Philippines.
L-46595, petitioner then wrote all the educational 4. The Court of Appeals erred in holding that Lyceum as a
institutions it could find using the word "Lyceum" as part of generic word cannot be appropriated by the petitioner to Etymologically, the word "Lyceum" is the Latin word for the
their corporate name, and advised them to discontinue the exclusion of others. 5 Greek lykeion which in turn referred to a locality on the
such use of "Lyceum." When, with the passage of time, it river Ilissius in ancient Athens "comprising an enclosure
became clear that this recourse had failed, petitioner We will consider all the foregoing ascribed errors, though dedicated to Apollo and adorned with fountains and
instituted before the SEC SEC-Case No. 2579 to enforce not necessarily seriatim. We begin by noting that the buildings erected by Pisistratus, Pericles and Lycurgus
what petitioner claims as its proprietary right to the word Resolution of the Court in G.R. No. L-46595 does not, of frequented by the youth for exercise and by the
"Lyceum." The SEC hearing officer rendered a decision course, constitute res adjudicata in respect of the case at philosopher Aristotle and his followers for teaching." 8 In
sustaining petitioner's claim to an exclusive right to use the bar, since there is no identity of parties. Neither is stare time, the word "Lyceum" became associated with schools
word "Lyceum." The hearing officer relied upon the SEC decisis pertinent, if only because the SEC En Banc itself has and other institutions providing public lectures and

Page 25 of 160
CORPORATION LAW CASES MAYORDO, M.A
LLB 3

concerts and public discussions. Thus today, the word institution in the mind of the general public (or at least this fact alone did not amount to mean that the said word
"Lyceum" generally refers to a school or an institution of that portion of the general public which has to do with had acquired secondary meaning in its favor because the
learning. While the Latin word "lyceum" has been schools). The Court of Appeals recognized this issue and appellant failed to prove that it had been using the same
incorporated into the English language, the word is also answered it in the negative: word all by itself to the exclusion of others. More so, there
found in Spanish (liceo) and in French (lycee). As the Court was no evidence presented to prove that confusion will
of Appeals noted in its Decision, Roman Catholic schools "Under the doctrine of secondary meaning, a word or surely arise if the same word were to be used by other
frequently use the term; e.g., "Liceo de Manila," "Liceo de phrase originally incapable of exclusive appropriation with educational institutions. Consequently, the allegations of
Baleno" (in Baleno, Masbate), "Liceo de Masbate," "Liceo de reference to an article in the market, because geographical the appellant in its first two assigned errors must
Albay." 9 "Lyceum" is in fact as generic in character as the or otherwise descriptive might nevertheless have been used necessarily fail." 13 (Underscoring partly in the original and
word "university." In the name of the petitioner, "Lyceum" so long and so exclusively by one producer with reference partly supplied)
appears to be a substitute for "university;" in other places, to this article that, in that trade and to that group of the
however, "Lyceum," or "Liceo" or "Lycee" frequently denotes purchasing public, the word or phrase has come to mean We agree with the Court of Appeals. The number alone of
a secondary school or a college. It may be (though this is a that the article was his produce (Ana Ang vs. Toribio the private respondents in the case at bar suggests strongly
question of fact which we need not resolve) that the use of Teodoro, 74 Phil. 56). This circumstance has been referred that petitioner's use of the word "Lyceum" has not been
the word "Lyceum" may not yet be as widespread as the use to as the distinctiveness into which the name or phrase has attended with the exclusivity essential for applicability of
of "university," but it is clear that a not inconsiderable evolved through the substantial and exclusive use of the the doctrine of secondary meaning. It may be noted also
number of educational institutions have adopted "Lyceum" same for a considerable period of time. Consequently, the that at least one of the private respondents, i.e., the
or "Liceo" as part of their corporate names. Since "Lyceum" same doctrine or principle cannot be made to apply where Western Pangasinan Lyceum, Inc., used the term "Lyceum"
or "Liceo" denotes a school or institution of learning, it is the evidence did not prove that the business (of the seventeen (17) years before the petitioner registered its
not unnatural to use this word to designate an entity which plaintiff) has continued for so long a time that it has own corporate name with the SEC and began using the word
is organized and operating as an educational institution. become of consequence and acquired a good will of "Lyceum." It follows that if any institution had acquired an
considerable value such that its articles and produce have exclusive right to the word "Lyceum," that institution would
It is claimed, however, by petitioner that the word acquired a well-known reputation, and confusion will result have been the Western Pangasinan Lyceum, Inc. rather than
"Lyceum" has acquired a secondary meaning in relation to by the use of the disputed name (by the defendant) (Ang Si the petitioner institution.
petitioner with the result that that word, although Heng vs. Wellington Department Store, Inc., 92 Phil. 448).
originally a generic, has become appropriable by petitioner In this connection, petitioner argues that because the
to the exclusion of other institutions like private With the foregoing as a yardstick, [we] believe the Western Pangasinan Lyceum, Inc. failed to reconstruct its
respondents herein. appellant failed to satisfy the aforementioned requisites. records before the SEC in accordance with the provisions of
No evidence was ever presented in the hearing before the R.A. No. 62, which records had been destroyed during
The doctrine of secondary meaning originated in the field Commission which sufficiently proved that the word World War II, Western Pangasinan Lyceum should be
of trademark law. Its application has, however, been 'Lyceum' has indeed acquired secondary meaning in favor of deemed to have lost all rights it may have acquired by
extended to corporate names sine the right to use a the appellant. If there was any of this kind, the same tend virtue of its past registration. It might be noted that the
corporate name to the exclusion of others is based upon to prove only that the appellant had been using the Western Pangasinan Lyceum, Inc. registered with the SEC
the same principle which underlies the right to use a disputed word for a long period of time. Nevertheless, its soon after petitioner had filed its own registration on 21
particular trademark or tradename. 10 In Philippine Nut (appellant) exclusive use of the word (Lyceum) was never September 1950. Whether or not Western Pangasinan
Industry, Inc. v. Standard Brands, Inc., 11 the doctrine of established or proven as in fact the evidence tend to Lyceum, Inc. must be deemed to have lost its rights under
secondary meaning was elaborated in the following terms: convey that the cross-claimant was already using the word its original 1933 registration, appears to us to be quite
'Lyceum' seventeen (17) years prior to the date the secondary in importance; we refer to this earlier
" . . . a word or phrase originally incapable of exclusive appellant started using the same word in its corporate registration simply to underscore the fact that petitioner's
appropriation with reference to an article on the market, name. Furthermore, educational institutions of the Roman use of the word "Lyceum" was neither the first use of that
because geographically or otherwise descriptive, might Catholic Church had been using the same or similar word term in the Philippines nor an exclusive use thereof.
nevertheless have been used so long and so exclusively by like 'Liceo de Manila,' 'Liceo de Baleno' (in Baleno, Petitioner's use of the word "Lyceum" was not exclusive but
one producer with reference to his article that, in that Masbate), 'Liceo de Masbate,' 'Liceo de Albay' long before was in truth shared with the Western Pangasinan Lyceum
trade and to that branch of the purchasing public, the word appellant started using the word 'Lyceum'. The appellant and a little later with other private respondent institutions
or phrase has come to mean that the article was his also failed to prove that the word 'Lyceum' has become so which registered with the SEC using "Lyceum" as part of
product." 12 identified with its educational institution that confusion their corporation names. There may well be other schools
will surely arise in the minds of the public if the same word using Lyceum or Liceo in their names, but not registered
The question which arises, therefore, is whether or not the were to be used by other educational institutions. with the SEC because they have not adopted the corporate
use by petitioner of "Lyceum" in its corporate name has form of organization.
been for such length of time and with such exclusivity as to In other words, while the appellant may have proved that it
have become associated or identified with the petitioner had been using the word 'Lyceum' for a long period of time,

Page 26 of 160
CORPORATION LAW CASES MAYORDO, M.A
LLB 3

We conclude and so hold that petitioner institution is not mortgage over some machineries in favor of Defendant
entitled to a legally enforceable exclusive right to use the On 10 May 1984, a Supplemental Complaint4  was filed to Bank; that thereafter, Plaintiff Corporation defaulted in the
word "Lyceum" in its corporate name and that other include additional defendants, namely: Pio Martinez, Acting payment of its IGLF loan with Defendant Bank hence
institutions may use "Lyceum" as part of their corporate Ex  Officio  Regional Sheriff of Antipolo, Rizal, and Nicanor Defendant Bank sent a demand letter dated November 22,
names. To determine whether a given corporate name is D. Blanco, Deputy Sheriff-in-Charge. 1983, reminding Plaintiff Javier to make payments because
"identical" or "confusingly or deceptively similar" with their accounts have been long overdue; that on May 2,
another entity's corporate name, it is not enough to The facts that gave rise to the aforesaid complaint, as 1984, Defendant Bank sent another demand letter to
ascertain the presence of "Lyceum" or "Liceo" in both found by Branch 62 of the RTC of Makati City, and adopted Plaintiff spouses informing them that since they have
names. One must evaluate corporate names in their by the respondent court, are as follows: defaulted in paying their obligation, their mortgage will
entirety and when the name of petitioner is juxtaposed now be foreclosed; that when Plaintiffs still failed to pay,
with the names of private respondents, they are not In February, 1981, Plaintiff P.C. Javier and Sons Services, Defendant Bank initiated extrajudicial foreclosure of the
reasonably regarded as "identical" or "confusingly or Inc., Plaintiff Corporation, for short, applied with First real estate mortgage executed by Plaintiff spouses and
deceptively similar" with each other. Summa Savings and Mortgage Bank, later on renamed as accordingly the auction sale of the property covered by TCT
PAIC Savings and Mortgage Bank, Defendant Bank, for short, No. 473216 was scheduled by the Ex–Officio Sheriff on May
WHEREFORE, the petitioner having failed to show any for a loan accommodation under the Industrial Guarantee 9, 1984.5
reversible error on the part of the public respondent Court Loan Fund (IGLF) for ₱1.5 Million. On March 21, 1981,
of Appeals, the Petition for Review is DENIED for lack of Plaintiff Corporation through Plaintiff Pablo C. Javier, The instant complaint was filed to forestall the
merit, and the Decision of the Court of Appeals dated 28 Plaintiff Javier for short, was advised that its loan extrajudicial foreclosure sale of a piece of land covered by
June 1991 is hereby AFFIRMED. No pronouncement as to application was approved and that the same shall be Transfer Certificate of Title (TCT) No. 4732166  mortgaged
costs. forwarded to the Central Bank (CB) for processing and by petitioner corporation in favor of First Summa Savings
SO ORDERED. release (Exhibit A also Exhibit 8). and Mortgage Bank which bank was later renamed as PAIC
Savings and Mortgage Bank, Inc.7  It likewise asked for the
G.R. No. 129552               June 29, 2005 The CB released the loan to Defendant Bank in two (2) nullification of the Real Estate Mortgages it entered into
P.C. JAVIER & SONS, INC., SPS. PABLO C. JAVIER, SR. and tranches of ₱750,000 each. The first tranche was released with First Summa Savings and Mortgage Bank. The
ROSALINA F. JAVIER, petitioners,  vs. HON. COURT OF to the Plaintiff Corporation on May 18, 1981 in the amount supplemental complaint added several defendants who
APPEALS, PAIC SAVINGS & MORTGAGE BANK, INC., of ₱750,000.00 and the second tranche was released to scheduled for public auction other real estate properties
SHERIFFS GRACE BELVIS, SOFRONIO VILLARIN, PIO Plaintiff Corporation on November 21, 1981 in the amount contained in the same real estate mortgages and covered
MARTINEZ and NICANOR BLANCO, respondents. of ₱750,000.00. From the second tranche release, the by TCTs No. N-5510, No. 426872, No. 506346 and Original
amount of ₱250,000.00 was deducted and deposited in the Certificate of Title No. 10146.8
DECISION name of Plaintiff Corporation under a time deposit.
CHICO-NAZARIO, J.: Several extrajudicial foreclosures of the mortgaged
Plaintiffs claim that the loan releases were delayed; that properties were scheduled but were temporarily restrained
Before Us is an appeal by  certiorari  under Rule 45 of the the amount of ₱250,000.00 was deducted from the IGLF by the RTC notwithstanding the denial9  of petitioners’
Rules of Court which seeks to set aside the decision1 of the loan of ₱1.5 Million and placed under time deposit; that prayer for a writ of preliminary injunction. In an
Court of Appeals dated 31 January 1997 which affirmed  in Plaintiffs were never allowed to withdraw the proceeds of Order10  dated 10 December 1990, the RTC ordered
toto  the decision of Branch 62 of the Regional Trial Court the time deposit because Defendant Bank intended this respondents-sheriffs to maintain the  status quo  and to
(RTC) of Makati City, dismissing the complaint for time deposit as automatic payments on the accrued desist from further proceeding with the extrajudicial
Annulment of Mortgage and Foreclosure with Preliminary principal and interest due on the loan. Defendant Bank, foreclosure of the mortgaged properties.
Injunction, Prohibition and Damages filed by petitioners, however, claims that only the final proceeds of the loan in
and its Resolution2 dated 20 June 1997 denying petitioners’ the amount of ₱750,000.00 was delayed the same having Among the issues raised by petitioners at the RTC are
motion for reconsideration. been released to Plaintiff Corporation only on November whether or not First Summa Savings and Mortgage Bank and
20, 1981, but this was because of the shortfall in the PAIC Savings and Mortgage Bank, Inc. are one and the same
A complaint3  for Annulment of Mortgage and Foreclosure collateral cover of Plaintiff’s loan; that this second tranche entity, and whether or not their obligation is already due
with Preliminary Injunction, Prohibition and Damages was of the loan was precisely released after a firm commitment and demandable at the time respondent bank commenced
filed by petitioners P.C. Javier & Sons, Inc. and spouses was made by Plaintiff Corporation to cover the collateral to extrajudicially foreclose petitioners’ properties in April
Pablo C. Javier, Sr. and Rosalina F. Javier against PAIC deficiency through the opening of a time deposit using a 1984.
Savings & Mortgage Bank, Inc., Grace S. Belvis, Acting portion of the loan proceeds in the amount of ₱250,000.00
Ex  Officio  Regional Sheriff of Pasig, Metro Manila and for the purpose; that in compliance with their commitment The RTC declared that First Summa Savings and Mortgage
Sofronio M. Villarin, Deputy Sheriff-in-Charge, before to submit additional security and open time deposit, Bank and PAIC Savings and Mortgage Bank, Inc. are one and
Branch 62 of the RTC of Makati City, on 07 May 1984. The Plaintiff Javier in fact opened a time deposit for the same entity and that petitioner corporation is liable to
case was docketed as Civil Case No. 7184. ₱250,000.00 and on February 15, 1983, executed a chattel respondent bank for the unpaid balance of its Industrial

Page 27 of 160
CORPORATION LAW CASES MAYORDO, M.A
LLB 3

Guarantee Loan Fund (IGLF) loans. The RTC further ruled COROLLARY TO THE ABOVE ARGUMENT, THE PUBLIC requirement. This being the case, this Court cannot impose
that respondent bank was justified in extrajudicially RESPONDENT COURT ALSO GRAVELY ERRED WHEN IT RULED on a bank that changes its corporate name to notify a
foreclosing the real estate mortgages executed by THAT THE PETITIONERS CANNOT WITHHOLD THEIR PAYMENT debtor of such change absent any law, circular or regulation
petitioner corporation in its favor because the loans were TO THE RESPONDENT BANK NOTWITHSTANDING THE requiring it. Such act would be judicial legislation. The
already due and demandable when it commenced ADMITTED INABILITY OF THE RESPONDENT BANK TO FURNISH formal notification is, therefore, discretionary on the bank.
foreclosure proceedings in April 1984. THE PETITIONERS THE SAID REQUESTED DOCUMENTS. Unless there is a law, regulation or circular from the SEC or
BSP requiring the formal notification of all debtors of banks
In its decision dated 06 July 1993, the RTC disposed of the b. PUBLIC RESPONDENT COURT GRAVELY ERRED WHEN IT of any change in corporate name, such notification remains
case as follows: SUSTAINED THE COLLECTION OF THE ENTIRE PROCEEDS OF to be a mere internal policy that banks may or may not
THE IGLF LOANS OF P1,500,000.00 DESPITE THE FACT THAT adopt.
Premises considered, judgment is hereby rendered THE P250,000.00 OF THIS LOAN WAS WITHHELD BY THE
dismissing the Complaint against Defendant Bank and FIRST SUMMA SAVINGS AND MORTGAGE BANK TO BECOME In the case at bar, though there was no evidence showing
ordering Plaintiffs to pay Defendant Bank jointly and PART OF THE COLLATERALS TO THE SAID P1,500,000.00 that petitioners were furnished copies of official documents
severally, the following: LOAN. showing the First Summa Savings and Mortgage Bank’s
c. PUBLIC RESPONDENT COURT GRAVELY ERRED WHEN IT change of corporate name to PAIC Savings and Mortgage
1. The principal amount of P700,453.45 under SUSTAINED THE DAMAGES AWARDED TO THE RESPONDENT Bank, Inc., evidence abound that they had notice or
P.N. No. 713 plus all the accrued interests, BANK DESPITE THE ABSENCE OF MALICE OR BAD FAITH ON knowledge thereof. Several documents establish this fact.
liquidated damages and other fees due thereon THE PART OF THE PETITIONERS IN FILING THIS CASE First, letter17  dated 16 July 1983 signed by Raymundo V.
from March 18, 1983 until fully paid as provided AGAINST THE RESPONDENT BANK. Blanco, Accountant of petitioner corporation, addressed to
in said PN; PAIC Savings and Mortgage Bank, Inc. Part of said letter
2. The principal amount of P749,879.38 under On the first assigned error, petitioners argue that they are reads: "In connection with your inquiry as to the utilization
P.N. No. 841 plus all the accrued interests, legally justified to withhold their amortized payments to of funds we obtained from the former First Summa Savings
liquidated damages and other fees due thereon the respondent bank until such time they would have been and Mortgage Bank, . . ." Second, Board Resolution18  of
from September 1, 1982 until fully paid as properly notified of the change in the corporate name of petitioner corporation signed by Pablo C. Javier, Sr. on 24
provided in such PN; First Summa Savings and Mortgage Bank. They claim that August 1983 authorizing him to execute a Chattel Mortgage
3. The amount of P40,000.00 as actual damages; they have never received any formal notice of the alleged over certain machinery in favor of PAIC Savings and
4. The amount of P30,000.00 as exemplary change of corporate name of First Summa Savings and Mortgage Bank, Inc. Third, Secretary’s Certificate19  signed
damages; Mortgage Bank to PAIC Savings & Mortgage Bank, Inc. They by Fortunato E. Gabriel, Corporate Secretary of petitioner
5. The amount of P50,000.00 as attorney’s fees; further claim that the only and first time they received corporation, on 01 September 1983, certifying that a board
plus formal evidence of a change in the corporate name of First resolution was passed authorizing Mr. Pablo C. Javier, Sr. to
6. Cost of suit.11 Summa Savings and Mortgage Bank surfaced when execute a chattel mortgage on the corporation’s equipment
respondent bank presented its witness, Michael Caguioa, on that will serve as collateral to cover the IGLF loan with
Petitioners filed a Motion for Reconsideration12  which was 03 April 1990, where he presented the Securities and PAIC Savings and Mortgage Bank, Inc. Fourth, undated
opposed13  by respondent bank. The motion was denied in Exchange Commission (SEC) Certificate of Filing of the letter20 signed by Pablo C. Javier, Sr. and addressed to PAIC
an Order dated 11 May 1994. Amended Articles of Incorporation of First Summa Savings Savings and Mortgage Bank, Inc., authorizing Mr. Victor F.
and Mortgage Bank,14  the Central Bank (CB) Certificate of Javier, General Manager of petitioner corporation, to
Petitioners appealed the decision to the Court of Appeals. Authority15 to change the name of First Summa Savings and secure from PAIC Savings and Mortgage Bank, Inc. certain
The latter affirmed in toto the decision of the lower court. Mortgage Bank to PAIC Savings and Mortgage Bank, Inc., and documents for his signature.
It also denied petitioners’ motion for reconsideration. the CB Circular Letter16 dated 27 June 1983.
From the foregoing documents, it cannot be denied that
Hence, this appeal by certiorari. Their argument does not hold water. Their defense that petitioner corporation was aware of First Summa Savings
they should first be formally notified of the change of and Mortgage Bank’s change of corporate name to PAIC
Petitioners assigned the following as errors: corporate name of First Summa Savings and Mortgage Bank Savings and Mortgage Bank, Inc. Knowing fully well of such
to PAIC Savings and Mortgage Bank, Inc., before they will change, petitioner corporation has no valid reason not to
a. PUBLIC RESPONDENT COURT GRAVELY ERRED WHEN IT continue paying their loan obligations to respondent bank pay because the IGLF loans were applied with and obtained
SUSTAINED THE DISMISSAL OF PETITIONERS’ COMPLAINT AND presupposes that there exists a requirement under a law or from First Summa Savings and Mortgage Bank. First Summa
IN AFFIRMING THE RIGHT OF THE RESPONDENT BANK TO regulation ordering a bank that changes its corporate name Savings and Mortgage Bank and PAIC Savings and Mortgage
COLLECT THE IGLF LOANS IN LIEU OF FIRST SUMMA SAVINGS to formally notify all its debtors. After going over the Bank, Inc., are one and the same bank to which petitioner
AND MORTGAGE BANK WHICH ORIGINALLY GRANTED SAID Corporation Code and Banking Laws, as well as the corporation is indebted. A change in the corporate name
LOANS. regulations and circulars of both the SEC and the Bangko does not make a new corporation, whether effected by a
Sentral ng Pilipinas (BSP), we find that there is no such special act or under a general law. It has no effect on the

Page 28 of 160
CORPORATION LAW CASES MAYORDO, M.A
LLB 3

identity of the corporation, or on its property, rights, or mortgage on additional machineries, equipment and tools IGLF loans that it had no choice but to comply with
liabilities.21 The corporation, upon such change in its name, and thru the opening of a time deposit with PAIC Bank using respondent bank’s requirement to put in time deposits the
is in no sense a new corporation, nor the successor of the a portion of the loan proceeds in the amount of said amount as additional collateral.
original corporation. It is the same corporation with a P250,000.00 to answer for its obligation to the defendant
different name, and its character is in no respect changed. bank under the IGLF loan was the final proceeds of the loan We agree with respondent court that the questioning of the
22 released in favor of the plaintiffs. The delay in the release propriety of the placing of the P250,000.00 in time
of the final proceeds of the IGLF loan was due to the deposits32 with respondent bank as additional collateral was
Anent the second assigned error, this Court rules that aforestated collateral deficiency.25 belatedly made. As above-discussed, the requirement to
respondent court did not err when it sustained the give additional collateral was warranted because the
collection of the entire proceeds of the IGLF loans As declared by the respondent court, the finding in said collateral petitioner corporation put up failed to cover its
amounting to P1,500,000.00 despite the withholding of order was not disputed in the appeal before it. It said that IGLF loans. If petitioner corporation was really bent on
P250,000.00 to become part of the collaterals to the said what was contained in petitioners’ brief was that "their questioning the reasonableness of putting up the
P1,500,000.00 IGLF loan. loans were ‘overcollateralized,’ and fail to specify why or aforementioned amount as additional collateral, it should
in what manner it was so."26 Having failed to raise this issue have done immediately after it made the time deposits on
Petitioners contend that the collaterals they submitted before the respondent court, petitioners thus cannot raise 26 November 1981. This, it did not do. It questioned the
were more than sufficient to cover the P1,500,000.00 IGLF this issue before this Court. Moreover, since the issue of placing of the time deposits only on 08 February 198433 or
loan. Such contention is untenable. Petitioner corporation whether or not the collateral put up by petitioners is long after defendant bank had already demanded full
was required to place P250,000.00 in a time deposit with sufficient is factual, the same is not proper for this Court’s payment of the loans, then amounting to P2,045,401.79 as
respondent bank for the simple reason that the collateral it consideration. The basic rule is that factual questions are of 22 November 1983. It is too late in the day for petitioner
put up was insufficient to cover the IGLF loans it has beyond the province of the Supreme Court in a petition for corporation to question the placing of the P250,000.00 in
received. It admitted the shortfall of its collateral when it review.27 time deposits after it failed to pay its loan obligations as
authorized petitioner Pablo C. Javier, Sr.,  via  a board scheduled, making them due and demandable, and after a
resolution,23  to execute a chattel mortgage over certain Petitioners maintain that to collect the P250,000.00 from demand for full payment has been made. We will not allow
machinery in favor of PAIC Savings and Mortgage Bank, Inc. them would be a clear case of unjust enrichment because petitioner corporation to have one’s cake and eat it too.
which was certified by its corporate secretary.24  If the they have not availed or used said amount for the same was As regards the payments made by petitioner corporation,
collateral it put up was sufficient, why then did it execute unlawfully withheld from them. respondent court has this to say:
another chattel mortgage?
In his order dated 07 September 1984, Hon. Rafael T. We do not agree. The fundamental doctrine of unjust The trial court held, based on plaintiffs’ own exhibits, that
Mendoza found that the loanable value of the lands, enrichment is the transfer of value without just cause or plaintiff[s] made the following payments:
buildings, machinery and equipment amounted only to consideration. The elements of this doctrine are:
P934,000.00. The order reads in part: enrichment on the part of the defendant; impoverishment On Promissory Note No. 713:
on the part of the plaintiff; and lack of cause. The main
Date
 Actual Date of
 Amount
The terms and conditions of the IGLF loan extended to objective is to prevent one to enrich himself at the expense
(Per P N Payment
plaintiff corporation are governed by the loan and security of another.28  It is commonly accepted that this doctrine
Schedule)
documents evidencing said loan. Although the loan simply means that a person shall not be allowed to profit or
agreement was approved by the defendant bank, the same enrich himself inequitably at another's expense.29  In the
has to be processed and be finally approved by the Central instant case, there is no unjust enrichment to speak of. The
Bank of the Philippines, in pursuance to the IGLF program, amount of P225,905.79 was applied as payment for July 6, 1981 August 3, 1981 ₱ 28,125.00
of which the defendant bank is an accredited participant. petitioner corporation’s loan which was taken from the
The defendant had to await Central Bank’s advise (sic) P250,000.00, together with its accrued interest, that was October 6, October 28,
28,836.13
regarding the final approval of the loan before the release placed in time deposit with First Summa Savings and 1981 1981
of the proceeds thereof. The proceeds of the loan was Mortgage Bank. The use of said amount as payment was
released to the plaintiff on 6 April and November 20, 1981, approved by petitioner Pablo C. Javier, Sr. on 17 March January 6, January 22,
29,227.38
and the final proceeds was released only on November 20, 1983.30  As further found by the RTC in its decision, the 1982 1982
1981, on account of short fall in the collateral covered by balance of the time deposit was withdrawn by petitioners.31
March 17, 1983 225,905.79
the lands and buildings as well as the machineries and Petitioner corporation faults respondent bank, then known
equipment then subject of the existing mortgages in favor as First Summa Savings and Mortgage Bank, for requiring it
of the defendant bank, having only a loanable value of to put up as additional collateral the amount of TOTAL ₱
P934,000.00, and only after a firm commitment made by P250,000.00 inasmuch as the CB never required it to do so. 312,094.30
plaintiff corporation to the defendant bank to correct the It added that respondent bank took advantage of its urgent
collateral deficiency thru the execution of a chattel and immediate need at the time for the proceeds of the And on Promissory Note No. 841:

Page 29 of 160
CORPORATION LAW CASES MAYORDO, M.A
LLB 3

its corporate name to PAIC Savings and Mortgage Bank, Inc. unfounded legal action, Defendant Bank should be entitled
Date

Actual Date of
 Despite knowledge that First Summa Savings and Mortgage to an award of exemplary damages.35
(Per PN Amount
Payment Bank and PAIC Savings and Mortgage Bank, Inc., are one and
Schedule)
the same entity, it pretended otherwise. It used this This Court finds that petitioners failed to comply with what
purported ignorance as an excuse to renege on its is incumbent upon them – to pay their loans when they
obligation to pay its loans after they became due and after became due. The lame excuse they belatedly advanced for
F e b r u a r y 2 0 , April 13, 1982 ₱ demands for payment were made, claiming that it never their non-payment cannot and should not prevent
1982 28,569.30 obtained the loans from respondent bank. respondent bank from exercising its right to foreclose the
real estate mortgages executed in its favor.
May 20, 1982 July 7, 1982 29,254.31 No good faith was shown by petitioner corporation. If it
were in good faith in complying with its loan obligations WHEREFORE, premises considered, the Court of Appeals
August 20, 1982 August 31, 36,795.44 since it believed that respondent bank had no right to the decision dated 31 January 1997 and its resolution dated 20
1982 payment, it should have made a valid consignation in court. June 1997 are hereby AFFIRMED  in toto.  Costs against
This, it did not do. If petitioner corporation were at a loss petitioners.
TOTAL ₱
as to who should receive the payment, it could have easily SO ORDERED.
94,619.05
taken steps and inquired from the SEC, CB of the
Philippines or from the bank itself from which it received G.R. No. L-26370 July 31, 1970
Plaintiff-appellant[s] does not dispute the finding, which is the loans and to where it made previous payments. Further, PHILIPPINE FIRST INSURANCE COMPANY, INC., plaintiff-
obvious from the foregoing summary, that plaintiff[s] the fact that it was respondent bank that was demanding appellant,  vs. MARIA CARMEN HARTIGAN, CGH, and O.
stopped payments on March 17, 1983 on Promissory Note payment for loans already due and demandable and not ENGKEE, defendants-appellees.
No. 713, and on August 31, 1982 on Promissory Note No. First Summa Savings and Mortgage Bank is sufficient to
841. make petitioner corporation wonder why this is so. It never BARREDO, J.:
took any initiative to clear the matter. Instead, it paid no Appeal from the decision dated 6 October 1962 of the Court
By simply looking at the amortization schedule attached to attention to the valid demands of respondent bank. of First Instance of Manila — dismissing the action in its
the two promissory notes, it is clear that plaintiff[s] Civil Case No. 48925 — brought by the herein plaintiff-
already defaulted on its loan obligations when the The awarding of actual and compensatory damages, as well appellant Philippine First Insurance Co., Inc. to the Court of
defendant Bank gave notice of the foreclosure proceedings as attorney’s fees, is justified under the circumstances. We Appeals which could, upon finding that the said appeal
on April 28, 1984. On amortization payments alone, quote with approval the reasons given by the RTC for the raises purely questions of law, declared itself without
plaintiff[s] should have paid a total of P459,339 as of April grant of the same: jurisdiction to entertain the same and, in its resolution
6, 1984 on Promissory [Note] No. 713, and a total of dated 15 July 1966, certified the records thereof to this
P328,173.00 as of February 20, 1984 on Promissory Note Considering that Defendant Bank had been prevented at Court for proper determination.
[No.] 841. No extended computation is necessary to least four (4) times from foreclosing the mortgages (i.e.,
demonstrate that, even without imputing the liquidated Temporary Restraining Orders of May 9 and 19 and October The antecedent facts are set forth in the pertinent portions
damages equivalent to 2% a month on the delayed 22, 1984 and status quo order of December 10, 1990 of the resolution of the Court of Appeals referred to as
payments (see second paragraph of the promissory notes), enjoining the extrajudicial foreclosure sales of May 9 and follows:
the plaintiffs were grossly deficient in amortization 16 and October 23, 1984 and December 20, 1990,
payments, and already in default when the foreclosure respectively), it is proper that Defendant Bank be According to the complaint, plaintiff was originally
proceedings were commenced. Further, we note that under reimbursed its actual expenses. The amount of P40,000.00 organized as an insurance corporation under the
the terms of the promissory note, "failure to pay an is reasonable reimbursement for the publication and other name of 'The Yek Tong Lin Fire and Marine Insurance
installment when due shall entitle the bank or its assign to expenses incurred in the four (4) extrajudicial foreclosures Co., Ltd.' The articles of incorporation originally
declare all the obligations as immediately due and payable" which were enjoined by the Court. Considering the wanton presented before the Security and Exchange
(second paragraph).34 and reckless filing of this clearly unfounded and baseless Commissioner and acknowledged before Notary
legal action and the fact that Defendant Bank had to Public Mr. E. D. Ignacio on June 1, 1953 state that
As to the third assigned error, petitioners argue that there defend itself against such suit, attorney’s fees in the the name of the corporation was 'The Yek Tong Lin
being no malice or bad faith on their part when they filed amount of P50,000.00 should be paid by the Plaintiffs to Fire and Marine Insurance Co., Ltd.' On May 26,
the instant case, no damages should have been awarded to the Defendant Bank. Defendant Bank failed to adduce 1961 the articles of incorporation were amended
respondent bank. indubitable proof on the moral and exemplary damages pursuant to a certificate of the Board of Directors
We cannot sustain such argument. The presence of malice that it seeks. Nevertheless, since such proof is not dated March 8, 1961 changing the name of the
or bad faith is very evident in the case before us. By the absolutely necessary and primarily as an example for the corporation to 'Philippine First Insurance Co., Inc.'.
documents it executed, petitioner corporation was well public good to deter others from filing a similar clearly
aware that First Summa Savings and Mortgage Bank changed

Page 30 of 160
CORPORATION LAW CASES MAYORDO, M.A
LLB 3

The complaint alleges that the plaintiff Philippine Ltd.' and not in favor of the plaintiff. They likewise Philippines First Insurance Co., Inc. is of dubious
First Insurance Co., Inc., doing business under the admit that they failed to pay the promissory note validity. Such change of name in effect dissolved
name of 'The Yek Tong Lin Fire and Marine Insurance when it fell due but they allege that since their the original corporation by a process of dissolution
Co., Lt.' signed as co-maker together with obligation with the China Banking Corporation not authorized by our corporation law (see Secs. 62
defendant Maria Carmen Hartigan, CGH, a based on the promissory note still subsists, the and 67, inclusive, of our Corporation Law).
promissory note for P5,000.00 in favor of the China surety who co-signed the promissory note is not Moreover, said change of name, amounting to a
Banking Corporation payable within 30 days after entitled to collect the value thereof from the dissolution of the Yek Tong Lin Fire & Marine
the date of the promissory note with the usual defendants otherwise they will be liable for double Insurance Co., Ltd., does not appear to have been
banking interest; that the plaintiff agreed to act as amount of their obligation, there being no effected with the written note or assent of
such co-maker of the promissory note upon the allegation that the surety has paid the obligation to stockholders representing at least two-thirds of the
application of the defendant Maria Carmen the creditor. subscribed capital stock of the corporation, a voting
Hartigan, CGH, who together with Antonio F. Chua proportion required not only for the dissolution of a
and Chang Ka Fu, signed an indemnity agreement in By way of special defense, defendants claim that corporation but also for any amendment of its
favor of the plaintiff, undertaking jointly and there is no privity of contract between the plaintiff articles of incorporation (Secs. 18 and 62,
severally, to pay the plaintiff damages, losses or and the defendants and consequently, the plaintiff Corporation Law). Furthermore, such change of
expenses of whatever kind or nature, including has no cause of action against them, considering corporate name appears to be against public policy
attorney's fees and legal costs, which the plaintiff that the complaint does not allege that the plaintiff and may be effected only by express authority of
may sustain as a result of the execution by the and the 'Yek Tong Lin Fire and Marine Insurance Co., law (Red Line Transportation Co. v. Rural Transit
plaintiff and co-maker of Maria Carmen Hartigan, Ltd.' are one and the same or that the plaintiff has Co., Ltd., 60 Phil. 549, 555; Cincinnati Cooperage
CGH, of the promissory note above-referred to; acquired the rights of the latter. The parties after Co., Ltd. vs. Vate, 26 SW 538, 539; Pilsen Brewing
that as a result of the execution of the promissory the admission of Exhibit A which is the amended Co. vs. Wallace, 125 NE 714), but there is nothing in
note by the plaintiff and Maria Carmen Hartigan, articles of incorporation and Exhibit 1 which is a our corporation law authorizing the change of
CGH, the China Banking Corporation delivered to demand letter dated August 16, 1962 signed by the corporate name in this jurisdiction.
the defendant Maria Carmen Hartigan, CGH, the manager of the loans and discount department of
sum of P5,000.00 which said defendant failed to the China Banking Corporation showing that the In the second place, assuming that the change of
pay in full, such that on August 31, 1961 the same promissory note up to said date in the sum of name of the Yek Tong Lin Fire & Marine Insurance
was. renewed and as of November 27, 1961 there P4,500.00 was still unpaid, submitted the case for Co. Ltd., to Philippines pine First Insurance Co.,
was due on account of the promissory note the sum decision based on the pleadings. Inc., as accomplished on March 8, 1961, is valid,
of P4,559.50 including interest. The complaint ends that would mean that the original corporation, the
with a prayer for judgment against the defendants, Under date of 6 October 1962, the Court of First Instance of Yek Tong Lin Fire & Marine Insurance Co., Ltd.,
jointly and severally, for the sum of P4,559.50 with Manila rendered the decision appealed. It dismissed the became dissolved and of no further existence since
interest at the rate of 12% per annum from action with costs against the plaintiff Philippine First March 8, 1961, so that on May 15, 1961, the date
November 23, 1961 plus P911.90 by way of Insurance Co., Inc., reasoning as follows: the indemnity agreement, Annex A, was executed,
attorney's fees and costs. said original corporation bad no more power to
... With these undisputed facts in mind, the parties enter into any agreement with the defendants, and
Although O. Engkee was made as party defendant in correctly concluded that the issues for resolution by the agreement entered into by it was ineffective
the caption of the complaint, his name is not this Court are as follows: for lack of capacity of said dissolved corporation to
mentioned in the body of said complaint. However, enter into said agreement. At any rate, even if we
his name Appears in the Annex A attached to the (a) Whether or not the plaintiff is the real party in hold that said change of name is valid, the fact
complaint which is the counter indemnity interest that may validly sue on the indemnity remains that there is no evidence showing that the
agreement supposed to have been signed according agreement signed by the defendants and the Yek new entity, the Philippine First Insurance Co., Inc.
to the complaint by Maria Carmen Hartigan, CGH, Tong Lin Fire & Marine Insurance Co., Ltd. (Annex A has with the consent of the original parties,
Antonio F. Chua and Chang Ka Fu. to plaintiff's complaint ); and assumed the obligations or was assigned the rights
of action in the original corporation, the Yek Tong
In their answer the defendants deny the allegation (b) Whether or not a suit for indemnity or Lin Fire & Marine Insurance Co., Ltd. In other
that the plaintiff formerly conducted business reimbursement may under said indemnity words, there is no evidence of conventional
under the name and style of 'The Yek Tong Lin Fire agreement prosper without plaintiff having yet paid subrogation of the Plaintiffs in the rights of the Yek
and Marine Insurance Co., Ltd.' They admit the the amount due under said promissory note. Tong Lin Fire & Marine Insurance Co., Ltd. under
execution of the indemnity agreement but they said indemnity agreement (Arts. 1300, 1301, New
claim that they signed said agreement in favor of In the first place, the change of name of the Yek Civil Code). without such subrogation assignment of
the Yek Tong Lin Fire and Marine Insurance Co., Tong Lin Fire & Marine Insurance Co., Ltd. to the rights, the herein plaintiff has no cause of action

Page 31 of 160
CORPORATION LAW CASES MAYORDO, M.A
LLB 3

against the defendants, and is, therefore, not the 13, Corp. Law). The enumeration of corporate Securities and Exchange Commissioner. We find no
right party in interest as plaintiff. powers made in our Corporation Law implies the justification, legal, moral, or practical, for
exclusion of all others (Thomas v. West Jersey R. adhering to the view taken by the Securities and
Last, but not least, assuming that the said change Co., 101 U.S. 71, 25 L. ed. 950). It is obvious, in Exchange Commissioner that the name of a
of name was legal and operated to dissolve the this connection, that change of name is not one of corporation in the Philippines may be changed by
original corporation, the dissolved corporation, the powers necessary to the exercise of the powers mere amendment of its Articles of Incorporation as
must pursuant to Sec. 77 of our corporation law, be conferred on corporations by said Sec. 13 (see Sec. to its corporate name. A change of corporate name
deemed as continuing as a body corporate for three 14, Corp. Law). would serve no useful purpose, but on the contrary
(3) years from March 8, 1961 for the purpose of would most probably cause confusion. Only a
prosecuting and defending suits. It is, therefore, To rule that Sec. 18 of our Corporation Law dubious purpose could inspire a change of a
the Yek Tong Lin Fire & Marine Insurance Co., Ltd. authorizes the change of name of a corporation by corporate. name which, unlike a natural person's
that is the proper party to sue the defendants amendment of its Articles of Incorporation is to name, was chosen by the incorporators themselves;
under said indemnity agreement up to March 8, indulge in judicial legislation. We have examined and our Courts should not lend their assistance to
1964. the cases cited in Volume 13 of American the accomplishment of dubious purposes.
Jurisprudence in support of the proposition that the WHEREFORE, we hereby deny plaintiff's motion for
Having arrived at the foregoing conclusions, this general power to alter or amend the charter of a reconsideration, dated November 8, 1962, for lack
Court need not squarely pass upon issue (b) corporation necessarily includes the power to alter of merit.
formulated above. the name of a corporation, and find no justification
WHEREFORE, plaintiff's action is hereby dismissed, for said conclusion arrived at by the editors of In this appeal appellant contends that —
with costs against the plaintiff. American Jurisprudence. On the contrary, the I
annotations in favor of plaintiff's view appear to THE TRIAL COURT ERRED IN HOLDING THAT IN THIS
In due time, the Philippine First Insurance Company, Inc. have been based on decisions in cases where the JURISDICTION, THERE IS NOTHING IN OUR
moved for reconsideration of the decision aforesaid, but statute itself expressly authorizes change of CORPORATION LAW AUTHORIZING THE CHANGE OF
said motion was denied on December 3, 1962 in an order corporate name by amendment of its Articles of CORPORATE NAME;
worded thus: Incorporation. The correct rule in harmony with the II
provisions of our Corporation Law is well expressed THE TRIAL COURT ERRED IN DECLARING THAT A
The motion for reconsideration, dated November 8, in an English case as follows: CHANGE OF CORPORATE NAME APPEARS TO BE
1962, raises no new issue that we failed to consider AGAINST PUBLIC POLICY;
in rendering our decision of October 6, 1962. After a company has been completely register III
However, it gives us an opportunity to amplify our without defect or omission, so as to be THE TRIAL COURT ERRED IN HOLDING THAT A
decision as regards the question of change of name incorporated by the name set forth in the deed CHANGE OF CORPORATE NAME HAS THE LEGAL
of a corporation in this jurisdiction. of settlement, such incorporated company has E F F E C T O F D I S S O LV I N G T H E O R I G I N A L
not the power to change its name ... Although CORPORATION:
We find nothing in our Corporation Law authorizing the King by his prerogative might incorporate IV
a change of name of a corporation organized by a new name, and the newly named THE TRIAL COURT ERRED IN HOLDING THAT THE
pursuant to its provisions. Sec. 18 of the corporation might retain former rights, and CHANGE OF NAME OF THE YEK TONG LIN FIRE &
Corporation Law authorizes, in our opinion, sometimes its former name also, ... it never MARINE INSURANCE CO., LTD. IS OF DUBIOUS
amendment to the Articles of Incorporation of a appears to be such an act as the corporation VALIDITY;
corporation only as to matters other than its could do by itself, but required the same power V
corporate name. Once a corporation is organized in as created the corporation. (Reg. v. Registrar of THE TRIAL COURT ERRED IN HOLDING THAT THE
this jurisdiction by the execution and registration Joint Stock Cos 10 Q.B. 839, 59 E.C.L. 839). APPELLANT HEREIN IS NOT THE RIGHT PARTY
of its Articles of Incorporation, it shall continue to INTEREST TO SUE DEFENDANTS-APPELLEES;
exist under its corporate name for the lifetime of The contrary view appears to represent the IV
its corporate existence fixed in its Articles of minority doctrine, judging from the annotations on THE TRIAL COURT FINALLY ERRED IN DISMISSING THE
Incorporation, unless sooner legally dissolved (Sec. decided cases on the matter. COMPLAINT.
11, Corp. Law). Significantly, change of name is not
one of the methods of dissolution of corporations The movant invokes as persuasive precedent the Appellant's Position is correct; all the above assignments of
expressly authorized by our Corporation Law. Also action of the Securities Commissioner in tacitly error are well taken. The whole case, however, revolves
significant is the fact that the power to change its approving the Amended, Articles of Incorporation around only one question. May a Philippine corporation
corporate name is not one of the general powers on May 26, 1961. We regret that we cannot in good change its name and still retain its original personality and
conferred on corporations in this jurisdiction (Sec. conscience lend approval to this action of the individuality as such?

Page 32 of 160
CORPORATION LAW CASES MAYORDO, M.A
LLB 3

corporation:  Provided,  however, That their own filing said copy of the amended articles of
The answer is not difficult to find. True, under Section 6 of shares of stock purchased or otherwise acquired by incorporation.  Provided, however, That when the
the Corporation Law, the first thing required to be stated in banks, trust companies, and insurance companies, amendment consists in extending the term of corporate
the Articles of Incorporation of any corn corporation is its should be disposed of within six months after existence, the Securities and Exchange Commissioner shall
name, but it is only one among many matters equally if not acquiring title thereto. be entitled to collect and receive for the filing of its
more important, that must be stated therein. Thus, it is amended articles of incorporation the same fees collectible
also required, for example, to state the number and names Unless and until such amendment to the articles of under existing law for the filing of articles of incorporation.
of and residences of the incorporators and the residence or incorporation shall have been abandoned or the The Securities & Exchange Commissioner shall not hereafter
location of the principal office of the corporation, its term action rescinded, the stockholder making such file any amendment to the articles of incorporation of any
of existence, the amount of its capital stock and the demand in writing shall cease to be a stockholder bank, banking institution, or building and loan association
number of shares into which it is divided, etc., etc. and shall have no rights with respect to such unless accompanied by a certificate of the Monetary Board
shares, except the right to receive payment (of the Central Bank) to the effect that such amendment is
On the other hand, Section 18 explicitly permits the therefor as aforesaid. in accordance with law. (As further amended by Act No.
articles of incorporation to be amended thus: 3610, Sec. 2 and Sec. 9.R.A. No. 337 and R.A. No. 3531.)
A stockholder shall not be entitled to payment for It can be gleaned at once that this section does not only
Sec. 18. — Any corporation may for legitimate his shares under the provisions of this section unless authorize corporations to amend their charter; it also lays
corporate purpose or purposes, amend its articles the value of the corporate assets which would down the procedure for such amendment; and, what is
of incorporation by a majority vote of its board of remain after such payment would be at least equal more relevant to the present discussion, it contains
directors or trustees and the vote or written assent to the aggregate amount of its debts and liabilities provisos restricting the power to amend when it comes to
of two-thirds of its members, if it be a nonstock and the aggregate par value and/or issued value of the term of their existence and the increase or decrease of
corporation or, if it be a stock corporation, by the the remaining subscribed capital stock. the capital stock. There is no prohibition therein against
vote or written assent of the stockholders the change of name. The inference is clear that such a
representing at least two-thirds of the subscribed A copy of the articles of incorporation as amended, change is allowed, for if the legislature had intended to
capital stock of the corporation Provided, however, duly certified to be correct by the president and enjoin corporations from changing names, it would have
That if such amendment to the articles of the secretary of the corporation and a majority of expressly stated so in this section or in any other provision
incorporation should consist in extending the the board of directors or trustees, shall be filed of the law.
corporate existence or in any change in the rights with the Securities and Exchange Commissioner,
of holders of shares of any class, or would authorize who shall attach the same to the original articles of No doubt, "(the) name (of a corporation) is peculiarly
shares with preferences in any respect superior to incorporation, on file in his office. From the time of important as necessary to the very existence of a
those of outstanding shares of any class, or would filing such copy of the amended articles of corporation. The general rule as to corporations is that
restrict the rights of any stockholder, then any incorporation, the corporation shall have the same each corporation shall have a name by which it is to sue
stockholder who did not vote for such corporate powers and it and the members and stockholders and be sued and do all legal acts. The name of a
action may, within forty days after the date upon thereof shall thereafter be subject to the same corporation in this respect designates the corporation in
which such action was authorized, object thereto in liabilities as if such amendment had been embraced the same manner as the name of an individual designates
writing and demand Payment for his shares. If, in the original articles of incorporation:  Provided, the person."1 Since an individual has the right to change his
after such a demand by a stockholder, the however, That should the amendment consist in name under certain conditions, there is no compelling
corporation and the stockholder cannot agree upon extending the corporate life, the extension shall reason why a corporation may not enjoy the same right.
the value of his share or shares at the time such not exceed 50 years in any one instance. Provided, There is nothing sacrosanct in a name when it comes to
corporate action was authorized, such values all be further, That the original articles and amended artificial beings. The sentimental considerations which
ascertained by three disinterested persons, one of articles together shall contain all provisions individuals attach to their names are not present in
whom shall be named by the stockholder, another required by law to be set out in the articles of corporations and partnerships. Of course, as in the case of
by the corporation, and the third by the two thus incorporation:  And provided, further, That nothing an individual, such change may not be made exclusively. by
chosen. The findings of the appraisers shall be in this section shall be construed to authorize any the corporation's own act. It has to follow the procedure
final, and if their award is not paid by the corporation to increase or diminish its capital stock prescribed by law for the purpose; and this is what is
corporation within thirty days after it is made, it or so as to effect any rights or actions which important and indispensably prescribed — strict adherence
may be recovered in an action by the stockholder accrued to others between the time of filing the to such procedure.
against the corporation. Upon payment by the original articles of incorporation and the filing of
corporation to the stockholder of the agreed or the amended articles. Local well known corporation law commentators are
awarded price of his share or shares, the unanimous in the view that a corporation may change its
stockholder shall forthwith transfer and assign the The Securities and, Exchange Commissioner shall be name by merely amending its charter in the manner
share or shares held by him as directed by the entitled to collect and receive the sum of ten pesos for prescribed by law.2  American authorities which have

Page 33 of 160
CORPORATION LAW CASES MAYORDO, M.A
LLB 3

persuasive force here in this regard because our carefully examined these authorities and We are satisfied Nat. F. i. C. vs. Swedish Nat. Assn., 205 Illinois
corporation law is of American origin, the same being a sort of their relevance. Even Lord Denman who has been quoted [Appellate Courts], 428, 434.)
of codification of American corporate law,3 are of the same by His Honor from  In Reg. v. Registrar of Joint Stock Cos.
opinion. 10, Q.B., 59 E.C.L. maintains merely that the change of its In other words, what We have held to be contrary to public
name never appears to be such an act as the corporation policy is the use by one corporation of the name of another
A general power to alter or amend the charter of a could do for itself, but required ;the same Power as corporation as its trade name. We are certain no one will
corporation necessarily includes the power to alter created a corporation." What seems to have been disagree that such an act can only "result in confusion and
the name of the corporation.  Ft. Pitt Bldg., etc., overlooked, therefore, is that the procedure prescribes by open the door to frauds and evasions and difficulties of
Assoc. v. Model Plan Bldg., etc., Assoc., 159 Pa. St. Section 18 of our Corporation Law for the amendment of administration and supervision." Surely, the Red Line case
308, 28 Atl. 215;  In re  Fidelity Mut. Aid Assoc., 12 corporate charters is practically identical with that for the was not one of change of name.
W.N.C. (Pa.) 271; Excelsior Oil Co., 3 Pa. Co. Ct. incorporation itself of a corporation.
184; Wetherill Steel Casting Co., 5 Pa. Co. Ct. 337. Neither can We share the posture of His Honor that the
In the appealed order of dismissal, the trial court, made change of name of a corporation results in its dissolution.
xxx xxx xxx the observation that, according to this Court in  Red Line There is unanimity of authorities to the contrary.
Under the General Laws of Rhode Island, c 176, sec. Transportation Co. v. Rural Transit Co., Ltd., 60 Phil, 549,
7, relating to an increase of the capital stock of a 555, change of name of a corporation is against public An authorized change in the name of a corporation
corporation, it is provided that 'such agreement policy. We must clarify that such is not the import of Our has no more effect upon its identity as a
may be amended in any other particular, excepting said decision. What this Court held in that case is simply corporation than a change of name of a natural
as provided in the following section', which relates that: person has upon his identity. It does not affect the
to a decrease of the capital stock This section has rights of the corporation or lessen or add to its
been held to authorize a change in the name of a We know of no law that empowers the Public obligations. After a corporation has effected a
corporation.  Armington v. Palmer, 21 R.I. 109, 42 Service Commission or any court in this jurisdiction change in its name it should sue and be sued in its
Atl. 308, 43, L.R.A. 95, 79 Am. St. Rep. 786.(Vol. to authorize one corporation to assume the name of new name ....(13 Am. Jur. 276-277, citing cases.)
19, American and English Annotated Cases, p. another corporation as a trade name. Both the
1239.) Rural Transit Company, Ltd., and the Bachrach A mere change in the name of a corporation, either
Motor Co., Inc., are Philippine corporations and the by the legislature or by the corporators or
Fletcher, a standard authority on American an corporation very law of their creation and continued existence stockholders under legislative authority, does not,
law also says: requires each to adopt and certify a distinctive generally speaking, affect the identity of the
name. The incorporators 'constitute a body politic corporation, nor in any way affect the rights,
Statutes are to be found in the various jurisdictions and corporate  under the name stated in the privileges, or obligations previously acquired or
dealing with the matter of change in corporate certificate.' (Section 11, Act No. 1459, as incurred by it. Indeed, it has been said that a
names. Such statutes have been subjected to amended.) A corporation has the power  'of change of name by a corporation has no more
judicial construction and have, in the main, been succession by its corporate name.'(Section effect upon the identity of the corporation than a
upheld as constitutional.  In direct terms or by 13,  ibid.) The name of a corporation is therefore change of name by a natural person has upon the
necessary implication, they authorize corporations essential to its existence. It cannot change its name identity of such person. The corporation, upon such
new names  and prescribe the mode of procedure except in the manner provided by the statute. By change in its name, is in no sense a new
for that purpose. The same steps must be taken that name alone is it authorized to transact corporation, nor the successor of the original one,
under some statutes to effect a change in a business. The law gives a corporation no express or but remains and continues to be the original
corporate name, as when any other amendment of implied authority to assume another name that is corporation. It is the same corporation with a
the corporate charter is sought .... When the unappropriated; still less that of another different name, and its character is in no respect
general law thus deals with the subject, a corporation, which is expressly set apart for it and changed. ... (6 Fletcher, Cyclopedia of the Law of
corporation can change its name only in the manner protected by the law. If any corporation could Private Corporations, 224-225, citing cases.)
provided. (6 Fletcher, Cyclopedia of the Law of assume at pleasure as an unregistered trade name
Private Corporations, 1968 Revised Volume, pp. the name of another corporation, this practice The change in the name of a corporation has no
212-213.) (Emphasis supplied) would result in confusion and open the door to more effect upon its identity as a corporation than
frauds and evasions and difficulties of a change of name of a natural person has upon his
The learned trial judge held that the above-quoted administration and supervision. The policy of the identity. It does not affect the rights of the
proposition are not supported by the weight of authority law as expressed our corporation statute and the corporation, or lessen or add to its obligations.
because they are based on decisions in cases where the Code of Commerce is clearly against such a
statutes expressly authorize change of corporate name by practice. (Cf. Scarsdale Pub.Co. — Colonial Press vs. England.  —  Doe v. Norton,  11 M. & W. 913, 7 Jur.
amendment of the articles of incorporation. We have Carter, 116 New York Supplement, 731; Svenska 751, 12 L. J. Exch. 418.

Page 34 of 160
CORPORATION LAW CASES MAYORDO, M.A
LLB 3

The fact that a corporation organized as a state in the original articles of incorporation." It goes without
United States. —  Metropolitan Nat. Bank v. bank afterwards becomes a national bank by saying then that appellant rightly acted in its old name
Claggett,  141 U.S. 520, 12 S. Ct. 60, 35 U.S. (L. complying with the provisions of the National when on May 15, 1961, it entered into the indemnity
ed.) 841. Banking Act, and changes its name accordingly, has agreement, Annex A, with the defendant-appellees; for
Alabama. —  Lomb v. Pioneer Sav., etc., Co.,  106 no effect on its right to sue upon obligations or only after the filing of the amended articles of
Ala. 591, 17 So. 670; North Birmingham Lumber Co. liabilities incurred to it by its former name. incorporation with the Securities & Exchange Commission
v. Sims, 157 Ala. 595, 48 So. 84. Michigan Ins. Bank v. Eldred 143 U.S. 293, 12 S. Ct. on May 26, 1961, did appellant legally acquire its new
450, 36 U.S. (L. ed.) 162. name; and it was perfectly right for it to file the present
Connecticut. — Trinity Church v. Hall, 22 Com. 125. case In that new name on December 6, 1961. Such is, but
Illinois. —  Mt. Palatine Academy v. Kleinschnitz  28 A deed of land to a church by a particular name has the logical effect of the change of name of the corporation
III, 133;  St. Louis etc. R. Co. v. Miller, 43 Ill. been held not to be affected by the fact that the upon its actions.
199; Reading v. Wedder, 66 III. 80. church afterwards took a different name. Cahill v.
Bigger, 8 B. Mon (ky) 211. Actions brought by a corporation after it has
Indiana. —  Rosenthal v. Madison etc., Plank Road changed its name should be brought under the new
Co., 10 Ind. 358. A change in the name of a corporation is not a name although for the enforcement of rights
divestiture of title or such a change as requires a existing at the time the change was made. Lomb v.
Kentucky. — Cahill v. Bigger, 8 B. Mon. 211; Wilhite regular transfer of title to property, whether real or Pioneer Sav., etc., Co.,  106 Ala. 591, 17 So.
v. Convent of Good Shepherd, 177 Ky. 251, 78 S. W. personal, from the corporation under one name to 670:  Newlan v. Lombard University, 62 III.
138. the same corporation under another 195;  Thomas v. Visitor of Frederick County School,
name.  McCloskey v. Doherty,  97 Ky. 300, 30 S. W. 7 Gill & J (Md.) 388; Delaware, etc., R. Co. v. Trick,
Maryland. —  Phinney v. Sheppard & Enoch Pratt 649. (19 American and English Annotated Cases 23 N. J. L. 321;  Northumberland Country Bank v.
Hospital, 88 Md. 633, 42 Atl. 58, writ of error 1242-1243.) Eyer, 60 Pa. St. 436; Wilson v. Chesapeake etc., R.
dismissed, 177 U.S. 170, 20 S. Ct. 573, 44 U.S. (L. Co., 21 Gratt (Va.) 654.
ed.) 720. As was very aptly said in  Pacific Bank v. De Ro  37 The change in the name of the corporation does not
Cal. 538, "The changing of the name of a affect its right to bring an action on a note given to
Missouri. — Dean v. La Motte Lead Co., 59 Mo. 523. corporation is no more the creation of a the corporation under its former name. Cumberland
corporation than the changing of the name of a College v. Ish, 22.Cal. 641; Northwestern College v.
Nebraska. — Carlon v. City Sav. Bank, 82 Neb. 582, natural person is the begetting of a natural person. Schwagler,  37 Ia. 577.(19 American and English
188 N. W. 334. New York First Soc of M.E. Church v. The act, in both cases, would seem to be what the Annotated Cases 1243.)
Brownell, 5 Hun 464. language which we use to designate it imports — a
change of name, and not a change of being. In consequence, We hold that the lower court erred in
Pennsylvania. — Com. v. Pittsburgh, 41 Pa. St. 278. dismissing appellant's complaint. We take this opportunity,
Having arrived at the above conclusion, We have agree with however, to express the Court's feeling that it is apparent
South Carolina. —  South Carolina Mut Ins. Co. v. appellant's pose that the lower court also erred in holding that appellee's position is more technical than otherwise.
Price 67 S.C. 207, 45 S.E. 173. that it is not the right party in interest to sue defendants- Nowhere in the record is it seriously pretended that the
appellees.4  As correctly pointed out by appellant, the indebtedness sued upon has already been paid. If appellees
Virginia. —  Wilson v. Chesapeake etc., R. Co., 21 approval by the stockholders of the amendment of its entertained any fear that they might again be made liable
Gratt 654; Wright-Caesar Tobacco Co. v. Hoen, 105 articles of incorporation changing the name "The Yek Tong to Yek Tong Lin Fire & Marine Insurance Co. Ltd., or to
Va. 327, 54 S.E. 309. Lin Fire & Marine Insurance Co., Ltd." to "Philippine First someone else in its behalf, a cursory examination of the
Insurance Co., Inc." on March 8, 1961, did not automatically records of the Securities & Exchange Commission would
Washington. — King v. Ilwaco R. etc., Co., 1 Wash. change the name of said corporation on that date. To be have sufficed to clear up the fact that Yek Tong Lin had just
127. 23 Pac. 924. effective, Section 18 of the Corporation Law, earlier changed its name but it had not ceased to be their creditor.
quoted, requires that "a copy of the articles of Everyone should realize that when the time of the courts is
Wisconsin. — Racine Country Bank v. Ayers, 12 Wis. incorporation as amended, duly certified to be correct by utilized for cases which do not involve substantial questions
512. the president and the secretary of the corporation and a and the claim of one of the parties, therein is based on
The fact that the corporation by its old name makes majority of the board of directors or trustees, shall be filed pure technicality that can at most delay only the ultimate
a format transfer of its property to the corporation with the Securities & Exchange Commissioner", and it outcome necessarily adverse to such party because it has
by its new name does not of itself show that the is  only from the time of such filing, that "the corporation no real cause on the merits, grave injustice is committed to
change in name has affected a change in the shall have the same powers and it and the members and numberless litigants whose meritorious cases cannot be
i d e n t i t y o f t h e c o r p o r a t i o n .  P a l f r e y v. stockholders thereof shall thereafter be subject to the given all the needed time by the courts. We address this
Association for Relief, etc., 110 La. 452, 34 So. 600. same liabilities as if such amendment had been embraced appeal once more to all members of the bar, in particular,

Page 35 of 160
CORPORATION LAW CASES MAYORDO, M.A
LLB 3

since it is their bounden duty to the profession and to our San Miguel contended that the amendments of the articles
country and people at large to help ease as fast as possible of incorporation of Zeta were for the purpose of changing The same respondent is ordered to pay the complainant
the clogged dockets of the courts. Let us not wait until the the corporate name, broadening the primary functions, and Ronaldo San Miguel attorney’s fees equivalent to ten
people resort to other means to secure speedy, just and increasing the capital stock; and that such amendments percent (10%) of the total award.
inexpensive determination of their cases. could not mean that Zeta had been thereby dissolved.3
All other claims are dismissed.
WHEREFORE, judgment of the lower court is reversed, and On its part, petitioner countered that San Miguel’s SO ORDERED.7
this case is remanded to the trial court for further termination from Zeta had been for a cause authorized by
proceedings consistent herewith With costs against the Labor Code; that its non-acceptance of him had not Decision of the NLRC
appellees. been by any means irregular or discriminatory; that its
predecessor-in-interest had complied with the Petitioner appealed, but the NLRC issued a resolution on
G.R. No. 157900               July 22, 2013 requirements for termination due to the cessation of April 4, 2001,8 affirming the decision of the Labor Arbiter.
ZUELLIG FREIGHT AND CARGO SYSTEMS, Petitioner, 
 business operations; that it had no obligation to employ San
vs. NATIONAL LABOR RELATIONS COMMISSION AND Miguel in the exercise of its valid management prerogative; The NLRC later on denied petitioner’s motion for
RONALDO V. SAN MIGUEL, Respondents. that all employees had been given sufficient time to make reconsideration via its resolution dated June 15, 2001.9
their decision whether to accept its offer of employment or
DECISION not, but he had not responded to its offer within the time Decision of the CA
BERSAMIN, J.: set; that because of his failure to meet the deadline, the
offer had expired; that he had nonetheless been hired on a Petitioner then filed a petition for certiorari in the CA,
The mere change in the corporate name is not considered temporary basis; and that when it decided to hire another imputing to the NLRC grave abuse of discretion amounting
under the law as the creation of a new corporation; hence, employee instead of San Miguel, such decision was not to lack or excess of jurisdiction, as follows:
the renamed corporation remains liable for the illegal arbitrary because of seniority considerations.4
dismissal of its employee separated under that guise. 1. In failing to consider the circumstances
Decision of the Labor Arbiter attendant to the cessation of business of Zeta;
The Case 2. In failing to consider that San Miguel failed to
On November 15, 1999, Labor Arbiter Francisco A. Robles meet the deadline Zeta fixed for its employees to
Petitioner employer appeals the decision promulgated on rendered a decision holding that San Miguel had been accept the offer of petitioner for re-employment;
November 6, 2001,1  whereby the Court of Appeals (CA) illegally dismissed,5 to wit: 3. In failing to consider that San Miguel’s
dismissed its petition for certiorari and upheld the adverse employment with petitioner from April 1 to 15,
decision of the National Labor Relations Commission (NLRC) Contrary to respondents’ claim that Zeta ceased operations 1994 could in no way be interpreted as a
finding respondent Ronaldo V. San Miguel to have been and closed its business, we believe that there was merely a continuation of employment with Zeta;
illegally dismissed. change of business name and primary purpose and 4. In admitting in evidence the letter dated
upgrading of stocks of the corporation. Zuellig and Zeta are January 21, 1994 of petitioner’s counsel to the
Antecedents therefore legally the same person and entity and this was Bureau of Internal Revenue; and
admitted by Zuellig’s counsel in its letter to the VAT 5. In awarding attorney’s fees to San Miguel
San Miguel brought a complaint for unfair labor practice, Department of the Bureau of Internal Revenue on 08 June based on Article 2208 of the Civil Code and
illegal dismissal, non-payment of salaries and moral 1994 (Reply, Annex "A"). As such, the termination of Article 111 of the Labor Code.
damages against petitioner, formerly known as Zeta complainant’s services allegedly due to cessation of
Brokerage Corporation (Zeta).2 He alleged that he had been business operations of Zeta is deemed illegal. On November 6, 2002, the CA promulgated its assailed
a checker/customs representative of Zeta since December Notwithstanding his receipt of separation benefits from decision dismissing the petition for certiorari,10 viz:
16, 1985; that in January 1994, he and other employees of respondents, complainant is not estopped from questioning
Zeta were informed that Zeta would cease operations, and the legality of his dismissal.6 A careful perusal of the records shows that the closure of
that all affected employees, including him, would be business operation was not validly made. Consider the
separated; that by letter dated February 28, 1994, Zeta xxxx Certificate of Filing of the Amended Articles of
informed him of his termination effective March 31, 1994; WHEREFORE, in view of the foregoing, complainant is found Incorporation which clearly shows that petitioner Zuellig is
that he reluctantly accepted his separation pay subject to to have been illegally dismissed. Respondent Zuellig Freight actually the former Zeta as per amendment dated January
the standing offer to be hired to his former position by and Cargo Systems, Inc. is hereby ordered to pay 21, 1994. The same observation can be deduced with
petitioner; and that on April 15, 1994, he was summarily complainant his backwages from April 1, 1994 up to respect to the Certificate of Filing of Amended By-Laws
terminated, without any valid cause and due process. November 15, 1999, in the amount of THREE HUNDRED dated May 10, 1994. As aptly pointed out by private
TWENTY FOUR THOUSAND SIX HUNDRED FIFTEEN PESOS respondent San Miguel, the amendment of the articles of
(₱324,615.00). incorporation merely changed its corporate name,

Page 36 of 160
CORPORATION LAW CASES MAYORDO, M.A
LLB 3

broadened its primary purpose and increased its authorized the Civil Code and Article 111 of the Labor Code as private
capital stocks. The requirements contemplated in Article respondent’s wages were not withheld. On the other hand, In its reply,13  petitioner reiterates that the cessation of
283 were not satisfied in this case. Good faith was not public respondent NLRC argues that paragraphs 2 and 3, Zeta’s business, which resulted in the severance of San
established by mere registration with the Securities and Article 2208 of the Civil Code and paragraph (a), Article 111 Miguel from his employment, was valid; that the CA erred
Exchange Commission (SEC) of the Amended Articles of of the Labor Code justify the award of attorney’s fees. in upholding the NLRC’s finding that San Miguel had been
Incorporation and ByLaws. The factual milleu of the case, NLRC was saying to the effect that by petitioner Zuellig’s illegally terminated; that his acknowledgment of the
considered in its totality, shows that there was no closure act of illegally dismissing private respondent San Miguel, validity of his separation from Zeta by signing a quitclaim
to speak of. The termination of services allegedly due to the latter was compelled to litigate and thus incurred and waiver estopped him from claiming that it had
cessation of business operations of Zeta was illegal. expenses to protect his interest. In the same passion, subsequently employed him; and that the award of
Notwithstanding private respondent San Miguel’s receipt of private respondent San Miguel contends that petitioner attorney’s fees had no basis in fact and in law.
separation benefits from petitioner Zuellig, the former is Zuellig acted in gross and evident bad faith in refusing to
not estopped from questioning the legality of his dismissal. satisfy his plainly valid, just and demandable claim. Ruling

Petitioner Zuellig’s allegation that the five employees who After careful and judicious evaluation of the arguments The petition for review on certiorari is denied for its lack of
refused to receive the termination letters were verbally advanced to support the propriety or impropriety of the merit.
informed that they had until 6:00 p.m. of March 1, 1994 to award of attorney’s fees to private respondent San Miguel,
receive the termination letters and sign the employment this Court finds the resolutions of public respondent NLRC First of all, the outcome reached by the CA that the NLRC
contracts, otherwise the former would be constrained to supported by laws and jurisprudence. It does not need did not commit any grave abuse of discretion was borne out
withdraw its offer of employment and seek for much imagination to see that by reason of petitioner by the records of the case. We cannot undo such finding
replacements in order to ensure the smooth operations of Zuellig’s feigned closure of business operations, private without petitioner making a clear demonstration to the
the new company from its opening date, is of no moment in respondent San Miguel incurred expenses to protect his Court now that the CA gravely erred in passing upon the
view of the foregoing circumstances. There being no valid rights and interests. Therefore, the award of attorney’s petition for certiorari of petitioner.
closure of business operations, the dismissal of private fees is in order.
respondent San Miguel on alleged authorized cause of Indeed, in a special civil action for certiorari brought
cessation of business pursuant to Article 283 of the Labor WHEREFORE, in view of the foregoing, the resolutions against a court or quasi-judicial body with jurisdiction over
Code, was utterly illegal. Despite verbal notice that the dated April 4, 2001 and June 15, 2001 of the National Labor a case, petitioner carries the burden of proving that the
employees had until 6:00 p.m. of March 1, 1994 to receive Relations Commission affirming the November 15, 1999 court or quasi-judicial body committed not a merely
the termination letters and sign the employment contracts, decision of the Labor Arbiter in NLRC NCR 05-03639-94 (CA reversible error but a grave abuse of discretion amounting
the dismissal was still illegal for the said condition is null No. 022861-00) are hereby AFFIRMED and the instant to lack or excess of jurisdiction in issuing the impugned
and void. In point of facts and law, private respondent San petition for certiorari is hereby DENIED and ordered order.14Showing mere abuse of discretion is not enough, for
Miguel remained an employee of petitioner Zuellig. If at DISMISSED. it is necessary to demonstrate that the abuse of discretion
all, the alleged closure of business operations merely SO ORDERED. was grave. Grave abuse of discretion means either that the
operates to suspend employment relation since it is not judicial or quasi-judicial power was exercised in an
permanent in character. Hence, petitioner appeals. arbitrary or despotic manner by reason of passion or
personal hostility, or that the respondent judge, tribunal or
Where there is no showing of a clear, valid, and legal cause Issues board evaded a positive duty, or virtually refused to
for the termination of employment, the law considers the perform the duty enjoined or to act in contemplation of
matter a case of illegal dismissal and the burden is on the Petitioner asserts that the CA erred in holding that the law, such as when such judge, tribunal or board exercising
employer to prove that the termination was for a valid or NLRC did not act with grave abuse of discretion in ruling judicial or quasi-judicial powers acted in a capricious or
authorized cause. that the closure of the business operation of Zeta had not whimsical manner as to be equivalent to lack of
been bona fide, thereby resulting in the illegal dismissal of jurisdiction.15  Under the circumstances, the CA committed
Findings of facts of the NLRC, particularly when both the San Miguel; and in holding that the NLRC did not act with no abuse of discretion, least of all grave, because its
NLRC and Labor Arbiter are in agreement, are deemed grave abuse of discretion in ordering it to pay San Miguel justifications were supported by the records and by the
binding and conclusive upon the Supreme Court. attorney’s fees.11 applicable laws and jurisprudence.

As regards the second and last argument advanced by In his comment,12 San Miguel counters that the CA correctly Secondly, it is worthy to point out that the Labor Arbiter,
petitioner Zuellig that private respondent San Miguel is not found no grave abuse of discretion on the part of the NLRC the NLRC, and the CA were united in concluding that the
entitled to attorney’s fees, this Court finds no reason to because the ample evidence on record showed that he had cessation of business by Zeta was not a bona fide closure to
disturb the ruling of the public respondent NLRC. Petitioner been illegally terminated; that such finding accorded with be regarded as a valid ground for the termination of
Zuellig maintains that the factual backdraft (sic) of this applicable laws and jurisprudence; and that he was entitled employment of San Miguel within the ambit of Article 283
petition does not call for the application of Article 2208 of to back wages and attorney’s fees. of the Labor Code. The provision pertinently reads:

Page 37 of 160
CORPORATION LAW CASES MAYORDO, M.A
LLB 3

different name, and its character is in no respect changed. Balabagan of the same province. Balabagan was formerly a
Article 283.Closure of establishment and reduction of (Bold underscoring supplied for emphasis) part of the municipality of Malabang, having been created
personnel. — The employer may also terminate the on March 15, 1960, by Executive Order 386 of the then
employment of any employee due to the installation of In short, Zeta and petitioner remained one and the same President Carlos P. Garcia, out of barrios and sitios1  of the
labor-saving devices, redundancy, retrenchment to prevent corporation. The change of name did not give petitioner latter municipality.
losses or the closing or cessation of operation of the the license to terminate employees of Zeta like San Miguel
establishment or undertaking unless the closing is for the without just or authorized cause. The situation was not The petitioners brought this action for prohibition to nullify
purpose of circumventing the provisions of this Title, by similar to that of an enterprise buying the business of Executive Order 386 and to restrain the respondent
serving a written notice on the workers and the another company where the purchasing company had no municipal officials from performing the functions of their
Department of Labor and Employment at least one (1) obligation to rehire terminated employees of the latter. respective office relying on the ruling of this Court
month before the intended date thereof. x x x. 18  Petitioner, despite its new name, was the mere in  Pelaez v. Auditor General  2  and  Municipality of San
continuation of Zeta's corporate being, and still held the Joaquin v. Siva. 3
The unanimous conclusions of the CA, the NLRC and the obligation to honor all of Zeta's obligations, one of which
Labor Arbiter, being in accord with law, were not tainted was to respect San Miguel's security of tenure. The In Pelaez this Court, through Mr. Justice (now Chief Justice)
with any abuse of discretion, least of all grave, on the part dismissal of San Miguel from employment on the pretext Concepcion, ruled: (1) that section 23 of Republic Act 2370
of the NLRC. Verily, the amendments of the articles of that petitioner, being a different corporation, had no [Barrio Charter Act, approved January 1, 1960], by vesting
incorporation of Zeta to change the corporate name to obligation to accept him as its employee, was illegal and the power to create  barrios  in the provincial board, is a
Zuellig Freight and Cargo Systems, Inc. did not produce the ineffectual. "statutory denial of the presidential authority to create a
dissolution of the former as a corporation. For sure, the new barrio [and] implies a negation of the bigger power to
Corporation Code defined and delineated the different And, lastly, the CA rightfully upheld the NLRC's affirmance create municipalities," and (2) that section 68 of the
modes of dissolving a corporation, and amendment of the of the grant of attorney's fees to San Miguel. Thereby, the Administrative Code, insofar as it gives the President the
articles of incorporation was not one of such modes. The NLRC did not commit any grave abuse of its discretion, power to create municipalities, is unconstitutional (a)
effect of the change of name was not a change of the considering that San Miguel had been compelled to litigate because it constitutes an undue delegation of legislative
corporate being, for, as well stated in Philippine First and to incur expenses to protect his rights and interest. In power and (b) because it offends against section 10 (1) of
Insurance Co., Inc. v. Hartigan:16 "The changing of the name Producers Bank of the Philippines v. Court of Appeals,19the article VII of the Constitution, which limits the President's
of a corporation is no more the creation of a corporation Court ruled that attorney's fees could be awarded to a power over local governments to mere supervision. As this
than the changing of the name of a natural person is party whom an unjustified act of the other party compelled Court summed up its discussion: "In short, even if it did not
begetting of a natural person. The act, in both cases, would to litigate or to incur expenses to protect his interest. It entail an undue delegation of legislative powers, as it
seem to be what the language which we use to designate it was plain that petitioner's refusal to reinstate San Miguel certainly does, said section 68, as part of the Revised
imports – a change of name, and not a change of being." with backwages and other benefits to which he had been Administrative Code, approved on March 10, 1917, must be
legally entitled was unjustified, thereby entitling him to deemed repealed by the subsequent adoption of the
The consequences, legal and otherwise, of the change of recover attorney's fees. Constitution, in 1935, which is utterly incompatible and
name were similarly dealt with in P.C. Javier & Sons, Inc. v. inconsistent with said statutory enactment."
Court of Appeals,17 with the Court holding thusly: WHEREFORE, the Court AFFIRMS the decision of the Court
of Appeals promulgated on November 6, 2002; and ORDERS On the other hand, the respondents, while admitting the
From the foregoing documents, it cannot be denied that petitioner to pay the costs of suit. facts alleged in the petition, nevertheless argue that the
petitioner corporation was aware of First Summa Savings SO ORDERED. rule announced in  Pelaez  can have no application in this
and Mortgage Bank’s change of corporate name to PAIC case because unlike the municipalities involved in  Pelaez,
Savings and Mortgage Bank, Inc. Knowing fully well of such G.R. No. L-28113               March 28, 1969 the municipality of Balabagan is at least a  de
change, petitioner corporation has no valid reason not to THE MUNICIPALITY OF MALABANG, LANAO DEL SUR, and facto  corporation, having been organized under color of a
pay because the IGLF loans were applied with and obtained AMER MACAORAO BALINDONG, petitioners, 
 statute before this was declared unconstitutional, its
from First Summa Savings and Mortgage Bank. First Summa vs. PANGANDAPUN BENITO, HADJI NOPODIN officers having been either elected or appointed, and the
Savings and Mortgage Bank and PAIC Savings and Mortgage MACAPUNUNG, HADJI HASAN MACARAMPAD, FREDERICK municipality itself having discharged its corporate functions
Bank, Inc., are one and the same bank to which petitioner V. DUJERTE MONDACO ONTAL, MARONSONG ANDOY, for the past five years preceding the institution of this
corporation is indebted. A change in the corporate name MACALABA INDAR LAO. respondents. action. It is contended that as a  de facto  corporation, its
does not make a new corporation, whether effected by a existence cannot be collaterally attacked, although it may
special act or under a general law. It has no effect on the CASTRO, J.: be inquired into directly in an action for  quo warranto  at
identity of the corporation, or on its property, rights, or The petitioner Amer Macaorao Balindong is the mayor of the instance of the State and not of an individual like the
liabilities. The corporation, upon to change in its name, is Malabang, Lanao del Sur, while the respondent petitioner Balindong.
in no sense a new corporation, nor the successor of the Pangandapun Bonito is the mayor, and the rest of the
original corporation. It is the same corporation with a respondents are the councilors, of the municipality of

Page 38 of 160
CORPORATION LAW CASES MAYORDO, M.A
LLB 3

It is indeed true that, generally, an inquiry into the legal is some other valid law under which the factocorporation, as, independently of the Administrative
existence of a municipality is reserved to the State in a organization may be effected, or at least an Code provision in question, there is no other valid statute
proceeding for  quo warranto  or other direct proceeding, authority in potentia by the state constitution, has to give color of authority to its creation. Indeed,
and that only in a few exceptions may a private person its counterpart in the negative propositions that in Municipality of San Joaquin v. Siva,  11  this Court granted
exercise this function of government.  4  But the rule there can be no color of authority in an a similar petition for prohibition and nullified an executive
disallowing collateral attacks applies only where the unconstitutional statute that plainly so appears on order creating the municipality of Lawigan in Iloilo on the
m u n i c i p a l c o r p o r a t i o n i s a t l e a s t a  d e its face or that attempts to authorize the ousting of basis of thePelaez  ruling, despite the fact that the
facto  corporations.  5  For where it is neither a a  de jure  or  de facto  municipal corporation upon municipality was created in 1961, before section 68 of the
corporation  de jure  nor  de facto, but a nullity, the rule is the same territory; in the one case the fact would Administrative Code, under which the President had acted,
that its existence may be, questioned collaterally or imply the imputation of bad faith, in the other the was invalidated. 'Of course the issue of de facto municipal
directly in any action or proceeding by any one whose new organization must be regarded as a mere corporation did not arise in that case.
rights or interests ate affected thereby, including the usurper....
citizens of the territory incorporated unless they are In  Norton v. Shelby Count,  12  Mr. Justice Field said: "An
estopped by their conduct from doing so. 6 As a result of this analysis of the cases the following unconstitutional act is not a law; it confers no rights; it
principles may be deduced which seem to reconcile imposes no duties; it affords no protection;  it creates no
And so the threshold question is whether the municipality the apparently conflicting decisions: office; it is, in legal contemplation, as inoperative as
of Balabagan is a  de facto  corporation. As earlier stated, though it had never been passed." Accordingly, he held that
the claim that it is rests on the fact that it was organized I. The color of authority requisite to the bonds issued by a board of commissioners created under an
before the promulgation of this Court's decision in Pelaez. 7 o r g a n i z a t i o n o f a  d e f a c t o  m u n i c i p a l invalid statute were unenforceable.
corporation may be:
Accordingly, we address ourselves to the question whether 1. A valid law enacted by the legislature. Executive Order 386 "created no office." This is not to say,
a statute can lend color of validity to an attempted 2. An unconstitutional law, valid on its face, however, that the acts done by the municipality of
organization of a municipality despite the fact that such which has either (a) been upheld for a time by Balabagan in the exercise of its corporate powers are a
statute is subsequently declared unconstitutional. the courts or (b) not yet been declared nullity because the executive order "is, in legal
void;  provided  that a warrant for its creation contemplation, as inoperative as though it had never been
This has been a litigiously prolific question, sharply dividing can be found in some other valid law or in the passed." For the existence of Executive, Order 386 is "an
courts in the United States. Thus, some hold that a  de recognition of its potential existence by the operative fact which cannot justly be ignored." As Chief
facto corporation cannot exist where the statute or charter general laws or constitution of the state. Justice Hughes explained in Chicot County Drainage District
creating it is unconstitutional because there can be no  de v. Baxter State Bank: 13
f a c t o  c o r p o r a t i o n w h e r e t h e r e c a n b e n o  d e II. There can be no  de facto  municipal The courts below have proceeded on the theory
jure one, 8 while others hold otherwise on the theory that a corporation unless either directly or potentially, that the Act of Congress, having been found to be
statute is binding until it is condemned as such a de jurecorporation is authorized by some unconstitutional, was not a law; that it was
unconstitutional. 9 legislative fiat. inoperative, conferring no rights and imposing no
duties, and hence affording no basis for the
An early article in the Yale Law Journal offers the following III. There can be no color of authority in an challenged decree. Norton v. Shelby County, 118
analysis: unconstitutional statute alone, the invalidity of U.S. 425, 442; Chicago, I. & L. Ry. Co. v. Hackett,
which is apparent on its face. 228 U.S. 559, 566. It is quite clear, however, that
It appears that the true basis for denying to the   such broad statements as to the effect of a
corporation a de facto status lay in the absence of IV. There can be no  de facto  corporation determination of unconstitutionality must be taken
any legislative act to give vitality to its creation. An created to take the place of an existing  de with qualifications. The actual existence of a
examination of the cases holding, some of them jure  corporation, as such organization would statute, prior to such a determination, is an
unreservedly, that a  de facto  office or municipal clearly be a usurper.10 operative fact and may have consequences which
corporation can exist under color of an cannot justly be ignored. The past cannot always be
unconstitutional statute will reveal that in no In the cases where a  de facto  municipal corporation was erased by a new judicial declaration. The effect of
instance did the invalid act give life to the recognized as such despite the fact that the statute the subsequent ruling as to invalidity may have to
corporation, but that either in other valid acts or in creating it was later invalidated, the decisions could fairly be considered in various aspects — with respect to
the constitution itself the office or the corporation be made to rest on the consideration that there was some particular relations, individual and corporate, and
was potentially created.... other valid law giving corporate vitality to the organization. particular conduct, private and official. Questions
Hence, in the case at bar, the mere fact that Balabagan of rights claimed to have become vested, of status
The principle that color of title under an was organized at a time when the statute had not been of prior determinations deemed to have finality and
unconstitutional statute can exist only where there i n v a l i d a t e d c a n n o t c o n c e i v a b l y m a k e i t a  d e acted upon accordingly, of public policy in the light

Page 39 of 160
CORPORATION LAW CASES MAYORDO, M.A
LLB 3

of the nature both of the statute and of its previous (3) On December 2, 1947, the said articles of incorporation Law). The complaining associates have not represented to
application, demand examination. These questions were filed in the office of the Securities and Exchange the others that they were incorporated any more than the
are among the most difficult of those which have Commissioner, for the issuance of the corresponding latter had made similar representations to them. And as
engaged the attention of courts, state and federal, certificate of incorporation. nobody was led to believe anything to his prejudice and
and it is manifest from numerous decisions that an damage, the principle of estoppel does not apply. Obviously
all-inclusive statement of a principle of absolute (4) On March 22, 1948, pending action on the articles of this is not an instance requiring the enforcement of
retroactive invalidity cannot be justified. incorporation by the aforesaid governmental office, the contracts  with the corporation  through the rule of
respondents Fred Brown, Emma Brown, Hipolita D. estoppel.
There is then no basis for the respondents' apprehension Chapman and Ceferino S. Abella filed before the Court of
that the invalidation of the executive order creating First Instance of Leyte the civil case numbered 381, The first proposition above stated is premised on the theory
Balabagan would have the effect of unsettling many an act entitled "Fred Brown et al.  vs. Arnold C. Hall et al.", that, inasmuch as the Far Eastern Lumber and Commercial
done in reliance upon the validity of the creation of that alleging among other things that the Far Eastern Lumber Co., is a de facto corporation, section 19 of the Corporation
municipality. 14 and Commercial Co. was an unregistered partnership; that Law applies, and therefore the court had not jurisdiction to
they wished to have it dissolved because of bitter take cognizance of said civil case number 381. Section 19
ACCORDINGLY, the petition is granted, Executive Order 386 dissension among the members, mismanagement and fraud reads as follows:
is declared void, and the respondents are hereby by the managers and heavy financial losses.
permanently restrained from performing the duties and . . . The due incorporation of any corporations
functions of their respective offices. No pronouncement as (5) The defendants in the suit, namely, C. Arnold Hall and claiming in good faith to be a corporation under
to costs. Bradley P. Hall, filed a motion to dismiss, contesting the this Act and its right to exercise corporate powers
court's jurisdiction and the sufficiently of the cause of shall not be inquired into collaterally in any private
G.R. No. L-2598             June 29, 1950 action. suit to which the corporation may be a party, but
C. ARNOLD HALL and BRADLEY P. HALL, petitioners, 
 such inquiry may be had at the suit of the Insular
vs. EDMUNDO S. PICCIO, Judge of the Court of First (6) After hearing the parties, the Hon. Edmund S. Piccio Government on information of the Attorney-
Instance of Leyte, FRED BROWN, EMMA BROWN, HIPOLITA ordered the dissolution of the company; and at the request General.
CAPUCIONG, in his capacity as receiver of the Far Eastern of plaintiffs, appointed of the properties thereof, upon the
Lumber and Commercial Co., Inc.,respondents. filing of a P20,000 bond. There are least two reasons why this section does not
govern the situation. Not having obtained the certificate of
BENGZON, J.: (7) The defendants therein (petitioners herein) offered to incorporation, the Far Eastern Lumber and Commercial Co.
This is petition to set aside all the proceedings had in civil file a counter-bond for the discharge of the receiver, but — even its stockholders — may not probably claim "in good
case No. 381 of the Court of First Instance of Leyte and to the respondent judge refused to accept the offer and to faith" to be a corporation.
enjoin the respondent judge from further acting upon the discharge the receiver. Whereupon, the present special civil
same. action was instituted in this court. It is based upon two Under our statue it is to be noted (Corporation Law,
main propositions, to wit: sec. 11) that it is the issuance of a certificate of
Facts: (1) on May 28, 1947, the petitioners C. Arnold Hall incorporation by the Director of the Bureau of
and Bradley P. Hall, and the respondents Fred Brown, Emma (a) The court had no jurisdiction in civil case No. 381 to Commerce and Industry which calls a corporation
Brown, Hipolita D. Chapman and Ceferino S. Abella, signed decree the dissolution of the company, because it being into being. The immunity if collateral attack is
and acknowledged in Leyte, the article of incorporation of a  de facto  corporation, dissolution thereof may only be granted to corporations "claiming in good faith to
the Far Eastern Lumber and Commercial Co., Inc., ordered in a  quo warranto  proceeding instituted in be a corporation under this act." Such a claim is
organized to engage in a general lumber business to carry accordance with section 19 of the Corporation Law. compatible with the existence of errors and
on as general contractors, operators and managers, etc. irregularities; but not with a total or substantial
Attached to the article was an affidavit of the treasurer (b) Inasmuch as respondents Fred Brown and Emma Brown disregard of the law. Unless there has been an
stating that 23,428 shares of stock had been subscribed and had signed the article of incorporation but only a evident attempt to comply with the law the claim
fully paid with certain properties transferred to the partnership. to be a corporation "under this act" could not be
corporation described in a list appended thereto. made "in good faith." (Fisher on the Philippine Law
Discussion: The second proposition may at once be o f S t o c k C o r p o r a t i o n s , p . 7 5 .  S e e
(2) Immediately after the execution of said articles of dismissed. All the parties are informed that the Securities also Humphreys vs. Drew, 59 Fla., 295; 52 So., 362.)
incorporation, the corporation proceeded to do business and Exchange Commission has not, so far, issued the
with the adoption of by-laws and the election of its corresponding certificate of incorporation. All of them Second, this is not a suit in which  the corporation  is a
officers. know, or sought to know, that the personality of a party. This is a litigation between stockholders of the
corporation begins to exist only from the moment such alleged corporation, for the purpose of obtaining its
certificate is issued — not before (sec. 11, Corporation dissolution. Even the existence of a  de jure  corporation

Page 40 of 160
CORPORATION LAW CASES MAYORDO, M.A
LLB 3

may be terminated in a private suit for its dissolution The court below rendered judgment in favor of the plaintiff HORNEY, J., delivered the opinion of the Court.
between stockholders, without the intervention of the for the sum demanded in the complaint, with interest on
state. the sum of P24,147.34 from November 1, 1923, at the rate On the theory that the Real Estate Service Bureau was
of 10 per cent per annum, and the costs. From this neither a de jure nor a de facto corporation and that Albion
There might be room for argument on the right of minority judgment the defendant appeals to this court. C. Cranson, Jr., was a partner in the business conducted by
stockholders to sue for dissolution;1 but that question does the Bureau and as such was personally liable for its debts,
not affect the court's jurisdiction, and is a matter for At the trial of the case the plaintiff failed to prove the International Business Machines Corporation brought
decision by the judge, subject to review on appeal. Whkch affirmatively the corporate existence of the parties and the this action against Cranson for the balance due on electric
brings us to one principal reason why this petition may not appellant insists that under these circumstances the court typewriters purchased by the Bureau. At the same time it
prosper, namely: the petitioners have their remedy by erred in finding that the parties were corporations with moved for summary judgment and supported the motion by
appealing the order of dissolution at the proper time. juridical personality and assigns same as reversible error. affidavit. In due course, Cranson filed a general issue plea
and an affidavit in opposition to summary judgment in
There is a secondary issue in connection with the There is no merit whatever in the appellant's contention. which he asserted in effect that the Bureau was a  de
appointment of a receiver. But it must be admitted that The general rule is that in the absence of fraud a person facto corporation and that he was not personally liable for
receivership is proper in proceedings for dissolution of a who has contracted or otherwise dealt with an association its debts.
company or corporation, and it was no error to reject the in such a way as to recognize and in effect admit its legal
counter-bond, the court having declared the dissolution. As existence as a corporate body is thereby estopped to deny The agreed statement of facts shows that in April 1961,
to the amount of the bond to be demanded of the receiver, its corporate existence in any action leading out of or Cranson was asked to invest in a new business corporation
much depends upon the discretion of the trial court, which involving such contract or dealing, unless its existence is which was about to be created. Towards this purpose he
in this instance we do not believe has been clearly abused. attacked for cause which have arisen since making the met with other interested individuals and an attorney and
contract or other dealing relied on as an estoppel and this agreed to purchase stock and become an officer and
Judgment: The petition will, therefore, be dismissed, with applies to foreign as well as to domestic corporations. (14 director. Thereafter, upon being advised by the attorney
costs. The preliminary injunction heretofore issued will be C. J., 227; Chinese Chamber of Commerce vs. Pua Te Ching, that the corporation had been formed under the laws of
dissolved. 14 Phil., 222.) Maryland, he paid for and received a stock certificate
Ozaeta, Pablo, Tuason, Montemayor, and Reyes, evidencing ownership of shares in the corporation, and was
JJ., concur. The defendant having recognized the corporate existence shown the corporate seal and minute book. The business of
of the plaintiff by making a promissory note in its favor and the new venture was conducted as if it were a corporation,
G.R. No. 22106           September 11, 1924 making partial payments on the same is therefore estopped through corporate bank accounts, with auditors maintaining
to deny said plaintiff's corporate existence. It is, of course, corporate books and records, and under a
ASIA BANKING CORPORATION, plaintiff-appellee, 
 also estopped from denying its own corporate existence. lease  *480  entered into by the corporation for the office
vs. STANDARD PRODUCTS, CO., INC., defendant-appellant. Under these circumstances it was unnecessary for the from which it operated its business. Cranson was elected
plaintiff to present other evidence of the corporate president and all transactions conducted by him for the
OSTRAND, J.: existence of either of the parties. It may be noted that corporation, including the dealings with I.B.M., were made
This action is brought to recover the sum of P24,736.47, there is no evidence showing circumstances taking the case as an officer of the corporation. At no time did he assume
the balance due on the following promissory note: out of the rules stated. any personal obligation or pledge his individual credit to
P37,757.22 I.B.M. Due to an oversight on the part of the attorney, of
The judgment appealed from is affirmed, with the costs which Cranson was not aware, the certificate of
MANILA, P. I.,     Nov. 28, 1921. against the appellant. So ordered. incorporation, which had been signed and acknowledged
MANILA, P. I., Nov. 28, 1921. prior to May 1, 1961, was not filed until November 24,
On demand, after date we promise to pay to the CRANSON v. INTERNATIONAL BUSINESS MACHINES 1961. Between May 17 and November 8, the Bureau
Asia Banking Corporation, or order, the sum of CORPORATION purchased eight typewriters from I.B.M., on account of
thirty-seven thousand seven hundred fifty-seven which partial payments were made, leaving a balance due
and 22/100 pesos at their office in Manila, for [No. 245, September Term, 1963.] of $4,333.40, for which this suit was brought.
value received, together with interest at the rate Court of Appeals of Maryland.
of ten per cent per annum. Decided April 30, 1964. Although a question is raised as to the propriety of making
No. ________ Due __________ *479 The cause was argued before HENDERSON, HAMMOND, use of a motion for summary judgment as the means of
HORNEY, MARBURY and SYBERT, JJ. determining the issues presented by the pleadings, we
THE STANDARD PRODUCTS CO., INC.
 think the motion was appropriate. Since there was no
William J. Brannan, Jr., with whom were Kardy, Brannan &
        By     (Sgd.) GEORGE H. SEAVER genuine dispute as to the material facts, the only question
Neumann on the brief, for the appellant.
Henry J. Noyes for the appellee. was whether I.B.M. was entitled to judgment as a matter of
                By     President
law. The trial court found that it was, but we disagree.

Page 41 of 160
CORPORATION LAW CASES MAYORDO, M.A
LLB 3

the proposition that substantial compliance with those corporation had no legal existence at the time of the
The fundamental question presented by the appeal is formalities of the corporation law, which are made a alleged libel. In referring to the Boycecase, it was said (at
whether an officer [1]  of a defectively incorporated condition precedent to corporate existence, was not only p. 320) that "it has been held by our predecessors that a
association may be subjected to personal liability under the necessary for the creation of a corporation de jure, but was corporation cannot be actually or virtually created by
circumstances of this case. We think not. a l s o a p r e r e q u i s i t e t o t h e e x i s t e n c e o f a  d e estoppel in Maryland." And, on the basis of the statements
facto corporation or a corporation by estoppel. in  Jones v. Aspen Hardware Co., supra  (also relied on in
Traditionally, two doctrines have been used by the courts to the  Maryland Tube  case), it was concluded that the
clothe an officer of a defectively incorporated association In the  Boyce  case, an action in assumpsit against a corporation could not maintain the action.
with the corporate attribute of limited liability. The first, defectively incorporated religious society, the Court (at p.
often referred to as the doctrine of de facto corporations, 373 and p. 374), in holding that the society was not On the other hand, where the corporation has obtained
has been applied in those cases where there are elements estopped to deny its corporate existence, said: legal existence but has failed to comply with a condition
showing: (1) the existence of law authorizing incorporation: subsequent to corporate existence, this Court has held that
(2) an effort in good faith to incorporate under the existing "We think it would be extending the doctrine of estoppel to such nonperformance afforded the State the right to
law; and (3) actual user or exercise of corporate powers. an extent, not justified by the principles of public policy, to institute proceedings for the forfeiture of the charter, but
B a l l a n t i n e ,  P r i v a t e C o r p o r a t i o n s ,  § 2 3 ; 8 allow it to operate through the conduct of the parties that such neglect or omission could never be set up by the
F l e t c h e r ,  C y c l o p e d i a o f t h e L a w o f concerned, to create substantially a  de facto  corporation, corporation itself, or by its members and stockholders, as a
P r i v a t e  * 4 8 1  C o r p o r a t i o n s ,  § 3 7 7 7 ; 1 3 A m . with just such powers as the parties may by their acts give defense to an action to enforce their liabilities.  C. & O.
Jur., Corporations, §§ 49-56; 18 C.J.S., Corporations, § 99. to it. Canal Co. v. B. & O. Railroad Co.,  4 G. & J. 1
The second, the doctrine of estoppel to deny the corporate *** (1832);  Hammond v. Straus,  53 Md. 1  (1880);  Murphy v.
existence, is generally employed where the person seeking "The statute law of the State, expressly requiring certain Wheatley, 102 Md. 501, 63 A. 62 (1906).
to hold the officer personally liable has contracted or prescribed acts to be done to constitute a corporation, to
otherwise dealt with the association in such a manner as to permit parties indirectly, or upon the principle of estoppel, *484 In the Hammond case, an action by a creditor against
recognize and in effect admit its existence as a corporate virtually to create a corporation for any purpose, or to have a stockholder of a state bank on his statutory liability, the
body. Ballantine,  op.cit.,  § 29; Machen,  Modern Law of acts so construed, would be in manifest opposition to the Court, after stating that a corporation or a stockholder
Corporations, §§ 278-282; 18 C.J.S., op.cit., § 109. statute law, and clearly against its policy, and justified could not defeat an action by showing noncompliance with
upon no sound principle in the administration of justice." the requirements of the corporation law unless the acts
It is not at all clear what Maryland has done with respect to required are conditions precedent to corporate existence,
the two doctrines. There have been no recent cases in this In the  Maryland Tube  case, an action by a corporation for said (at p. 15):
State on the subject and some of the seemingly specific performance of a contract to convey land which it
irreconcilable earlier cases offer little to clarify the had entered into prior to its becoming a legal entity, the "By holding otherwise, parties might avail themselves of the
problem.[2] Court, having cited (at p. 217) the statements in  Jones v. powers and privileges of a corporation, without in any
Aspen Hardware Co.,  40 P. 457  (Colo. 1895),[3]  with manner subjecting themselves to its duties and obligations,
In one line of cases, the Court, in determining the rights approval for the  *483proposition that "`the doctrine of and might set up their own neglect of duty, of wilful
and liabilities of a defectively organized corporation, or a estoppel cannot be successfully invoked, unless the omission to comply with the requirements of the statute, as
member or stockholder thereof, seems to have drawn a corporation has at least a de facto existence,'" that "`a de means of discharge from all their just obligations under the
distinction between those acts or requirements which are a factocorporation can never be recognized in violation of a law. This is forbidden by every principle of law and justice,
condition precedent to corporate existence and those acts positive law'" and that "`there is a broad distinction and hence such a defense could never be tolerated."
prescribed by law to be done after incorporation. In so between those acts made necessary by the statute  as a
doing, it has been generally held that where there had prerequisite to the exercise of corporate powers,  and It seems clear therefore that when a defect in the
been a failure to comply with a requirement which the law those acts required of individuals seeking incorporation but incorporation process resulted from a failure to comply
declared to be a condition precedent to the existence of not made prerequisite to the exercise of such powers,'" with a condition subsequent, the doctrine of estoppel may
the corporation, the corporation was not a legal entity and went on to say (at p. 218) that "these principles were be applied for the benefit of a creditor to estop the
was therefore precluded from suing or being sued as clearly recognized and applied" in the Boyce case. corporation, or the members or stockholders thereof, from
such. Boyce v. M.E. Church, 46 Md. 359 (1877); Regester v. denying its corporate existence. See Brune (Herbert M.,
Medcalf,  71 Md. 528,  18 A. 966  (1889);  Bonaparte v. Lake In the National Shutter Bar case, an action by a corporation Jr.),  Maryland Corporation Law and Practice  (rev. ed.), §
Roland R.R. Co.,  75 Md. 340,  23 A. 784  (1892);  Jones v. for an alleged libel which had occurred before the 339.
Linden Building Asso., 79 Md. 73, 29 A. 76 (1894); Maryland performance of a condition precedent necessary for legal
Tube Works v. West End Imp. Co.,  87 Md. 207,  39 A. incorporation, it was held — citing the Maryland Tube case In another line of Maryland cases which determined the
620(1898);  Cleaveland v. Mullin,  96 Md.  *482  598,  54 A. for the proposition that statutory conditions precedent rights and liabilities of a defectively organized corporation,
665  (1903);  National Shutter Bar Co. v. Zimmerman,  110 must have been complied with to give existence to or a member or stockholder thereof, the Court, apparently
Md. 313,  73 A. 19  (1909). These cases appear to stand for corporations formed under general laws — that the disregarding the distinction made between those

Page 42 of 160
CORPORATION LAW CASES MAYORDO, M.A
LLB 3

requirements which are conditions precedent and those the cases. The debt was not created with them individually, some cases tend to assimilate the doctrines of
which are conditions subsequent to corporate existence, but with a company acting  *486  under a formal incorporation  de facto  and by estoppel, each is a distinct
has generally precluded, on the grounds of estoppel or incorporation, and in the exercise of its corporate powers. theory and they are not dependent on one another in their
collateral attack, inquiry into the question of corporate This [creditor] dealt with it and gave it credit as a application. See 8 Fletcher,  op.cit.,  § 3763;  France on
existence.  Maltby v. Northwestern Va. R.R. Co.,  16 Md. corporation. If its assets are not ample to pay, it is the Corporations  (2nd ed.), § 29; 18 C.J.S.,  op.cit.,  § 111h.
422  (1860);  Franz v. Teutonia Building Asso.,  24 Md. misfortune of the creditor."[4] Where there is a concurrence of the three elements
259 (1866); Grape Sugar & Vinegar Mfg. Co. v. Small,40 Md. necessary for the application of the  de facto  corporation
395 (1874); Laflin & Rand Powder Co. v. Sinsheimer, 46 Md. See also the  Franz  case at p. 270 (of 24 Md.) and doctrine, there exists an entity which is a corporation  de
315 (1877); Keene v. Van Reuth, 48 Md. 184 (1878); Bartlett the  Bartlett  case at p. 498 (of 53 Md.) for similar jure  against all persons but the state. On the other hand,
v. Wilbur,53 Md. 485  (1880);  Pott & Co. v. Schmucker,  84 statements of the law. From these cases it appears that the estoppel theory is applied only to the facts of each
Md. 535,  36 A. 592  (1897). In the  Grape Sugar  case, an where the parties have assumed corporate existence and particular case and may be invoked even where there is no
action against a defectively organized corporation dealt with each other on that basis, the Court will apply corporation de facto.Accordingly, even though one or more
to  *485  recover the balance due for work done and the estoppel doctrine on the theory that the parties by of the requisites of a  de facto  corporation are absent, we
materials furnished, the Court said (at p. 400): recognizing the organization as a corporation were think that this factor does not preclude the application of
thereafter prevented from raising a question as to its the estoppel doctrine[5] in a proper case, such as the one at
"The second prayer proceeds upon the assumption that the corporate existence. bar.
[corporation] is not liable, provided the work was done
prior to the recording of the certificate of incorporation. It When summarized, the law in Maryland pertaining to the de *488  I.B.M. contends that the failure of the Bureau to file
is true, that under the general incorporation law of this facto and estoppel doctrines reveals that the cases seem to its certificate of incorporation debarred  all  corporate
State, the recording of the certificate was necessary to fall into one or the other of two categories. In one line of existence. But, in spite of the fact that the omission might
constitute the [corporation] a body politic. If, however, the cases, the Court, choosing to disregard the nature of the have prevented the Bureau from being either a
contract was made with the [creditor] through * * * [the] dealings between the parties, refused to recognize both corporation  de jure  or  de facto,[6]Jones v. Linden Building
President of the [corporation], after the certificate had doctrines where there had been a failure to comply with a Asso., supra,  we think that I.B.M. having dealt with the
been signed by the members of the proposed corporation, condition precedent to corporate existence, but, whenever Bureau as if it were a corporation and relied on its credit
but before it was recorded, and the company, after its such noncompliance concerned a condition subsequent to rather than that of Cranson, is estopped to assert that the
incorporation was complete, accepted the work done under incorporation, the Court often applied the estoppel Bureau was not incorporated at the time the typewriters
the contract, it will be estopped, both in law and equity, doctrine. In the other line of cases, the Court, choosing to were purchased.  Laflin & Rand Powder Co. v. Sinsheimer,
from denying its liability, on account of the same." make no distinction between defects which  *487were supra. See also Tulane Improvement Co. v. S.A. Chapman &
conditions precedent and those which were conditions Co., 56 So. 509 (La. 1911). In 1 Clark and Marshall, Private
Cf.  Hammond v. Straus, supra.  And see to the subsequent, emphasized the course of conduct between Corporations, § 89, it is stated:
contrary  Boyce v. M.E. Church, supra,  which might be the parties and applied the estoppel doctrine when there
distinguishable in that it involved an effort to impose had been substantial dealings between them on a corporate "The doctrine in relation to estoppel is based upon the
liability on a religious society and not a business basis. ground that it would generally be inequitable
corporation. to *489 permit the corporate existence of an association to
Whether or not the decisions in the  Boyce  and  Maryland be denied by persons who have represented it to be a
In the Laflin & Rand case, decided in the same year (1877) Tube  cases had the effect of repudiating the  de corporation, or held it out as a corporation, or by any
as the  Boyce  case, the Court, in an action against certain facto  doctrine in this state, as some of the text writers persons who have recognized it as a corporation by dealing
members of a corporation to make them individually liable seem to think, is a question we do not reach in this case with it as such; and by the overwhelming weight of
for goods sold and delivered to the corporation, said (at p. and therefore need not consider at this time. On the other authority, therefore, a person may be estopped to deny the
321): hand, since it is clear that the Maryland Tube and National legal incorporation of an association which is not even a
Shutter Bar  cases are inconsistent with other Maryland corporation de facto."
"[The company] has been clothed with all the forms of a cases insofar as they held (in relying on the statements
corporation by the laws of a neighboring State, and was in in Jones v. Aspen Hardware Co., supra) that the doctrine of In cases similar to the one at bar, involving a failure to file
the exercise and use of the franchises conferred upon it. It estoppel cannot be invoked unless a corporation has at articles of incorporation, the courts of other jurisdictions
was a corporation de facto at the time the goods were sold least a  de facto  existence, both cases —  Maryland have held that where one has recognized the corporate
and delivered to it * * * and its existence as a corporation Tube and National Shutter Bar — should be, and are hereby, existence of an association, he is estopped to assert the
cannot be collaterally drawn into question. overruled to the extent of the inconsistency. There is, as contrary with respect to a claim arising out of such
we see it, a wide difference between creating a dealings. See, for example,  Tarbell v. Page,  24 Ill.
"To permit a recovery against the defendants, and thereby corporation by means of the  de facto  doctrine and 46 (1860); Magnolia Shingle Co. v. J. Zimmern's Co., 58 So.
to say that they are to be regarded in law as a voluntary estopping a party, due to his conduct in a particular case, 90 (Ala. 1912); Lockwood v. Wynkoop, 144 N.W. 846 (Mich.
unincorporated association, would be a departure from all from setting up the claim of no incorporation. Although

Page 43 of 160
CORPORATION LAW CASES MAYORDO, M.A
LLB 3

1914); John Lucas Co. v. Bernhardt's Estate, 100 So.399 (La. property which crop was, at the time of the complained of, acted with grave abuse of discretion and
1924). commencement of the action, already harvested, processed prayed that same be declared a nullity.
and sold by defendants; that notwithstanding that fact,
Since I.B.M. is estopped to deny the corporate existence of defendants refused to render an accounting of the income From the foregoing narration of facts, it is clear that the
the Bureau, we hold that Cranson was not liable for the derived therefrom and to deliver the lessor's share; that order sought to be nullified was issued by tile respondent
balance due on account of the typewriters. the estimated gross income was P4,500, and the deductible Judge upon motion of defendant Refuerzo, obviously
expenses amounted to P1,000; that as defendants' refusal pursuant to Rule 38 of the Rules of Court. Section 3 of said
Judgment reversed; the appellee to pay the costs. to undertake such task was in violation of the terms of the Rule, however, in providing for the period within which
covenant entered into between the plaintiff and defendant such a motion may be filed, prescribes that:
G.R. No. L-11442             May 23, 1958 corporation, a rescission was but proper.
MANUELA T. VDA. DE SALVATIERRA, petitioner, 
 SEC. 3.WHEN PETITION FILED; CONTENTS AND
vs. HON. LORENZO C. GARLITOS, in his capacity as Judge As defendants apparently failed to file their answer to the VERIFICATION. — A petition provided for in either of
of the Court of First Instance of Leyte, Branch II, and complaint, of which they were allegedly notified, the Court the preceding sections of this rule must be verified,
SEGUNDINO REFUERZO, respondents. declared them in default and proceeded to receive filed  within sixty days after the petitioner learns
plaintiff's evidence. On  June 8, 1955, the lower Court of the judgment, order, or other proceeding to be
FELIX, J.: rendered judgment granting plaintiff's prayer, and required set aside, and not more than six months after such
This is a petition for certiorari filed by Manuela T. Vda. de defendants to render a complete accounting of the harvest judgment or order was entered, or such proceeding
Salvatierra seeking to nullify the order of the Court of First of the land subject of the proceeding within 15 days from was taken; and must be must be accompanied with
Instance of Leyte in Civil Case No. 1912, dated March 21, receipt of the decision and to deliver 30 per cent of the net affidavit showing the fraud, accident, mistake, or
1956, relieving Segundino Refuerzo of liability for the income realized from the last harvest to plaintiff, with excusable negligence relied upon, and the facts
contract entered into between the former and the legal interest from the date defendants received payment constituting the petitioner is good and substantial
Philippine Fibers Producers Co., Inc., of which Refuerzo is for said crop. It was further provide that upon defendants' cause of action or defense, as the case may be,
the president. The facts of the case are as follows: failure to abide by the said requirement, the gross income which he may prove if his petition be granted".
would be fixed at P4,200 or a net income of P3,200 after (Rule 38)
Manuela T. Vda. de Salvatierra appeared to be the owner of deducting the expenses for production, 30 per cent of
a parcel of land located at Maghobas, Poblacion, Burauen, which or P960 was held to be due the plaintiff pursuant to The aforequoted provision treats of 2 periods, i.e., 60 days
Teyte. On March 7, 1954, said landholder entered into a the aforementioned contract of lease, which was declared after petitioner learns of the judgment, and not more than
contract of lease with the Philippine Fibers Producers Co., rescinded. 6 months after the judgment or order was rendered, both
Inc., allegedly a corporation "duly organized and existing of which must be satisfied. As the decision in the case at
under the laws of the Philippines, domiciled at Burauen, No appeal therefrom having been perfected within the bar was under date of June 8, 1955, whereas the motion
Leyte, Philippines, and with business address therein, reglementary period, the Court, upon motion of plaintiff, filed by respondent Refuerzo was dated January 31, 1956,
represented in this instance by Mr. Segundino Q. Refuerzo, issued a writ of execution, in virtue of which the Provincial or after the lapse of 7 months and 23 days, the filing of the
the President". It was provided in said contract, among Sheriff of Leyte caused the attachment of 3 parcels of land aforementioned motion was clearly made beyond the
other things, that the lifetime of the lease would be for a registered in the name of Segundino Refuerzo. No property prescriptive period provided for by the rules. The remedy
period of 10 years; that the land would be planted to of the Philippine Fibers Producers Co., Inc., was found allowed by Rule 38 to a party adversely affected by a
kenaf, ramie or other crops suitable to the soil; that the available for attachment. On January 31, 1956, defendant decision or order is certainly an alert of grace or
lessor would be entitled to 30 per cent of the net income Segundino Refuerzo filed a motion claiming that the benevolence intended to afford said litigant a penultimate
accruing from the harvest of any, crop without being decision rendered in said Civil Case No. 1912 was null and opportunity to protect his interest. Considering the nature
responsible for the cost of production thereof; and that void with respect to him, there being no allegation in the of such relief and the purpose behind it, the periods fixed
after every harvest, the lessee was bound to declare at the complaint pointing to his personal liability and thus prayed by said rule are non-extendible and never interrupted; nor
earliest possible time the income derived therefrom and to that an order be issued limiting such liability to defendant could it be subjected to any condition or contingency
deliver the corresponding share due the lessor. corporation. Over plaintiff's opposition, the Court  a because it is of itself devised to meet a condition or
quo granted the same and ordered the Provincial Sheriff of contingency (Palomares vs. Jimenez,*  G.R. No. L-4513,
Apparently, the aforementioned obligations imposed on the Leyte to release all properties belonging to the movant that January 31, 1952). On this score alone, therefore, the
alleged corporation were not complied with because on might have already been attached, after finding that the petition for a writ of  certiorari  filed herein may be
April 5, 1955, Alanuela T. Vda, de Salvatierra filed with the evidence on record made no mention or referred to any granted. However, taking note of the question presented by
Court of First Instance of Leyte a complaint against the fact which might hold movant personally liable therein. As the motion for relief involved herein, We deem it wise to
Philippine Fibers Producers Co., Inc., and Segundino Q. plaintiff's petition for relief from said order was denied, delve in and pass upon the merit of the same.
Refuerzo, for accounting, rescission and damages (Civil Manuela T. Vda.de Salvatierra instituted the instant action Refuerzo, in praying for his exoneration from any liability
Case No. 1912). She averred that sometime in April, 1954, asserting that the trial Judge in issuing the order resulting from the non-fulfillment of the obligation imposed
defendants planted kenaf on 3 hectares of the leased on defendant Philippine Fibers Producers Co., Inc.,

Page 44 of 160
CORPORATION LAW CASES MAYORDO, M.A
LLB 3

interposed the defense that the complaint filed with the their own risk. And as it is an elementary principle of law The Court of First Instance of Sorsogon dismissed the
lower court contained no allegation which would hold him that a person who acts as an agent without authority or complaint.  3  On appeal, its decision was set aside by the
liable personally, for while it was stated therein that he without a principal is himself regarded as the principal, respondent court, which held the school suable and liable
was a signatory to the lease contract, he did so in his possessed of all the rights and subject to all the liabilities while absolving the other defendants.  4  The motion for
capacity as president of the corporation. And this allegation of a principal, a person acting or purporting to act on reconsideration having been denied,  5  the school then
was found by the Court a quo to be supported by the behalf of a corporation which has no valid existence came to this Court in this petition for review on certiorari.
records. Plaintiff on the other hand tried to refute this assumes such privileges and obligations and comes
averment by contending that her failure to specify personally liable for contracts entered into or for other acts The issues raised in the petition are:
defendant's personal liability was due to the fact that all performed as such, agent (Fay vs. Noble, 7 Cushing [Mass.]
the time she was under the impression that the Philippine 188. Cited in II Tolentino's Commercial Laws of the 1. Whether or not a school that has not been incorporated
Fibers Producers Co., Inc., represented by Refuerzo was a Philippines, Fifth Ed., P. 689-690). Considering that may be sued by reason alone of its long continued
duly registered corporation as appearing in the contract, defendant Refuerzo, as president of the unregistered existence and recognition by the government,
but a subsequent inquiry from the Securities and Exchange corporation Philippine Fibers Producers Co., Inc., was the
Commission yielded otherwise. While as a general rule a moving spirit behind the consummation of the lease 2. Whether or not a complaint filed against persons
person who has contracted or dealt with an association in agreement by acting as its representative, his liability associated under a common name will justify a judgment
such a way as to recognize its existence as a corporate cannot be limited or restricted that imposed upon against the association itself and not its individual
body is estopped from denying the same in an action arising corporate shareholders. In acting on behalf of a corporation members.
out of such transaction or dealing, (Asia Banking which he knew to be unregistered, he assumed the risk of
Corporation vs. Standard Products Co., 46 Phil., 114; reaping the consequential damages or resultant rights, if 3. Whether or not the collection of tuition fees and book
Compania Agricola de Ultramar vs. Reyes, 4 Phil., 1; Ohta any, arising out of such transaction. rentals will make a school profit-making and not charitable.
Development Co.; vs. Steamship Pompey, 49 Phil., 117), yet
this doctrine may not be held to be applicable where fraud Wherefore, the order of the lower Court of March 21, 1956, 4. Whether or not the Termination Pay Law then in force
takes a part in the said transaction. In the instant case, on amending its previous decision on this matter and ordering was available to the private respondent who was employed
plaintiff's charge that she was unaware of the fact that the the Provincial Sheriff of Leyte to release any and all on a year-to-year basis.
Philippine Fibers Producers Co., Inc., had no juridical properties of movant therein which might have been
personality, defendant Refuerzo gave no confirmation or attached in the execution of such judgment, is hereby set 5. Whether or not the awards made by the respondent
denial and the circumstances surrounding the execution of aside and nullified as if it had never been issued. With costs court were warranted.
the contract lead to the inescapable conclusion that against respondent Segundino Refuerzo. It is so ordered.
plaintiff Manuela T. Vda. de Salvatierra was really made to We hold against the petitioner on the first question. It is
believe that such corporation was duly organized in G.R. No. L-58028 April 18, 1989 true that Rule 3, Section 1, of the Rules of Court clearly
accordance with law. CHIANG KAI SHEK SCHOOL, petitioner, 
 provides that "only natural or juridical persons may be
vs. COURT OF APPEALS and FAUSTINA FRANCO parties in a civil action." It is also not denied that the
There can be no question that a corporation with registered OH, respondents. school has not been incorporated. However, this omission
has a juridical personality separate and distinct from its should not prejudice the private respondent in the
component members or stockholders and officers such that CRUZ, J.: assertion of her claims against the school.
a corporation cannot be held liable for the personal An unpleasant surprise awaited Fausta F. Oh when she
indebtedness of a stockholder even if he should be its reported for work at the Chiang Kai Shek School in Sorsogon As a school, the petitioner was governed by Act No. 2706 as
president (Walter A. Smith Co. vs. Ford, SC-G.R. No. 42420) on the first week of July, 1968. She was told she had no amended by C.A. No. 180, which provided as follows:
and conversely, a stockholder or member cannot be held assignment for the next semester. Oh was shocked. She had
personally liable for any financial obligation be, the been teaching in the school since 1932 for a continuous Unless exempted for special reasons by the
corporation in excess of his unpaid subscription. But this period of almost 33 years. And now, out of the blue, and for Secretary of Public Instruction, any private school
rule is understood to refer merely to registered no apparent or given reason, this abrupt dismissal. or college recognized by the government shall be
corporations and cannot be made applicable to the liability incorporated under the provisions of Act No. 1459
of members of an unincorporated association. The reason Oh sued. She demanded separation pay, social security known as the Corporation Law, within 90 days after
behind this doctrine is obvious-since an organization which benefits, salary differentials, maternity benefits and moral the date of recognition, and shall file with the
before the law is non-existent has no personality and would and exemplary damages. 1 The original defendant was the Secretary of Public Instruction a copy of its
be incompetent to act and appropriate for itself the powers Chiang Kai Shek School but when it filed a motion to dismiss incorporation papers and by-laws.
and attribute of a corporation as provided by law; it cannot on the ground that it could not be sued, the complaint was
create agents or confer authority on another to act in its amended.  2  Certain officials of the school were also Having been recognized by the government, it was under
behalf; thus, those who act or purport to act as its impleaded to make them solidarily liable with the school. obligation to incorporate under the Corporation Law within
representatives or agents do so without authority and at 90 days from such recognition. It appears that it had not

Page 45 of 160
CORPORATION LAW CASES MAYORDO, M.A
LLB 3

done so at the time the complaint was filed yearly basis and the school was not required to rehire her The employer, upon whom no such notice was
notwithstanding that it had been in existence even earlier in 1968. The argument is that her services were terminable served in case of termination of employment
than 1932. The petitioner cannot now invoke its own non- at the end of each year at the discretion of the school. without just cause may hold the employee liable
compliance with the law to immunize it from the private Significantly, no explanation was given by the petitioner, for damages.
respondent's complaint. and no advance notice either, of her relief after teaching
year in and year out for all of thirty-two years, the private The employee, upon whom no such notice was
There should also be no question that having contracted respondent was simply told she could not teach any more. served in case of termination of employment
with the private respondent every year for thirty two years without just cause shall be entitled to
and thus represented itself as possessed of juridical The Court holds, after considering the particular compensation from the date of termination of his
personality to do so, the petitioner is now estopped from circumstance of Oh's employment, that she had become a employment in an I amount equivalent to his
denying such personality to defeat her claim against it. permanent employee of the school and entitled to security salaries or wages correspond to the required period
According to Article 1431 of the Civil Code, "through of tenure at the time of her dismissal. Since no cause was of notice. ... .
estoppel an admission or representation is rendered shown and established at an appropriate hearing, and the
conclusive upon the person making it and cannot be denied notice then required by law had not been given, such The respondent court erred, however, in awarding her one
or disproved as against the person relying on it." dismissal was invalid. month pay instead of only one-half month salary for every
year of service. The law is quite clear on this matter.
As the school itself may be sued in its own name, there is The private respondent's position is no different from that Accordingly, the separation pay should be computed at
no need to apply Rule 3, Section 15, under which the of the rank-and-file employees involved in  Gregorio P90.00 times 32 months, for a total of P2,880.00.
persons joined in an association without any juridical Araneta University Foundation v. NLRC,  9  of whom the
personality may be sued with such association. Besides, it Court had the following to say: Parenthetically, R.A. No. 4670, otherwise known as the
has been shown that the individual members of the board Magna Carta for Public School Teachers, confers security of
of trustees are not liable, having been appointed only after Undoubtedly, the private respondents' positions as tenure on the teacher upon appointment as long as he
the private respondent's dismissal. 6 deans and department heads of the petitioner possesses the required qualification.  10  And under the
university are necessary in its usual business. present policy of the Department of Education, Culture and
It is clear now that a charitable institution is covered by Moreover, all the private respondents have been Sports, a teacher becomes permanent and automatically
the labor laws  7  although the question was still unsettled serving the university from 18 to 28 years. All of acquires security of tenure upon completion of three years
when this case arose in 1968. At any rate, there was no law them rose from the ranks starting as instructors in the service. 11
even then  exempting  such institutions from the operation until they became deans and department heads of
of the labor laws (although they were exempted by the the university. A person who has served the While admittedly not applicable to the case at bar, these I
Constitution from ad valorem taxes). Hence, even assuming University for 28 years and who occupies a high rules nevertheless reflect the attitude of the government
that the petitioner was a charitable institution as it claims, administrative position in addition to teaching on the protection of the worker's security of tenure, which
the private respondent was nonetheless still entitled to the duties could not possibly be a temporary employee is now guaranteed by no less than the Constitution
protection of the Termination Pay Law, which was then in or a casual. itself. 12
force.
The applicable law is the Termination Pay Law, which We find that the private respondent was arbitrarily treated
While it may be that the petitioner was engaged in provided: by the petitioner, which has shown no cause for her
charitable works, it would not necessarily follow that those removal nor had it given her the notice required by the
in its employ were as generously motivated. Obviously, SECTION 1. In cases of employment, without a Termination Pay Law. As the respondent court said, the
most of them would not have the means for such charity. definite period, in a commercial, industrial, or contention that she could not report one week before the
The private respondent herself was only a humble school agricultural establishment or enterprise, the start of classes is a flimsy justification for replacing
teacher receiving a meager salary of Pl80. 00 per month. employer or the employee may terminate at any her.  13  She had been in its employ for all of thirty-two
time the employment with just cause; or without years. Her record was apparently unblemished. There is no
At that, it has not been established that the petitioner is a just cause in the case of an employee by serving showing of any previous strained relations between her and
charitable institution, considering especially that it charges written notice on the employer at least one month the petitioner. Oh had every reason to assume, as she had
tuition fees and collects book rentals from its in advance, or in the case of an employer, by done in previous years, that she would continue teaching as
students.  8  While this alone may not indicate that it is serving such notice to the employee at least one usual.
profit-making, it does weaken its claim that it is a non- month in advance or one-half month for every year
profit entity. of service of the employee, whichever, is longer, a It is easy to imagine the astonishment and hurt she felt
fraction of at least six months being considered as when she was flatly and without warning told she was
The petitioner says the private respondent had not been one whole year. dismissed. There was not even the amenity of a formal
illegally dismissed because her teaching contract was on a notice of her replacement, with perhaps a graceful

Page 46 of 160
CORPORATION LAW CASES MAYORDO, M.A
LLB 3

expression of thanks for her past services. She was simply was the president of the Samahang Angeles-Mabalacat "Section 5. x x x [T]he Securities and Exchange Commission
informed she was no longer in the teaching staff. To put it Jeepney Operators' and Drivers' Association, Inc. [has] original and exclusive jurisdiction to hear and decide
bluntly, she was fired. (SAMAJODA);  in August 1995, upon the request of the cases involving:
Sangguniang Bayan of Mabalacat, Pampanga, petitioner and (a)  Devices or schemes employed by or any acts of the
For the wrongful act of the petitioner, the private private respondent  agreed to consolidate their respective board of directors, business associates, its officers or
respondent is entitled to moral damages. 14 As a proximate associations and form the Unified Mabalacat-Angeles partners, amounting to fraud and misrepresentation which
result of her illegal dismissal, she suffered mental anguish, Jeepney Operators' and Drivers' Association, Inc. may be detrimental to the interest of the public and/or of
serious anxiety, wounded feelings and even besmirched (UMAJODA); petitioner and private respondent also agreed the stockholders, partners, members of associations or
reputation as an experienced teacher for more than three to elect one set of officers who shall be given the sole organizations registered with the Commission.
decades. We also find that the respondent court did not err authority to collect the daily dues from the members of the
in awarding her exemplary damages because the petitioner consolidated association; elections were held on October (b) Controversies arising out of intracorporate or
acted in a wanton and oppressive manner when it dismissed 29, 1995 and both petitioner and private respondent ran for partnership relations, between and among stockholders,
her. 15 president;  petitioner won;  private respondent protested members or associates; between any or all of them and the
and, alleging fraud, refused to recognize the results of the corporation, partnership or association of which they are
The Court takes this opportunity to pay a sincere tribute to election; private respondent also refused to abide by their stockholders, members, or associates, respectively;  and
the grade school teachers, who are always at the forefront agreement and continued collecting the dues from the between such corporation, partnership or association and
in the battle against illiteracy and ignorance. If only members of his association despite several demands to the state insofar as it concerns their individual franchise or
because it is they who open the minds of their pupils to an desist. Petitioner was thus constrained to file the complaint right to exist as such entity.
unexplored world awash with the magic of letters and to restrain private respondent from collecting the dues and
numbers, which is an extraordinary feat indeed, these to order him to pay damages in the amount of  P25,000.00 (c) Controversies in the election or appointment of
humble mentors deserve all our respect and appreciation. and attorney's fees of P500.00.[1] directors, trustees, officers or managers of such
corporations, partnerships or associations.
WHEREFORE, the petition is DENIED. The appealed decision Private respondent moved to dismiss the complaint for lack
is AFFIRMED except for the award of separation pay, which of jurisdiction, claiming that jurisdiction was lodged with (d) Petitions of corporations, partnerships or associations to
is reduced to P2,880.00. All the other awards are approved. the Securities and Exchange Commission (SEC).  The MCTC be declared in the state of suspension of payments in cases
Costs against the petitioner. denied the motion on February 9, 1996.[2]  It denied where the corporation, partnership or association possesses
reconsideration on March 8, 1996.[3] sufficient property to cover all its debts but foresees the
This decision is immediately executory. impossibility of meeting them when they respect very fall
SO ORDERED. Private respondent filed a petition for certiorari before the due or in cases where the corporation, partnership or
Regional Trial Court, Branch 58, Angeles City.[4]  The trial association has no sufficient assets to cover its liabilities,
[G.R. No. 125221. June 19, 1997] court found the dispute to be intracorporate, hence, but is under the management of a Rehabilitation Receiver
REYNALDO M. LOZANO, petitioner, vs. HON. ELIEZER R. subject to the jurisdiction of the SEC, and ordered the or Management Committee created pursuant to this
DE LOS SANTOS, Presiding Judge, RTC, Br. 58, MCTC to dismiss Civil Case No. 1214 accordingly.[5] It denied Decree."
Angeles City; and ANTONIO ANDA, respondents. reconsideration on May 31, 1996.[6]
The grant of jurisdiction to the SEC must be viewed in the
DECISION Hence this petition. Petitioner claims that: light of its nature and function under the law.[8]  This
PUNO, J.: jurisdiction is determined by a concurrence of two
"THE RESPONDENT JUDGE ACTED WITH GRAVE ABUSE OF elements: (1) the status or relationship of the parties; and
This petition for certiorari seeks to annul and set aside the DISCRETION AMOUNTING TO LACK OR EXCESS OF (2) the nature of the question that is the subject of their
decision of the Regional Trial Court, Branch 58, Angeles City JURISDICTION AND SERIOUS ERROR OF LAW IN CONCLUDING controversy.[9]
which ordered the Municipal Circuit Trial Court, Mabalacat THAT THE SECURITIES AND EXCHANGE COMMISSION HAS
and Magalang, Pampanga to dismiss Civil Case No. 1214 for JURISDICTION OVER A CASE OF DAMAGES BETWEEN HEADS/ The first element requires that the controversy must arise
lack of jurisdiction. PRESIDENTS OF TWO (2) ASSOCIATIONS WHO INTENDED TO out of intracorporate or partnership relations between and
CONSOLIDATE/MERGE THEIR ASSOCIATIONS BUT NOT YET among stockholders, members, or associates; between any
The facts are undisputed.  On December 19, 1995, [SIC] APPROVED AND REGISTERED WITH THE SECURITIES AND or all of them and the corporation, partnership or
petitioner Reynaldo M. Lozano filed Civil Case No. 1214 for EXCHANGE COMMISSION."[7] association of which they are stockholders, members or
damages against respondent Antonio Anda before associates, respectively; and between such corporation,
the  Municipal Circuit Trial Court (MCTC), Mabalacat The jurisdiction of the Securities and Exchange Commission partnership or association and the State in so far as it
and  Magalang, Pampanga.  Petitioner alleged that he was (SEC) is set forth in Section 5 of Presidential Decree No. concerns their individual franchises.[10] The second element
the president of the Kapatirang Mabalacat-Angeles Jeepney 902-A. Section 5 reads as follows: requires that the dispute among the parties be intrinsically
Drivers' Association, Inc. (KAMAJDA) while respondent Anda connected with the regulation of the corporation,

Page 47 of 160
CORPORATION LAW CASES MAYORDO, M.A
LLB 3

partnership or association or deal with the internal affairs involved and the conflict arises only among those assuming and circumstances and the proceedings that transpired
of the corporation, partnership or association.[11] After all, the form of a corporation, who therefore know that it has during the trial of this case;
the principal function of the SEC is the supervision and not been registered, there is no corporation by estoppel.[20]
control of corporations, partnerships and associations with a. P532,045.00 representing [the] unpaid purchase price of
the end in view that investments in these entities may be IN VIEW WHEREOF, the petition is granted and the decision the fishing nets covered by the Agreement plus P68,000.00
encouraged and protected, and their activities pursued for dated April 18, 1996 and the order dated May 31, 1996 of representing the unpaid price of the floats not covered by
the promotion of economic development.[12] the Regional Trial Court, Branch 58, Angeles City are set said Agreement;
aside.The Municipal Circuit Trial Court of Mabalacat and
There is no intracorporate nor partnership relation between Magalang, Pampanga is ordered to proceed with dispatch in b.  12% interest per annum counted from date of plaintiffs
petitioner and private respondent.  The controversy resolving Civil Case No. 1214. No costs. invoices and computed on their respective amounts as
between them arose out of their plan to consolidate their SO ORDERED. follows:
respective jeepney drivers' and operators' associations into
a single common association.  This unified association was, [G.R. No. 136448. November 3, 1999] i. Accrued interest of  P73,221.00 on Invoice No. 14407
however, still a proposal. It had not been approved by the LIM TONG LIM, petitioner, vs. PHILIPPINE FISHING GEAR for P385,377.80 dated February 9, 1990;
SEC, neither had its officers and members submitted their INDUSTRIES, INC., respondent. ii. Accrued interest of P27,904.02 on Invoice No. 14413
articles of consolidation in accordance with Sections 78 and for P146,868.00 dated February 13, 1990;
79 of the Corporation Code.  Consolidation becomes DECISION iii. Accrued interest of P12,920.00 on Invoice No. 14426
effective not upon mere agreement of the members but PANGANIBAN, J.: for P68,000.00 dated February 19, 1990;
only upon issuance of the certificate of consolidation by the
SEC.[13]  When the SEC, upon processing and examining the A partnership may be deemed to exist among parties who c.  P50,000.00 as and for attorneys fees, plus  P8,500.00
articles of consolidation, is satisfied that the consolidation agree to borrow money to pursue a business and to divide representing P500.00 per appearance in court;
of the corporations is not inconsistent with the provisions the profits or losses that may arise therefrom, even if it is
of the Corporation Code and existing laws, it issues a shown that they have not contributed any capital of their d.  P65,000.00 representing  P5,000.00 monthly rental for
certificate of consolidation which makes the reorganization own to a "common fund." Their contribution may be in the storage charges on the nets counted from September 20,
official.[14]  The new consolidated corporation comes into form of credit or industry, not necessarily cash or fixed 1990 (date of attachment) to September 12, 1991 (date of
existence and the constituent corporations dissolve and assets. Being partners, they are all liable for debts incurred auction sale);
cease to exist.[15] by or on behalf of the partnership.  The liability for a
contract entered into on behalf of an unincorporated e. Cost of suit.
The  KAMAJDA and SAMAJODA to which petitioner and association or ostensible corporation may lie in a person With respect to the joint liability of defendants for the
private respondent belong are duly registered with the SEC, who may not have directly transacted on its behalf, but principal obligation or for the unpaid price of nets and
but these associations are two separate entities.  The reaped benefits from that contract. floats in the amount of  P532,045.00 and  P68,000.00,
dispute between petitioner and private respondent is not respectively, or for the total amount of  P600,045.00, this
within the KAMAJDA nor the SAMAJODA.  It is between The Case Court noted that these items were attached to guarantee
members of separate and distinct associations.  Petitioner In the Petition for Review on Certiorari before us, Lim Tong any judgment that may be rendered in favor of the plaintiff
and private respondent have no intracorporate relation Lim assails the November 26, 1998 Decision of the Court of but, upon agreement of the parties, and, to avoid further
much less do they have an intracorporate dispute. The SEC Appeals in CA-GR CV 41477,[1] which disposed as follows: deterioration of the nets during the pendency of this case,
therefore has no jurisdiction over the complaint. WHEREFORE, [there being] no reversible error in the it was ordered sold at public auction for not less
appealed decision, the same is hereby affirmed.[2] than  P900,000.00 for which the plaintiff was the sole and
The doctrine of corporation by estoppel[16]  advanced by winning bidder.  The proceeds of the sale paid for by
private respondent cannot override jurisdictional The decretal portion of the Quezon City Regional Trial plaintiff was deposited in court.  In effect, the amount
requirements. Jurisdiction is fixed by law and is not subject Court (RTC) ruling, which was affirmed by the CA, reads as of  P900,000.00 replaced the attached property as a
to the agreement of the parties.[17]  It cannot be acquired follows: guaranty for any judgment that plaintiff may be able to
through or waived, enlarged or diminished by, any act or WHEREFORE, the Court rules: secure in this case with the ownership and possession of the
omission of the parties, neither can it be conferred by the nets and floats awarded and delivered by the sheriff to
acquiescence of the court.[18] 1.  That plaintiff is entitled to the writ of preliminary plaintiff as the highest bidder in the public auction sale. It
attachment issued by this Court on September 20, 1990; has also been noted that ownership of the nets [was]
Corporation by estoppel is founded on principles of equity retained by the plaintiff until full payment [was] made as
and is designed to prevent injustice and unfairness.[19]  It 2.  That defendants are jointly liable to plaintiff for the stipulated in the invoices; hence, in effect, the plaintiff
applies when persons assume to form a corporation and following amounts, subject to the modifications as attached its own properties.  It [was] for this reason also
exercise corporate functions and enter into business hereinafter made by reason of the special and unique facts that this Court earlier ordered the attachment bond filed
relations with third persons. Where there is no third person by plaintiff to guaranty damages to defendants to be

Page 48 of 160
CORPORATION LAW CASES MAYORDO, M.A
LLB 3

cancelled and for the P900,000.00 cash bidded and paid for respondent some of the nets which were in his Lim appealed to the Court of Appeals (CA) which, as
by plaintiff to serve as its bond in favor of defendants. possession.  Peter Yao filed an Answer, after which he was already stated, affirmed the RTC.
deemed to have waived his right to cross-examine
From the foregoing, it would appear therefore that witnesses and to present evidence on his behalf, because of Ruling of the Court of Appeals
whatever judgment the plaintiff may be entitled to in this his failure to appear in subsequent hearings. Lim Tong Lim,
case will have to be satisfied from the amount on the other hand, filed an Answer with Counterclaim and In affirming the trial court, the CA held that petitioner was
of  P900,000.00 as this amount replaced the attached nets Crossclaim and moved for the lifting of the Writ of a partner of Chua and Yao in a fishing business and may
and floats.  Considering, however, that the total judgment Attachment.[6]  The trial court maintained the Writ, and thus be held liable as a such for the fishing nets and floats
obligation as computed above would amount to upon motion of private respondent, ordered the sale of the purchased by and for the use of the partnership.  The
only  P840,216.92, it would be inequitable, unfair and fishing nets at a public auction.  Philippine Fishing Gear appellate court ruled:
unjust to award the excess to the defendants who are not Industries won the bidding and deposited with the said The evidence establishes that all the defendants including
entitled to damages and who did not put up a single court the sales proceeds of P900,000.[7] herein appellant Lim Tong Lim undertook a partnership for
centavo to raise the amount of P900,000.00 aside from the a specific undertaking, that is for commercial fishing x x
fact that they are not the owners of the nets and On November 18, 1992, the trial court rendered its x.  Obviously, the ultimate undertaking of the defendants
floats. For this reason, the defendants are hereby relieved Decision, ruling that Philippine Fishing Gear Industries was was to divide the profits among themselves which is what a
from any and all liabilities arising from the monetary entitled to the Writ of Attachment and that Chua, Yao and partnership essentially is x x x.  By a contract of
judgment obligation enumerated above and for plaintiff to Lim, as general partners, were jointly liable to pay partnership, two or more persons bind themselves to
retain possession and ownership of the nets and floats and respondent.[8] contribute money, property or industry to a common fund
for the reimbursement of the  P900,000.00 deposited by it with the intention of dividing the profits among themselves
with the Clerk of Court. The trial court ruled that a partnership among Lim, Chua (Article 1767, New Civil Code).[13]
SO ORDERED. [3] and Yao existed based (1) on the testimonies of the
witnesses presented and (2) on a Compromise Agreement Hence, petitioner brought this recourse before this Court.
The Facts executed by the three[9]  in Civil Case No. 1492-MN which [14]

Chua and Yao had brought against Lim in the RTC of


On behalf of "Ocean Quest Fishing Corporation," Antonio Malabon, Branch 72, for (a) a declaration of nullity of The Issues
Chua and Peter Yao entered into a Contract dated February commercial documents; (b) a reformation of contracts; (c)
7, 1990, for the purchase of fishing nets of various sizes a declaration of ownership of fishing boats; (d) an In his Petition and Memorandum, Lim asks this Court to
from the Philippine Fishing Gear Industries, Inc. (herein injunction and (e) damages.[10] The Compromise Agreement reverse the assailed Decision on the following grounds:
respondent).  They claimed that they were engaged in a provided:
business venture with Petitioner Lim Tong Lim, who I  THE COURT OF APPEALS ERRED IN HOLDING, BASED ON A
however was not a signatory to the agreement.  The total a) That the parties plaintiffs & Lim Tong Lim agree to have COMPROMISE AGREEMENT THAT CHUA, YAO AND PETITIONER
price of the nets amounted to  P532,045.  Four hundred the four (4) vessels sold in the amount of  P5,750,000.00 LIM ENTERED INTO IN A SEPARATE CASE, THAT A
pieces of floats worth  P68,000 were also sold to the including the fishing net.  This  P5,750,000.00 shall be PARTNERSHIP AGREEMENT EXISTED AMONG THEM.
Corporation.[4] applied as full payment for  P3,250,000.00 in favor of JL
Holdings Corporation and/or Lim Tong Lim; II  SINCE IT WAS ONLY CHUA WHO REPRESENTED THAT HE
The buyers, however, failed to pay for the fishing nets and WAS ACTING FOR OCEAN QUEST FISHING CORPORATION
the floats; hence, private respondent filed a collection suit b) If the four (4) vessel[s] and the fishing net will be sold at WHEN HE BOUGHT THE NETS FROM PHILIPPINE FISHING, THE
against Chua, Yao and Petitioner Lim Tong Lim with a prayer a higher price than  P5,750,000.00 whatever will be the COURT OF APPEALS WAS UNJUSTIFIED IN IMPUTING
for a writ of preliminary attachment. The suit was brought excess will be divided into 3: 1/3 Lim Tong Lim; 1/3 Antonio LIABILITY TO PETITIONER LIM AS WELL.
against the three in their capacities as general partners, on Chua; 1/3 Peter Yao;
the allegation that Ocean Quest Fishing Corporation was a III  THE TRIAL COURT IMPROPERLY ORDERED THE SEIZURE
nonexistent corporation as shown by a Certification from c)  If the proceeds of the sale the vessels will be less AND ATTACHMENT OF PETITIONER LIMS GOODS.
the Securities and Exchange Commission.[5]  On September than  P5,750,000.00 whatever the deficiency shall be
20, 1990, the lower court issued a Writ of Preliminary shouldered and paid to JL Holding Corporation by 1/3 Lim In determining whether petitioner may be held liable for
Attachment, which the sheriff enforced by attaching the Tong Lim; 1/3 Antonio Chua; 1/3 Peter Yao.[11] the fishing nets and floats purchased from respondent, the
fishing nets on board  F/B Lourdes  which was then docked Court must resolve this key issue:  whether by their acts,
at the Fisheries Port, Navotas, Metro Manila. The trial court noted that the Compromise Agreement was Lim, Chua and Yao could be deemed to have entered into a
silent as to the nature of their obligations, but that joint partnership.
Instead of answering the Complaint, Chua filed a liability could be presumed from the equal distribution of
Manifestation admitting his liability and requesting a the profit and loss.[12] This Courts Ruling
reasonable time within which to pay. He also turned over to

Page 49 of 160
CORPORATION LAW CASES MAYORDO, M.A
LLB 3

The Petition is devoid of merit. acquisition of the aforesaid equipment, without which the
(5)  That Lim, Chua and Yao agreed that the refurbishing , business could not have proceeded.
First and Second Issues:  Existence of a Partnership and re-equipping, repairing, dry docking and other expenses for
Petitioner's Liability the boats would be shouldered by Chua and Yao; Given the preceding facts, it is clear that there was, among
petitioner, Chua and Yao, a partnership engaged in the
In arguing that he should not be held liable for the (6)  That because of the unavailability of funds, Jesus Lim fishing business.  They purchased the boats, which
equipment purchased from respondent, petitioner again extended a loan to the partnership in the amount of constituted the main assets of the partnership, and they
controverts the CA finding that a partnership existed P1 million secured by a check, because of which, Yao and agreed that the proceeds from the sales and operations
between him, Peter Yao and Antonio Chua. He asserts that Chua entrusted the ownership papers of two other boats, thereof would be divided among them.
the CA based its finding on the Compromise Agreement Chuas FB Lady Anne Mel and Yaos FB Tracy to Lim Tong Lim.
alone. Furthermore, he disclaims any direct participation in We stress that under Rule 45, a petition for review like the
the purchase of the nets, alleging that the negotiations (7) That in pursuance of the business agreement, Peter Yao present case should involve only questions of law. Thus, the
were conducted by Chua and Yao only, and that he has not and Antonio Chua bought nets from Respondent Philippine foregoing factual findings of the RTC and the CA are binding
even met the representatives of the respondent Fishing Gear, in behalf of "Ocean Quest Fishing on this Court, absent any cogent proof that the present
company. Petitioner further argues that he was a lessor, not Corporation," their purported business name. action is embraced by one of the exceptions to the rule.
a partner, of Chua and Yao, for the "Contract of Lease" [16] In assailing the factual findings of the two lower courts,

dated February 1, 1990, showed that he had merely leased (8)  That subsequently, Civil Case No. 1492-MN was filed in petitioner effectively goes beyond the bounds of a petition
to the two the main asset of the purported partnership -- the Malabon RTC, Branch 72 by Antonio Chua and Peter Yao for review under Rule 45.
the fishing boat F/B Lourdes. The lease was for six months, against Lim Tong Lim for (a) declaration of nullity of
with a monthly rental of  P37,500 plus 25 percent of the commercial documents; (b) reformation of contracts; (c) Compromise Agreement Not the Sole Basis of
gross catch of the boat. declaration of ownership of fishing boats; (4) injunction; Partnership
and (e) damages.
We are not persuaded by the arguments of petitioner.  The Petitioner argues that the appellate courts sole basis for
facts as found by the two lower courts clearly showed that (9)  That the case was amicably settled through a assuming the existence of a partnership was the
there existed a partnership among Chua, Yao and him, Compromise Agreement executed between the parties- Compromise Agreement. He also claims that the settlement
pursuant to Article 1767 of the Civil Code which provides: litigants the terms of which are already enumerated above. was entered into only to end the dispute among them, but
Article 1767 - By the contract of partnership, two or more not to adjudicate their preexisting rights and
persons bind themselves to contribute money, property, or From the factual findings of both lower courts, it is clear obligations.  His arguments are baseless.  The Agreement
industry to a common fund, with the intention of dividing that Chua, Yao and Lim had decided to engage in a fishing was but an embodiment of the relationship extant among
the profits among themselves. business, which they started by buying boats worth  P3.35 the parties prior to its execution.
million, financed by a loan secured from Jesus Lim who was
Specifically, both lower courts ruled that a partnership petitioners brother.  In their Compromise Agreement, they A proper adjudication of claimants rights mandates that
among the three existed based on the following factual subsequently revealed their intention to pay the loan with courts must review and thoroughly appraise all relevant
findings:[15] the proceeds of the sale of the boats, and to divide equally facts.  Both lower courts have done so and have found,
among them the excess or loss. These boats, the purchase correctly, a preexisting partnership among the parties.  In
(1)  That Petitioner Lim Tong Lim requested Peter Yao who and the repair of which were financed with borrowed implying that the lower courts have decided on the basis of
was money, fell under the term common fund under Article one piece of document alone, petitioner fails to appreciate
engaged in commercial fishing to join him, while Antonio 1767.  The contribution to such fund need not be cash or that the CA and the RTC delved into the history of the
Chua was already Yaos partner; fixed assets; it could be an intangible like credit or document and explored all the possible consequential
industry.  That the parties agreed that any loss or profit c o m b i n a t i o n s i n h a r m o n y w i t h l a w, l o g i c a n d
(2) That after convening for a few times, Lim Chua, and Yao from the sale and operation of the boats would be divided fairness.  Verily, the two lower courts factual findings
verbally agreed to acquire two fishing boats, the  FB equally among them also shows that they had indeed mentioned above nullified petitioners argument that the
Lourdes and the FB Nelson for the sum of P3.35 million; formed a partnership. existence of a partnership was based only on the
Compromise Agreement.
(3)  That they borrowed P3.25 million from Jesus Lim, Moreover, it is clear that the partnership extended not only
brother of Petitioner Lim Tong Lim, to finance the venture. to the purchase of the boat, but also to that of the nets Petitioner Was a Partner, Not a Lessor
and the floats.  The fishing nets and the floats, both
(4)  That they bought the boats from CMF Fishing essential to fishing, were obviously acquired in furtherance We are not convinced by petitioners argument that he was
Corporation, which executed a Deed of Sale over these two of their business. It would have been inconceivable for Lim merely the lessor of the boats to Chua and Yao, not a
(2) boats in favor of Petitioner Lim Tong Lim only to serve to involve himself so much in buying the boat but not in the partner in the fishing venture. His argument allegedly finds
as security for the loan extended by Jesus Lim; support in the Contract of Lease and the registration papers

Page 50 of 160
CORPORATION LAW CASES MAYORDO, M.A
LLB 3

showing that he was the owner of the boats, including F/B One who assumes an obligation to an ostensible corporation contracts and since he never directly transacted with the
Lourdes where the nets were found. as such, cannot resist performance thereof on the ground respondent corporation, ergo, he cannot be held liable.
that there was in fact no corporation.
His allegation defies logic.  In effect, he would like this Unquestionably, petitioner benefited from the use of the
Court to believe that he consented to the sale of  his Thus, even if the ostensible corporate entity is proven to nets found inside  F/B Lourdes,  the boat which has earlier
own boats to pay a debt of Chua and Yao, with the excess be legally nonexistent, a party may be estopped from been proven to be an asset of the partnership.  He in fact
of the proceeds to be divided among the three of them. No denying its corporate existence. The reason behind this questions the attachment of the nets, because the Writ has
lessor would do what petitioner did. Indeed, his consent to doctrine is obvious - an unincorporated association has no effectively stopped his use of the fishing vessel.
the sale proved that there was a preexisting partnership personality and would be incompetent to act and
among all three. appropriate for itself the power and attributes of a It is difficult to disagree with the RTC and the CA that Lim,
corporation as provided by law; it cannot create agents or Chua and Yao decided to form a corporation.  Although it
Verily, as found by the lower courts, petitioner entered into confer authority on another to act in its behalf; thus, those was never legally formed for unknown reasons, this fact
a business agreement with Chua and Yao, in which debts who act or purport to act as its representatives or agents alone does not preclude the liabilities of the three as
were undertaken in order to finance the acquisition and the do so without authority and at their own risk.  And as it is contracting parties in representation of it.  Clearly, under
upgrading of the vessels which would be used in their an elementary principle of law that a person who acts as an the law on estoppel, those acting on behalf of a
fishing business.  The sale of the boats, as well as the agent without authority or without a principal is himself corporation and those benefited by it, knowing it to be
division among the three of the balance remaining after the regarded as the principal, possessed of all the right and without valid existence, are held liable as general partners.
payment of their loans, proves beyond cavil that  F/B subject to all the liabilities of a principal, a person acting
Lourdes, though registered in his name, was not his own or purporting to act on behalf of a corporation which has no Technically, it is true that petitioner did not directly act on
property but an asset of the partnership.  It is not valid existence assumes such privileges and obligations and behalf of the corporation.  However, having reaped the
uncommon to register the properties acquired from a loan becomes personally liable for contracts entered into or for benefits of the contract entered into by persons with
in the name of the person the lender trusts, who in this other acts performed as such agent.[17] whom he previously had an existing relationship, he is
case is the petitioner himself. After all, he is the brother of deemed to be part of said association and is covered by the
the creditor, Jesus Lim. The doctrine of corporation by estoppel may apply to the scope of the doctrine of corporation by estoppel.  We
alleged corporation and to a third party.  In the first reiterate the ruling of the Court in Alonso v. Villamor:[19]
We stress that it is unreasonable indeed, it is absurd -- for instance, an unincorporated association, which represented
petitioner to sell his property to pay a debt he did not itself to be a corporation, will be estopped from denying its A litigation is not a game of technicalities in which one,
incur, if the relationship among the three of them was corporate capacity in a suit against it by a third person who more deeply schooled and skilled in the subtle art of
merely that of lessor-lessee, instead of partners. relied in good faith on such representation. It cannot allege movement and position , entraps and destroys the other. It
lack of personality to be sued to evade its responsibility for is, rather, a contest in which each contending party fully
Corporation by Estoppel a contract it entered into and by virtue of which it received and fairly lays before the court the facts in issue and then,
advantages and benefits. brushing aside as wholly trivial and indecisive all
Petitioner argues that under the doctrine of corporation by imperfections of form and technicalities of procedure, asks
estoppel, liability can be imputed only to Chua and Yao, On the other hand,  a third party who, knowing an that justice be done upon the merits.  Lawsuits, unlike
and not to him. Again, we disagree. association to be unincorporated, nonetheless treated it as duels, are not to be won by a rapiers thrust.  Technicality,
a corporation and received benefits from it, may be barred when it deserts its proper office as an aid to justice and
Section 21 of the Corporation Code of the Philippines from denying its corporate existence in a suit brought becomes its great hindrance and chief enemy, deserves
provides: against the alleged corporation. In such case, all those who scant consideration from courts. There should be no vested
benefited from the transaction made by the ostensible rights in technicalities.
Sec. 21. Corporation by estoppel. - All persons who assume corporation, despite knowledge of its legal defects, may be
to act as a corporation knowing it to be without authority held liable for contracts they impliedly assented to or took Third Issue: Validity of Attachment
to do so shall be liable as general partners for all debts, advantage of.
liabilities and damages incurred or arising as a result Finally, petitioner claims that the Writ of Attachment was
thereof:  Provided however,  That when any such ostensible There is no dispute that the respondent, Philippine Fishing improperly issued against the nets.  We agree with the
corporation is sued on any transaction entered by it as a Gear Industries, is entitled to be paid for the nets it Court of Appeals that this issue is now moot and
corporation or on any tort committed by it as such, it shall sold.  The only question here is whether petitioner should academic.  As previously discussed,  F/B Lourdes  was an
not be allowed to use as a defense its lack of corporate be held jointly[18]  liable with Chua and Yao.  Petitioner asset of the partnership and that it was placed in the name
personality. contests such liability, insisting that only those who dealt in of petitioner, only to assure payment of the debt he and his
the name of the ostensible corporation should be held partners owed.  The nets and the floats were specifically
liable.  Since his name does not appear on any of the manufactured and tailor-made according to their own
design, and were bought and used in the fishing venture

Page 51 of 160
CORPORATION LAW CASES MAYORDO, M.A
LLB 3

they agreed upon. Hence, the issuance of the Writ to assure ground that Henri Kahn allegedly guaranteed the said computed from July 5, 1990, the date the complaint was
the payment of the price stipulated in the invoices is obligation.6 filed, until the principal obligation is fully liquidated; and
proper.  Besides, by specific agreement, ownership of the another sum of P15,000.00 for attorney's fees.
nets remained with Respondent Philippine Fishing Gear, Henri Kahn filed his answer with counterclaim. While not
until full payment thereof. denying the allegation that the Federation owed the The complaint of the plaintiff against the Philippine
amount P207,524.20, representing the unpaid balance for Football Federation and the counterclaims of the defendant
WHEREFORE, the Petition is  DENIED  and the assailed the plane tickets, he averred that the petitioner has no Henri Kahn are hereby dismissed.
Decision AFFIRMED. Costs against petitioner. cause of action against him either in his personal capacity
SO ORDERED. or in his official capacity as president of the Federation. He With the costs against defendant Henri Kahn.10
maintained that he did not guarantee payment but merely
G.R. No. 119002               October 19, 2000 acted as an agent of the Federation which has a separate Only Henri Kahn elevated the above decision to the Court
INTERNATIONAL EXPRESS TRAVEL & TOUR SERVICES, and distinct juridical personality.7 of Appeals. On 21 December 1994, the respondent court
INC., petitioner, 
 rendered a decision reversing the trial court, the decretal
vs. HON. COURT OF APPEALS, HENRI KAHN, PHILIPPINE On the other hand, the Federation failed to file its answer, portion of said decision reads:
FOOTBALL FEDERATION, respondents. hence, was declared in default by the trial court.8
WHEREFORE, premises considered, the judgment appealed
DECISION In due course, the trial court rendered judgment and ruled from is hereby REVERSED and SET ASIDE and another one is
KAPUNAN, J.: in favor of the petitioner and declared Henri Kahn rendered dismissing the complaint against defendant Henri
On June 30 1989, petitioner International Express Travel personally liable for the unpaid obligation of the S. Kahn.11
and Tour Services, Inc., through its managing director, Federation. In arriving at the said ruling, the trial court In finding for Henri Kahn, the Court of Appeals recognized
wrote a letter to the Philippine Football Federation rationalized: the juridical existence of the Federation. It rationalized
(Federation), through its president private respondent that since petitioner failed to prove that Henri Kahn
Henri Kahn, wherein the former offered its services as a Defendant Henri Kahn would have been correct in his guaranteed the obligation of the Federation, he should not
travel agency to the latter.1 The offer was accepted. contentions had it been duly established that defendant be held liable for the same as said entity has a separate
Petitioner secured the airline tickets for the trips of the Federation is a corporation. The trouble, however, is that and distinct personality from its officers.
athletes and officials of the Federation to the South East neither the plaintiff nor the defendant Henri Kahn has
Asian Games in Kuala Lumpur as well as various other trips adduced any evidence proving the corporate existence of Petitioner filed a motion for reconsideration and as an
to the People's Republic of China and Brisbane. The total the defendant Federation. In paragraph 2 of its complaint, alternative prayer pleaded that the Federation be held
cost of the tickets amounted to P449,654.83. For the plaintiff asserted that "Defendant Philippine Football liable for the unpaid obligation. The same was denied by
tickets received, the Federation made two partial Federation is a sports association xxx." This has not been the appellate court in its resolution of 8 February 1995,
payments, both in September of 1989, in the total amount denied by defendant Henri Kahn in his Answer. Being the where it stated that:
of P176,467.50.2 President of defendant Federation, its corporate existence
is within the personal knowledge of defendant Henri Kahn. As to the alternative prayer for the Modification of the
On 4 October 1989, petitioner wrote the Federation, He could have easily denied specifically the assertion of the Decision by expressly declaring in the dispositive portion
through the private respondent a demand letter requesting plaintiff that it is a mere sports association, if it were a thereof the Philippine Football Federation (PFF) as liable
for the amount of P265,894.33.3  On 30 October 1989, the domestic corporation. But he did not. for the unpaid obligation, it should be remembered that
Federation, through the Project Gintong Alay, paid the the trial court dismissed the complaint against the
amount of P31,603.00.4 xxx Philippine Football Federation, and the plaintiff did not
A voluntary unincorporated association, like defendant appeal from this decision. Hence, the Philippine Football
On 27 December 1989, Henri Kahn issued a personal check Federation has no power to enter into, or to ratify, a Federation is not a party to this appeal and consequently,
in the amount of P50,000 as partial payment for the contract. The contract entered into by its officers or agents no judgment may be pronounced by this Court against the
outstanding balance of the Federation.5  Thereafter, no on behalf of such association is not binding on, or PFF without violating the due process clause, let alone the
further payments were made despite repeated demands. enforceable against it. The officers or agents are fact that the judgment dismissing the complaint against it,
themselves personally liable. had already become final by virtue of the plaintiff's failure
This prompted petitioner to file a civil case before the x x x9 to appeal therefrom. The alternative prayer is therefore
Regional Trial Court of Manila. Petitioner sued Henri Kahn in similarly DENIED.12
his personal capacity and as President of the Federation The dispositive portion of the trial court's decision reads:
and impleaded the Federation as an alternative defendant. Petitioner now seeks recourse to this Court and alleges that
Petitioner sought to hold Henri Kahn liable for the unpaid WHEREFORE, judgment is rendered ordering defendant the respondent court committed the following assigned
balance for the tickets purchased by the Federation on the Henri Kahn to pay the plaintiff the principal sum of errors:13
P207,524.20, plus the interest thereon at the legal rate

Page 52 of 160
CORPORATION LAW CASES MAYORDO, M.A
LLB 3

A. THE HONORABLE COURT OF APPEALS ERRED IN 3. To purchase, sell, lease or otherwise encumber a juridical personality. The power to purchase, sell, lease
HOLDING THAT PETITIONER HAD DEALT WITH THE property both real and personal, for the and encumber property are acts which may only be done by
PHILIPPINE FOOTBALL FEDERATION (PFF) AS A accomplishment of their purpose; persons, whether natural or artificial, with juridical
CORPORATE ENTITY AND IN NOT HOLDING THAT PRIVATE capacity. However, while we agree with the appellate court
RESPONDENT HENRI KAHN WAS THE ONE WHO 4. To affiliate with international or regional sports' that national sports associations may be accorded
REPRESENTED THE PFF AS HAVING A CORPORATE Associations after due consultation with the executive corporate status, such does not automatically take place by
PERSONALITY. committee; the mere passage of these laws.
xxx
B. THE HONORABLE COURT OF APPEALS ERRED IN NOT It is a basic postulate that before a corporation may
H O L D I N G P R I VAT E R E S P O N D E N T H E N R I K A H N 13. To perform such other acts as may be necessary for acquire juridical personality, the State must give its
PERSONALLY LIABLE FOR THE OBLIGATION OF THE the proper accomplishment of their purposes and not consent either in the form of a special law or a general
UNINCORPORATED PFF, HAVING NEGOTIATED WITH inconsistent with this Act. enabling act. We cannot agree with the view of the
PETITIONER AND CONTRACTED THE OBLIGATION IN appellate court and the private respondent that the
BEHALF OF THE PFF, MADE A PARTIAL PAYMENT AND Section 8 of P.D. 604, grants similar functions to these Philippine Football Federation came into existence upon
ASSURED PETITIONER OF FULLY SETTLING THE sports associations: the passage of these laws. Nowhere can it be found in R.A.
OBLIGATION. 3135 or P.D. 604 any provision creating the Philippine
SEC. 8.Functions, Powers, and Duties of National Sports Football Federation. These laws merely recognized the
C. ASSUMING ARGUENDO THAT PRIVATE RESPONDENT Association. - The National sports associations shall have existence of national sports associations and provided the
KAHN IS NOT PERSONALLY LIABLE, THE HONORABLE the following functions, powers, and duties: manner by which these entities may acquire juridical
COURT OF APPEALS ERRED IN NOT EXPRESSLY personality. Section 11 of R.A. 3135 provides:
DECLARING IN ITS DECISION THAT THE PFF IS SOLELY 1. Adopt a Constitution and By-Laws for their internal
LIABLE FOR THE OBLIGATION. organization and government which shall be submitted SEC. 11.National Sports' Association; organization and
to the Department and any amendment thereto shall recognition. - A National Association shall be organized for
The resolution of the case at bar hinges on the take effect upon approval by the Department: each individual sports in the Philippines in the manner
determination of the existence of the Philippine Football Provided, however, That no team, school, club, hereinafter provided to constitute the Philippine Amateur
Federation as a juridical person. In the assailed decision, organization, or entity shall be admitted as a voting Athletic Federation. Applications for recognition as a
the appellate court recognized the existence of the member of an association unless 60 per cent of the National Sports' Association shall be filed with the
Federation. In support of this, the CA cited Republic Act athletes composing said team, school, club, executive committee together with, among others, a copy
3135, otherwise known as the Revised Charter of the organization, or entity are Filipino citizens; of the constitution and by-laws and a list of the members of
Philippine Amateur Athletic Federation, and Presidential the proposed association, and a filing fee of ten pesos.
Decree No. 604 as the laws from which said Federation 2. Raise funds by donations, benefits, and other means
derives its existence. for their purpose subject to the approval of the The Executive Committee shall give the recognition applied
Department; for if it is satisfied that said association will promote the
As correctly observed by the appellate court, both R.A. purposes of this Act and particularly section three thereof.
3135 and P.D. No. 604 recognized the juridical existence of 3. Purchase, sell, lease, or otherwise encumber No application shall be held pending for more than three
national sports associations. This may be gleaned from the p r o p e r t y, b o t h r e a l a n d p e r s o n a l , f o r t h e months after the filing thereof without any action having
powers and functions granted to these associations. Section accomplishment of their purpose; been taken thereon by the executive committee. Should
14 of R.A. 3135 provides: 4. Conduct local, interport, and international the application be rejected, the reasons for such rejection
competitions, other than the Olympic and Asian Games, shall be clearly stated in a written communication to the
SEC. 14.Functions, powers and duties of Associations. - The for the promotion of their sport; applicant. Failure to specify the reasons for the rejection
National Sports' Association shall have the following shall not affect the application which shall be considered as
functions, powers and duties: 5. Affiliate with international or regional sports unacted upon: Provided, however, That until the executive
associations after due consultation with the committee herein provided shall have been formed,
1. To adopt a constitution and by-laws for their internal Department; applications for recognition shall be passed upon by the
organization and government; xxx duly elected members of the present executive committee
of the Philippine Amateur Athletic Federation. The said
2. To raise funds by donations, benefits, and other 13. Perform such other functions as may be provided by executive committee shall be dissolved upon the
means for their purposes. law. organization of the executive committee herein provided:
Provided, further, That the functioning executive
The above powers and functions granted to national sports committee is charged with the responsibility of seeing to it
associations clearly indicate that these entities may acquire that the National Sports' Associations are formed and

Page 53 of 160
CORPORATION LAW CASES MAYORDO, M.A
LLB 3

organized within six months from and after the passage of performed as such agent.14  As president of the Federation, LGVHAI was organized on February 8, 1983 as the
this Act. Henri Kahn is presumed to have known about the corporate association of homeowners and residents of the Loyola
existence or non-existence of the Federation. We cannot Grand Villas. It was registered with the Home Financing
Section 7 of P.D. 604, similarly provides: subscribe to the position taken by the appellate court that Corporation, the predecessor of herein respondent HIGC, as
even assuming that the Federation was defectively the sole homeowners' organization in the said subdivision
SEC. 7.National Sports Associations. - Application for incorporated, the petitioner cannot deny the corporate under Certificate of Registration No. 04-197. It was
accreditation or recognition as a national sports association existence of the Federation because it had contracted and organized by the developer of the subdivision and its first
for each individual sport in the Philippines shall be filed dealt with the Federation in such a manner as to recognize president was Victorio V. Soliven, himself the owner of the
with the Department together with, among others, a copy and in effect admit its existence.15  The doctrine of developer. For unknown reasons, however, LGVHAI did not
of the Constitution and By-Laws and a list of the members corporation by estoppel is mistakenly applied by the file its corporate by-laws.
of the proposed association. respondent court to the petitioner. The application of the
doctrine applies to a third party only when he tries to Sometime in 1988, the officers of the LGVHAI tried to
The Department shall give the recognition applied for if it escape liability on a contract from which he has benefited register its by-laws. They failed to do so.  2 To the officers'
is satisfied that the national sports association to be on the irrelevant ground of defective incorporation.16 In the consternation, they discovered that there were two other
organized will promote the objectives of this Decree and case at bar, the petitioner is not trying to escape liability organizations within the subdivision — the North Association
has substantially complied with the rules and regulations of from the contract but rather is the one claiming from the and the South Association. According to private
the Department: Provided, That the Department may contract. respondents, a non-resident and Soliven himself,
withdraw accreditation or recognition for violation of this respectively headed these associations. They also
Decree and such rules and regulations formulated by it. WHEREFORE, the decision appealed from is REVERSED and discovered that these associations had five (5) registered
SET ASIDE. The decision of the Regional Trial Court of homeowners each who were also the incorporators,
The Department shall supervise the national sports Manila, Branch 35, in Civil Case No. 90-53595 is hereby directors and officers thereof. None of the members of the
association: Provided, That the latter shall have exclusive REINSTATED. LGVHAI was listed as member of the North Association
technical control over the development and promotion of SO ORDERED. while three (3) members of LGVHAI were listed as members
the particular sport for which they are organized. of the South Association.3  The North Association was
G.R. No. 117188 August 7, 1997 registered with the HIGC on February 13, 1989 under
Clearly the above cited provisions require that before an LOYOLA GRAND VILLAS HOMEOWNERS (SOUTH) Certificate of Registration No. 04-1160 covering Phases
entity may be considered as a national sports association, ASSOCIATION, INC., petitioner, 
 West II, East III, West III and East IV. It submitted its by-laws
such entity must be recognized by the accrediting vs. HON. COURT OF APPEALS, HOME INSURANCE AND on December 20, 1988.
organization, the Philippine Amateur Athletic Federation GUARANTY CORPORATION, EMDEN ENCARNACION and
under R.A. 3135, and the Department of Youth and Sports HORATIO AYCARDO, respondents. In July, 1989, when Soliven inquired about the status of
Development under P.D. 604. This fact of recognition, LGVHAI, Atty. Joaquin A. Bautista, the head of the legal
however, Henri Kahn failed to substantiate. In attempting ROMERO, J.: department of the HIGC, informed him that LGVHAI had
to prove the juridical existence of the Federation, Henri May the failure of a corporation to file its by-laws within been automatically dissolved for two reasons. First, it did
Kahn attached to his motion for reconsideration before the one month from the date of its incorporation, as mandated not submit its by-laws within the period required by the
trial court a copy of the constitution and by-laws of the by Section 46 of the Corporation Code, result in its Corporation Code and, second, there was non-user of
Philippine Football Federation. Unfortunately, the same automatic dissolution? corporate charter because HIGC had not received any
does not prove that said Federation has indeed been This is the issue raised in this petition for review report on the association's activities. Apparently, this
recognized and accredited by either the Philippine Amateur on  certiorari  of the Decision1  of the Court of Appeals information resulted in the registration of the South
Athletic Federation or the Department of Youth and Sports affirming the decision of the Home Insurance and Guaranty Association with the HIGC on July 27, 1989 covering Phases
Development. Accordingly, we rule that the Philippine Corporation (HIGC). This quasi-judicial body recognized West I, East I and East II. It filed its by-laws on July 26,
Football Federation is not a national sports association Loyola Grand Villas Homeowners Association (LGVHA) as the 1989.
within the purview of the aforementioned laws and does sole homeowners' association in Loyola Grand Villas, a duly
not have corporate existence of its own. registered subdivision in Quezon City and Marikina City that These developments prompted the officers of the LGVHAI
was owned and developed by Solid Homes, Inc. It revoked to lodge a complaint with the HIGC. They questioned the
Thus being said, it follows that private respondent Henry the certificates of registration issued to Loyola Grand Villas revocation of LGVHAI's certificate of registration without
Kahn should be held liable for the unpaid obligations of the homeowners (North) Association Incorporated (the North due notice and hearing and concomitantly prayed for the
unincorporated Philippine Football Federation. It is a Association for brevity) and Loyola Grand Villas cancellation of the certificates of registration of the North
settled principal in corporation law that any person acting Homeowners (South) Association Incorporated (the South and South Associations by reason of the earlier issuance of
or purporting to act on behalf of a corporation which has no Association). a certificate of registration in favor of LGVHAI.
valid existence assumes such privileges and becomes
personally liable for contract entered into or for other acts

Page 54 of 160
CORPORATION LAW CASES MAYORDO, M.A
LLB 3

On January 26, 1993, after due notice and hearing, private provide or at least imply that failure to file the by-laws
respondents obtained a favorable ruling from HIGC Hearing results in an automatic dissolution of the corporation. Undaunted, the South Association filed the instant petition
Officer Danilo C. Javier who disposed of HIGC Case No. While Section 46, in prescribing that by-laws must be for review on  certiorari. It elevates as sole issue for
RRM-5-89 as follows: adopted within the period prescribed therein, may be resolution the first issue it had raised before the Court of
interpreted as a mandatory provision, particularly Appeals, i.e., whether or not the LGVHAI's failure to file its
WHEREFORE, judgment is hereby rendered recognizing because of the use of the word "must," its meaning by-laws within the period prescribed by Section 46 of the
the Loyola Grand Villas Homeowners Association, Inc., cannot be stretched to support the argument that Corporation Code had the effect of automatically dissolving
under Certificate of Registration No. 04-197 as the duly automatic dissolution results from non-compliance. the said corporation.
registered and existing homeowners association for
Loyola Grand Villas homeowners, and declaring the We realize that Section 46 or other provisions of the Petitioner contends that, since Section 46 uses the word
Certificates of Registration of Loyola Grand Villas Corporation Code are silent on the result of the failure "must" with respect to the filing of by-laws, noncompliance
Homeowners (North) Association, Inc. and Loyola Grand to adopt and file the by-laws within the required therewith would result in "self-extinction" either due to
Villas Homeowners (South) Association, Inc. as hereby period. Thus, Section 46 and other related provisions of non-occurrence of a suspensive condition or the occurrence
revoked or cancelled; that the receivership be the Corporation Code are to be construed with Section of a resolutory condition "under the hypothesis that (by)
terminated and the Receiver is hereby ordered to 6 (1) of P.D. 902-A. This section empowers the SEC to the issuance of the certificate of registration alone the
render an accounting and turn-over to Loyola Grand suspend or revoke certificates of registration on the corporate personality is deemed already formed." It asserts
Villas Homeowners Association, Inc., all assets and grounds listed therein. Among the grounds stated is the that the Corporation Code provides for a "gradation of
records of the Association now under his custody and failure to file by-laws (see also II Campos: The violations of requirements." Hence, Section 22 mandates
possession. Corporation Code, 1990 ed., pp. 124-125). Such that the corporation must be formally organized and should
suspension or revocation, the same section provides, commence transaction within two years from date of
The South Association appealed to the Appeals Board of the should be made upon proper notice and hearing. incorporation. Otherwise, the corporation would be
HIGC. In its Resolution of September 8, 1993, the Although P.D. 902-A refers to the SEC, the same deemed dissolved. On the other hand, if the corporation
Board 4 dismissed the appeal for lack of merit. principles and procedures apply to the public commences operations but becomes continuously
respondent HIGC as it exercises its power to revoke or inoperative for five years, then it may be suspended or its
Rebuffed, the South Association in turn appealed to the suspend the certificates of registration or homeowners corporate franchise revoked.
Court of Appeals, raising two issues.  First, whether or not association. (Section 2 [a], E.O. 535, series 1979,
LGVHAI's failure to file its by-laws within the period transferred the powers and authorities of the SEC over Petitioner concedes that Section 46 and the other
prescribed by Section 46 of the Corporation Code resulted homeowners associations to the HIGC.) provisions of the Corporation Code do not provide for
in the automatic dissolution of LGVHAI. Second, whether or sanctions for non-filing of the by-laws. However, it insists
not two homeowners' associations may be authorized by the We also do not agree with the petitioner's that no sanction need be provided "because the mandatory
HIGC in one "sprawling subdivision." However, in the interpretation that Section 46, Corporation Code nature of the provision is so clear that there can be no
Decision of August 23, 1994 being assailed here, the Court prevails over Section 6, P.D. 902-A and that the latter is doubt about its being an essential attribute of corporate
of Appeals affirmed the Resolution of the HIGC Appeals invalid because it contravenes the former. There is no birth." To petitioner, its submission is buttressed by the
Board. basis for such interpretation considering that these two facts that the period for compliance is "spelled out
provisions are not inconsistent with each other. They distinctly;" that the certification of the SEC/HIGC must
In resolving the first issue, the Court of Appeals held that are, in fact, complementary to each other so that one show that the by-laws are not inconsistent with the Code,
under the Corporation Code, a private corporation cannot be considered as invalidating the other. and that a copy of the by-laws "has to be attached to the
commences to have corporate existence and juridical articles of incorporation." Moreover, no sanction is provided
personality from the date the Securities and Exchange The Court of Appeals added that, as there was no showing for because "in the first place, no corporate identity has
Commission (SEC) issues a certificate of incorporation that the registration of LGVHAI had been validly revoked, it been completed." Petitioner asserts that "non-provision for
under its official seal. The requirement for the filing of by- continued to be the duly registered homeowners' remedy or sanction is itself the tacit proclamation that
laws under Section 46 of the Corporation Code within one association in the Loyola Grand Villas. More importantly, non-compliance is fatal and no corporate existence had yet
month from official notice of the issuance of the certificate the South Association did not dispute the fact that LGVHAI evolved," and therefore, there was "no need to proclaim its
of incorporation presupposes that it is already had been organized and that, thereafter, it transacted demise."  6 In a bid to convince the Court of its arguments,
incorporated, although it may file its by-laws with its business within the period prescribed by law. petitioner stresses that:
articles of incorporation. Elucidating on the effect of a
delayed filing of by-laws, the Court of Appeals said: On the second issue, the Court of Appeals reiterated its . . . the word  MUST  is used in Sec. 46 in its universal
previous ruling  5  that the HIGC has the authority to order literal meaning and corollary human implication — its
We also find nothing in the provisions cited by the the holding of a referendum to determine which of two compulsion is integrated in its very essence —  MUST  is
petitioner, i.e., Section 46 and 22, Corporation Code, or contending associations should represent the entire always enforceable by the inevitable consequence —
in any other provision of the Code and other laws which community, village or subdivision. that is, "OR ELSE". The use of the word MUST in Sec. 46

Page 55 of 160
CORPORATION LAW CASES MAYORDO, M.A
LLB 3

is no exception — it means file the by-laws within one "rise above and change" the substantive provisions of the Ordinarily, the word "must" connotes an imperative act or
month after notice of issuance of certificate of Code. operates to impose a duty which may be enforced.  9  It is
registration  OR ELSE. The  OR ELSE, though not The pertinent provision of the Corporation Code that is the synonymous with "ought" which connotes compulsion or
specified, is inextricably a part of MUST . Do this or if focal point of controversy in this case states: mandatoriness.  10  However, the word "must" in a statute,
you do not you are "Kaput". The importance of the by- like "shall," is not always imperative. It may be consistent
laws to corporate existence compels such meaning for Sec. 46.Adoption of by-laws. — Every corporation with an exercise of discretion. In this jurisdiction, the
as decreed the by-laws is "the government" of the formed under this Code, must within one (1) month tendency has been to interpret "shall" as the context or a
corporation. Indeed, how can the corporation do any after receipt of official notice of the issuance of its reasonable construction of the statute in which it is used
lawful act as such without by-laws. Surely, no law is certificate of incorporation by the Securities and demands or requires.  11  This is equally true as regards the
indeed to create chaos. 7 Exchange Commission, adopt a code of by-laws for its word "must." Thus, if the languages of a statute considered
government not inconsistent with this Code. For the as a whole and with due regard to its nature and object
Petitioner asserts that P.D. No. 902-A cannot exceed the adoption of by-laws by the corporation, the affirmative reveals that the legislature intended to use the words
scope and power of the Corporation Code which itself does vote of the stockholders representing at least a "shall" and "must" to be directory, they should be given that
not provide sanctions for non-filing of by-laws. For the majority of the outstanding capital stock, or of at least meaning.12
petitioner, it is "not proper to assess the true meaning of a majority of the members, in the case of non-stock
Sec. 46 . . . on an unauthorized provision on such matter corporations, shall be necessary. The by-laws shall be In this respect, the following portions of the deliberations
contained in the said decree." signed by the stockholders or members voting for them of the Batasang Pambansa No. 68 are illuminating:
and shall be kept in the principal office of the
In their comment on the petition, private respondents corporation, subject to the stockholders or members MR. FUENTEBELLA. Thank you, Mr. Speaker.
counter that the requirement of adoption of by-laws is not voting for them and shall be kept in the principal office On page 34, referring to the adoption of by-laws, are
mandatory. They point to P.D. No. 902-A as having resolved of the corporation, subject to inspection of the we made to understand here, Mr. Speaker, that by-laws
the issue of whether said requirement is mandatory or stockholders or members during office hours; and a must immediately be filed within one month after the
merely directory. Citing  Chung Ka Bio v.  Intermediate copy thereof, shall be filed with the Securities and issuance? In other words, would this be mandatory or
Appellate Court,  8  private respondents contend that Exchange Commission which shall be attached to the directory in character?
Section 6(I) of that decree provides that non-filing of by- original articles of incorporation.
laws is only a ground for suspension or revocation of the MR. MENDOZA. This is mandatory.
certificate of registration of corporations and, therefore, it Notwithstanding the provisions of the preceding
may not result in automatic dissolution of the corporation. paragraph, by-laws may be adopted and filed prior to MR. FUENTEBELLA. It being mandatory, Mr. Speaker,
Moreover, the adoption and filing of by-laws is a condition incorporation; in such case, such by-laws shall be what would be the effect of the failure of the
subsequent which does not affect the corporate personality approved and signed by all the incorporators and corporation to file these by-laws within one month?
of a corporation like the LGVHAI. This is so because Section submitted to the Securities and Exchange Commission,
9 of the Corporation Code provides that the corporate together with the articles of incorporation. MR. MENDOZA. There is a provision in the latter part of
existence and juridical personality of a corporation begins the Code which identifies and describes the
from the date the SEC issues a certificate of incorporation In all cases, by-laws shall be effective only upon the consequences of violations of any provision of this
under its official seal. Consequently, even if the by-laws issuance by the Securities and Exchange Commission of Code. One such consequences is the dissolution of the
have not yet been filed, a corporation may be considered a certification that the by-laws are not inconsistent corporation for its inability, or perhaps, incurring
a  de facto  corporation. To emphasize the fact the LGVHAI with this Code. certain penalties.
was registered as the sole homeowners' association in the
Loyola Grand Villas, private respondents point out that The Securities and Exchange Commission shall not MR. FUENTEBELLA. But it will not automatically amount
membership in the LGVHAI was an "unconditional restriction accept for filing the by-laws or any amendment thereto to a dissolution of the corporation by merely failing to
in the deeds of sale signed by lot buyers." of any bank, banking institution, building and loan file the by-laws within one month. Supposing the
association, trust company, insurance company, public corporation was late, say, five days, what would be the
In its reply to private respondents' comment on the utility, educational institution or other special mandatory penalty?
petition, petitioner reiterates its argument that the word " corporations governed by special laws, unless
must" in Section 46 of the Corporation Code is mandatory. It accompanied by a certificate of the appropriate MR. MENDOZA. I do not think it will necessarily result in
a d d s t h a t , b e f o r e t h e r u l i n g i n  C h u n g K a B i o government agency to the effect that such by-laws or the automatic or  ipso facto  dissolution of the
v.  Intermediate Appellate Court  could be applied to this amendments are in accordance with law. corporation. Perhaps, as in the case, as you suggested,
case, this Court must first resolve the issue of whether or in the case of El Hogar Filipino where a  quo
not the provisions of P.D. No. 902-A prescribing the rules As correctly postulated by the petitioner, interpretation of warrantoaction is brought, one takes into account the
and regulations to implement the Corporation Code can this provision of law begins with the determination of the gravity of the violation committed. If the by-laws were
meaning and import of the word "must" in this section late — the filing of the by-laws were late by, perhaps, a

Page 56 of 160
CORPORATION LAW CASES MAYORDO, M.A
LLB 3

day or two, I would suppose that might be a tolerable It has been said that the by-laws of a corporation are of procedures to govern the proceedings, hearings and
delay, but if they are delayed over a period of months the rule of its life, and that until by-laws have been appeals of cases falling with its jurisdiction.
— as is happening now — because of the absence of a adopted the corporation may not be able to act for the
clear requirement that by-laws must be completed purposes of its creation, and that the first and most The aggrieved party may appeal the order, decision or
within a specified period of time, the corporation must important duty of the members is to adopt them. This ruling of the Commission sitting en banc to the Supreme
suffer certain consequences. 13 would seem to follow as a matter of principle from the Court by petition for review in accordance with the
office and functions of by-laws. Viewed in this light, pertinent provisions of the Rules of Court.
This exchange of views demonstrates clearly that automatic the adoption of by-laws is a matter of practical, if not
corporate dissolution for failure to file the by-laws on time one of legal, necessity. Moreover, the peculiar Even under the foregoing express grant of power and
was never the intention of the legislature. Moreover, even circumstances attending the formation of a corporation authority, there can be no  automatic corporate
without resorting to the records of deliberations of the may impose the obligation to adopt certain by-laws, as dissolutionsimply because the incorporators failed to abide
Batasang Pambansa, the law itself provides the answer to in the case of a close corporation organized for specific by the required filing of by-laws embodied in Section 46 of
the issue propounded by petitioner. purposes. And the statute or general laws from which the Corporation Code. There is no outright "demise" of
the corporation derives its corporate existence may corporate existence. Proper notice and hearing are cardinal
Taken as a whole and under the principle that the best expressly require it to make and adopt by-laws and components of due process in any democratic institution,
interpreter of a statute is the statute itself (optima statuli specify to some extent what they shall contain and the agency or society. In other words, the incorporators must
interpretatix est ipsum statutum),  14  Section 46 manner of their adoption.  The mere fact, however, of be given the chance to explain their neglect or omission
aforequoted reveals the legislative intent to attach a the existence of power in the corporation to adopt by- and remedy the same.
directory, and not mandatory, meaning for the word "must" laws does not ordinarily and of necessity make the
in the first sentence thereof. Note should be taken of the exercise of such power essential to its corporate life, That the failure to file by-laws is not provided for by the
second paragraph of the law which allows the filing of the or to the validity of any of its acts. 17 Corporation Code but in another law is of no moment. P.D.
by-laws even  prior  to incorporation. This provision in the No. 902-A, which took effect immediately after its
same section of the Code rules out mandatory compliance Although the Corporation Code requires the filing of by- promulgation on March 11, 1976, is very much apposite to
with the requirement of filing the by-laws "within one (1) laws, it does not expressly provide for the consequences of the Code. Accordingly, the provisions abovequoted supply
month after receipt of official notice of the issuance of its the non-filing of the same within the period provided for in the law governing the situation in the case at bar, inasmuch
certificate of incorporation by the Securities and Exchange Section 46. However, such omission has been rectified by as the Corporation Code and P.D. No. 902-A are statutes
Commission." It necessarily follows that failure to file the Presidential Decree No. 902-A, the pertinent provisions on in  pari materia.  Interpretare et concordare legibus est
by-laws within that period does not imply the "demise" of the jurisdiction of the SEC of which state: optimus interpretandi. Every statute must be so construed
the corporation. By-laws may be necessary for the and harmonized with other statutes as to form a uniform
"government" of the corporation but these are subordinate Sec. 6. In order to effectively exercise such system of jurisprudence. 18
to the articles of incorporation as well as to the jurisdiction, the Commission shall possess the following
Corporation Code and related statutes.15  There are in fact powers: As the "rules and regulations or private laws enacted by the
cases where by-laws are unnecessary to corporate xxx xxx xxx corporation to regulate, govern and control its own actions,
existence or to the valid exercise of corporate powers, (1) To  suspend, or revoke, after proper notice and affairs and concerns and its stockholders or members and
thus: hearing, the franchise or certificate of registration of directors and officers with relation thereto and among
corporations, partnerships or associations, upon any of themselves in their relation to it,"  19  by-laws are
In the absence of charter or statutory provisions to the the grounds provided by law, including the following: indispensable to corporations in this jurisdiction. These
contrary, by-laws are not necessary either to the xxx xxx xxx may not be essential to corporate birth but certainly, these
existence of a corporation or to the valid exercise of 5. Failure to file by-laws within the required period; are required by law for an orderly governance and
the powers conferred upon it, certainly in all cases xxx xxx xxx management of corporations. Nonetheless, failure to file
where the charter sufficiently provides for the In the exercise of the foregoing authority and them within the period required by law by no means tolls
government of the body; and even where the governing jurisdiction of the Commission or by a Commissioner or the automatic dissolution of a corporation.
statute in express terms confers upon the corporation by such other bodies, boards, committees and/or any
the power to adopt by-laws, the failure to exercise the officer as may be created or designated by the In this regard, private respondents are correct in relying on
power will be ascribed to mere nonaction which will Commission for the purpose. The decision, ruling or the pronouncements of this Court in  Chung Ka Bio
not render void any acts of the corporation which order of any such Commissioner, bodies, boards, v.Intermediate Appellate Court, 20 as follows:
would otherwise be valid. 16 (Emphasis supplied.) committees and/or officer may be appealed to the
Commission sitting en banc within thirty (30) days after . . . . Moreover, failure to file the by-laws does not
As Fletcher aptly puts it: receipt by the appellant of notice of such decision, automatically operate to dissolve a corporation but is
ruling or order. The Commission shall promulgate rules now considered only a ground for such dissolution.

Page 57 of 160
CORPORATION LAW CASES MAYORDO, M.A
LLB 3

Section 19 of the Corporation Law, part of which is now notice of the issuance of its certificate of Plaintiff prayed for a judgment ordering the Botica
Section 22 of the Corporation Code, provided that the incorporation." 21 Nolasco, Inc. to register in his name in the books of the
powers of the corporation would cease if it did not corporation the five shares of stock recorded in said books
formally organize and commence the transaction of its That the corporation involved herein is under the in the name of Manuel Gonzalez, and to indemnify him in
business or the continuation of its works within two supervision of the HIGC does not alter the result of this the sum of P500 as damages, and to pay the costs. The
years from date of its incorporation. Section 20, which case. The HIGC has taken over the specialized functions of defendant again filed a demurrer on the ground that the
has been reproduced with some modifications in the former Home Financing Corporation by virtue of amended complaint did not state facts sufficient to
Section 46 of the Corporation Code, expressly declared Executive Order No. 90 dated December 17, 1989.  22  With constitute a cause of action, and that said amended
that "every corporation formed under this Act, must respect to homeowners associations, the HIGC shall complaint was ambiguous, unintelligible, uncertain, which
within one month after the filing of the articles of "exercise all the powers, authorities and responsibilities demurrer was overruled by the court.
incorporation with the Securities and Exchange that are vested on the Securities and Exchange
Commission, adopt a code of by-laws." Whether this Commission . . . , the provision of Act 1459, as amended by The defendant answered the amended complaint denying
provision should be given mandatory or only directory P.D. 902-A, to the contrary notwithstanding." 23 generally and specifically each and every one of the
effect remained a controversial question until it material allegations thereof, and, as a special defense,
became academic with the adoption of PD 902-A. Under WHEREFORE, the instant petition for review on certiorari is alleged that the defendant, pursuant to article 12 of its by-
this decree, it is now clear that the failure to file by- hereby DENIED and the questioned Decision of the Court of laws, had preferential right to buy from the plaintiff said
laws within the required period is only a ground for Appeals AFFIRMED. This Decision is immediately executory. shares at the par value of P100 a share, plus P90 as
suspension or revocation of the certificate of Costs against petitioner. dividends corresponding to the year 1922, and that said
registration of corporations. SO ORDERED. offer was refused by the plaintiff. The defendant prayed
for a judgment absolving it from all liability under the
Non-filing of the by-laws will not result in automatic G.R. No. L-23241             March 14, 1925 complaint and directing the plaintiff to deliver to the
dissolution of the corporation. Under Section 6(I) of PD HENRY FLEISCHER, plaintiff-appellee,  vs. BOTICA defendant the five shares of stock in question, and to pay
902-A, the SEC is empowered to "suspend or revoke, NOLASCO CO., INC., defendant-appellant. damages in the sum of P500, and the costs.
after proper notice and hearing, the franchise or
certificate of registration of a corporation" on the JOHNSON, J.: Upon the issue presented by the pleadings above stated,
ground  inter alia  of "failure to file by-laws within the This action was commenced in the Court of First Instance of the cause was brought on for trial, at the conclusion of
required period." It is clear from this provision that the Province of Oriental Negros on the 14th day of August, which, and on August 21, 1924, the Honorable N.
there must first of all be a hearing to determine the 1923, against the board of directors of the Botica Nolasco, Capistrano, judge, held that, in his opinion, article 12 of
existence of the ground, and secondly, assuming such Inc., a corporation duly organized and existing under the the by-laws of the corporation which gives it preferential
finding, the penalty is not necessarily revocation but laws of the Philippine Islands. The plaintiff prayed that said right to buy its shares from retiring stockholders, is in
may be only suspension of the charter. In fact, under board of directors be ordered to register in the books of the conflict with Act No. 1459 (Corporation Law), especially
the rules and regulations of the SEC, failure to file the corporation five shares of its stock in the name of Henry with section 35 thereof; and rendered a judgment ordering
by-laws on time may be penalized merely with the Fleischer, the plaintiff, and to pay him the sum of P500 for the defendant corporation, through its board of directors,
imposition of an administrative fine without affecting damages sustained by him resulting from the refusal of said to register in the books of said corporation the said five
the corporate existence of the erring firm. body to register the shares of stock in question. The shares of stock in the name of the plaintiff, Henry
defendant filed a demurrer on the ground that the facts Fleischer, as the shareholder or owner thereof, instead of
It should be stressed in this connection that substantial alleged in the complaint did not constitute sufficient cause the original owner, Manuel Gonzalez, with costs against the
compliance with conditions subsequent will suffice to of action, and that the action was not brought against the defendant.
perfect corporate personality. Organization and proper party, which was the Botica Nolasco, Inc. The
commencement of transaction of corporate business demurrer was sustained, and the plaintiff was granted five The defendant appealed from said judgment, and now
are but conditions subsequent and not prerequisites for days to amend his complaint. makes several assignment of error, all of which, in
acquisition of corporate personality. The adoption and substance, raise the question whether or not article 12 of
filing of by-laws is also a condition subsequent. Under On November 15, 1923, the plaintiff filed an amended the by-laws of the corporation is in conflict with the
Section 19 of the Corporation Code, a Corporation complaint against the Botica Nolasco, Inc., alleging that he provisions of the Corporation Law (Act No. 1459).
commences its corporate existence and juridical became the owner of five shares of stock of said
personality and is deemed incorporated from the date corporation, by purchase from their original owner, one There is no controversy as to the facts of the present case.
the Securities and Exchange Commission issues Manuel Gonzalez; that the said shares were fully paid; and They are simple and may be stated as follows:
certificate of incorporation under its official seal. This that the defendant refused to register said shares in his
may be done even before the filing of the by-laws, name in the books of the corporation in spite of repeated That Manuel Gonzalez was the original owner of the five
which under Section 46 of the Corporation Code, must demands to that effect made by him upon said corporation, shares of stock in question, Nos. 16, 17, 18, 19 and 20 of
be adopted "within one month after receipt of official which refusal caused him damages amounting to P500. the Botica Nolasco, Inc.; that on March 11, 1923, he

Page 58 of 160
CORPORATION LAW CASES MAYORDO, M.A
LLB 3

assigned and delivered said five shares to the plaintiff, accionista a cuyo nombre se ha expedido la accion o Section 13, paragraph 7, above-quoted, empowers a
Henry Fleischer, by accomplishing the form of endorsement acciones que se transfieran, o un documento de corporation to make  by-laws, not inconsistent with any
provided on the back thereof, together with other credits, transferencia.  Entendiendose que, ningun accionista existing law, for the transferring of its stock. It follows
in consideration of a large sum of money owed by Gonzalez transferira accion alguna a otra persona sin participar from said provision, that a by-law adopted by a corporation
to Fleischer (Exhibits A, B, B-1, B-2, B-3, B-4); that on antes por escrito al Secretario-Tesorero. En igualdad de relating to transfer of stock should be in harmony with the
March 13, 1923, Dr. Eduardo Miciano, who was the condiciones, la sociedad tendra el derecho de adquirir law on the subject of transfer of stock. The law on this
secretary-treasurer of said corporation, offered to buy from para si la accion o acciones que se traten de transferir. subject is found in section 35 of Act No. 1459 above
Henry Fleischer, on behalf of the corporation, said shares of (Exhibit 2.) quoted. Said section specifically provides that the shares of
stock, at their par value of P100 a share, for P500; that by stock  "are personal property and may be transferred by
virtue of article 12 of the by-laws of Botica Nolasco, Inc., The above-quoted article constitutes a by-law or regulation delivery of the certificate indorsed by the owner, etc." Said
said corporation had the preferential right to buy from adopted by the Botica Nolasco, Inc., governing the transfer section 35 defines the nature, character and transferability
Manuel Gonzalez said shares (Exhibit 2); that the plaintiff of shares of stock of said corporation. The latter part of of shares of stock. Under said section they are personal
refused to sell them to the defendant; that the plaintiff said article creates in favor of the Botica Nolasco, Inc., a property and may be transferred as therein provided. Said
requested Doctor Miciano to register said shares in his preferential right to buy, under the same conditions, the section contemplates no restriction as to whom they may
name; that Doctor Miciano refused to do so, saying that it share or shares of stock of a retiring shareholder. Has said be transferred or sold. It does not suggest that any
would be in contravention of the by-laws of the corporation any power, under the Corporation Law (Act.No. discrimination may be created by the corporation in favor
corporation. 1459), to adopt such by-law? or against a certain purchaser. The holder of shares, as
owner of personal property, is at liberty, under said section,
It also appears from the record that on the 13th day of The particular provisions of the Corporation Law referring to dispose of them in favor of whomsoever he pleases,
March, 1923, two days after the assignment of the shares to to transfer of shares of stock are as follows: without any other limitation in this respect, than the
the plaintiff, Manuel Gonzales made a written statement to general provisions of law. Therefore, a stock corporation in
the Botica Nolasco, Inc., requesting that the five shares of SEC. 13. Every corporation has the power: adopting a by-law governing transfer of shares of stock
stock sold by him to Henry Fleischer be noted transferred xxx     xxx     xxx should take into consideration the specific provisions of
to Fleischer's name. He also acknowledged in said written (7) To make by-laws, not inconsistent with any existing section 35 of Act No. 1459, and said by-law should be made
statement the preferential right of the corporation to buy law, for the fixing or changing of the number of its to harmonize with said provisions. It should not be
said five shares (Exhibit 3). On June 14, 1923, Gonzalez officers and directors within the limits prescribed by inconsistent therewith.
wrote a letter to the Botica Nolasco, withdrawing and law, and for the  transferring of its stock, the
cancelling his written statement of March 13, 1923 (Exhibit administration of its corporate affairs, etc. The by-law now in question was adopted under the power
C), to which letter the Botica Nolasco on June 15, 1923, conferred upon the corporation by section 13, paragraph 7,
replied, declaring that his written statement was in xxx     xxx     xxx above quoted; but in adopting said by-law the corporation
conformity with the by-laws of the corporation; that his SEC. 35. The capital stock of stock corporations shall de has transcended the limits fixed by law in the same
letter of June 14th was of no effect, and that the shares in divided into shares for which certificates signed by the section, and has not taken into consideration the provisions
question had been registered in the name of the Botica president or the vice-president, countersigned by the of section 35 of Act No. 1459.
Nolasco, Inc., (Exhibit X). secretary or clerk and sealed with the seal of the As a general rule, the by-laws of a corporation are valid if
corporation, shall be issued in accordance with the by- they are reasonable and calculated to carry into effect the
As indicated above, the important question raised in this laws.  Shares of stock so issued are personal property objects of the corporation, and are not contradictory to the
appeal is whether or not article 12 of the by-laws of the and may be transferred by delivery of the certificate general policy of the laws of the land. (Supreme
Botica Nolasco, Inc., is in conflict with the provisions of the indorsed by the owner  or his attorney in fact or other Commandery of the Knights of the Golden Rule  vs.
Corporation Law (Act No. 1459). Appellant invoked said person legally authorized to make the transfer.  No Ainsworth, 71 Ala., 436; 46 Am. Rep., 332.)
article as its ground for denying the request of the plaintiff transfer, however, shall be valid, except as between
that the shares in question be registered in his (plaintiff's) the parties, until the transfer is entered and noted On the other hand, it is equally well settled that by-laws of
name, and for claiming that it (Botica Nolasco, Inc.) had upon the books of the corporation so as to show the a corporation must be reasonable and for a corporate
the preferential right to buy said shares from Gonzalez. names of the parties to the transaction, that date of purpose, and always within the charter limits. They must
Appellant now contends that article 12 of the said by-laws the transfer, the number of the certificate, and the always be strictly subordinate to the constitution and the
is in conformity with the provisions of Act No. 1459. Said number of shares transferred. general laws of the land. They must not infringe the policy
article is as follows: of the state, nor be hostile to public welfare. (46 Am. Rep.,
No share of stock against which the corporation holds 332.) They must not disturb vested rights or impair the
ART. 12. Las acciones de la Corporacion pueden ser any unpaid claim shall be transferable on the books of obligation of a contract, take away or abridge the
transferidas a otra persona, pero para que estas the corporation. substantial rights of stockholder or member, affect rights of
transferencias tengan validez legal, deben constar en property or create obligations unknown to the law.
los registros de la Corporacion con el debido endoso del (People's Home Savings Bank  vs. Superior Court, 104 Cal.,

Page 59 of 160
CORPORATION LAW CASES MAYORDO, M.A
LLB 3

649; 43 Am. St. Rep., 147; Ireland vs. Globe Milling Co., 79 It follows from the foregoing that a corporation has no by any contractual restriction of which he had no
Am. St. Rep., 769.) power to prevent or to restrain transfers of its shares, notice. (Brinkerhoff-Farris Trust and Savings Co.  vs.
unless such power is expressly conferred in its charter Home Lumber Co., 118 Mo., 447.)
The validity of the by-law of a corporation is purely a or governing statute. This conclusion follows from the
question of law. (South Florida Railroad Co. vs. Rhodes, 25 further consideration that by-laws or other regulations The assignment of shares of stock in a corporation by
Fla., 40.) restraining such transfers, unless derived from one who has assented to an unauthorized by-law has
authority expressly granted by the legislature, would only the effect of a contract by, and enforceable
The power to enact by-laws restraining the sale and be regarded as impositions in restraint of trade. (10 against, the assignor; the assignee is not bound by such
transfer of stock must be found in the governing Cyc., p. 578.) by-law by virtue of the assignment alone. (Ireland  vs.
statute or the charter.  Restrictions upon the traffic in Globe Milling Co., 21 R.I., 9.)
stock must have their source in legislative enactment, The foregoing authorities go farther than the stand we are
as the corporation itself cannot create such taking on this question. They hold that the power of a A by-law of a corporation which provides that transfers
impediments. By-law are intended merely for the corporation to enact by-laws restraining the sale and of stock shall not be valid unless approved by the board
protection of the corporation, and prescribe regulation transfer of shares, should not only be in harmony with the of directors, while it may be enforced as a reasonable
and not restriction; they are always subject to the law or charter of the corporation, but such power should be regulation for the protection of the corporation against
charter of the corporation. The corporation, in the expressly granted in said law or charter. worthless stockholders, cannot be made available to
absence of such a power, cannot ordinarily inquire into The only restraint imposed by the Corporation Law upon defeat the rights of third persons. (Farmers' and
or pass upon the legality of the transaction by which its transfer of shares is found in section 35 of Act No. 1459, Merchants' Bank of Lineville vs. Wasson, 48 Iowa, 336.)
stock passes from one person to another, nor can it quoted above, as follows: "No transfer, however, shall be
question the consideration upon which a sale is based. valid, except as between the parties, until the transfer is Counsel for defendant incidentally argues in his brief, that
A by-law cannot take away or abridge the substantial entered and noted upon the books of the corporation so as the plaintiff does not have any right of action against the
rights of stockholder.  Under a statute authorizing by- to show the names of the parties to the transaction, the defendant corporation, but against the president and
laws for the transfer of stock, a corporation can do no date of the transfer, the number of the certificate, and the secretary thereof, inasmuch as the signing and registration
more than prescribe a general mode of transfer on the number of shares transferred." This restriction is necessary of shares is incumbent upon said officers pursuant to
corporate books and cannot justify an unreasonable in order that the officers of the corporation may know who section 35 of the Corporation Law. This contention cannot
restriction upon the right of sale. (4 Thompson on are the stockholders, which is essential in conducting be sustained now. The question should have been raised in
Corporations, sec. 4137, p. 674. elections of officers, in calling meeting of stockholders, the lower court. It is too late to raise it now in this appeal.
and for other purposes. but any restriction of the nature of Besides, as stated above, the corporation was made
The right of unrestrained transfer of shares inheres in that imposed in the by-law now in question, is ultra vires, defendant in this action upon the demurrer of the attorney
the very nature of a corporation, and courts will violative of the property rights of shareholders, and in of the original defendant in the lower court, who
carefully scrutinize any attempt to impose restrictions restraint of trade. contended that the Botica Nolasco, Inc., should be made
or limitations upon the right of stockholders to sell and the party defendant in this action. Accordingly, upon order
assign their stock. The right to impose any restraint in And moreover, the by-laws now in question cannot have any of the court, the complaint was amended and the said
this respect must be conferred upon the corporation effect on the appellee. He had no knowledge of such by- corporation was made the party defendant.
either by the governing statute or by the articles of law when the shares were assigned to him. He obtained Whenever a corporation refuses to transfer and register
the corporation. It cannot be done by a by-law without them in good faith and for a valuable consideration. He was stock in cases like the present, mandamus will lie to
statutory or charter authority. (4 Thompson on not a privy to the contract created by said by-law between compel the officers of the corporation to transfer said stock
Corporations, sec. 4334, pp. 818, 819.) the shareholder Manuel Gonzalez and the Botica Nolasco, upon the books of the corporation. (26 Cyc. 347; Hager vs.
Inc. Said by-law cannot operate to defeat his rights as a Bryan, 19 Phil., 138.)
The jus disponendi, being an incident of the ownership purchaser.
of property, the general rule (subject to exceptions In view of all the foregoing, we are of the opinion, and so
hereafter pointed out and discussed) is that  every An unauthorized by-law forbidding a shareholder to sell hold, that the decision of the lower court is in accordance
owner of corporate shares has the same uncontrollable his shares without first offering them to the corporation with law and should be and is hereby affirmed, with costs.
right to alien them which attaches to the ownership of for a period of thirty days is not binding upon an So ordered.
any other species of property. A shareholder is under assignee of the stock as a personal contract, although
no obligation to refrain from selling his shares at the his assignor knew of the by-law and took part in its G.R. No. L-26649             July 13, 1927
sacrifice of his personal interest, in order to secure the adoption. (10 Cyc., 579; Ireland  vs. Globe Milling Co., THE GOVERNMENT OF THE PHILIPPINE ISLANDS (on
welfare of the corporation, or to enable another 21 R.I., 9.) relation of the Attorney-General), plaintiff,  vs. EL HOGAR
shareholder to make gains and profits. (10 Cyc., p. FILIPINO, defendant.
577.) When no restriction is placed by public law on the
transfer of corporate stock, a purchaser is not affected STREET, J.:

Page 60 of 160
CORPORATION LAW CASES MAYORDO, M.A
LLB 3

This is a  quo warranto  proceeding instituted originally in enactment by amending its articles so as to provide that The foregoing document was received in this office at
this court by the Government of the Philippine Islands on the capital should be in an amount not exceeding the then 4.10 p. m., December 28, 1920, according to entry
the relation of the Attorney-General against the building lawful limit. From the time of its first organization the 1898, page 50 of Book One of the Day Book and
and loan association known as El Hogar Filipino, for the number of shareholders has constantly increased, with the registered on the back of certificate of title No. 2211
purpose of depriving it of its corporate franchise, excluding result that on December 31, 1925, the association had and its duplicate, folio 193 of Book A-10 of the register
it from all corporate rights and privileges, and effecting a 5,826 shareholders holding 125,750 shares, with a total of original certificate. Tarlac, Tarlac, January 12, 1921.
final dissolution of said corporation. The complaint paid-up value of P8,703,602.25. During the period of its (Sgd.) SILVINO LOPEZ DE JESUS, Register of Deeds.
enumerates seventeen distinct causes of action, to all of existence prior to the date last above-mentioned the
which the defendant has answered upon the merits, first association paid to withdrawing stockholders the amount of For months no reply was received by El Hogar Filipino from
admitting the averments of the first paragraph in the P7,618,257,.72; and in the same period it distributed in the the register of deeds of Tarlac, and letters were written to
statement of the first cause of action, wherein it is alleged form of dividends among its stockholders the sum of him by El Hogar Filipino on the subject in March and April,
that the defendant was organized in the year 1911 as a P7,621,565.81. 1921, requesting action. No answer having been received to
building and loan association under the laws of the these letters, a complaint was made by El Hogar Filipino to
Philippine Islands, and that, since its organization, the First cause of action. — The first cause of action is based the Chief of the General Land Registration Office; and on
corporation has been doing business in the Philippine upon the alleged illegal holding by the respondent of the May 7, 1921, the certificate of title to the San Clemente
Islands, with its principal office in the City of Manila. Other title to real property for a period in excess of five years land was received by El Hogar Filipino from the register of
facts alleged in the various causes of action in the after the property had been bought in by the respondent at deeds of Tarlac.
complaint are either denied in the answer or controverted one of its own foreclosure sales. The provision of law
in legal effect by other facts. relevant to the matter is found in section 75 of Act of On March 10, 1921, the board of directors of El Hogar
Congress of July 1, 1902 (repeated in subsection 5 of Filipino adopted a resolution authorizing Vicente Bengzon,
After issue had been thus joined upon the merits, the section 13 of the Corporation Law.) In both of these an agent of the corporation, to endeavor to find a buyer for
attorneys entered into an elaborate agreement as to the provisions it is in substance declared that while the San Clemente land. On July 27, 1921, El Hogar Filipino
fact, thereby removing from the field of dispute such corporations may loan funds upon real estate security and authorized one Jose Laguardia to endeavor to find a
matters of fact as are necessary to the solution of the purchase real estate when necessary for the collection of purchaser for the San Clemente land for the sum of P23,000
controversy. It follows that we are here confronted only loans, they shall dispose of real estate so obtained within undertaking to pay the said Laguardia a commission of 5
with the legal questions arising upon the agreed statement. five years after receiving the title. per centum of the selling price for his services, but no
offers to purchase were obtained through this agent or
On March 1, 1906, the Philippine Commission enacted what In this connection it appears that in the year 1920 El Hogar through the agent Bengzon. In July, 1923, plans of the San
is known as the Corporation Law (Act No. 1459) effective Filipino was the holder of a recorded mortgage upon a tract Clemente land were sent to Mr. Luis Gomez, Mr. J. Gonzalez
upon April 1 of the same year. Section 171 to 190, inclusive, of land in the municipality of San Clemente, Province of and Mr. Alfonso de Castelvi, as prospective purchasers, but
of this Act are devoted to the subject of building and loan Tarlac, as security for a loan of P24,000 to the shareholders no offers were received from them. In January, 1926, the
associations, defining their objects making various of El Hogar Filipino who were the owners of said property. agent not having succeeded in finding a buyer, the San
provisions governing their organization and administration, The borrowers having defaulted in their payments, El Hogar Clemente land was advertised for sale by El Hogar Filipino
and providing for the supervision to be exercised over Filipino foreclosed the mortgage and purchased the land at in  El Debate, La Vanguardia  and  Taliba, three newspapers
them. These provisions appear to be adopted from the foreclosure sale for the net amount of the of general circulation in the Philippine Islands published in
American statutes governing building and loan associations indebtedness, namely, the sum of P23,744.18. The auction the City of Manila. On March 16, 1926, the first offer for
and they of course reflect the ideals and principles found in sale of the mortgaged property took place November 18, the purchase of the San Clemente land was received by El
American law relative to such associations. The respondent, 1920, and the deed conveying the property to El Hogar Hogar Filipino. This offer was made to it in writing by one
El Hogar Filipino, was apparently the first corporation Filipino was executed and delivered December 22, 1920. On Alcantara, who offered to buy it for the sum of P4,000,
organized in the Philippine Islands under the provisions December 27, 1920, the deed conveying the property to El Philippine currency, payable P500 in cash, and the
cited, and the association has been favored with Hogar Filipino was sent to the register of deeds of the remainder within thirty days. Alcantara's offer having been
extraordinary success. The articles of incorporation bear Province of Tarlac, with the request that the certificate of reported by the manager of El Hogar Filipino to its board of
the date of December 28, 1910, at which time capital stock title then standing in the name of the former owners be directors, it was decided, by a resolution adopted at a
in the association had been subscribed to the amount of cancelled and that a new certificate of title be issued in meeting of the board held on March 25, 1926, to accept the
P150,000 of which the sum of P10,620 had been paid in. the name of El Hogar Filipino. Said deed was received in offer, and this acceptance was communicated to the
Under the law as it then stood, the capital of the the office of the register of deeds of Tarlac on December prospective buyer. Alcantara was given successive
Association was not permitted to exceed P3,000,000, but 28, 1920, together with the old certificate of title, and extensions of the time, the last of which expired April 30,
by Act No. 2092, passed December 23, 1911, the statute thereupon the register made upon the said deed the 1926, within which to make the payment agreed upon; and
was so amended as to permit the capitalization of building following annotation: upon his failure to do so El Hogar Filipino treated the
and loan associations to the amount of ten millions. Soon contract with him as rescinded, and efforts were made at
thereafter the association took advantage of this once to find another buyer. Finally the land was sold to

Page 61 of 160
CORPORATION LAW CASES MAYORDO, M.A
LLB 3

Doña Felipa Alberto for P6,000 by a public instrument the property to Alcantara, prior to the rescission of the that it be outset from the continuance of such offense
executed before a notary public at Manila, P. I., on July 30, contract by reason of Alcantara's failure to make the or the exercise of such power.
1926. stipulated first payment. Upon this point the contention of
the respondent is, in our opinion, well founded. The This provision clearly shows that the court has a discretion
Upon consideration of the facts above set forth it is evident acceptance by it of Alcantara's offer obligated the with respect to the infliction of capital punishment upon
that the strict letter of the law was violated by the respondent to Alcantara; and if it had not been for the corporation and that there are certain misdemeanors and
respondent; but it is equally obvious that its conduct has default of Alcantara, the effective sale of the property misuses of franchises which should not be recognized as
not been characterized by obduracy or pertinacity in would have resulted. The respondent was not at all requiring their dissolution. In Government of the Philippine
contempt of the law. Moreover, several facts connected chargeable with the collapse of these negotiations; and Islands vs. Philippine Sugar Estates Development Co.(38
with the incident tend to mitigate the offense. The hence in any equitable application of the law this period Phil., 15), it was found that the offending corporation had
Attorney-General points out that the respondent acquired should be deducted from the five-year period within which been largely (though indirectly) engaged in the buying and
title on December 22, 1920, when the deed was executed the respondent ought to have made the sale. Another holding or real property for speculative purposes in
and delivered, by which the property was conveyed to it as circumstance explanatory of the respondent's delay in contravention of its charter and contrary to the express
purchaser at its foreclosure sale, and this title remained in selling the property is found in the fact that it purchased provisions of law. Moreover, in that case the offending
it until July 30, 1926, when the property was finally sold to the property for the full amount of the indebtedness due to corporation was found to be still interested in the
Felipa Alberto. The interval between these two it from the former owner, which was nearly P24,000. It was properties so purchased for speculative at the time the
conveyances is thus more than five years; and it is subsequently found that the property was not salable for action was brought. Nevertheless, instead of making an
contended that the five year period did not begin to run anything like that amount and in the end it had to be sold absolute and unconditional order for the dissolution of the
against the respondent until May 7, 1921, when the register for P6,000, notwithstanding energetic efforts on the part of corporation, the judgment of ouster was made conditional
of deeds of Tarlac delivered the new certificate of title to the respondent to find a purchaser upon better terms. upon the failure of the corporation to discontinue its
the respondent pursuant to the deed by which the property unlawful conduct within six months after final decision. In
was acquired. As an equitable consideration affecting the The question then arises whether the failure of the the case before us the respondent appears to have rid itself
case this contention, though not decisive, is in our opinion respondent to get rid of the San Clemente property within of the San Clemente property many months prior to the
more than respectable. It has been held by this court that a five years after it first acquired the deed thereto, even institution of this action. It is evident from this that the
purchaser of land registered under the Torrens system supposing the five-year period to be properly counted from dissolution of the respondent would not be an appropriate
cannot acquire the status of an innocent purchaser for that date, is such a violation of law as should work a remedy in this case. We do not of course undertake to say
value unless his vendor is able to place in his hands an forfeiture of its franchise and require a judgment to be that a corporation might not be dissolved for offenses of
owner's duplicate showing the title of such land to be in the entered for its dissolution in this action of  quo warranto. this nature perpetrated in the past, especially if its conduct
vendor (Director of Lands  vs.  Addison, 49, Phil., 19; Upon this point we do not hesitate to say that in our had exhibited a willful obduracy and contempt of law. We
Rodriguez  vs.  Llorente, G. R. No. 266151). It results that opinion the corporation has not been shown to have content ourselves with holding that upon the facts here
prior to May 7, 1921, El Hogar Filipino was not really in a offended against the law in a manner that should entail a before us the penalty of dissolution would be excessively
position to pass an indefeasible title to any purchaser. In forfeiture of its charter. Certainly no court with any severe and fraught with consequences altogether
this connection it will be noted that section 75 of the Act of discretion to use in the matter would visit upon the disproportionate to the offense committed.
Congress of July 1, 1902, and the similar provision in respondent and its thousands of shareholders the extreme
section 13 of the Corporation Law, allow the corporation penalty of the law as a consequence of the delinquency The evident purpose behind the law restricting the rights of
"five years after receiving the title," within which to here shown to have been committed. corporations with respect to the tenure of land was to
dispose of the property. A fair interpretation of these prevent the revival of the entail (mayorazgo) or other
provisions would seem to indicate that the date of the The law applicable to the case is in our opinion found in similar institution by which land could be fettered and its
receiving of the title in this case was the date when the section 212 of the Code of Civil Procedure, as applied by alienation hampered over long periods of time. In the case
respondent received the owner's certificate, or May 7, this court in  Government of the Philippine Islands vs. before us the respondent corporation has in good faith
1921, for it was only after that date that the respondent Philippine Sugar Estates Development Co.  (38 Phil., 15). disposed of the piece of property which appears to have
had an unequivocal and unquestionable power to pass a This section (212), in prescribing the judgment to be been in its hands at the expiration of the period fixed by
complete title. The failure of the respondent to receive the rendered against a corporation in an action of  quo law, and a fair explanation is given of its failure to dispose
certificate sooner was not due in any wise to its fault, but warranto, among other things says: of it sooner. Under these circumstances the destruction of
to unexplained delay on the part of the register of deeds. the corporation would bring irreparable loss upon the
For this delay the respondent cannot be held accountable. . . . When it is found and adjudged that a corporation thousand of innocent shareholders of the corporation
has offended in any matter or manner which does not without any corresponding benefit to the public. The
Again, it is urged for the respondent that the period by law work as a surrender or forfeiture, or has misused discretion permitted to this court in the application of the
between March 25, 1926, and April 30, 1926, should not be a franchise or exercised a power not conferred by law, remedy of  quo warranto  forbids so radical a use of the
counted as part of the five-year period. This was the period but not of such a character as to work a surrender or remedy.
during which the respondent was under obligation to sell forfeiture of its franchise, judgment shall be rendered

Page 62 of 160
CORPORATION LAW CASES MAYORDO, M.A
LLB 3

But the case for the plaintiff supposes that the discretion When the statute says, "If the violation is committed by a upon such corporations, such courts, it exercising
of this court in matters like that now before us has been corporation, the same shall, upon such violation being equitable power,  shall, by injunction, at suit of the
expressly taken away by the third section of Act No. 2792, proved, be dissolved by  quo warranto  proceedings . . .," private parties or other corporations, restrain such
and that the dissolution of the corporation is obligatory the intention was to indicate that the remedy against the injurious acts.
upon the court a mere finding that the respondent has corporation shall be by action of quo warranto. There was
violated the provision of the Corporation Law in any no intention to define the principles governing said remedy, In an action based on this statute the plaintiff claimed
respect. This makes necessary to examine the Act last and it must be understood that in applying the remedy the injunctive relief as a matter of right. But this was denied
above-mentioned with some care. Upon referring thereto, court is still controlled by the principles established in the court saying:
we find that it consists of three sections under the immemorial jurisprudence. The interpretation placed upon
following style: this language in the brief of the Attorney-General would be Notwithstanding, therefore, the use of the imperative
dangerous in the extreme, since it would actually place the "shall" the injunction is not to be granted unless a
No. 2792. — An Act to amend certain sections of the life of all corporate investments in the official. No proper case for injunction be made out, in accordance
Corporation Law, Act Numbered Fourteen hundred and corporate enterprise of any moment can be conducted with the principles and practice of equity. The word
fifty-nine, providing for the publication of the assets perpetually without some trivial misdemeanor against "shall" when used by the legislature to a court, is
and liabilities of corporations registering in the Bureau corporate law being committed by some one or other of its usually a grant of authority and means "may", and even
of Commerce and Industry, determining the liability of numerous employees. As illustrations of the preposterous if it be intended to be mandatory it must be subject to
the officers of corporations with regard to the issuance effects of the provision, in the sense contended for by the the necessary limitation that a proper case has been
of stock or bonus, establishing penalties for certain Attorney-General, the attorneys for the respondent have made out for the exercise of the power.
things, and for other purposes. called attention to the fact that under section 52 of the
Corporation Law, a business corporation is required to keep Other authorities amply sustain this view
The first two section contain amendments to the a stock book and a transfer book in which the names of (People  vs.  Nusebaum, 66 N. Y. Supp., 129, 133; West
Corporation Law with respect to matters with which we are stockholders shall kept in alphabetical order. Again, under Wisconsin R. Co. vs. Foley, 94 U. S., 100, 103; 24 Law. Ed.,
not here concurred. The third section contains anew section 94, railroad corporations are required to cause all 71; Clancy  vs.  McElroy, 30 Wash., 567; 70 Pac., 1095;
enactment to be inserted as section 190 (A) in the employees working on passenger trains or at a station for State vs. West, 3 Ohio State, 509, 511; In re Lent, 40 N. Y.
corporation Law immediately following section 190. This passengers to wear a badge on his cap or hat which will Supp., 570, 572; 16 Misc. Rep., 606; Ludlow  vs.  Ludlow's
new section reads as follows: indicate his office. Can it be supposed that the Legislature Executors, 4 N. J. Law [1 Sothard], 387, 394;
intended to penalize the violation of such provisions as Whipple  vs.  Eddy, 161 Ill., 114;43 N. E., 789, 790;
SEC. 190. (A). Penalties. — The violation of any of the these by dissolution of the corporation involved? Evidently Borkheim  vs.  Fireman's Fund Ins. Co., 38 Cal., 505, 506;
provisions of this Act and its amendments not otherwise such could not have been the intention; and the only way Beasley  vs.  People, 89 Ill., 571, 575; Donnelly  vs.  Smith,
penalized therein, shall be punished by a fine of not to avoid the consequence suggested is to hold, as we now 128 Iowa, 257; 103 N. W., 776).
more than one thousand pesos, or by imprisonment for hold, that the provision now under consideration has not
not more than five years, or both, in the discretion of impaired the discretion of this court in applying the writ But section 3 of Act No. 2792 is challenged by the
the court. If the violation being proved, be dissolved of quo warranto. respondent on the ground that the subject-matter of this
by  quo warranto  proceedings instituted by the section is not expressed in the title of the Act, with the
Attorney-General or by any provincial fiscal, by order of Another way to put the same conclusion is to say that the result that the section is invalid. This criticism is in our
said Attorney-General: Provided, That nothing in this expression "shall be dissolved by quo warrantoproceedings" opinion well founded. Section 3 of our organic law (Jones
section provided shall be construed to repeal the other m e a n s i n e f f e c t , " m a y b e d i s s o l v e d b y  q u o Bill) declares, among other things, that "No bill which may
causes for the dissolution of corporation prescribed by warranto  proceedings in the discretion of the court." The be enacted into law shall embrace more than one subject,
existing law, and the remedy provided for in this proposition that the word "shall" may be construed as and that subject shall be expressed in the title of the bill."
section shall be considered as additional to the "may", when addressed by the Legislature to the courts, is Any law or part of a law passed by the Philippine
remedies already existing. well supported in jurisprudence. In the case of  Becker vs. Legislature since this provision went into effect and
Lebanon and M. St. Ry. Co.,  (188 Pa., 484), the Supreme offending against its requirement is necessarily void.
The contention for the plaintiff is to the effect that the Court of Pennsylvania had under consideration a statute
second sentence in this enactment has entirely abrogated providing as follows: Upon examining the entire Act (No. 2792), we find that it is
the discretion of this court with respect to the application directed to three ends which are successively dealt with in
of the remedy of qou warranto, as expressed in section 212 It shall be the duty of the court . . . to examine, the first three sections of the Act. But it will be noted that
of the Code of Civil Procedure, and that it is now inquire and ascertain whether such corporation does in these three matters all relate to the Corporation Law; and
mandatory upon us to dissolved any corporation whenever fact posses the right or franchise to do the act from it is at once apparent that they might properly have been
we find that it has committed any violation of the which such alleged injury to private rights or to the embodied in a single Act if a title of sufficient unity and
Corporation Law, however trivial. In our opinion in this rights and franchises of other corporations results; and generality had been prefixed thereto. Furthermore, it is
radical view of the meaning of the enactment is untenable. if such rights or franchises have not been conferred obvious, even upon casual inspection, that the subject-

Page 63 of 160
CORPORATION LAW CASES MAYORDO, M.A
LLB 3

matter of each of the first two sections is expressed and by the simple device of putting into the title of an act purposes") is frequently found in the title of acts adopted
defined with sufficient precision in the title. With respect words which denote a subject "broad" enough to cover by the Philippine Legislature; and its presence in our laws is
to the subject-matter of section 3 the only words in the everything. Under that view, the title, "An act due to the adoption by our Legislature of the style used in
title which can be taken to refer to the subject-matter of concerning the laws of the state," would be good, and Congression allegation. But it must be remembered that
said section are these, "An Act . . . establishing penalties the convention and people who framed and adopted the legislation of Congress is subject to no constitutional
for certain things, and for other purposes." These words the constitution would be convicted of the folly of restriction with respect to the title of bills. Consequently,
undoubtedly have sufficient generality to cover the elaborately constructing a grave constitutional in Congressional legislation the words "and for other
subject-matter of section 3 of the Act. But this is not limitation of legislative power upon a most important purposes" at least serve the purpose of admonishing the
enough. The Jones Law requires that the subject-matter of subject, which the legislature could at once circumvent public that the bill whose heading contains these words
the bill "shall be expressed in the title of the bill." by a mere verbal trick. The word "subject" is used in contains legislation upon other subjects than that
the constitution embrace but "one subject" it expressed in the title. Now, so long as the Philippine
When reference is had to the expression "establishing necessarily implies — what everybody knows — that Legislature was subject to no restriction with respect to
penalties for certain things," it is obvious that these words there are numerous subjects of the legislation, and the title of bills intended for enactment into general laws,
express nothing. The constitutional provision was declares that only one of these subjects shall embraced the expression "for other purposes" could be appropriately
undoubtedly adopted in order that the public might be in any one act. All subjects cannot be conjured into one used in titles, not precisely for the purpose of conveying
informed as to what the Legislature is about while bills are subject by the mere magic of a word in a title. information as to the matter legislated upon, but for the
in process of passage. The expression "establishing purpose ad admonishing the public that any bill containing
penalties for certain things" would give no definite In Rader vs. Township of Union (39 N. J. L., 509, 515), the such words in the title might contain other subjects than
information to anybody as to the project of legislation Supreme Court of New Jersey made the following that expressed in the definitive part of the title. But, when
intended under this expression. An examination of the observation: congress adopted the Jones Law, the restriction with which
decided cases shows that courts have always been we are now dealing became effective here and the words
indulgent of the practices of the Legislature with respect to * * * It is true, that it may be difficult to indicate, by a "for other purposes" could no longer be appropriately used
the form and generality of title, for if extreme refinements formula, how specialized the title of a statute must be; in the title of legislative bills. Nevertheless, the custom of
were indulged by the courts, the work of legislation would but it is not difficult to conclude that it must mean using these words has still been followed, although they
be unnecessarily hampered. But, as has been observed by something in the way of being a notice of what is doing. can no longer serve to cover matter not germane to the bill
the California court, there must be some reasonable limit Unless it does not enough that it embraces the in the title of which they are used. But the futility of
to the generality of titles that will be allowed. The legislative purpose — it must express it; and where the adding these words to the style of any act is now obvious
measure of legality is whether the title is sufficient to give language is too general, it will accomplish the former, (Cooley, Const. Lims., 8th ed., p. 302)
notice of the general subject of the proposed legislation to but not the latter. Thus, a law entitled "An act for a
the persons and interests likely to be affected. certain purpose," would embrace any subject, but In the brief for the plaintiff it is intimated that the
would express none, and, consequently, it would not constitutional restriction which we have been discussing is
In Lewis vs. Dunne (134 Cal., 291), the court had before it a stand the constitutional test. more or less of a dead letter in this jurisdiction; and it
statute entitled "An Act to revise the Code of Civil seems to be taken for granted that no court would ever
Procedure of the State of California, by amending certain The doctrine properly applicable in matters of this kind is, presume to hold a legislative act or part of a legislative act
sections, repealing others, and adding certain new we think, fairly summed up in a current repository of invalid for non-compliance with the requirement. This is a
sections." This title was held to embrace more than one jurisprudence in the following language: mistake; and no utterance of this court can be cited as
subject, which were not sufficiently expressed in the title. giving currency to any such notion. On the contrary the
In discussing the question the court said: * * * While it may be difficult to formulate a rule by discussion contained in Central Capiz vs. Ramirez (40 Phil.,
which to determine the extent to which the title of a 883), shows that when a case arises where a violation of
* * * It is apparent that the language of the title of the bill must specialize its object, it may be safely assumed the restriction is apparent, the court has no alternative but
act in question, in and of itself, express no subject that the title must not only embrace the subject of to declare the legislation affected thereby to be invalid.
whatever. No one could tell from the title alone what proposed legislation, but also express it clearly and
subject of legislation was dealt with in the body of the fully enough to give notice of the legislative purpose. Second cause of action. — The second cause of action is
act; such subject so far as the title of the act informs (25 R. C. L., p. 853.) based upon a charge that the respondent is owning and
us, might have been entirely different from anything to holding a business lot, with the structure thereon, in the
be found in the act itself. In dealing with the problem now before us the words "and financial district of the City of Manila is excess of its
for other purposes "found at the end of the caption of Act reasonable requirements and in contravention of subsection
We cannot agree with the contention of some of No. 2792, must be laid completely out of consideration. 5 of section 13 of the corporation Law. The facts on which
respondent's counsel — apparently to some extent They express nothing, and amount to nothing as a this charge is based appear to be these:
countenanced by a few authorities — that the provision compliance with the constitutional requirement to which
of the constitution in question can be entirely avoided attention has been directed. This expression "(for other

Page 64 of 160
CORPORATION LAW CASES MAYORDO, M.A
LLB 3

On August 28, 1913, the respondent purchased 1,413 square corporation has the power to purchase, hold and lease such
meters of land at the corner of Juan Luna Street and the real property as the transaction of the lawful business of In Rector vs. Hartford Deposit Co., a question was raised as
Muelle de la Industria, in the City of Manila, immediately the corporation may reasonably and necessarily require. to the power of the Deposit Company to erect and own a
adjacent to the building then occupied by the Hongkong When this property was acquired in 1916, the business of El fourteen-story building — containing eight storerooms, one
and Shanghai Banking Corporation. At the time the Hogar Filipino had developed to such an extent, and its hundred suites of offices, and one safety deposit vault,
respondent acquired this lot there stood upon it a building, prospects for the future were such as to justify its directors under a statute authorizing the corporation to possess so
then nearly fifty years old, which was occupied in part by in acquiring a lot in the financial district of the City of much real estate "as shall be necessary for the transaction
the offices of an importing firm and in part by warehouses Manila and in constructing thereon a suitable building as of their business." The court said:
of the same firm. The material used in the construction was the site of its offices; and it cannot be fairly said that the
Guadalupe stone and hewn timber, and the building area of the lot — 1,413 square meters — was in excess of its That the appellee company possessed ample power to
contained none of the facilities usually found in a modern reasonable requirements. The law expressly declares that acquire real property and construct a building thereon
office building. corporations may acquire such real estate as is reasonably for the purpose of transacting therein the legitimate
necessary to enable them to carry out the purposes for business of the corporation is beyond the range of
In purchase of a design which had been formed prior to the which they were created; and we are of the opinion that debate. Nor is the contrary contended, but the
purchase of the property, the directors of the El Hogar the owning of a business lot upon which to construct and insistence is that, under the guise of erecting a building
Filipino caused the old building to be demolished; and they maintain its offices is reasonably necessary to a building for corporate purposes, the appellee company
erected thereon a modern reinforced concrete office and loan association such as the respondent was at the purposely constructed a much larger building than its
building. As at first constructed the new building was three time this property was acquired. A different ruling on this business required, containing many rooms intended to
stories high in the main, but in 1920, in order to obtain point would compel important enterprises to conduct their be rented to others for offices and business purposes, —
greater advantage from the use of the land, an additional business exclusively in leased offices — a result which could among them, the basement rooms contracted to be
story was added to the building, making a structure of four serve no useful end but would retard industrial growth and leased to the appellant, — and that in so doing it
stories except in one corner where an additional story was be inimical to the best interests of society. designedly exceeded its corporate powers. The position
place, making it five stories high over an area of 117.52 off appellant therefore is that the appellee corporation
square meters. It is admitted in the plaintiffs brief that this We are furthermore of the opinion that, inasmuch as the has flagrantly abused its general power to acquire real
"noble and imposing structure" — to use the words of the lot referred to was lawfully acquired by the respondent, it estate and construct a building thereon . . . It was
Attorney-General — "has greatly improved the aspect of the is entitled to the full beneficial use thereof. No legitimate within the general scope of the express powers of the
banking and commercial district of Manila and has greatly principle can discovered which would deny to one owner appellee corporation to own and possess a building
contributed to the movement and campaign for the Manila the right to enjoy his (or its) property to the same extent necessary for its proper corporate purposes. In planning
Beautiful." It is also admitted that the competed building is that is conceded to any other owner; and an intention to and constructing such a building, as was said in People
reasonably proportionate in value and revenue producing discriminate between owners in this respect is not lightly to vs. Pullman's Palace Car Co., supra, the corporation
capacity to the value of the land upon which it stands. The be imputed to the Legislature. The point here involved has should not necessarily be restricted to a building
total outlay of the respondent for the land and the been the subject of consideration in many decisions of containing the precise number of rooms its then
improvements thereon was P690,000 and at this valuation American courts under statutes even more restrictive than business might require, and no more, but that the
the property is carried on the books of the company, while that which prevails in this jurisdiction; and the conclusion future probable growth and volume of its business
the assessed valuation of the land and improvements is at has uniformly been that a corporations whose business may might be considered and anticipated, and a larger
P786,478. properly be conducted in a populous center may acquire an building, and one containing more rooms than the
appropriate lot and construct thereon an edifice with present volume of business required be erected, and
Since the new building was completed the respondent has facilities in excess of its own immediate requirements. the rooms not needed might be rented by the
used about 324 square meters of floor space for its own corporation, — provided, of course, such course should
offices and has rented the remainder of the office space in Thus in  People vs. Pullman's Palace-Car Co.  (175 Ill., 125; be taken in good faith, and not as a mere evasion of
said building, consisting of about 3,175 square meters, to 64 L. R. A., 366), it appeared that the respondent the public law and the policy of the state relative to
other persons and entities. In the second cause of action of corporation owned and controlled a large ten-story business the ownership of real estate by corporations. In such
the complaint it is supposed that the acquisition of this lot, block in the City of Chicago, worth $2,000,000, and that it state of case the question is whether the corporation
the construction of the new office building thereon, and occupied only about one-fourth thereof for its own has abused or excessively and unjustifiably used the
the subsequent renting of the same in great part to third purposes, leasing the remainder to others at heavy rentals. power and authority granted it by the state to
persons, are ultra vires acts on the part of the corporation, The corporate charter merely permitted the holding of such construct buildings and own real estate necessary for
and that the proper penalty to be enforced against it in this real estate by the respondent as might be necessary for the its corporate purposes.
action is that if dissolution. successful prosecution of its business. An attempt was
made to obtain the dissolution of the corporation in a  quo In Home savings building  Association vs. Driver  (129 Ky.,
With this contention we are unable to agree. Under warranto proceeding similar to that now before us, but the 754), one of the questions before the court was precisely
subsection 5 of section 13 of the Corporation Law, every remedy was denied.

Page 65 of 160
CORPORATION LAW CASES MAYORDO, M.A
LLB 3

the same as that now before us. Upon this the Supreme are few of such institutions that do not, at times, rent making improvements in the bank building and inquiring
Court of Kentucky said: out or lease the unneeded portions of the building if there is anything in the national banking laws
occupied by them as homes. We do not think that in so prohibiting the construction of a building which will
The third question is, has the association the right to doing they are violating any provisions of the law, but contain floors for offices to be rented out by the bank
erect, remodel, or own a building of more than that the renting out of the unused or unoccupied as well as the banking room. Your attention is called to
sufficient capacity to accommodate its own business portions of their buildings is but an incident in the the case of Brown vs. Schleier, 118 Fed., 981 [55 C. C.
and to rent out the excess? There is nothing in the conduct of their business. A, 475], in which the court held that: 'If the land which
Constitution, charter of the association, or statutes a national bank purchases or leases for the
placing any limitation upon the character of a building In Wingert vs. First National Bank of Hagerstown, Md. (175 accommodation of its business is very valuable it may
which a corporation may erect as a home in which to Fed., 739, 741), a stockholder sought to enjoin the bank exercise the same rights that belong to other
conduct its business. A corporation conducting a from building a six-story building owned by the bank in the landowners of improving it in a way that will yield the
business of the character of that in which appellant is commercial district of Hagerstown of which only the first largest income, lessen its own rent, and render that
engaged naturally expects its business to grow and story was to be used by the bank, the remaining stories to part of its funds which are invested in realty most
expand from time to time, and, in building a home it be rented out for offices and places of business, on the productive.'" This seems to be the common sense
would be exercising but a short-sighted judgment if it theory that such action was  ultra vires  and in violation of interpretation of the act of Congress and is the one
did not make provision for the future by building a the provisions of the national banking act confining such which prevails.'
home large enough to take care of its expanding corporations to the holding, only, of such real estate "as
business, and hence, even if it should build a house shall be necessary for its immediate accommodation in the It would seem to be unnecessary to extend the opinion by
larger and roomier than its present needs or interests transaction of its business." lengthy citations upon the point under consideration,
require, it would be acting clearly with the exercise of butBrown vs. Schleier  (118 Fed., 981), may be cited as
its corporate right and power. The limitation which the The injunction was denied, the court adopting the opinion being in harmony with the foregoing authorities. In dealing
statute imposes is that proper conduct of its business, of the lower court in which the following was said: with the powers of a national bank the court, in this case,
but it does not attempt to place any restriction or said:
limitation upon the right of the corporation or 'The other ground urged by the complainant is that the
association as to the character of building it shall erect proposed action is violative of the restriction which When an occasion arises for an investment in real
on said real estate; and, while the Constitution and the permits a national bank to hold only such real estate as property for either of the purposes specified in the
statutes provide that no corporation shall engage in any shall be necessary for its immediate accommodation in statute the national bank act permits banking
business other than that expressly authorized by its the transaction of its business, and that, therefore, the associations to act as any prudent person would act in
charter, we are of opinion that, in renting out the erection of a building which will contain offices not making an investment in real estate, and to exercise
unoccupied and unused portions of the building so necessary for the business of the bank is not permitted the same measure of judgment and discretion. The act
erected, the association could not be said to engaged in by the law, although that method of improving the lot ought not to be construed in such as way as to compel a
any other business than that authorized by its charter. may be the most beneficial use that can be made of it. national bank, when it acquires real property for a
The renting of the unused portions of the building is a It is matter of common knowledge that the actual legitimate purpose, to deal with it otherwise than a
mere incident in the conduct of its real business. We practice of national banks is to the contrary. Where prudent land owner would ordinarily deal with such
would not say that a building association might embark ground is valuable, it may probably be truly said that property.
in the business of building houses and renting or leasing the majority of national bank buildings are built with
them, but there is quite a difference in building or accommodations in excess of the needs of the bank for In the brief of the Attorney-General reliance is place almost
renting a house in which to conduct its own business the purpose of lessening the bank's expense by renting entirely upon two Illinois cases, namely  Africani Home
and leasing the unused portion thereof for the time out the unused portion. If that were not allowable, Purchase and Loan Association vs. Carroll  (267 Ill., 380),
being, or until such time as they may be needed by the many smaller banks in cities would be driven to become and  First Methodist Episcopal Church of Chicago vs.
association, and in building houses for the purpose of tenants as the great cost of the lot would be prohibitive Dixon  (178 Ill., 260). In our opinion these cases are either
renting or leasing them. The one might properly be said of using it exclusively for the banking accommodation distinguishable from that now before us, or they reflect a
to be the proper exercise of a power incident to the of a single bank. As indicative of the interpretation of view of the law which is incorrect. At any rate the weight
conduct of its legitimate business, whereas the other the law commonly received and acted upon, reference of judicial opinion is so overwhelmingly in favor of
would be a clear violation of that provision of the may be made to the reply of the Comptroller of the sustaining the validity of the acts alleged in the second
statute which denies to any corporation the right to Currency to the injury by the bank in this case asking cause of action to have been done by the respondent in
conduct any business other than that authorized by its whether the law forbids the bank constructing such a excess of its powers that we refrain from commenting at
charter. To hold otherwise would be to charge most of building as was contemplated. any length upon said cases. The ground stated in the
the banking institutions, trust companies and other second cause of action is in our opinion without merit.
corporations, such as title guaranty companies, etc., 'The reply was follows: "Your letter of the 9th instant
doing with violating the law; for it is known that there received, stating that the directors contemplate

Page 66 of 160
CORPORATION LAW CASES MAYORDO, M.A
LLB 3

Third cause of action. — Under the third cause of action cause of action is therefore without merit, as was true of 21, 1925, wherein the practice in question was criticized.
the respondent is charged with engaging in activities the first. The administration of property in the manner described is
foreign to the purposes for which the corporation was more befitting to the business of a real estate agent or
created and not reasonable necessary to its legitimate The third specification under this cause of action relates to trust company than to the business of a building and loan
ends. The specifications under this cause of action relate to certain activities which are described in the following association. The practice to which this criticism is directed
three different sorts of activities. The first consist of the paragraphs contained in the agreed statements of facts:. relates of course solely to the management and
administration of the offices in the El Hogar building not administration of properties which are not mortgaged to
used by the respondent itself and the renting of such El Hogar Filipino has undertaken the management of the association. The circumstance that the owner of the
offices to the public. As stated in the discussion connected some parcels of improved real estate situated in Manila property may have been required to subscribe to one or
with the second cause of action, the respondent uses only not under mortgage to it, but owned by shareholders, more shares of the association with a view to qualifying him
about ten per cent of the office space in the El Hogar and has held itself out by advertisement as prepared to to receive this service is of no significance. It is a general
building for its own purposes, and it leases the remainder do so. The number of properties so managed during the rule of law that corporations possess only such express
to strangers. In the years 1924 and 1925 the respondent years 1921 to 1925, inclusive, was as follows: powers. The management and administration of the
received as rent for the leased portions of the building the 1921 eight properties property of the shareholders of the corporation is not
sums of P75,395.06 and P58,259.27, respectively. The 1922 six properties expressly authorized by law, and we are unable to see that,
activities here criticized clearly fall within the legitimate 1923 ten properties upon any fair construction of the law, these activities are
powers of the respondent, as shown in what we have said 1924 fourteen properties necessary to the exercise of any of the granted powers. The
above relative to the second cause of action. This matter 1925 fourteen properties. corporation, upon the point now under the criticism, has
will therefore no longer detain us. If the respondent had clearly extended itself beyond the legitimate range of its
the power to acquire the lot, construct the edifice and hold This service is limited to shareholders; but some of the powers. But it does not result that the dissolution of the
it beneficially, as there decided, the beneficial persons whose properties are so managed for them corporation is in order, and it will merely be enjoined from
administration by it of such parts of the building as are let became shareholders only to enable them to take further activities of this sort.
to others must necessarily be lawful. advantage thereof.
Fourth cause of action. — It appears that among the by
The second specification under the third cause of action The services rendered in the management of such laws of the association there is an article (No. 10) which
has reference to the administration and management of improved real estate by El Hogar Filipino consist in the reads as follows:
properties belonging to delinquent shareholders of the renting of the same, the payment of real estate taxes
association. In this connection it appears that in case of and insurance for the account of the owner, causing the The board of directors of the association, by the vote of
delinquency on the part of its shareholders in the payment necessary repairs for upkeep to be made, and an absolute majority of its members, is empowered to
of interest, premium, and dues, the association has been collecting rents due from tenants. For the services so cancel shares and to return to the owner thereof the
accustomed (pursuant to clause 8 of its standard mortgage) rendered in the management of such properties El balance resulting from the liquidation thereof
to take over and manage the mortgaged property for the Hogar Filipino receives compensation in the form of whenever, by reason of their conduct, or for any other
purpose of applying the income to the obligations of the commissions upon the gross receipts from such motive, the continuation as members of the owners of
debtor party. For these services the respondent charges a properties at rates varying from two and one-half per such shares is not desirable.
commission at the rate of 2½ per centum on sums centum to five per centum of the sums so collected,
collected. The case for the government supposes that the according to the location of the property and the effort This by-law is of course a patent nullity, since it is in direct
only remedy which the respondent has in case of default on involved in its management. conflict with the latter part of section 187 of the
the part of its shareholders is to proceed to enforce Corporation Law, which expressly declares that the board
collection of the whole loan in the manner contemplated in The work of managing real estate belonging to non- of directors shall not have the power to force the surrender
section 185 of the Corporation Law. It will be noted, borrowing shareholders administered by El Hogar and withdrawal of unmatured stock except in case of
however, that, according to said section, the association Filipino is carried on by the same members of the staff liquidation of the corporation or of forfeiture of the stock
may treat the whole indebtedness as due, "at the option of who attend to the details of the management of for delinquency. It is agreed that this provision of the by-
the board of directors," and this remedy is not made properties administered by the manager of El Hogar laws has never been enforced, and in fact no attempt has
exclusive. We see no reason to doubt the validity of the Filipino under the provisions of paragraph 8 of the ever been made by the board of directors to make use of
clause giving the association the right to take over the standard mortgage form, and of properties bought in on the power therein conferred. In November, 1923, the Acting
property which constitutes the security for the delinquent foreclosure of mortgage. Insular Treasurer addressed a letter to El Hogar Filipino,
debt and to manage it with a view to the satisfaction of the calling attention to article 10 of its by-laws and expressing
obligations due to the debtor than the immediate The practice described in the passage above quoted from the view that said article was invalid. It was therefore
enforcement of the entire obligation, and the validity of the agreed facts is in our opinion unauthorized by law. Such suggested that the article in question should be eliminated
the clause allowing this course to be taken appears to us to was the view taken by the bank examiner of the Treasury from the by-laws. At the next meeting of the board of
be not open to doubt. The second specification under this Bureau in his report to the Insular Treasurer on December directors the matter was called to their attention and it

Page 67 of 160
CORPORATION LAW CASES MAYORDO, M.A
LLB 3

was resolved to recommend to the shareholders that in the meeting was attended only by shareholders, in person vacant at the expiration of the term of office for which the
their next annual meeting the article in question be and by proxy, representing 3,889 shares, out of a total of officer was elected, the general rule is to allow the officer
abrogated. It appears, however, that no annual meeting of 106,491 then outstanding and entitled to vote. to holdover until his successor is duly qualified. Mere
the shareholders called since that date has been attended failure of a corporation to elect officers does not terminate
by a sufficient number of shareholders to constitute a Owing to the failure of a quorum at most of the general the terms of existing officers nor dissolve the corporation
quorum, with the result that the provision referred to has meetings since the respondent has been in existence, it has (Quitman Oil Company  vs. Peacock, 14 Ga. App., 550;
no been eliminated from the by-laws, and it still stands been the practice of the directors to fill vacancies in the Jenkins vs. Baxter, 160 Pa. State, 199; New York B. & E. Ry.
among the by-laws of the association, notwithstanding its directorate by choosing suitable persons from among the Co.  vs.  Motil, 81 Conn., 466; Hatch  vs.  Lucky Bill Mining
patent conflict with the law. stockholders. This custom finds its sanction in article 71 of Company, 71 Pac., 865; Youree  vs.  Home Town Matual Ins.
the by-laws, which reads as follows: Company, 180 Missouri, 153; Cassell vs. Lexington, H. and P.
It is supposed, in the fourth cause of action, that the Turnpike Road Co., 10 Ky. L. R., 486). The doctrine above
existence of this article among the by-laws of the ART. 71. The directors shall elect from among the stated finds expressions in article 66 of the by-laws of the
association is a misdemeanor on the part of the respondent shareholders members to fill the vacancies that respondent which declares in so many words that directors
which justifies its dissolution. In this view we are unable to may occur in the board of directors until the shall hold office "for the term of one year on until their
concur. The obnoxious by-law, as it stands, is a mere nullity, election at the general meeting. successors shall have been elected and taken possession of
and could not be enforced even if the directors were to their offices."
attempt to do so. There is no provision of law making it a The person thus chosen to fill vacancies in the directorate
misdemeanor to incorporate an invalid provision in the by- have, it is admitted, uniformly been experienced and It result that the practice of the directorate of filling
laws of a corporation; and if there were such, the hazards successful business and professional men of means, vacancies by the action of the directors themselves is valid.
incident to corporate effort would certainly be largely enjoying earned incomes of from P12,000 to P50,000 per Nor can any exception be taken to then personality of the
increased. There is no merit in this cause of action. annum, with an annual average of P30,000 in addition to individuals chosen by the directors to fill vacancies in the
such income as they derive from their properties. Moreover, body. Certainly it is no fair criticism to say that they have
Fifth cause of action. — In section 31 of the Corporation it appears that several of the individuals constituting the chosen competent businessmen of financial responsibility
Law it is declared that, "at all elections of directors there original directorate and persons chosen to supply vacancies instead of electing poor persons to so responsible a
must be present, either in person or by representative therein belong to prominent Filipino families, and that they position. The possession of means does not disqualify a man
authorized to act by written proxy, the owners of the are more or less related to each other by blood or for filling positions of responsibility in corporate affairs.
majority of the subscribed capital stock entitled to vote. . . marriage. In addition to this it appears that it has been the
." Conformably with this requirement it is declared in policy of the directorate to keep thereon some member or Sixth cause of action. — Under the sixth cause of action it
article 61 of the by-laws of El Hogar Filipino that, "the another of a single prominent American law firm in the city. is alleged that the directors of El Hogar Filipino, instead of
attendance in person or by proxy of shareholders owning serving without pay, or receiving nominal pay or a fixed
one-half plus one of the shareholders shall be necessary to It is supposed in the statement of the fifth cause of action salary, — as the complaint supposes would be proper, —
constitute a quorum for the election of directors. At the in the complaint that the failure of the corporation to hold have been receiving large compensation, varying in amount
general annual meetings of the El Hogar Filipino held in the annual meetings and the filling of vacancies in the from time to time, out of the profits of the respondent.
years 1911 and 1912, there was a quorum of shares present directorate in the manner described constitute The facts relating to this cause of action are in substance
or represented at the meetings and directors were duly misdemeanors on the part of the respondent which justify these:
elected accordingly. As the corporation has grown, the resumption of the franchise by the Government and
however, it has been fond increasingly difficult to get dissolution of the corporation; and in this connection it is Under section 92 of the by-laws of El Hogar Filipino 5 per
together a quorum of the shareholders, or their proxies, at charge that the board of directors of the respondent has centum of the net profit shown by the annual balance sheet
the annual meetings; and with the exception of the annual become a permanent and self perpetuating body composed is distributed to the directors in proportion to their
meeting held in 1917, when a new directorate was elected, of wealthy men instead of wage earners and persons of attendance at meetings of the board. The compensation
the meetings have failed for lack of quorum. It has been moderate means. We are unable to see the slightest merit paid to the directors from time to time since the
foreseen by the officials in charge of the respondent that in the charge. No fault can be imputed to the corporation organization was organized in 1910 to the end of the year
this condition of affairs would lead to embarrassment, and on account of the failure of the shareholders to attend the 1925, together with the number of meetings of the board
a special effort was made by the management to induce a annual meetings; and their non-attendance at such held each year, is exhibited in the following table:
sufficient number of shareholders to attend the annual meetings is doubtless to be interpreted in part as
meeting for February, 1923. In addition to the publication expressing their satisfaction of the way in which things
Compensatio
of notices in the newspapers, as required by the by-laws, a have been conducted. Upon failure of a quorum at any Number Rate per 

n 

letter of notification was sent to every shareholder at his annual meeting the directorate naturally holds over and of 
 meeting 

Year p a i d
last known address, together with a blank form of proxy to continues to function until another directorate is chosen meetings 
 as a
directors 

be used in the event the shareholder could not personally and qualified. Unless the law or the charter of a held whole
as a whole
attend the meeting. Notwithstanding these special efforts corporation expressly provides that an office shall become

Page 68 of 160
CORPORATION LAW CASES MAYORDO, M.A
LLB 3

1925, inclusive, when the board was composed of nine the by-laws meeting to change the rule. The remedy, if any,
191
P 4,167.96 25 P 166.71 members, the attendance has regularly been eight meeting seems to lie rather in publicity and competition, rather
1
with the exception of two years when the average than in a court proceeding. The sixth cause of action is in
attendance was seven. It is insisted in the brief for the our opinion without merit.
191
10,511.87 29 362.47 Attorney-General that the payment of the compensation
2
indicated is excessive and prejudicial to he interests of the Seventh cause of action. — It appears that the promoter
191 shareholders at large. For the respondent, attention is and organizer of El Hogar Filipino was Mr. Antonio Melian,
15,479.29 27 573.30 directed to the fact that the liberal policy adopted by the and in the early stages of the organization of the
3
association with respect to the compensation of the association the board of directors authorized the
191 directors has had highly beneficial results, not only in association to make a contract with him with regard to the
19,164.72 27 709.80
4 securing a constant attendance on the part of the services him therefor. Pursuant to this authority the
membership, but in obtaining their intelligent attention to president of the corporation, on January 11, 1911, entered
191 the affairs of the association. Certainly, in this connection, into a written agreement with Mr. Melian, which is
24,032.85 25 961.31
5 the following words from the report of the government reproduced in the agreed statement of facts and of which
examiners for 1918 to the Insular Treasurer contain matter the important clauses are these:
191 worthy of consideration:
27,539.50 28 983.55
6 1. The corporation "El Hogar Filipino Sociedad Mutua de
The management of the association is entrusted to men of Construccion y Prestamos," and on its behalf its
191 recognized ability in financial affairs and it is believed that president, Don Antonio R. Roxas, hereby confers on Don
31,327.00 26 1,204.88
7 they have long foreseen all possible future contingencies Antonio Melian the office of manager of said association
and that under such men the interests of the stockholders for the period of one year from the date of this
191
32,858.35 20 1,642.91 are duly protected. The steps taken by the directorate to contract.
8 curtail the influx of unnecessary capital into the
association's coffers, as mentioned above, reveals how the 2. Don Antonio Melian accepts said office and
191
36,318.78 21 1,729.46 men at grasp the situation and to apply the necessary undertakes to render the services thereto
9
remedy as the circumstances were found in the same corresponding for the period of one year, as prescribed
excellent condition as in the previous examination. by the by-laws of the corporation, without salary.
192
63,517.01 28 2,268.46
0
In so far as this court is concerned the question here before 3. Don Antonio Melian furthermore undertakes to pay
192 us is not one concerning the propriety and wisdom of the for his own account, all the expenses incurred in the
36,815.33 25 1,472.61 measure of compensation adopted by the respondent but organization of the corporation.
1
rather the question of the validity of the measure. Upon
192 this point there can, it seems to us, be no difference of 4. Don Antonio Melian further undertakes to lend to the
43,133.73 25 1,725.34
2 intelligent opinion. The Corporation Law does not corporation, without interest the sum of six thousand
undertake to prescribe the rate of compensation for the pesos (P6,000), Philippine Currency, for the purpose of
192 directors of corporations. The power to fixed the meeting the expense of rent, office supplies, etcetera,
39,773.61 27 1,473.09
3 compensation they shall receive, if any, is left to the until such time as the association has sufficient funds of
corporation, to be determined in its by-laws(Act No. 1459, its own with which to return this loan:  Provided,
192 sec. 21). Pursuant to this authority the compensation for nevertheless, That the maximum period thereof shall
38,651.92 26 1,486.61
4 the directors of El Hogar Filipino has been fixed in section not exceed three (3) years.
92 of its by-laws, as already stated. The justice and
192 property of this provision was a proper matter for the 5. Don Antonio Melian undertakes that the capital of
35,719.27 26 1,373.81
5 shareholders when the by-laws were framed; and the the association shall amount to the sum of four hundred
circumstance that, with the growth of the corporation, the thousand pesos (P400,000), Philippine currency, par
It will be note that the compensation above indicated as amount paid as compensation to the directors has increased value, during the first year of its duration.
accruing to the directorate as a whole has been divided beyond what would probably be necessary to secure
among the members actually present at the different adequate service from them is matter that cannot be 6. In compensation of the studies made and services
meetings. As a result of this practice, and the liberal corrected in this action; nor can it properly be made a basis rendered by Don Antonio Melian for its organization,
measure of compensation adopted, we find that the for depriving the respondent of its franchise, or even for the expenses incurred by him to that end, and in
attendance of the membership at the board meetings has enjoining it from compliance with the provisions of its own further consideration of the said loan of six thousand
been extraordinarily good. Thus, during the years 1920 to by-laws. If a mistake has been made, or the rule adopted in pesos (P6,000), and of the services to be rendered by

Page 69 of 160
CORPORATION LAW CASES MAYORDO, M.A
LLB 3

him as manager, and of the obligation assumed by him the association should be enjoined from performing the the pit of populism and bolshevism. The seventh count is
that the nominal value of the capital of the association agreement. not sustainable.
shall reach the sum of four hundred thousand pesos It is our opinion that this contention is entirely without
(P400,000) during the first year of its duration, the merit. Stated in its true simplicity, the primary question Eight cause of action. — Under the fourth cause of action
corporation 'El Hogar Filipino Sociedad Mutua de here is whether the making of a (possibly) indiscreet we had case where the alleged ground for the revocation of
Construccion y Prestamos' hereby grants him five per contract is a capital offense in a corporation, — a question the respondent's charter was based upon the presence in
centum (5%) of the net profits to be earned by it in which answers itself. No possible doubt exists as to the the by-laws of article 10 that was found to be inconsistent
each year during the period fixed for the duration of power of a corporation to contract for services rendered with the express provisions of law. Under the eight cause of
the association by its articles of and to be rendered by a promoter in connection with action the alleged ground for putting an end to the
incorporation;  Provided, that this participation in the organizing and maintaining the corporation. It is true that corporate life of the respondent is found in the presence of
profits shall be transmitted to the heirs of Señor Melian contracts with promoters must be characterized by good other articles in the by-laws, namely, articles 70 and 76,
in the event of his death;  And provided further, that faith; but could it be said with certainty, in the light of which are alleged to be unlawful but which, as will
the performance of all the obligations assumed by facts existing at the time this contract was made, that the presently be seen, are entirely valid. Article 70 of the by-
Señor Melian in favor of the association, in accordance compensation therein provided was excessive? If the laws in effect requires that persons elected to the board of
with this contract, shall and does constitute a condition amount of the compensation now appears to be a subject directors must be holders of shares of the paid up value of
precedent to the acquisition by Señor Melian of the of legitimate criticism, this must be due to the P5,000 which shall be held as security may be put up in the
right to the said participation in the profits of the extraordinary development of the association in recent behalf of any director by some other holder of shares in the
association, unless the non-performance of such years. amount stated. Article 76 of the by-laws declares that the
obligations shall be due to a fortuitous event or  force directors waive their right as shareholders to receive loans
majeure. If the Melian contract had been clearly ultra vires — which from the association.
is not charged and is certainly untrue — its continued
In conformity with this agreement there was inserted in performance might conceivably be enjoined in such a It is asserted, under the eight cause of action, that article
section 92 of the by-laws of the association a provision proceeding as this; but if the defect from which it suffers is 70 is objectionable in that, under the requirement for
recognizing the rights of Melian, as founder, to 5 per mere matter for an action because Melian is not a party. It security, a poor member, or wage-earner, cannot serve as
centum of the net profits shown by the annual balance is rudimentary in law that an action to annul a contract director, irrespective of other qualifications and that as a
sheet, payment of the same to be made to him or his heirs cannot be maintained without joining both the contracting matter of fact only men of means actually sit on the board.
during the life of the association. It is declared in said parties as defendants. Moreover, the proper party to bring Article 76 is criticized on the ground that the provision
article that this portion of the earnings of the association is such an action is either the corporation itself, or some requiring directors to renounce their right to loans
conceded to him in compensation for the studies, work and shareholder who has an interest to protect. unreasonably limits their rights and privileges as members.
contributions made by him for the organization of El Hogar There is nothing of value in either of theses suggestions.
Filipino and the performance on his part of the contract of The mere fact that the compensation paid under this Section 21 of the Corporation Law expressly gives the
January 11, 1911, above quoted. During the whole life of contract is in excess of what, in the full light of history, power to the corporation to provide in its by-laws for the
the association, thus far, it has complied with the may be considered appropriate is not a proper qualifications of directors; and the requirement of security
obligations assumed by it in the contract above- consideration for this court, and supplies no ground for from them for the proper discharge of the duties of their
mentioned; and during the years 1911 to 1925, inclusive, it interfering with its performance. In the case of  El Hogar office, in the manner prescribed in article 70, is highly
paid to him as founder's royalty the sum of P459,011.19, in Filipino vs. Rafferty  (37 Phil., 995), which was before this prudent and in conformity with good practice. Article 76,
addition to compensation received from the association by court nearly ten years ago, this court held that the El Hogar prohibiting directors from making loans to themselves, is of
him in to remuneration of services to the association in Filipino is contract with Mr. Melian did not affect the course designed to prevent the possibility of the looting of
various official capacities. association's legal character. The inference is that the the corporation by unscrupulous directors. A more discreet
contract under consideration was then considered binding, provision to insert in the by-laws of a building and loan
As a seventh cause of action it is alleged in the complaint and it occurred to no one that it was invalid. It would be a association would be hard to imagine. Clearly, the eighth
that this royalty of the founder is "unconscionable, radical step indeed for a court to attempt to substitute its cause of action cannot be sustained.
excessive and out of all proportion to the services judgment for the judgment of the contracting parties and
rendered, besides being contrary to and incompatible with to hold, as we are invited to hold under this cause of Ninth cause of action. — The specification under this head
the spirit and purpose of building and loan associations." It action, that the making of such a contract as this removes is in effect that the respondent has abused its franchise in
is not alleged that the making of this contract was beyond the respondent association from the pale of the law. The issuing "special" shares. The issuance of these shares is
the powers of the association (ultra vires); nor it alleged majority of the court is of the opinion that our traditional allege to be illegal and inconsistent with the plan and
that it is vitiated by fraud of any kind in its procurement. respect for the sanctity of the contract obligation should purposes of building and loan associations; and in
Nevertheless, it is pretended that in making and observing prevail over the radical and innovating tendencies which particular, it is alleged and inconsistent with the plan and
said contract the respondent committed an offense find acceptance with some and which, if given full rein, purposes of building and loan associations; and in
requiring its dissolution, or, as is otherwise suggested, that would go far to sink legitimate enterprise in the Islands into particular, it is alleged that they are, in the main, held by

Page 70 of 160
CORPORATION LAW CASES MAYORDO, M.A
LLB 3

well-to-wage-earners for accumulating their modest savings to each share being computed on the basis of the capital payment shares; in if close attention be paid to the
for the building of homes. paid in, plus the accumulated dividends pertaining to each language used in the last sentence of section 178 of the
share at the end of the year. The total number of shares of Corporation Law, it will be found that special shares where
In the articles of incorporation we find the special shares El Hogar Filipino outstanding on December 31, 1925, was evidently created for the purpose of meeting the condition
described as follows: 125,750, owned by 5,826 shareholders, and dividend into cause by the prepayment of dues that is there permitted.
classes as follows: The language of this provision is as follow "payment of dues
"Special" shares shall be issued upon the payment of 80 or interest may be made in advance, but the corporation
P r e f e r r e d
per cent of their par value in cash, or in monthly dues 1,503 shall not allow interest on such advance payment at a
shares ..................................
of P10. The 20 per cent remaining of the par value of greater rate than six per centum per annum nor for a
such shares shall be completed by the accumulation S p e c i a l longer period than one year." In one sort of special shares
thereto of their proportionate part of the profits of the shares .................................. 20,884 the dues are prepaid to the extent of P160 per share; in the
corporation. At the end of each quarter the holders of ... other sort prepayment is made in the amount of P10 per
special shares shall be entitled to receive in cash such share, and the subscribers assume the obligation to pay P10
part of the net profits of the corporation corresponding O r d i n a r y 103,36 monthly until P160 shall have been paid.
to the amount on such date paid in by the holders of shares .................................. 3
special shares, on account thereof, as shall be It will escape notice that the provision quoted say that
The matter of the propriety of the issuance of special
determined by the directors, and at the end of each interest shall not be allowed on the advance payments at a
shares by El Hogar Filipino has been before this court in
year the full amount of the net profits available for greater rate than six per centum per annum nor for a
two earlier cases, in both of which the question has
distribution corresponding to the special shares. The longer period than one year. The word "interest " as there
received the fullest consideration from this court. In  El
directors shall apply such part as they deem advisable used must be taken in its true sense of compensation for
Hogar Filipino vs. Rafferty  (37 Phil., 995), it was insisted
to the amortization of the subscription to capital with the used of money loaned, and it not must not be confused
that the issuance of such shares constituted a departure on
respect to shares not fully paid up, and the remainder with the dues upon which it is contemplated that the
the part of the association from the principle of mutuality;
of the profits, if any, corresponding to such shares, interest may be paid. Now, in the absence of any showing
and it was claimed by the Collector of Internal Revenue
shall be delivered to the holders thereof in accordance to the contrary, we infer that no interest is ever paid by
that this rendered the association liable for the income tax
with the provision of the by-laws. the association in any amount for the advance payments
to which other corporate entities are subject. It was held made on these shares; and the reason is to be found in the
that this contention was untenable and that El Hogar
The ground for supposing the issuance of the "special" fact that the participation of the special shares in the
Filipino was a legitimate building and loan association
shares to be unlawful is that special shares are not earnings of the corporation, in accordance with section 188
notwithstanding the issuance of said shares. In Sevireno vs.
mentioned in the Corporation Law as one of the forms of of the Corporation Law, sufficiently compensates the
El Hogar Filipino (G. R. No. 24926),2 and the related cases
security which may be issued by the association. In the shareholder for the advance payments made by him; and no
of Gervasio Miraflores and Gil Lopes against the same
agreed statement of facts it is said that special shares are other incentive is necessary to induce inventors to purchase
entity, it was asserted by the plaintiffs that the emission of
issued upon two plans. By the second, the shareholder, the stock.
special shares deprived the herein responded of the
upon subscribing, pays in cash P10 for each share taken,
privileges and immunities of a building and loan association
and undertakes to pay P10 a month, as dues, until the total It will be observed that the final 20 per centum of the par
and that as a consequence the loans that had been made to
so paid in amounts to P160 per share. On December 31, value of each special share is not paid for by the
the plaintiffs in those cases were usurious. Upon an
1925, there were outstanding 20,844 special shares of a shareholder with funds out of the pocket. The amount is
elaborate review of the authorities, the court, though
total paid value (including accumulations ) of satisfied by applying a portion of the shareholder's
divided, adhered to the principle announced in the earlier
P3,680,162.51. The practice of El Hogar Filipino, since participation in the annual earnings. But as the right of
case and held that the issuance of the special shares did
1915, has been to accumulate to each special share, at the every shareholder to such participation in the earnings is
not affect the respondent's character as a building and loan
end of the year, one-tenth of the divident declared and to undeniable, the portion thus annually applied is as much
association nor make its loans usurious. In view of the
pay the remainder of the divident in cash to the holders of the property of the shareholder as if it were in fact taken
lengthy discussion contained in the decisions above-
shares. Since the same year dividend have been declared out of his pocket. It follows that the mission of the special
mentioned, it would appear to be an act of supererogation
on the special and common shares at the rate of 10 per shares does not involve any violation of the principle that
on our part to go over the same ground again. The
centum per annum. When the amount paid in upon any the shares must be sold at par.
discussion will therefore not be repeated, and what is now
special share plus the accumulated dividends accruing to it, From what has been said it will be seen that there is
to be said should be considered supplemental thereto.
amounts to the par value of the share (P200), such share express authority, even in the very letter of the law, for the
matures and ceases to participate further in the earning. emission of advance-payment or "special" shares, and the
Upon examination of the nature of the special shares in the
The amount of the par value of the share (P200) is then argument that these shares are invalid is seen to be
light of American usage, it will be found that said shares
returned to the shareholder and the share cancelled. baseless. In addition to this it is satisfactorily demonstrated
are precisely the same kind of shares that, in some
Holders of special and ordinary shares participate ratably in in Severino vs. El Hogar Filipino, supra, that even assuming
American jurisdictions, are generally known as advance
the dividends declared and distributed, the part pertaining that the statute has not expressly authorized such shares,

Page 71 of 160
CORPORATION LAW CASES MAYORDO, M.A
LLB 3

yet the association has implied authority to issue them. The and for the management of its business, as well as the and the payment of a dividend of 10 per centum to all
complaint consequently fails also as regards the stated in care, control and disposition of its property (Act No. 1459, special and ordinary shares (with accumulated dividends).
the ninth cause of action. sec. 13 [7]). But the Attorney-General questions the As things stood in 1926 the general reserve contained an
exercise of the direction confided to the board; and it is amount equivalent to about 5 per centum of the paid-in
Tenth cause of action. — Under this head of the complaint insisted that the excessive depreciation of the property of value of shared. This fund has never been drawn upon for
it is alleged that the defendant is pursuing a policy of the association is objectionable in several respects, but the purpose of maintaining the regular annual dividend; but
depreciating, at the rate of 10 per centum per annum, the mainly because it tends to increase unduly the reserves of recourse has been had to the special reserve on three
value of the real properties acquired by it at its sales; and the association, thereby frustrating the right of the different occasions to make good the amount necessary to
it is alleged that this rate is excessive. From the agreed shareholders to participate annually and equally in the pay dividends. It appears that in the last five years the
statement it appears that since its organization in 1910 El earnings of the association. reserves have declined from something over 9 per cent to
Hogar Filipino, prior to the end of the year 1925, had made something over 7.
1,373 loans to its shareholders secured by first mortgages This count for the complaint proceeds, in our opinion, upon
on real estate as well as by the pledge of the shares of the an erroneous notion as to what a court may do in It is insisted in the brief of the Attorney-General that the
borrowers. In the same period the association has determining the internal policy of a business corporation. If maintenance of reserve funds is unnecessary in the case of
purchased at foreclosure sales the real estate constituting the criticism contained in the brief of the Attorney-General building and loan associations, and at any rate the keeping
the security for 54 of the aforesaid loans. In making these upon the practice of the respondent association with of reserves is inconsistent with section 188 of the
purchases the association has always bid the full amount respect to depreciation be well founded, the Legislature Corporation Law. Moreover, it is said that the practice of
due to it from the debtor, after deducting the withdrawal should supply the remedy by defining the extent to which the association in declaring regularly a 10 per cent dividend
value of the shares pledged as collateral, with the result depreciation may be allowed by building and loan is in effect a guaranty by the association of a fixed dividend
that in no case has the shareholder been called upon to pay associations. Certainly this court cannot undertake to which is contrary to the intention of the statute.
a deficiency judgement on foreclosure. control the discretion of the board of directors of the
association about an administrative matter as to which they Upon careful consideration of the questions involved we
El Hogar Filipino places real estate so purchased in its have legitimate power of action. The tenth cause of action find no reason to doubt the right of the respondent to
inventory at actual cost, as determined by the amount bid is therefore not well founded. maintain these reserves. It is true that the corporation law
on foreclosure sale; and thereafter until sold the book does not expressly grant this power, but we think it is to be
value of such real estate is depreciated at the rate fixed by Eleventh and twelfth causes of action. — The same implied. It is a fact of common observation that all
the directors in accordance with their judgment as to each comment is appropriate with respect to the eleventh and commercial enterprises encounter periods when earnings
parcel, the annual average depreciation having varied from twelfth causes of action, which are treated together in the fall below the average, and the prudent manager makes
nothing to a maximum of 14.138 per cent. The sales briefs, and will be here combined. The specification in the provision for such contingencies. To regard all surplus as
thereof, but sales are made for the best prices obtainable, eleventh cause of action is that the respondent maintains profit is to neglect one of the primary canons of good
whether greater or less than the book value. excessive reserve funds, and in the twelfth cause of action business practice. Building and loan associations, though
that the board of directors has settled upon the unlawful among the most solid of financial institutions, are
It is alleged in the complaint that depreciation is charged policy of paying a straight annual dividend of 10 per nevertheless subject to vicissitudes. Fluctuations in the
by the association at the rate of 10 per centum per annum. centum, regardless of losses suffered and profits made by dividend rate are highly detrimental to any fiscal
The agreed statement of facts on this point shows that the the corporation and in contravention of the requirements institutions, while uniformity in the payments of dividends,
annual average varies from nothing to a maximum of of section 188 of the Corporation Law. The facts relating to continued over long periods, supplies the surest
something over 14 per centum. We are thus left in the dark these two counts in the complaint, as set forth in the foundations of public confidence.
as to the precise depreciation allowed from year to year. It stipulation, are these:
is not claimed for the Government that the association is The question now under consideration is not new in
without power to allow some depreciation; and it is quite In article 92 of the by-laws of El Hogar Filipino it is jurisprudence, for the American courts have been called
clear that the board of directors possesses a discretion in provided that 5 per centum of the net profits earned each upon more than once to consider the legality of the
this matter. There is no positive provision of law prohibiting year, as shown by the annual balance sheet shall be carried maintenance of reserves by institutions of this or similar
the association from writing off a reasonable amount for to a reserve fund. The fund so created is called the General character.
depreciation on its assets for the purpose of determining its Reserve. Article 93 of the by-laws authorizes the directors
real profits; and article 74 of its by-laws expressly to carry funds to a special reserve, whenever in their In  Greeff vs. Equitable Life Assurance Society, the court
authorizes the board of directors to determine each year judgment it is advisable to do so, provided that the annual had under consideration a charter provision of a life
the amount to be written down upon the expenses of dividend in the year in which funds are carried to special insurance company, organized on the mutual plan, in its
installation and the property of the corporation. There can reserve exceeds 8 per centum. It appears to have been the relation to the power of the company to provide reserves.
be no question that the power to adopt such a by-law is policy of the board of directors for several years past to There the statute provided that "the officers of the
embraced within the power to make by-laws for the place in the special reserve any balance in the profit and company, within sixty days from the expiration of the first
administration of the corporate affairs of the association loss account after the satisfaction of preferential charges five years, from December 31, 1859, and within the first

Page 72 of 160
CORPORATION LAW CASES MAYORDO, M.A
LLB 3

sixty days of every subsequent period of five years, shall association's assets, such as defective titles, Thirteenth cause of action. — The specification under this
cause a balance to be struck of the affairs of the company, undisclosed defalcations on the part of an officer, a head is, in effect, that the respondent association has
which shall exhibit its assets and liabilities, both present miscalculation of assets and liabilities, and many other made loans which, to the knowledge of the associations
and contingent, and also the net surplus, after deducting a errors and omissions which must always be reckoned officers were intended to be used by the borrowers for
sufficient amount to cover all outstanding risks and other within the conduct of human affairs. other purposes than the building of homes. In this
obligations. Each policy holder shall be credited with an connection it appears that, though loans have been made
equitable share of the said surplus." The contingent fund is merely insurance against by the association exclusively to its shareholders, no
possible loss. That losses may occur from time to time attempt has been made by it to control the borrowers with
The court said: seems almost inevitable and it is, therefore, respect to the use made of the borrowed funds, the
inequitable that the remaining stockholders should be association being content to see that the security given for
No prudent person would be inclined to take a policy in compelled to accept all securities at par, so, to say the the loan in each case is sufficient. On December 31, 1925,
a company which had so improvidently conducted its least, the maintenance of this fund is justified. The the respondent had five hundred forty-four loans
affairs that it only retained a fund barely sufficient to association teaches the duty of providing for the outstanding secured by mortgages upon real estate and by
pay its present liabilities, and, therefore, was in a proverbial rainy day. Why should it not provide for the the pledge of the borrowers' shares in an amount sufficient
condition where any change by the reduction of hour of adversity? The reserve fund has protected the at maturity to amortize the loans. With respect to the
interest upon, or depreciation in, the value of its maturing or withdrawing member during the period of nature of the real estate upon which these loans were
securities, or any increase of mortality, would render it his membership. In case of loss it has or would have made it appears that three hundred fifty-one loans were
insolvent and subject to be placed in the hands of a reimbursed him and, at all times, it has protected him secured by mortgages upon city residences, seven by
receiver. The evident purpose of the provisions of the and given strength and standing to the association. mortgages upon commercial building in cities, and three
defendant's charter and policy relating to this subject Losses may occur, after his membership ceases, that mortgages upon unimproved city lots. At the same time one
was to vest in the directors of the corporation a arose from some mistake or mismanagement committed hundred eighty-three of the loans were secured by
discretion to determine the proportion of its surplus during the period of his membership, and in fairness mortgages upon groves, sugar land, and rice land, with a
which should be dividend each year. and equity the remaining members should have some total area of about 7,558 hectares. From information
protection against this. (Sundheim, Law of Building and gathered by the association from voluntary statements of
In a friendly suit tried in a circuit court of Wisconsin in Loan Association, sec. 53.) borrowers given at the time of application with respect to
1916, entitled  Boheman Bldg. and Loan Association vs. the use intended to be made of the borrowed funds, it
Knolt, the court, in commenting on the nature of these The government insists, we thing, upon an interpretation of appears that the amount of P693,200 was borrowed to
reserves, said: section 188 of the Corporation Law that is altogether too redeem real property from existing mortgages or pactos de
strict and literal. From the fact that the statute provides retro, P280,800 to buy real estate, P449,100 to erect
The apparent function of this fund is to insure the that profits and losses shall be annually apportioned among buildings, P24,000 to improve and repair buildings,
stockholders against losses. Its purpose is not unlike the shareholders it is argued that all earnings should be P1,480,900 for agricultural purposes, while the amount of
that of the various forms of insurance now in such distributed without carrying anything to the reserve. But it P5,763,700 was borrowed for purposes not disclosed.
common use. This contribution is as legitimate an item will be noted that it is provided in the same section that
of expense as are the premiums paid on any insurance the profits and losses shall be determined by the board of Upon these facts an elaborate argument has been
policy. (See  Clarks and Chase, Building and Loan directors: and this means that they shall exercise the usual constructed in behalf of the plaintiff to the effect that in
Association, footnote, page 344.) discretion of good businessmen in allocating a portion of making loans for other purposes than the building of
the annual profits to purposes needful to the welfare of the residential houses the association has illegally departed
In commenting on the necessity of such funds, Sundheim association. The law contemplates the distribution of from its character and made itself amenable to the penalty
says: earnings and losses after other legitimate obligations have of dissolution. Aside from being directly opposed to the
been met. decision of this court in  Lopez and Javelona vs. El Hogar
It is optional with the association whether to maintain Filipino and Registrar of Deeds of Occidental Negros  (47
such a fund or not, but justice and good business policy Our conclusion is that the respondent has the power to Phil., 249), this contention finds no substantial support in
seem to require it. The retiring stockholder must be maintain the reserves criticized in the eleventh and twelfth the prevailing decisions made in American courts; and our
paid the value of his stock in cash and leave for those counts of the complaint; and at any rate, if it be supposed attention has not been directed to a single case wherein
remaining a large number of securities and perhaps that the reserves referred to have become excessive, the the dissolution of a building and loan association has been
some real estate purchased to protect the associations remedy is in the hands of the Legislature. It is no proper decreed in a  quo warranto  proceeding because the
interest. How much will be realized on these securities, function of the court to arrogate to itself the control of association allowed its borrowers to use the loans for other
or real estate, no human foresight can tell. Further, the administrative matters which have been confided to the purposes than the acquisition of homes.
realizing on these securities may entail considerable discretion of the board of directors. The causes of action
litigation and expense. There are many other under discussion must be pronounced to be without merit. The case principally relied upon for the Government
contingencies which might cause a shrinkage in the appears to be Pfeister vs. Wheeling Building Association (19

Page 73 of 160
CORPORATION LAW CASES MAYORDO, M.A
LLB 3

W. Va., 676, 716),which involved the question whether a In the brief of the plaintiff a number of excerpts from fundamental purposes. The conclusion of the court was that
building and loan association could recover the full amount textbooks and decisions have been collated in which the the loan was valid and could be lawfully enforced by a
of a note given to it by a member and secured by a idea is developed that the primary design of building and nonjudicial foreclosure in conformity with the terms of the
mortgage from a stranger. At the time the case arose there loan associations should be to help poor people to procure contract between the association and the borrowing
was a statute in force in the State of West Virginia homes of their own. This beneficent end is undoubtedly member. We now find no reason to depart from the
expressly forbidding building and loan associations to use or served by these associations, and it is not to be denied that conclusion reached in that case, and it is unnecessary to
direct their funds for or to any other object or purpose they have been generally fostered with this end in view. repeat what was then said. The thirteenth cause of action
than the buying of lots or houses or in building and But in this jurisdiction at least the lawmaker has taken care must therefore be pronounced unfounded.
repairing houses, and it was declared that in case the funds not to limit the activities of building and loan associations
should be improperly directed to other objects, the in an exclusive manner, and the exercise of the broader Fourteenth cause of action. — The specification under this
offending association should forfeit all rights and privileges powers must in the end approve itself to the business head is that the loans made by the defendant for purposes
as a corporation. Under the statute so worded the court community. Judging from the past history of these other than building or acquiring homes have been extended
held that the plaintiff could only recover the amount institutions it can be truly said that they have done more to in extremely large amounts and to wealthy persons and
actually advanced by it with lawful interest and fines, encourage thrift, economy and saving among the people at large companies. In this connection attention is directed to
without premium; and judgment was given accordingly. The large than any other institution of modern times, not eight loans made at different times in the last several years
suggestion in that case that the result would have been the excepting even the saving banks. In this connection Mr. to different persons or entities, ranging in amounts from
same even in the absence of statute was mere dictum and Sundheim, in a late treatise upon the subject of the law of P120,000 to P390,000 and to two large loans made to the
is not supported by respectable authority. building and loan associations, makes the following Roxas Estate and to the Pacific Warehouse Company in the
comment: amounts of P1,122,000 and P2,320,000, respectively. In
Reliance is also placed in the plaintiff's brief upon McCauley connection with the larger of the two after this loan was
vs. Building & Saving Association. The statute in force in They have grown to such an extent in recent years that made the available funds of El Hogar Filipino were reduced
the State of Tennessee at the time this action arose they no longer restrict their money to the home buyer, to the point that the association was compelled to take
provided that all loans should be made to the members of but loan their money to the mere investor or dealer in advantage of certain provisions of its by-laws authorizing
the association at open stated meetings and that the real estate. They are the holder of large mortgages the postponement of the payment of claims resulting from
money should be lent to the highest bidder. Inconsistently secured upon farms, factories and other business withdrawals, whereas previously the association had always
with this provision, there was inserted in the by-laws of the properties and rows of stores and dwellings. This is not settled these claims promptly from current funds. At no
association a provision to the effect that no loan should be an abuse of their powers or departure from their main time was there apparently any delay in the payment of
made at a greater premium than 30 per cent, nor at a less purposes, but only a natural and proper expansion matured shares; but in four or five cases there was as much
premium than 29 7/8 per cent. It was held that this by-law along healthy and legitimate lines. (Sundheim, Building as ten months delay in the payment of withdrawal
made free and open competition impossible and that it in and Loan Associations, sec. 7.) applications.
effect established a fixed premium. It was accordingly
held, in the case cited, that an association could not Speaking of the purpose for which loans may be made, the There is little that can be said upon the legal aspects of
recover such part of the loan as had been applied by it to same author adds: this cause of action. In so far, as it relates to the purposes
the satisfaction of a premium of 30 per centum. for which these loans were made, the matter is covered by
Loans are made for the purpose of purchasing a what was said above with reference to the thirteenth cause
We have no criticism to make upon the result reached in homestead, or other real estate, or for any lawful of action; and in so far as it relates to the personality of
either of the two decisions cited, but it is apparent that purpose or business, but there is no duty or obligation the borrowers, the question belongs more directly to the
much of the discussion contained in the opinions in those of the association to inquire for what purpose the loan discussion under the sixteenth cause of action, which will
cases does not reflect the doctrine now prevailing in the is obtained, or to require any stipulation from the be found below. The point, then, which remains for
United States; and much less are those decisions applicable borrower as to what use he will make of the money, or consideration here is whether it is a suicidal act on the part
in this jurisdiction. There is no statute here expressly in any manner to supervise or control its disbursement. of a building and loan association to make loans in large
declaring that loans may be made by these (Sundheim, Building and Loan Association, sec. 111.) amount. If the loans which are here the subject of criticism
associations  solely  for the purpose of building homes. On had been made upon inadequate security, especially in case
the contrary, the building of homes is mentioned in section In Lopez and Javelona vs. El Hogar Filipino and Registrar of of the largest two, the consequences certainly would have
171 of the Corporation Law as only one among several ends Deeds of Occidental Negros, this court had before it the been disastrous to the association in the extreme; but no
which building and loan associations are designed to question whether a loan made by the respondent such fact is alleged; and it is to be assumed that none of
promote. Furthermore, section 181 of the Corporation Law association upon the security of a mortgage upon the ten borrowers have defaulted in their contracts.
expressly authorities the Board of directors of the agricultural land, — where the loan was doubtless used for
association from time to time to fix the premium to be agricultural purposes, — was usurious or not; and the case Now, it must be admitted that two of these loans at least
charged. turned upon the point whether, in making such loans, the are of a very large size, considering the average range of
association had violated the law and departed from its financial transaction in this country; and the making of the

Page 74 of 160
CORPORATION LAW CASES MAYORDO, M.A
LLB 3

largest loan was followed, as we have already see, with It will be noted that the cause of action with which we are the word "person" or persons," is to be taken in this broad
unpleasant consequences to the association in dealing with now concerned is not directed to any positive misdemeanor sense in every part of the Corporation Law. For instance, it
current claims. Nevertheless the agreed statement of facts supposed to have been committed by the association. It has would seem reasonable to say that the incorporators of a
shoes that all of the loan referred to are only ten out of a exclusive relation to what may happen some thirty-five corporation ought to be natural persons, although in
total of five hundred forty-four outstanding on December years hence when the franchise expires, supposing of section 6 it is said that five or more "persons", although in
31, 1925; and the average of all the loans taken together is course that the corporation should not be reorganized and section 6 it is said that five or more "persons," not
modest enough. It appears that the chief examiner of banks continued after that date. There is nothing in article 95 of exceeding fifteen, may form a private corporation. But the
and corporations of the Philippine Treasury, after his the by-laws which is, in our opinion, subject to criticism. context there, as well as the common sense of the
examination of El Hogar Filipino at the end of the year The real point of criticism is that upon the final liquidation situation, suggests that natural persons are meant. When it
1925, made a report concerning this association as of of the corporation years hence there may be in existence a is said, however, in section 173, that "any person" may
January 31, 1926, in which he criticized the Pacific reserve fund out of all proportion to the requirements that become a stockholder in a building and loan association, no
Warehouse Company loan as being so large that it may then fall upon it in the liquidation of the company. It reason is seen why the phrase may not be taken in its
temporarily crippled the lending power of the association seems to us that this is matter that may be left to the proper broad sense of either a natural or artificial person.
for some time. This criticism was apparently justified as prevision of the directors or to legislative action if it should At any rate the question whether these loans and the
proper comment on the activities of the association; but be deemed expedient to require the gradual suppression of attendant subscriptions were properly made involves a
the question for use here to decide is whether the making the reserve funds as the time for dissolution approaches. It consideration of the power of the subscribing corporations
of this and the other large loans constitutes such a misuser is no matter for judicial interference, and much less could and partnerships to own the stock and take the loans; and
of the franchise as would justify us in depriving the the resumption of the franchise on this ground be justified. it is not alleged in the complaint that they were without
association of its corporate life. This question appears to us There is no merit in the fifteenth cause of action. power in the premises. Of course the mere motive with
to be so simple as almost to answer itself. The law states which loan or for some other reason, is of no moment in
no limit with respect to the size of the loans to be made by Sixteenth cause of action. — This part of the complaint determining subscriptions are made, whether to qualify the
the association. That matter is confided to the discretion of assigns as cause of action that various loans now stockholders to take a whether the subscribers were
the board of directors; and this court cannot arrogate to outstanding have been made by the respondent to competent to make the contracts. The result is that we
itself a control over the discretion of the chosen officials of corporations and partnerships, and that these entities have find nothing in the allegations of the sixteenth cause of
the company. If it should be thought wise in the future to in some instances subscribed to shares in the respondent action, or in the facts developed in connection therewith,
put a limit upon the amount of loans to be made to a single for the sole purpose of obtaining such loans. In this that would justify us in granting the relief.
person or entity, resort should be had to the Legislature; it connection it appears from the stipulation of facts that of
is not a matter amenable to judicial control. The the 5,826 shareholders of El Hogar Filipino, which Seventeenth cause of action. — Under the seventeenth
fourteenth cause of action is therefore obviously withoutt composed its membership on December 31, 1925, twenty- cause of action, it is charged that in disposing of real
merit. eight are juridical entities, comprising sixteen corporations estates purchased by it in the collection of its loans, the
and fourteen partnerships; while of the five hundred forty- defendant has no various occasions sold some of the said
Fifteenth cause of action. — The criticism here comes back four loans of the association outstanding on the same date, real estate on credit, transferring the title thereto to the
to the supposed misdemeanor of the respondent in nine had been made to corporations an five to purchaser; that the properties sold are then mortgaged to
maintaining its reserve funds, — a matter already discussed partnerships. It is also admitted that some of these the defendant to secure the payment of the purchase
under the eleventh and twelfth causes of action. Under the juridical entities became shareholders merely for the price, said amount being considered as a loan, and carried
fifteenth cause of action it is claimed that upon the purpose of qualifying themselves to take loans from the as such in the books of the defendant, and that several
expiration of the franchise of the association through the association, and the same is said with respect to many such obligations are still outstanding. It is further charged
effluxion of time, or earlier liquidation of its business, the natural persons who have taken shares in the association. that the persons and entities to which said properties are
accumulated reserves and other properties will accrue to Nothing is said in the agreed statement of facts on the sold under the condition charged are not members or
the founder, or his heirs, and the then directors of the point whether the corporations and partnerships that have shareholders nor are they made members or shareholders
corporation and to those persons who may at that time to taken loans from the respondent are qualified by law of the defendant.
be holders of the ordinary and special shares of the governing their own organization to enter into these
corporation. In this connection we note that article 95 of contracts with the respondent. This part of the complaint is based upon a mere
the by-laws reads as follows: technicality of bookkeeping. The central idea involved in
In section 173 of the Corporation Law it is declared that the discussion is the provision of the Corporation Law
ART. 95. The funds obtained by the liquidation of the "any person" may become a stockholder in building and loan requiring loans to be stockholders only and on the security
association shall be applied in the first place to the associations. The word "person" appears to be here used in of real estate and shares in the corporation, or of shares
repayment of shares and the balance, if any, shall be its general sense, and there is nothing in the context to alone. It seems to be supposed that, when the respondent
distribute in accordance with the system established indicate that the expression is used in the restricted sense sells property acquired at its own foreclosure sales and
for the distribution of annual profits. of both natural and artificial persons, as indicated in takes a mortgage to secure the deferred payments, the
section 2 of the Administrative Code. We would not say that obligation of the purchaser is a true loan, and hence

Page 75 of 160
CORPORATION LAW CASES MAYORDO, M.A
LLB 3

prohibited. But in requiring the respondent to sell real of buying, ginning, and selling cotton,and to organize a that it is duly incorporated. When the fact appears,as it
estate which it acquires in connection with the collection corporation for this purpose. They transacted abusiness does in the case at bar, by indisputable evidence that
of its loans within five years after receiving title to the with the plaintiff consisting of the purchase of partiesassociated and knowingly incurred liabilities under a
same, the law does not prescribe that the property must be lumber,materials, and labor for their buildings and of given name, thelegal presumption is that they are governed
sold for cash or that the purchaser shall be a shareholder in dealing in cotton with itwhich amounted to several tens of by the general rule, andthe burdens is upon them to prove
the corporation. Such sales can of course be made upon thousands of dollars, and theyremained indebted to it over that they fall under some exceptionto it.For the exception
terms and conditions approved by the parties; and when $5,000, of which $4,700.On September 3, 1902, three of to apply, under the general law of Arkansas in forcein the
the association takes a mortgage to secure the deferred the defendants met and signedarticles of incorporation as Indian Territory, the filing of articles of incorporation with
payments, the obligation of the purchaser cannot be fairly the "Coweta Cotton & Milling Company"and a declaration of theclerk of the Court of Appeals was a sine qua non of any
described as arising out of a loan. Nor does the fact that it the purpose of the incorporation, which thestatutes color of alegal corporation. Without that there was not,
is carried as a loan on the books of the respondent make it required to be verified by the signers and to be filed with and there could not be,an apparent corporation or the
a loan on the books of the respondent make it a loan in law. theclerk of the Court of Appeals and with the clerk of the color of a corporation, Agreements toform one, statements
The contention of the Government under this head is judicial district in which the contemplated corporation was that there was one, signed articles of association to make
untenable. to do business. Thisdeclaration was verified by Mann on one, acts as one, created no color of incorporation,
November 10, 1902, and byFrank M. Davis on December 10, because there could be no incorporation or color of itunder
In conclusion, the respondent is enjoined in the future from 1902, and it was filed with the clerkof the Court of Appeals the law until the articles were filed.The defendants never
administering real property not owned by itself, except as on December 22, 1902, and was never filedelsewhere. became a corporation de facto prior toDecember 22, 1902,
may be permitted to it by contract when a borrowing Frank M. Davis, as general manager of the that they never became a corporation de jure,that the
shareholder defaults in his obligation. In all other respects investmentcompany, treated the milling company as a indebtedness here in question was not incurred under
the complaint is dismissed, without costs. So ordered. corporation all the timeduring which this indebtedness was anypromise or assurance of the defendants as promoters
contracted, and never chargedany of it to himself or his that it shouldbecome the obligation of a corporation to be
associates.The Western Investment Company brought this formed, that a large partof it was incurred in the conduct
HARRILL v. DAVIS et al. action for a balancedue It upon an account for lumber and of a general commercial business,and not to prepare for
materials sold, cotton handled,and services rendered to the commencement of such a business or for the
Doctrine: Walter B. Mann, Frank Davis, Robert S.Davis, and James G. organization of a corporation
Parties who actively engage in business for profit under Knight, as partners doing business under thefirm name the
thename and pretense of a corporation which they know "Coweta Cotton & Milling Company. The defendantsdenied
neither exists nor has any color of existence may not the partnership and their liability, and averred that THE CORPORATE ENTITY
escape individualliability because strangers are led by their theindebtedness in question was that of the milling
pretense to con- tractwith their pretended entity as a company and thatthat company was a corporation. G.R. No. L-18216            October 30, 1962
corporation.
Issue/s: STOCKHOLDERS OF F. GUANZON AND SONS,
Color of legal organization as a corporation, such as a Was there ‘colorable’ compliance enough to give the INC., petitioners-appellants,  vs. REGISTER OF DEEDS OF
charter or thefiling of articles of incorporation under some supposed corporation at least the status of a ‘de facto’ MANILA, respondent-appellee.
law, and user of thesupposed corporate franchise in good corporation?
faith, are indispensable to theexistence of a de facto BAUTISTA ANGELO, J.:
corporation which will exempt from individualliability those Held:
who actively conduct it.Neither the execution of articles There was none.The defendants cannot escape individual On September 19, 1960, the five stockholders of the F.
which are not filed, nor statementsnor beliefs of the liability on the ground thatthe Coweta Cotton & Milling Guanzon and Sons, Inc. executed a certificate of liquidation
promoters that they are a corporation, nor thetreatment of Company was a corporation de factowhen that portion of of the assets of the corporation reciting, among other
themselves by themselves and by those who deal withthem the plaintiff's claim was incurred, because it thenhad no things, that by virtue of a resolution of the stockholders
as a corporation, nor all these together, will exempt those color of incorporation, and they knew it and yet, actively adopted on September 17, 1960, dissolving the corporation,
whoactively conduct the business under the assumed name usedits name to incur the obligation.The general rule is they have distributed among themselves in proportion to
of such anonexistent corporation from individual liability that parties who associate themselves together and actively their shareholdings, as liquidating dividends, the assets of
for the debts theyincur. engage in business for profit under any name are liableas said corporation, including real properties located in
partners for the debts they incur under that name. It is Manila.
Facts: anexception to this rule that such associates may escape
The four defendants(Walter B. Mann, Frank Davis, Robert S. individualliability for such debts by a compliance with The certificate of liquidation, when presented to the
Davis,and James G. Knight) agreed in April or June, 1902, incorporation laws or by areal attempt to comply with them Register of Deeds of Manila, was denied registration on
to take specifiedshares in a $10,000 enterprise for the which gives the color of a legalcorporation, and by the user seven grounds, of which the following were disputed by the
purpose of building a cotton ginand carrying on the business of the franchise of such a corporation inthe honest belief stockholders:

Page 76 of 160
CORPORATION LAW CASES MAYORDO, M.A
LLB 3

personal property they do not represent property of the that led to the organization of the defendant corporation,
3. The number of parcels not certified to in the corporation. The corporation has property of its own which plus P10,000.00 attorney's fees; 2
acknowledgment; consists chiefly of real estate (Nelson v. Owen, 113 Ala.,
5. P430.50 Reg. fees need be paid; 372, 21 So. 75; Morrow v. Gould, 145 Iowa 1, 123 N.W. 743). The petitioners claim that this order has no support in fact
6. P940.45 documentary stamps need be attached to the A share of stock only typifies an aliquot part of the and law because they had no contract whatsoever with the
document; corporation's property, or the right to share in its proceeds private respondent regarding the above-mentioned
7. The judgment of the Court approving the dissolution and to that extent when distributed according to law and equity services. Their position is that as mere subsequent
directing the disposition of the assets of the corporation (Hall & Faley v. Alabama Terminal, 173 Ala 398, 56 So., investors in the corporation that was later created, they
need be presented (Rules of Court, Rule 104, Sec. 3). 235), but its holder is not the owner of any part of the should not be held solidarily liable with the Filipinas Orient
capital of the corporation (Bradley v. Bauder 36 Ohio St., Airways, a separate juridical entity, and with Barretto and
Deciding the  consulta  elevated by the stockholders, the 28). Nor is he entitled to the possession of any definite Garcia, their co-defendants in the lower court, ** who were
Commissioner of Land Registration overruled ground No. 7 portion of its property or assets (Gottfried v. Miller, 104 the ones who requested the said services from the private
and sustained requirements Nos. 3, 5 and 6. U.S., 521; Jones v. Davis, 35 Ohio St., 474). The stockholder respondent. 3
is not a co-owner or tenant in common of the corporate
The stockholders interposed the present appeal. property (Halton v. Hohnston, 166 Ala 317, 51 So 992). We are not concerned here with the petitioners' co-
defendants, who have not appealed the decision of the
As correctly stated by the Commissioner of Land On the basis of the foregoing authorities, it is clear that the respondent court and may, for this reason, be presumed to
Registration, the propriety or impropriety of the three act of liquidation made by the stockholders of the F. have accepted the same. For purposes of resolving this case
grounds on which the denial of the registration of the Guanzon and Sons, Inc. of the latter's assets is not and before us, it is not necessary to determine whether it is the
certificate of liquidation was predicated hinges on whether cannot be considered a partition of community property, promoters of the proposed corporation, or the corporation
or not that certificate merely involves a distribution of the but rather a transfer or conveyance of the title of its assets itself after its organization, that shall be responsible for
corporation's assets or should be considered a transfer or to the individual stockholders. Indeed, since the purpose of the expenses incurred in connection with such organization.
conveyance. the liquidation, as well as the distribution of the assets of The only question we have to decide now is whether or not
the corporation, is to transfer their title from the the petitioners themselves are also and personally liable for
Appellants contend that the certificate of liquidation is not corporation to the stockholders in proportion to their such expenses and, if so, to what extent.
a conveyance or transfer but merely a distribution of the shareholdings, — and this is in effect the purpose which
assets of the corporation which has ceased to exist for they seek to obtain from the Register of Deeds of Manila, — The reasons for the said order are given by the respondent
having been dissolved. This is apparent in the minutes for that transfer cannot be effected without the corresponding court in its decision in this wise:
dissolution attached to the document. Not being a deed of conveyance from the corporation to the
conveyance the certificate need not contain a statement of stockholders. It is, therefore, fair and logical to consider As to the 4th assigned error we hold that as to the
the number of parcel of land involved in the distribution in the certificate of liquidation as one in the nature of a remuneration due the plaintiff for the preparation of the
the acknowledgment appearing therein. Hence the amount transfer or conveyance. project study and the pre-organizational services in the
of documentary stamps to be affixed thereon should only amount of P50,000.00, not only the defendant corporation
be P0.30 and not P940.45, as required by the register of WHEREFORE, we affirm the resolution appealed from, with but the other defendants including defendants Caram
deeds. Neither is it correct to require appellants to pay the costs against appellants. should be jointly and severally liable for this amount. As we
amount of P430.50 as registration fee. G.R. No. L-48627 above related it was upon the request of defendants
FERMIN Z. CARAM, JR. and ROSA O. DE CARAM, Barretto and Garcia that plaintiff handled the preparation
The Commissioner of Land Registration, however, petitioners 
 of the project study which project study was presented to
entertained a different opinion. He concurred in the view vs. THE HONORABLE COURT OF APPEALS and ALBERTO V. defendant Caram so the latter was convinced to invest in
expressed by the register of deed to the effect that the ARELLANO, respondents. the proposed airlines. The project study was revised for
certificate of liquidation in question, though it involves a purposes of presentation to financiers and the banks. It was
distribution of the corporation's assets, in the last analysis CRUZ, J.: on the basis of this study that defendant corporation was
represents a transfer of said assets from the corporation to We gave limited due course to this petition on the question actually organized and rendered operational. Defendants
the stockholders. Hence, in substance it is a transfer or of the solidary liability of the petitioners with their co- Garcia and Caram, and Barretto became members of the
conveyance. defendants in the lower court 1 because of the challenge to Board and/or officers of defendant corporation. Thus, not
the following paragraph in the dispositive portion of the only the defendant corporation but all the other
We agree with the opinion of these two officials. A decision of the respondent court: * defendants who were involved in the preparatory stages of
corporation is a juridical person distinct from the members the incorporation, who caused the preparation and/or
composing it. Properties registered in the name of the 1. Defendants are hereby ordered to jointly and severally benefited from the project study and the technical services
corporation are owned by it as an entity separate and pay the plaintiff the amount of P50,000.00 for the of plaintiff must be liable. 4
distinct from its members. While shares of stock constitute preparation of the project study and his technical services

Page 77 of 160
CORPORATION LAW CASES MAYORDO, M.A
LLB 3

It would appear from the above justification that the reconsideration of our Resolution dated September 24, 2003
petitioners were not really involved in the initial steps that In the light of these circumstances, we hold that the and to elevate this case to the Court En Banc. The
finally led to the incorporation of the Filipinas Orient petitioners cannot be held personally liable for the petitioner questions the Resolution which reversed our
Airways. Elsewhere in the decision, Barretto was described compensation claimed by the private respondent for the Decision of November 20, 2000, which in turn reversed and
as "the moving spirit." The finding of the respondent court services performed by him in the organization of the set aside a Decision of the Court of Appeals promulgated on
is that the project study was undertaken by the private corporation. To repeat, the petitioners did not contract July 18, 1995.
respondent at the request of Barretto and Garcia who, such services. It was only the results of such services that
upon its completion, presented it to the petitioners to Barretto and Garcia presented to them and which I. Facts
induce them to invest in the proposed airline. The study persuaded them to invest in the proposed airline. The most
could have been presented to other prospective investors. that can be said is that they benefited from such services, The undisputed facts of the case, as set forth in our
At any rate, the airline was eventually organized on the but that surely is no justification to hold them personally Resolution of September 24, 2003, are as follows:
basis of the project study with the petitioners as major liable therefor. Otherwise, all the other stockholders of the
stockholders and, together with Barretto and Garcia, as corporation, including those who came in later, and On January 27, 1997, the National Investment and
principal officers. regardless of the amount of their share holdings, would be Development Corporation (NIDC), a government
equally and personally liable also with the petitioners for corporation, entered into a Joint Venture Agreement (JVA)
The following portion of the decision in question is also the claims of the private respondent. with Kawasaki Heavy Industries, Ltd. of Kobe, Japan
worth considering: (KAWASAKI) for the construction, operation and
The petition is rather hazy and seems to be flawed by an management of the Subic National Shipyard, Inc. (SNS)
... Since defendant Barretto was the moving spirit in the ambiguous ambivalence. Our impression is that it is which subsequently became the Philippine Shipyard and
pre-organization work of defendant corporation based on opposed to the imposition of solidary responsibility upon Engineering Corporation (PHILSECO). Under the JVA, the
his experience and expertise, hence he was logically the Carams but seems to be willing, in a vague, NIDC and KAWASAKI will contribute P330 million for the
compensated in the amount of P200,000.00 shares of stock unexpressed offer of compromise, to accept joint liability. capitalization of PHILSECO in the proportion of 60%-40%
not as industrial partner but more for his technical services While it is true that it does here and there disclaim total respectively. One of its salient features is the grant to the
that brought to fruition the defendant corporation. By the liability, the thrust of the petition seems to be against the parties of the right of first refusal should either of them
same token, We find no reason why the plaintiff should not imposition of solidary liability only rather than against any decide to sell, assign or transfer its interest in the joint
be similarly compensated not only for having actively liability at all, which is what it should have categorically venture, viz:
participated in the preparation of the project study for argued.
several months and its subsequent revision but also in his 1.4 Neither party shall sell, transfer or assign all or any part
having been involved in the pre-organization of the Categorically, the Court holds that the petitioners are not of its interest in SNS [PHILSECO] to any third party without
defendant corporation, in the preparation of the franchise, liable at all, jointly or jointly and severally, under the first giving the other under the same terms the right of first
in inviting the interest of the financiers and in the training paragraph of the dispositive portion of the challenged refusal. This provision shall not apply if the transferee is a
and screening of personnel. We agree that for these special decision. So holding, we find it unnecessary to examine at corporation owned or controlled by the GOVERNMENT or by
services of the plaintiff the amount of P50,000.00 as this time the rules on solidary obligations, which the a KAWASAKI affiliate.
compensation is reasonable. 5 parties-needlessly, as it turns out have belabored unto
death. On November 25, 1986, NIDC transferred all its rights, title
The above finding bolsters the conclusion that the and interest in PHILSECO to the Philippine National Bank
petitioners were not involved in the initial stages of the WHEREFORE, the petition is granted. The petitioners are (PNB). Such interests were subsequently transferred to the
organization of the airline, which were being directed by declared not liable under the challenged decision, which is National Government pursuant to Administrative Order No.
Barretto as the main promoter. It was he who was putting hereby modified accordingly. It is so ordered. 14. On December 8, 1986, President Corazon C. Aquino
all the pieces together, so to speak. The petitioners were issued Proclamation No. 50 establishing the Committee on
merely among the financiers whose interest was to be [G.R. No. 124293. January 31, 2005] Privatization (COP) and the Asset Privatization Trust (APT)
invited and who were in fact persuaded, on the strength of J.G. SUMMIT HOLDINGS, INC., petitioner, vs. COURT OF to take title to, and possession of, conserve, manage and
the project study, to invest in the proposed airline. APPEALS; COMMITTEE ON PRIVATIZATION, its Chairman dispose of non-performing assets of the National
and Members; ASSET PRIVATIZATION TRUST; and Government. Thereafter, on February 27, 1987, a trust
Significantly, there was no showing that the Filipinas Orient PHILYARDS HOLDINGS, INC., respondents. agreement was entered into between the National
Airways was a fictitious corporation and did not have a Government and the APT wherein the latter was named the
separate juridical personality, to justify making the RESOLUTION trustee of the National Government's share in PHILSECO. In
petitioners, as principal stockholders thereof, responsible PUNO, J.: 1989, as a result of a quasi-reorganization of PHILSECO to
for its obligations. As a bona fide corporation, the Filipinas settle its huge obligations to PNB, the National
Orient Airways should alone be liable for its corporate acts For resolution before this Court are two motions filed by Government's shareholdings in PHILSECO increased to
as duly authorized by its officers and directors. t h e p e t i t i o n e r, J . G . S u m m i t H o l d i n g s , I n c . f o r

Page 78 of 160
CORPORATION LAW CASES MAYORDO, M.A
LLB 3

97.41% thereby reducing KAWASAKI's shareholdings to should be endorsed by the APT Board of Trustees to the the COP approve the highest bid, APT shall advise Kawasaki
2.59%. COP, and the latter approves the same. The APT shall advise Heavy Industries, Inc. and/or its nominee, [PHILYARDS]
Kawasaki Heavy Industries, Inc. and/or its nominee, Holdings, Inc. that the highest bid is acceptable to the
In the interest of the national economy and the [PHILYARDS] Holdings, Inc., that the highest bid is National Government. Kawasaki Heavy Industries, Inc. and/
government, the COP and the APT deemed it best to sell acceptable to the National Government. Kawasaki Heavy or [PHILYARDS] Holdings, Inc. shall then have a period of
the National Government's share in PHILSECO to private Industries, Inc. and/or [PHILYARDS] Holdings, Inc. shall then thirty (30) calendar days from the date of receipt of such
entities. After a series of negotiations between the APT and have a period of thirty (30) calendar days from the date of advice from APT within which to exercise their "Option to
KAWASAKI, they agreed that the latter's right of first refusal receipt of such advice from APT within which to exercise Top the Highest Bid" by offering a bid equivalent to the
under the JVA be "exchanged" for the right to top by five their "Option to Top the Highest Bid" by offering a bid highest bid plus five (5%) percent thereof.
percent (5%) the highest bid for the said shares. They equivalent to the highest bid plus five (5%) percent thereof.
further agreed that KAWASAKI would be entitled to name a As petitioner was declared the highest bidder, the COP
company in which it was a stockholder, which could 6.1 Should Kawasaki Heavy Industries, Inc. and/or approved the sale on December 3, 1993 "subject to the
exercise the right to top. On September 7, 1990, KAWASAKI [PHILYARDS] Holdings, Inc. exercise their "Option to Top the right of Kawasaki Heavy Industries, Inc./[PHILYARDS]
informed APT that Philyards Holdings, Inc. (PHI)[1] would Highest Bid," they shall so notify the APT about such Holdings, Inc. to top JGSMI's bid by 5% as specified in the
exercise its right to top. exercise of their option and deposit with APT the amount bidding rules."
equivalent to ten percent (10%) of the highest bid plus five
At the pre-bidding conference held on September 18, 1993, percent (5%) thereof within the thirty (30)-day period On December 29, 1993, petitioner informed APT that it was
interested bidders were given copies of the JVA between mentioned in paragraph 6.0 above. APT will then serve protesting the offer of PHI to top its bid on the grounds
NIDC and KAWASAKI, and of the Asset Specific Bidding Rules notice upon Kawasaki Heavy Industries, Inc. and/or that: (a) the KAWASAKI/PHI consortium composed of
(ASBR) drafted for the National Government's 87.6% equity [PHILYARDS] Holdings, Inc. declaring them as the preferred KAWASAKI, [PHILYARDS], Mitsui, Keppel, SM Group, ICTSI and
share in PHILSECO. The provisions of the ASBR were bidder and they shall have a period of ninety (90) days from Insular Life violated the ASBR because the last four (4)
explained to the interested bidders who were notified that the receipt of the APT's notice within which to pay the companies were the losing bidders thereby circumventing
the bidding would be held on December 2, 1993. A portion balance of their bid price. the law and prejudicing the weak winning bidder; (b) only
of the ASBR reads: KAWASAKI could exercise the right to top; (c) giving the
6.2 Should Kawasaki Heavy Industries, Inc. and/or same option to top to PHI constituted unwarranted benefit
1.0 The subject of this Asset Privatization Trust (APT) sale [PHILYARDS] Holdings, Inc. fail to exercise their "Option to to a third party; (d) no right of first refusal can be
through public bidding is the National Government's equity Top the Highest Bid" within the thirty (30)-day period, APT exercised in a public bidding or auction sale; and (e) the JG
in PHILSECO consisting of 896,869,942 shares of stock will declare the highest bidder as the winning bidder. Summit consortium was not estopped from questioning the
(representing 87.67% of PHILSECO's outstanding capital proceedings.
stock), which will be sold as a whole block in accordance xxx xxxxxx
with the rules herein enumerated. On February 2, 1994, petitioner was notified that PHI had
12.0 The bidder shall be solely responsible for examining fully paid the balance of the purchase price of the subject
xxx xxxxxx with appropriate care these rules, the official bid forms, bidding. On February 7, 1994, the APT notified petitioner
including any addenda or amendments thereto issued that PHI had exercised its option to top the highest bid and
2.0 The highest bid, as well as the buyer, shall be subject during the bidding period. The bidder shall likewise be that the COP had approved the same on January 6, 1994.
to the final approval of both the APT Board of Trustees and responsible for informing itself with respect to any and all On February 24, 1994, the APT and PHI executed a Stock
the Committee on Privatization (COP). conditions concerning the PHILSECO Shares which may, in Purchase Agreement. Consequently, petitioner filed with
any manner, affect the bidder's proposal. Failure on the this Court a Petition for Mandamus under G.R. No. 114057.
2.1 APT reserves the right in its sole discretion, to reject part of the bidder to so examine and inform itself shall be On May 11, 1994, said petition was referred to the Court of
any or all bids. its sole risk and no relief for error or omission will be given Appeals. On July 18, 1995, the Court of Appeals denied the
by APT or COP. . . . same for lack of merit. It ruled that the petition for
3.0 This public bidding shall be on an Indicative Price mandamus was not the proper remedy to question the
Bidding basis. The Indicative price set for the National At the public bidding on the said date, petitioner J.G. constitutionality or legality of the right of first refusal and
Government's 87.67% equity in PHILSECO is PESOS: ONE Summit Holdings, Inc.[2] submitted a bid of Two Billion and the right to top that was exercised by KAWASAKI/PHI, and
BILLION THREE HUNDRED MILLION (P1,300,000,000.00). Thirty Million Pesos (P2,030,000,000.00) with an that the matter must be brought "by the proper party in the
acknowledgment of KAWASAKI/[PHILYARDS'] right to top, proper forum at the proper time and threshed out in a full
xxx xxxxxx viz: blown trial." The Court of Appeals further ruled that the
right of first refusal and the right to top are prima facie
6.0 The highest qualified bid will be submitted to the APT 4. I/We understand that the Committee on Privatization legal and that the petitioner, "by participating in the public
Board of Trustees at its regular meeting following the (COP) has up to thirty (30) days to act on APT's bidding, with full knowledge of the right to top granted to
bidding, for the purpose of determining whether or not it recommendation based on the result of this bidding. Should KAWASAKI/[PHILYARDS] isestopped from questioning the

Page 79 of 160
CORPORATION LAW CASES MAYORDO, M.A
LLB 3

validity of the award given to [PHILYARDS] after the latter (d) return to private respondent PHGI the amount of Two 2. Whether the motion for reconsideration raises any new
exercised the right to top and had paid in full the purchase Billion One Hundred Thirty-One Million Five Hundred matter or cogent reason to warrant a reconsideration of
price of the subject shares, pursuant to the ASBR." Thousand Pesos (P2,131,500,000.00); and this Courts Resolution of September 24, 2003.
Petitioner filed a Motion for Reconsideration of said
Decision which was denied on March 15, 1996. Petitioner (e) cause the cancellation of the stock certificates issued to Motion to Elevate this Case to the
thus filed a Petition for Certiorari with this Court alleging PHI.
grave abuse of discretion on the part of the appellate Court En Banc
court. SO ORDERED.
The petitioner prays for the elevation of the case to the
On November 20, 2000, this Court rendered x xx [a] In separate Motions for Reconsideration, respondents Court en banc on the following grounds:
Decision ruling among others that the Court of Appeals submit[ted] three basic issues for x xx resolution: (1)
erred when it dismissed the petition on the sole ground of Whether PHILSECO is a public utility; (2) Whether under the 1. The main issue of the propriety of the bidding process
the impropriety of the special civil action of mandamus 1977 JVA, KAWASAKI can exercise its right of first refusal involved in the present case has been confused with the
because the petition was also one of certiorari. It further only up to 40% of the total capitalization of PHILSECO; and policy issue of the supposed fate of the shipping industry
ruled that a shipyard like PHILSECO is a public utility whose (3) Whether the right to top granted to KAWASAKI violates which has never been an issue that is determinative of this
capitalization must be sixty percent (60%) Filipino-owned. the principles of competitive bidding.[3] (citations omitted) case.[10]
Consequently, the right to top granted to KAWASAKI under
the Asset Specific Bidding Rules (ASBR) drafted for the sale In a Resolution dated September 24, 2003, this Court ruled 2. The present case may be considered under the Supreme
of the 87.67% equity of the National Government in in favor of the respondents. On the first issue, we held that Court Resolution dated February 23, 1984 which included
PHILSECO is illegal not only because it violates the rules on Philippine Shipyard and Engineering Corporation (PHILSECO) among en banc cases those involving a novel question of
competitive bidding but more so, because it allows foreign is not a public utility, as by nature, a shipyard is not a law and those where a doctrine or principle laid down by
corporations to own more than 40% equity in the shipyard. public utility[4] and that no law declares a shipyard to be a the Court en banc or in division may be modified or
It also held that "although the petitioner had the public utility.[5] On the second issue, we found nothing in reversed.[11]
opportunity to examine the ASBR before it participated in the 1977 Joint Venture Agreement (JVA) which prevents
the bidding, it cannot be estopped from questioning the Kawasaki Heavy Industries, Ltd. of Kobe, Japan (KAWASAKI) 3. There was clear executive interference in the judicial
unconstitutional, illegal and inequitable provisions from acquiring more than 40% of PHILSECOs total functions of the Court when the Honorable Jose Isidro
thereof." Thus, this Court voided the transfer of the capitalization.[6] On the final issue, we held that the right Camacho, Secretary of Finance, forwarded to Chief Justice
national government's 87.67% share in PHILSECO to to top granted to KAWASAKI in exchange for its right of first Davide, a memorandum dated November 5, 2001, attaching
Philyard[s] Holdings, Inc., and upheld the right of JG refusal did not violate the principles of competitive a copy of the Foreign Chambers Report dated October 17,
Summit, as the highest bidder, to take title to the said bidding.[7] 2001, which matter was placed in the agenda of the Court
shares, viz: and noted by it in a formal resolution dated November 28,
On October 20, 2003, the petitioner filed a Motion for 2001.[12]
WHEREFORE, the instant petition for review on certiorari is Reconsideration[8] and a Motion to Elevate This Case to the
GRANTED. The assailed Decision and Resolution of the Court Court En Banc.[9] Public respondents Committee on Opposing J.G. Summits motion to elevate the case en banc,
of Appeals are REVERSED and SET ASIDE. Petitioner is Privatization (COP) and Asset Privatization Trust (APT), and PHILYARDS points out the petitioners inconsistency in
ordered to pay to APT its bid price of Two Billion Thirty private respondent Philyards Holdings, Inc. (PHILYARDS) previously opposing PHILYARDS Motion to Refer the Case to
Million Pesos (P2,030,000,000.00), less its bid deposit plus filed their Comments on J.G. Summit Holdings, Inc.s (JG the Court En Banc. PHILYARDS contends that J.G. Summit
interests upon the finality of this Decision. In turn, APT is Summits) Motion for Reconsideration and Motion to Elevate should now be estopped from asking that the case be
ordered to: This Case to the Court En Banc on January 29, 2004 and referred to the Court en banc. PHILYARDS further contends
February 3, 2004, respectively. that the Supreme Court en banc is not an appellate court to
(a) accept the said amount of P2,030,000,000.00 less bid which decisions or resolutions of its divisions may be
deposit and interests from petitioner; II. Issues appealed citing Supreme Court Circular No. 2-89 dated
February 7, 1989.[13] PHILYARDS also alleges that there is
(b) execute a Stock Purchase Agreement with petitioner; Based on the foregoing, the relevant issues to resolve to no novel question of law involved in the present case as the
end this litigation are the following: a s s a i l e d Re s o l u t i o n w a s b a s e d o n w e l l - s e t t l e d
(c) cause the issuance in favor of petitioner of the jurisprudence. Likewise, PHILYARDS stresses that the
certificates of stocks representing 87.6% of PHILSECO's total 1. Whether there are sufficient bases to elevate the case at Resolution was merely an outcome of the motions for
capitalization; bar to the Court en banc. reconsideration filed by it and the COP and APT and is
consistent with the inherent power of courts to amend and
control its process and orders so as to make them
conformable to law and justice. (Rule 135, sec. 5)[14]

Page 80 of 160
CORPORATION LAW CASES MAYORDO, M.A
LLB 3

Private respondent belittles the petitioners allegations The right to top was merely a condition or a reservation awarded the CISS contract to Respondent SGS. In the
regarding the change in ponente and the alleged executive made in the bidding rules which was fully disclosed to all "Invitation to Prequalify and Bid" (Annex "C," supra), the
interference as shown by former Secretary of Finance Jose bidding parties. In Bureau Veritas, represented by Theodor CISS Committee made an express reservation of the right of
Isidro Camachos memorandum dated November 5, 2001 H. Hunermann v. Office of the President, et al., [19]we the Government to "reject any or all bids or any part
arguing that these do not justify a referral of the present dealt with this conditionality, viz: thereof or waive any defects contained thereon and accept
case to the Court en banc. an offer most advantageous to the Government." It is a
x xx It must be stressed, as held in the case of A.C. well-settled rule that where such reservation is made in an
In insisting that its Motion to Elevate This Case to the Court Esguerra & Sons v. Aytona, et al., (L-18751, 28 April 1962, 4 Invitation to Bid, the highest or lowest bidder, as the case
En Banc should be granted, J.G. Summit further argued SCRA 1245), that in an "invitation to bid, there is a may be, is not entitled to an award as a matter of right (C
that: its Opposition to the Office of the Solicitor Generals condition imposed upon the bidders to the effect that the & C Commercial Corp. v. Menor, L-28360, 27 January 1983,
Motion to Refer is different from its own Motion to Elevate; bidding shall be subject to the right of the government to 120 SCRA 112). Even the lowest Bid or any Bid may be
different grounds are invoked by the two motions; there reject any and all bids subject to its discretion. In the case rejected or, in the exercise of sound discretion, the award
was unwarranted executive interference; and the change in at bar, the government has made its choice and unless an may be made to another than the lowest bidder (A.C.
ponente is merely noted in asserting that this case should unfairness or injustice is shown, the losing bidders have no Esguerra & Sons v. Aytona, supra, citing 43 Am. Jur., 788).
be decided by the Court en banc.[15] cause to complain nor right to dispute that choice. This is a (emphases supplied)
well-settled doctrine in this jurisdiction and elsewhere."
We find no merit in petitioners contention that the Like the condition in the Bureau Veritas case, the right to
propriety of the bidding process involved in the present The discretion to accept or reject a bid and award top was a condition imposed by the government in the
case has been confused with the policy issue of the fate of contracts is vested in the Government agencies entrusted bidding rules which was made known to all parties. It was a
the shipping industry which, petitioner maintains, has with that function. The discretion given to the authorities condition imposed on all bidders equally, based on the APTs
never been an issue that is determinative of this case. The on this matter is of such wide latitude that the Courts will exercise of its discretion in deciding on how best to
Courts Resolution of September 24, 2003 reveals a clear not interfere therewith, unless it is apparent that it is used privatize the governments shares in PHILSECO. It was not a
and definitive ruling on the propriety of the bidding as a shield to a fraudulent award (Jalandoni v. NARRA, 108 whimsical or arbitrary condition plucked from the ether and
process. In discussing whether the right to top granted to Phil. 486 [1960]). x xxThe exercise of this discretion is a inserted in the bidding rules but a condition which the APT
KAWASAKI in exchange for its right of first refusal violates policy decision that necessitates prior inquiry, approved as the best way the government could comply
the principles of competitive bidding, we made an investigation, comparison, evaluation, and deliberation. with its contractual obligations to KAWASAKI under the JVA
exhaustive discourse on the rules and principles of public This task can best be discharged by the Government and its mandate of getting the most advantageous deal for
bidding and whether they were complied with in the case agencies concerned, not by the Courts. The role of the the government. The right to top had its history in the
at bar.[16] This Court categorically ruled on the petitioners Courts is to ascertain whether a branch or instrumentality mutual right of first refusal in the JVA and was reached by
argument that PHILSECO, as a shipyard, is a public utility of the Government has transgressed its constitutional agreement of the government and KAWASAKI.
which should maintain a 60%-40% Filipino-foreign equity boundaries. But the Courts will not interfere with executive
ratio, as it was a pivotal issue. In doing so, we recognized or legislative discretion exercised within those boundaries. Further, there is no executive interference in the functions
the impact of our ruling on the shipbuilding industry which Otherwise, it strays into the realm of policy decision- of this Court by the mere filing of a memorandum by
was beyond avoidance.[17] making. Secretary of Finance Jose Isidro Camacho. The
memorandum was merely noted to acknowledge its filing. It
We reject petitioners argument that the present case may It is only upon a clear showing of grave abuse of discretion had no further legal significance. Notably too, the assailed
be considered under the Supreme Court Resolution dated that the Courts will set aside the award of a contract made Resolution dated September 24, 2003 was decided
February 23, 1984 which included among en banc cases by a government entity. Grave abuse of discretion implies a unanimously by the Special First Division in favor of the
those involving a novel question of law and those where a capricious, arbitrary and whimsical exercise of power respondents.
doctrine or principle laid down by the court en banc or in (Filinvest Credit Corp. v. Intermediate Appellate Court, No.
division may be modified or reversed. The case was 65935, 30 September 1988, 166 SCRA 155). The abuse of Again, we emphasize that a decision or resolution of a
resolved based on basic principles of the right of first discretion must be so patent and gross as to amount to an Division is that of the Supreme Court[20] and the Court en
refusal in commercial law and estoppel in civil law. evasion of positive duty or to a virtual refusal to perform a banc is not an appellate court to which decisions or
Contractual obligations arising from rights of first refusal duty enjoined by law, as to act at all in contemplation of resolutions of a Division may be appealed.[21]
are not new in this jurisdiction and have been recognized in law, where the power is exercised in an arbitrary and
numerous cases.[18] Estoppel is too known a civil law despotic manner by reason of passion or hostility (Litton For all the foregoing reasons, we find no basis to elevate
concept to require an elongated discussion. Fundamental Mills, Inc. v. Galleon Trader, Inc., et al[.], L-40867, 26 July this case to the Court en banc.
principles on public bidding were likewise used to resolve 1988, 163 SCRA 489).
the issues raised by the petitioner. To be sure, petitioner Motion for Reconsideration
leans on the right to top in a public bidding in arguing that The facts in this case do not indicate any such grave abuse
the case at bar involves a novel issue. We are not swayed. of discretion on the part of public respondents when they

Page 81 of 160
CORPORATION LAW CASES MAYORDO, M.A
LLB 3

Three principal arguments were raised in the petitioners top and it was of no moment that losing bidders later c. The benefits derived from the right to top were
Motion for Reconsideration. First, that a fair resolution of joined PHILYARDS in raising the purchase price.[32] unwarranted.
the case should be based on contract law, not on policy
considerations; the contracts do not authorize the right to In cadence with the private respondent PHILYARDS, public 2. The landholding issue has been a legitimate issue since
top to be derived from the right of first refusal.[22] respondents COP and APT contend: the start of this case but is shamelessly ignored by the
Second, that neither the right of first refusal nor the right respondents.
to top can be legally exercised by the consortium which is 1. The conversion of the right of first refusal into a right to
not the proper party granted such right under either the top by 5% does not violate any provision in the JVA between a. The landholding issue is not a non-issue.
JVA or the Asset Specific Bidding Rules (ASBR).[23] Third, NIDC and KAWASAKI.
that the maintenance of the 60%-40% relationship between b. The landholding issue does not pose questions of fact.
the National Investment and Development Corporation 2. PHILSECO is not a public utility and therefore not
(NIDC) and KAWASAKI arises from contract and from the governed by the constitutional restriction on foreign c. That PHILSECO owned land at the time that the right of
Constitution because PHILSECO is a landholding corporation ownership. first refusal was agreed upon and at the time of the bidding
and need not be a public utility to be bound by the 60%-40% are most relevant.
constitutional limitation.[24] 3. The petitioner is legally estopped from assailing the
validity of the proceedings of the public bidding as it d. Whether a shipyard is a public utility is not the core
On the other hand, private respondent PHILYARDS asserts voluntarily submitted itself to the terms of the ASBR which issue in this case.
that J.G. Summit has not been able to show compelling included the provision on the right to top.
reasons to warrant a reconsideration of the Decision of the 3. Fraud and bad faith attend the alleged conversion of an
Court.[25] PHILYARDS denies that the Decision is based 4. The right to top was exercised by PHILYARDS as the inexistent right of first refusal to the right to top.
mainly on policy considerations and points out that it is nominee of KAWASAKI and the fact that PHILYARDS formed a
premised on principles governing obligations and contracts consortium to raise the required amount to exercise the a. The history behind the birth of the right to top shows
and corporate law such as the rule requiring respect for right to top the highest bid by 5% does not violate the JVA fraud and bad faith.
contractual stipulations, upholding rights of first refusal, or the ASBR.
and recognizing the assignable nature of contracts rights. b. The right of first refusal was, indeed, effectively useless.
[26] Also, the ruling that shipyards are not public utilities 5. The 60%-40% Filipino-foreign constitutional requirement
relies on established case law and fundamental rules of for the acquisition of lands does not apply to PHILSECO 4. Petitioner is not legally estopped to challenge the right
statutory construction. PHILYARDS stresses that KAWASAKIs because as admitted by petitioner itself, PHILSECO no to top in this case.
right of first refusal or even the right to top is not limited longer owns real property.
to the 40% equity of the latter.[27] On the landholding issue a. Estoppel is unavailing as it would stamp validity to an act
raised by J.G. Summit, PHILYARDS emphasizes that this is a 6. Petitioners motion to elevate the case to the Court en that is prohibited by law or against public policy.
non-issue and even involves a question of fact. Even banc is baseless and would only delay the termination of
assuming that this Court can take cognizance of such this case.[33] b. Deception was patent; the right to top was an attractive
question of fact even without the benefit of a trial, nuisance.
PHILYARDS opines that landholding by PHILSECO at the time In a Consolidated Comment dated March 8, 2004, J.G.
of the bidding is irrelevant because what is essential is that Summit countered the arguments of the public and private c. The 10% bid deposit was placed in escrow.
ultimately a qualified entity would eventually hold respondents in this wise:
PHILSECOs real estate properties.[28] Further, given the J.G. Summits insistence that the right to top cannot be
assignable nature of the right of first refusal, any 1. The award by the APT of 87.67% shares of PHILSECO to sourced from the right of first refusal is not new and we
applicable nationality restrictions, including landholding PHILYARDS with losing bidders through the exercise of a have already ruled on the issue in our Resolution of
limitations, would not affect the right of first refusal itself, right to top, which is contrary to law and the constitution is September 24, 2003. We upheld the mutual right of first
but only the manner of its exercise.[29] Also, PHILYARDS null and void for being violative of substantive due process refusal in the JVA.[34] We also ruled that nothing in the JVA
argues that if this Court takes cognizance of J.G. Summits and the abuse of right provision in the Civil Code. prevents KAWASAKI from acquiring more than 40% of
allegations of fact regarding PHILSECOs landholding, it must PHILSECOs total capitalization.[35] Likewise, nothing in the
also recognize PHILYARDS assertions that PHILSECOs a. The bidders[] right to top was actually exercised by JVA or ASBR bars the conversion of the right of first refusal
landholdings were sold to another corporation.[30] As losing bidders. to the right to top. In sum, nothing new and of significance
regards the right of first refusal, private respondent in the petitioners pleading warrants a reconsideration of
explains that KAWASAKIs reduced shareholdings (from 40% b. The right to top or the right of first refusal cannot co- our ruling.
to 2.59%) did not translate to a deprivation or loss of its exist with a genuine competitive bidding.
contractually granted right of first refusal.[31] Also, the Likewise, we already disposed of the argument that neither
bidding was valid because PHILYARDS exercised the right to the right of first refusal nor the right to top can legally be

Page 82 of 160
CORPORATION LAW CASES MAYORDO, M.A
LLB 3

exercised by the consortium which is not the proper party constitutional provision on landholdings as its shares are separate juridical entities. In this vein, the right of first
granted such right under either the JVA or the ASBR. Thus, more than 40% foreign-owned.[38] PHILYARDS admits that it refusal over shares pertains to the shareholders whereas
we held: may have previously held land but had already divested the capacity to own land pertains to the corporation.
such landholdings.[39] It contends, however, that even if Hence, the fact that PHILSECO owns land cannot deprive
The fact that the losing bidder, Keppel Consortium PHILSECO owned land, this would not affect the right of stockholders of their right of first refusal. No law
(composed of Keppel, SM Group, Insular Life Assurance, first refusal but only the exercise thereof. If the land is disqualifies a person from purchasing shares in a
Mitsui and ICTSI), has joined PHILYARDS in the latter's effort retained, the right of first refusal, being a property right, landholding corporation even if the latter will exceed the
to raise P2.131 billion necessary in exercising the right to could be assigned to a qualified party. In the alternative, allowed foreign equity, what the law disqualifies is the
top is not contrary to law, public policy or public morals. the land could be divested before the exercise of the right corporation from owning land. This is the clear import of
There is nothing in the ASBR that bars the losing bidders of first refusal. In the case at bar, respondents assert that the following provisions in the Constitution:
from joining either the winning bidder (should the right to since the right of first refusal was validly converted into a
top is not exercised) or KAWASAKI/PHI (should it exercise its right to top, which was exercised not by KAWASAKI, but by Section 2. All lands of the public domain, waters, minerals,
right to top as it did), to raise the purchase price. The PHILYARDS which is a Filipino corporation (i.e., 60% of its coal, petroleum, and other mineral oils, all forces of
petitioner did not allege, nor was it shown by competent shares are owned by Filipinos), then there is no violation of potential energy, fisheries, forests or timber, wildlife, flora
evidence, that the participation of the losing bidders in the the Constitution.[40] At first, it would seem that questions and fauna, and other natural resources are owned by the
public bidding was done with fraudulent intent. Absent any of fact beyond cognizance by this Court were involved in State. With the exception of agricultural lands, all other
proof of fraud, the formation by [PHILYARDS] of a the issue. However, the records show that PHILYARDS natural resources shall not be alienated. The exploration,
consortium is legitimate in a free enterprise system. The admits it had owned land up until the time of the bidding. development, and utilization of natural resources shall be
appellate court is thus correct in holding the petitioner [41] Hence, the only issue is whether KAWASAKI had a valid under the full control and supervision of the State. The
estopped from questioning the validity of the transfer of right of first refusal over PHILSECO shares under the JVA State may directly undertake such activities, or it may
the National Government's shares in PHILSECO to considering that PHILSECO owned land until the time of the enter into co-production, joint venture, or production-
respondent.[36] bidding and KAWASAKI already held 40% of PHILSECOs sharing agreements with Filipino citizens, or corporations
equity. or associations at least sixty per centum of whose capital is
Further, we see no inherent illegality on PHILYARDS act in owned by such citizens. Such agreements may be for a
seeking funding from parties who were losing bidders. This We uphold the validity of the mutual rights of first refusal period not exceeding twenty-five years, renewable for not
is a purely commercial decision over which the State should under the JVA between KAWASAKI and NIDC. First of all, the more than twenty-five years, and under such terms and
not interfere absent any legal infirmity. It is emphasized right of first refusal is a property right of PHILSECO conditions as may be provided by law. In cases of water
that the case at bar involves the disposition of shares in a shareholders, KAWASAKI and NIDC, under the terms of their rights for irrigation, water supply, fisheries, or industrial
corporation which the government sought to privatize. As JVA. This right allows them to purchase the shares of their uses other than the development of water power, beneficial
such, the persons with whom PHILYARDS desired to enter co-shareholder before they are offered to a third party. The use may be the measure and limit of the grant.
into business with in order to raise funds to purchase the agreement of co-shareholders to mutually grant this right
shares are basically its business. This is in contrast to a to each other, by itself, does not constitute a violation of xxx xxxxxx
case involving a contract for the operation of or the provisions of the Constitution limiting land ownership to
construction of a government infrastructure where the Filipinos and Filipino corporations. As PHILYARDS correctly Section 7. Save in cases of hereditary succession, no private
identity of the buyer/bidder or financier constitutes an puts it, if PHILSECO still owns land, the right of first refusal lands shall be transferred or conveyed except to
important consideration. In such cases, the government can be validly assigned to a qualified Filipino entity in order individuals, corporations, or associations qualified to
would have to take utmost precaution to protect public to maintain the 60%-40% ratio. This transfer, by itself, does acquire or hold lands of the public domain.[42] (emphases
interest by ensuring that the parties with which it is not amount to a violation of the Anti-Dummy Laws, absent supplied)
contracting have the ability to satisfactorily construct or proof of any fraudulent intent. The transfer could be made
operate the infrastructure. either to a nominee or such other party which the holder of The petitioner further argues that an option to buy land is
the right of first refusal feels it can comfortably do void in itself (Philippine Banking Corporation v. Lui She, 21
On the landholding issue, J.G. Summit submits that since business with. Alternatively, PHILSECO may divest of its SCRA 52 [1967]). The right of first refusal granted to
PHILSECO is a landholding company, KAWASAKI could landholdings, in which case KAWASAKI, in exercising its KAWASAKI, a Japanese corporation, is similarly void. Hence,
exercise its right of first refusal only up to 40% of the right of first refusal, can exceed 40% of PHILSECOs equity. the right to top, sourced from the right of first refusal, is
shares of PHILSECO due to the constitutional prohibition on In fact, it can even be said that if the foreign shareholdings also void.[43] Contrary to the contention of petitioner, the
landholding by corporations with more than 40% foreign- of a landholding corporation exceeds 40%, it is not the case of Lui She did not that say an option to buy land is
owned equity. It further argues that since KAWASAKI already foreign stockholders ownership of the shares which is void in itself, for we ruled as follows:
held at least 40% equity in PHILSECO, the right of first adversely affected but the capacity of the corporation to
refusal was inutile and as such, could not subsequently be own land that is, the corporation becomes disqualified to x xxTo be sure, a lease to an alien for a reasonable period
converted into the right to top. [37] Petitioner also asserts own land. This finds support under the basic corporate law is valid. So is an option giving an alien the right to buy real
that, at present, PHILSECO continues to violate the principle that the corporation and its stockholders are property on condition that he is granted Philippine

Page 83 of 160
CORPORATION LAW CASES MAYORDO, M.A
LLB 3

citizenship. As this Court said in Krivenko vs. Register of as its foreign equity is above 40% and yet owns long-term WHEREFORE, in view of the foregoing, the petitioners
Deeds: leasehold rights which are real rights.[45] It cites Article Motion for Reconsideration is DENIED WITH FINALITY and
415 of the Civil Code which includes in the definition of the decision appealed from is AFFIRMED. The Motion to
[A]liens are not completely excluded by the Constitution immovable property, contracts for public works, and Elevate This Case to the Court En Banc is likewise DENIED
from the use of lands for residential purposes. Since their servitudes and other real rights over immovable property. for lack of merit.
residence in the Philippines is temporary, they may be [46] Any existing landholding, however, is denied by
granted temporary rights such as a lease contract which is PHILYARDS citing its recent financial statements.[47] First, SO ORDERED.
not forbidden by the Constitution. Should they desire to these are questions of fact, the veracity of which would
remain here forever and share our fortunes and require introduction of evidence. The Court needs to
misfortunes, Filipino citizenship is not impossible to validate these factual allegations based on competent and G.R. No. 111008 November 7, 1994
acquire. reliable evidence. As such, the Court cannot resolve the TRAMAT MERCANTILE, INC. AND DAVID ONG, petitioners,
questions they pose. Second, J.G. Summit misreads the vs. HON. COURT OF APPEALS AND MELCHOR DE LA
But if an alien is given not only a lease of, but also an provisions of the Constitution cited in its own pleadings, to CUESTA, respondents.
option to buy, a piece of land, by virtue of which the wit:
Filipino owner cannot sell or otherwise dispose of his VITUG, J.:
property, this to last for 50 years, then it becomes clear 29.2 Petitioner has consistently pointed out in the past that
that the arrangement is a virtual transfer of ownership private respondent is not a 60%-40% corporation, and this This petition for review on certiorari challenges the 04th
whereby the owner divests himself in stages not only of the violates the Constitution x xxThe violation continues to this March 1993 decision of the Court of Appeals and its
right to enjoy the land (jus possidendi, jus utendi, jus day because under the law, it continues to own real resolution of 01 July 1993 denying the motion for
fruendi and jus abutendi) but also of the right to dispose of property reconsideration.
it (jus disponendi) rights the sum total of which make up
ownership. It is just as if today the possession is xxx xxxxxx On 09 April 1984, Melchor de la Cuesta, doing business
transferred, tomorrow, the use, the next day, the 32. To review the constitutional provisions involved, Section under the name and style of "Farmers Machineries," sold to
disposition, and so on, until ultimately all the rights of 14, Article XIV of the 1973 Constitution (the JVA was signed Tramat Mercantile, Inc. ("Tramat"), one (1) unit HINOMOTO
which ownership is made up are consolidated in an alien. in 1977), provided: TRACTOR Model MB 1100D powered by a 13 H.P. diesel
And yet this is just exactly what the parties in this case did engine. In payment, David Ong, Tramat's president and
within this pace of one year, with the result that Justina Save in cases of hereditary succession, no private lands manager, issued a check for P33,500.00 (apparently
Santos'[s] ownership of her property was reduced to a shall be transferred or conveyed except to individuals, replacing an earlier postdated check for P33,080.00).
hollow concept. If this can be done, then the Constitutional corporations, or associations qualified to acquire or hold Tramat, in turn, sold the tractor, together with an attached
ban against alien landholding in the Philippines, as lands of the public domain. lawn mower fabricated by it, to the Metropolitan
announced in Krivenko vs. Register of Deeds, is indeed in Waterworks and Sewerage System ("NAWASA") for
grave peril.[44] (emphases supplied; Citations omitted) 32.1 This provision is the same as Section 7, Article XII of P67,000.00. David Ong caused a "stop payment" of the
the 1987 Constitution. check when NAWASA refused to pay the tractor and lawn
In Lui She, the option to buy was invalidated because it mower after discovering that, aside from some stated
amounted to a virtual transfer of ownership as the owner 32.2 Under the Public Land Act, corporations qualified to defects of the attached lawn mower, the engine (sold by de
could not sell or dispose of his properties. The contract in acquire or hold lands of the public domain are corporations la Cuesta) was a reconditioned unit.
Lui She prohibited the owner of the land from selling, at least 60% of which is owned by Filipino citizens (Sec. 22,
donating, mortgaging, or encumbering the property during Commonwealth Act 141, as amended). (emphases supplied) On 28 May 1985, de la Cuesta filed an action for the
the 50-year period of the option to buy. This is not so in the recovery of P33,500.00, as well as attorney's fees of
case at bar where the mutual right of first refusal in favor As correctly observed by the public respondents, the P10,000.00, and the costs of suit. Ong, in his answer,
of NIDC and KAWASAKI does not amount to a virtual transfer prohibition in the Constitution applies only to ownership of averred, among other things, that de la Cuesta had no
of land to a non-Filipino. In fact, the case at bar involves a land.[48] It does not extend to immovable or real property cause of action; that the questioned transaction was
right of first refusal over shares of stock while the Lui She as defined under Article 415 of the Civil Code. Otherwise, between plaintiff and Tramat Mercantile, Inc., and not with
case involves an option to buy the land itself. As discussed we would have a strange situation where the ownership of Ong in his personal capacity; and that the payment of the
earlier, there is a distinction between the shareholders immovable property such as trees, plants and growing fruit check was stopped because the subject tractor had been
ownership of shares and the corporations ownership of land attached to the land[49] would be limited to Filipinos and priced as a brand new, not as a reconditioned unit.
arising from the separate juridical personalities of the Filipino corporations only.
corporation and its shareholders. On 02 November 1989, after the reception of evidence, the
III. trial court rendered a decision, the dispositive portions of
We note that in its Motion for Reconsideration, J.G. Summit which read:
alleges that PHILSECO continues to violate the Constitution

Page 84 of 160
CORPORATION LAW CASES MAYORDO, M.A
LLB 3

WHEREFORE, in view of the foregoing consideration, April 1984, and the first check was drawn at about the
judgment is hereby rendered: same time. The freight charges cannot be said to have been No wonder, then, it was a gasket Soledad Cac had to
incurred when the tractor engine was delivered back to the replace, at a cost chargeable to the appellants. No wonder,
1. Ordering the defendants, jointly and severally, to supplier for repairs. The appellants admitted that the furthermore, the appellants' witness declared that even
pay the plaintiff the sum of P33,500.00 with legal interest engine was not brought back to Isabela. The repairs were after the replacement of that one gasket, the engine still
thereon at the rate of 12% per annum from July 7, 1984 done at Soledad Cac's gasoline station in Quezon City. leaked oil after being torture-tested. The integrity of the
until fully paid; and other engine gaskets might have been impaired, too. Such
Anent the first assigned error, We sustain the trial court's was the burden placed on the engine. The engine
2. Ordering the defendants, jointly and severally, to finding that at the time of the purchase, the appellants did malfunctioned not necessarily because the engine, as
pay the plaintiff the sum of P10,000.00 as attorney's fees, not reveal to the appellee the true purpose for which the alleged by the appellants, had been a reconditioned, and
and the costs of this suit. tractor would be used. Granting that the appellants not a brand new, one. It malfunctioned because it was
informed the appellee that they would be reselling the unit made to do what it simply could not.2
SO ORDERED. 1 to the MWSS, an entity admittedly not engaged in farming,
and that they ordered the tractor without the power tiller, It was, nevertheless, an error to hold David Ong jointly and
An appeal was timely interposed by the defendants. On 04 an indispensable accessory if the tractor would be used in severally liable with TRAMAT to de la Cuesta under the
March 1993, the Court of Appeals affirmed in toto the farming, these in themselves would not constitute the questioned transaction. Ong had there so acted, not in his
decision of the trial court. Defendant-appellants' motion for required implied notice to the appellee as seller. personal capacity, but as an officer of a corporation,
reconsideration was denied. TRAMAT, with a distinct and separate personality. As such, it
xxx xxx xxx should only be the corporation, not the person acting for
Hence, the instant petition. In regard to the second assigned error, We do not agree and on its behalf, that properly could be made liable
that the appellee should have been held liable for the thereon.3
We could find no reason to reverse the factual findings of tractor's alleged hidden defects. . . .
both the trial court and the appellate court, particularly in Personal liability of a corporate director, trustee or officer
holding that the contract between de la Cuesta and It has to be noted in this regard that, to satisfy the along (although not necessarily) with the corporation may
TRAMAT was one of absolute, not conditional, sale of the requirements of the MWSS, the appellants borrowed a lawn so validly attach, as a rule, only when —
tractor and that de la Cuesta did not violate any warranty mower from the MWSS so they could fabricate one such
on the sale of the tractor to TRAMAT. The appellate court, mower. The appellants' witness stated that the kind of mid- 1. He assents (a) to a patently unlawful act of the
in its decision, adequately explained: mounted lawn mower was being manufactured by their corporation, or (b) for bad faith, or gross negligence in
competitor, Alpha Machinery, which had by then stopped directing its affairs, or (c) for conflict of interest, resulting
If the perfection of the sale was dependent upon supplying the same (tsn, Nov. 29, 1988, pp. 73-74). There is in damages to the corporation, its stockholders or other
acceptance by the MWSS of the subject tractor why did the no showing that the appellants had had any previous persons;4
appellants issue a check in payment of the item to the experience in the fabrication of this lawn mower. In fact, as
appellee? And long after MWSS had complained about the aforesaid, they had to borrow one from the MWSS which 2. He consents to the issuance of watered stocks or
defective tractor engine, and after the appellee had failed they could copy. But although they made a copy with the who, having knowledge thereof, does not forthwith file
to remedy the defect, why did the appellants still draw and same specifications and design, there was no assurance with the corporate secretary his written objection thereto;
deliver a replacement check to the appellee for the that the copy would function as well as with the model. 5
increased amount of P33,500.00?
xxx xxx xxx 3. He agrees to hold himself personally and
These payments argue against the claim now made by the Although the trial court discussed it in a different light, We solidarily liable with the corporation;6 or
defendants that the sale was conditional. view the matter in the same way the trial court did — that
the lawn mower as fabricated by the appellants was the 4. He is made, by a specific provision of law, to
According to the appellee, the additional amount covered root of the parties' problems. personally answer for his corporate action.7
the cost of replacing the oil gasket of the tractor engine
when it was repaired in Soledad Cac's gasoline station in Having had no previous experience in the manufacture of In the case at bench, there is no indication that petitioner
Quezon City. The appellants, on the other hand, claims the lawn mowers of the same type as that in litigation, and in a David Ong could be held personally accountable under any
amount represented the freight charges for transporting possibly patent-infringing effort to undercut their of the abovementioned cases.
the tractor from Cauayan, Isabela to Metro Manila. competition, the appellants gathered enough daring to do
the fabrication themselves. But the product might have WHEREFORE, the petition is given DUE COURSE and the
The appellants should have explained why they failed to proved too much for the subject tractor to power, and the decision of the trial court, affirmed by the appellate court,
include the freight charges in the first check. The tractor tractor's engine was strained beyond its limits, causing it to is MODIFIED insofar as it holds petitioner David Ong jointly
was transported from Isabela to Metro Manila as early as overheat and damage its gaskets. and severally liable with Tramat Mercantile, Inc., which

Page 85 of 160
CORPORATION LAW CASES MAYORDO, M.A
LLB 3

portion of the questioned judgment is SET ASIDE. In all trial, the Court of First Instance of Manila rendered Of the incorporators of the Marvel Building Corporation,
other respects, the decision appealed from is AFFIRMED. No judgment ordering the release of the properties mentioned, Maximo Cristobal and Antonio Cristobal are half-brothers of
costs. and enjoined the Collector of Internal Revenue from selling Maria B. Castro, Maria Cristobal is a half-sister, and Segundo
the same. The Collector of Internal Revenue has appealed Esguerra, Sr. a brother-in-law, husband of Maria Cristobal,
SO ORDERED. to this Court against the judgment. Maria B. Castro's half-sister. Maximo B. Cristobal did not file
any income tax returns before the year 1946, except for
G.R. No. L-5081 February 24, 1954 The following facts are not disputed, or are satisfactorily three years 1939 and 1940, but in these years he was
MARVEL BUILDING CORPORATION, ET AL., plaintiffs- proved by the evidence: exempted from the tax. He has not filed any war profits tax
appellees, vs. SATURNINO DAVID, in his capacity as return (Exhibit 54). Antonio Cristobal, Segundo Esguerra, Sr.
Collector, Bureau of Internal Revenue, defendant- The Articles of Incorporation of the Marvel Building and Jose T. Lopez did not file any income tax returns for
appellant. Corporation is dated February 12, 1947 and according to it the years prior to 1946, and neither did they file any war
the capital stock is P2,000,000, of which P1,025,000 was profits tax returns (Exhibit 52). Maria Cristobal filed income
LABRADOR, J.: (at the time of incorporation) subscribed and paid for by tax returns for the year 1929 to 1942, but they were
the following incorporators: exempt from the tax (Exhibit 53). Benita A. Lamagna did
This action was brought by plaintiffs as stockholders of the not file any income tax returns prior to 1945, except for
Marvel Building Corporation to enjoin the defendant 1942 which was exempt. She did not file any was profits tax
Maria B. Castro 250 shares P 250,000.00
Collector of Internal Revenue from selling at public auction (Exhibit 55). Ramon M. Sangalang did not file income tax
various properties described in the complaint, including Amado A. Yatco 100       " 100,000.00 returns up to 1945 except for the years 1936, 1937, 1938,
three parcels of land, with the buildings situated thereon, 1939 and 1940. He has not filed any war profits tax return
known as the Aguinaldo Building, the Wise Building, and the Santiago Tan 100       " 100,000.00 (Exhibit 57). Amada A. Yatco did not file income tax returns
Dewey Boulevard-Padre Faura Mansion, all registered in the prior to 1945, except for the years 1937, 1938, 1939, 1941
Jose T. Lopez   90       " 90,000.00 and 1942, but these were exempt. He did not file any war
name of the said corporation. Said properties were seized
and distrained by defendant to collect war profits taxes profits tax return (Exhibit 58).
Benita Lamagna   90       " 90,000.00
assessed against plaintiff Maria B. Castro (Exhibit B).
Plaintiffs allege that the said three properties (lands and C.S. Gonzales   80       " 80,000.00 Antonio Cristobal's income in 1946 is P15,630, and in 1947,
buildings) belong to Marvel Building Corporation and not to P4,550 (Exhibits 59-60); Maximo B. Cristobal's income in
Maria B. Castro, while the defendant claims that Maria B. Maria Cristobal   70       " 70,000.00 1946 is P19,759.10, in 1947, P9,773.47 (Exhibits 61-62);
Castro is the true and sole owner of all the subscribed stock Segundo Esquerra's income in 1946 is P5,500, in 1947,
Segundo Esguerra, Sr.   75       " 75,000.00 P7,754.32 (Exhibits 63-64); Jose T. Lopez's income in 1946 is
of the Marvel Building Corporation, including those
appearing to have been subscribed and paid for by the P20,785, in 1947, P14,302.77 (Exhibits 69-70); Benita A.
Ramon Sangalang   70       " 70,000.00
other members, and consequently said Maria B. Castro is Lamagna's income in 1945 is P1,559, in 1946, P6,463.36, in
also the true and exclusive owner of the properties seized. Maximo Cristobal   55       " 55,000.00 1947, P6,189.79 and her husband's income in 1947 is
The trial court held that the evidence, which is mostly P10,825.53 (Exhibits 65-68); Ramon M. Sangalang's income
circumstantial, fails to show to its satisfaction that Maria B. Antonio Cristobal   45       "       45,000.00 in 1945 is P5,500, in 1946, P18,300.00 (Exhibits 71-72);
Castro is the true owner of all the stock certificates of the Santiago Tan's income in 1945 is P456, in 1947 is P9,167.95,
P1,025,000.00 and in 1947, P7,620.11 (Exhibits 73-75); and Amado Yatco's
corporation, because the evidence is susceptible of two
interpretations and an interpretation may not be made income in 1945 is P12,600, in 1946, P23,960, and in 1947,
which would deprive one of the property without due Maria B Castro was elected President and Maximo Cristobal, P11,160 (Exhibits 76-78).
process of law. Secretary-Treasurer (Exhibit A).
In October, 1945 Maria B. Castro, NicasioYatco, Maxima
It appears that on September 15, 1950, the Secretary of The Wise Building was purchased on September 4, 1946, Cristobal de Esquerra, Maria Cristobal Lopez and Maximo
Finance, upon consideration of the report of a special the purchase being made in the name of Dolores Trinidad, Cristobal organized the Maria B. Castro, Inc. with capital
committee assigned to study the war profits tax case of wife of Amado A. Yatco (Exhibit V), and the Aguinaldo stock of P100,000, of which Maria B. Castro subscribed for
Mrs. Maria B. Castro, recommended the collection of Building, on January 17, 1947, in the name of Segundo P99,600 and all others for P100 each. This was increased in
P3,593,950.78 as war profits taxes for the latter, and on Esguerra, Sr. (Exhibit M). Both building were purchased for 1950 to P500,000 and Maria B. Castro subscribed P76,000
September 22, 1953 the President instructed the Collector P1,800,000, but as the corporation had only P1,025,000, and the others P1,000 each (Exhibit 126).
that steps be taken to collect the same (Exhibits 114, 114-A the balance of the purchase price was obtained as loans
to 114-D). Pursuant thereto various properties, including from the Insular Life Assurance Co., Ltd. and the Philippine It does not appear that the stockholders or the board of
the three above mentioned, were seized by the Collector of Guaranty Co., Inc. (Exhibit C). directors of the Marvel Building Corporation have ever held
Internal Revenue on October 31, 1950. On November 13, a business meeting, for no books thereof or minutes
1950, the original complaint in this case was filed. After meeting were ever mentioned by the officers thereof or

Page 86 of 160
CORPORATION LAW CASES MAYORDO, M.A
LLB 3

presented by them at the trial. The by-laws of the Treasurer when he had the certificates; and that after the daughter, and this at the time when as a bookkeeper he
corporation, if any had ever been approved, has not been checking he returned all of them to Mrs. Castro. He was entrusted with checking up the certificates of stock.
presented. Neither does it appear that any report of the recognized the photostats, Exhibit 4 to 13 as photostats of When the older Llamado kept secret the existence of the
affairs of the corporation has been made, either of its the said originals. He also declared that he also prepared a endorsed certificates, the friendship between the two
transactions or accounts. set of stock certificates, similar to the certificates which families was yet intact; hence, the existence of the
were copied in the photostats, the number of shares, and endorsed certificates must have been kept to himself by
From the book of accounts of the corporation, advances to the date issue, and that the certificates he had prepared the older Llamado. All the above circumstances reinforce
the Marvel Building Corporation of P125,000 were made by are Exhibits H, H-1 to H-7 and J (Exhibits 30-38). This set of our belief that the Llamados had personal knowledge of the
Maria B. Castro in 1947, P102,916.05 in 1948 and certificates was made by him first and the set of which facts they testify to, and the existence of this knowledge in
P102,916.05 in 1948, and P160,910.96 in 1949 (Exhibit 118). photostats were taken, a few days later. turn renders improbable plaintiffs' claim that their
testimonies were biased.
The main issue involved in these proceedings is: Is Maria B. The plaintiffs offered a half-hearted denial of the existence
Castro the owner of all the shares of stocks of Marvel of the endorsed blank certificates, Maximo Cristobal, Attempt was also made by the plaintiffs to show by expert
Building Corporation and the other stockholders mere secretary of the corporation, saying that no investigation evidence that the endorsement could have been
dummies of hers? was ever made by Aquino and Mariano in which said superimposed, i.e., that the signatures made on other
certificates were discovered by the latter. They, however, papers and these were pasted and thereafter the
The most important evidence presented by the Collector of vigorously attack the credibility of the witnesses for the documents photographed. Judicial experience is to the
Internal Revenue to prove his claim that Maria B. Castro is defendant, imputing to the Llamados, enmity against Maria effect that the expert witnesses can always be obtained for
the sole and exclusive owner of the shares of stock of the B. Castro, and to Aquino and Mariano, a very doubtful both sides of an issue, most likely because expert witnesses
Marvel Building Corporation is supposed endorsement in conduct in not divulging the existence of the certificates are no longer impermeable to the influence of fees (II
blank of the shares of stock issued in the name of the other either Lobrin, Chief Income Tax Examiner, or to the Wigmore, Sec. 563(2), p. 646). And if parties are capable of
incorporators, and the possession thereof by Maria B. Collector of Internal Revenue, both their immediate chiefs. paying fees, expert opinion should be received with
Castro. The existence of said endorsed certificates was Reliance is also placed on a certificate, Exhibit W, wherein caution. In the case at bar, the opinion on the supposed
testified to by witnesses Felipe Aquino, internal revenue Aquino and others declare that the certificates (Exhibits 30 superimposition was merely a possibility, and we note
examiner, Antonio Mariano, examiner, and Crispin Llamado, to 38, or H, H-1 to H-7 and J) were regular and were not various circumstances which proved that the signatures
Under Secretary of Finance, who declared as follows: endorsed when the same were examined. In connection were not superimposed and corroborate defendant's claim
Towards the end of the year 1948 and about the beginning with this certificate, Exhibit W, we note that it states that that they were genuine. In the first place,, the printed
of the year 1949, while Aquino and Mariano were examining the certificates examined were Exhibits 30 to 38, the endorsement contains a very heavy line at the bottom for
the books and papers were furnished by its secretary, existence or character of which are not disputed. But the the signature of the endorsee. This line in almost all the
Maximo Cristobal, they came across an envelope containing statement contains nothing to the effect that the above endorsements is as clear as the printed letters above it,
eleven stock certificates, bound together by an Acco certificates were the only one existence, according to their and at the points where the letters of the signature extend
fastener, which (certificates) corresponded in number and knowledge. Again the certificate was issued for an down and transversed it (the line), there is no indication
in amount on their face to the subscriptions of the examination on September 1949, not by Aquino and Mariano that the line is covered by a superimposed paper. Again in
stockholders that all the certificates, except that in the at the end of 1948 or the beginning of 1949. The these places both the signatures and the lines are clear and
name of Maria B. Castro, were endorsed in the bank by the certificate, therefore, neither denies the existence of the distinct where they cross one another. Had there been
subscribers; that as the two revenue agents could not agree endorsed certificates, nor that Aquino and Mariano had superimpositions the above features could not have been
what to do with the certificates, Aquino brought them to made an examination of the papers of the corporation at possible. In the second place, Maria B. Castro admitted
Under-Secretary of Finance Llamado, who thereupon the end of the year 1948. It ca not, therefore, discredit the having signed 25 stock certificates, but only eleven were
suggested that photostatic copies thereof be taken; that testimonies of the defendant's witnesses. issued (t. s. n., p. 662). No explanation is given by her why
this was done, and the photostatic copies taken are she had to sign as many as 25 forms when there were only
(Exhibits 4, 5, 6, 7, 8, 9, 10, 11, 12 and 13; and in that As to the supposed enmity of the Llamados towards the eleven subscribers and eleven forms to be filed. This
July, 1950, copy-cat copies of the above photostats were plaintiff Maria B. Castro, we note that, supposing that circumstances corroborate the young Llamado's declaration
taken, and said copy-cat copies are Exhibits 40-49. there really was such enmity, it does not appear that it was that two sets of certificates had been prepared. The
of such magnitude or force as could have induced the nineteen issue must be Exhibits H, H-1 to H-7 and J, or Nos.
Julio Llamado, bookkeeper of the Marvel Building Llamados and Maria B. Castro were close friends way back 30 to 38, and the stock certificates endorsed whose
Corporation from 1947 to May, 1948, also testified that he in 1947 and up to 1949; but that at the time of the trial the photostatic copies are Exhibits 4 to 13. It is to be
was the one who had prepared the original certificates, friendship had been marred by misunderstandings. We remembered also, that it is a common practice among
putting therein the number of shares in words in handprint; believe that in 1948 and 1949 the Llamados were trusted unscrupulous merchants to carry two sets of books, one set
that the originals were given to him by Maria B. Castro for friends of Maria B. Castro, and this explains why they had for themselves and another to be shown to tax collectors.
comparison with the articles of incorporation; that they knowledge of her secret transactions. The younger Llamado This practice could not have been unknown to Maria B.
were not yet signed by the President and by the Secretary- even made advances for the hand of Maria B. Castro's Castro, who apparently had been able to evade the

Page 87 of 160
CORPORATION LAW CASES MAYORDO, M.A
LLB 3

payment of her war profits taxes. These circumstances, who admitted that Maria B. Castro, paid for his (II Wigmore, Sec. 285, p.162). A party's silence to adverse
coupled with the testimony of Julio Llamado that two sets subscription. After a careful study of the above testimony is equivalent to an admission of its truth (Ibid,
of certificates were given to him for checking, show to an testimonies, however, we find them subject to various Sec. 289, p. 175).
impartial mind the existence of the set of certificates objections. Maximo Cristobal declared that he issued
endorsed in blank, thus confirming the testimonies of the provincial receipts for the subscriptions supposedly paid to Our consideration of the evidence submitted on both sides
defendant's witnesses, Aquino, Mariano and Crispin him in 1946; but none of the supposed receipts were leads us to a conclusion exactly opposite that arrived at by
Llamado, and thus discrediting the obviously partial presented. If the subscriptions were really received by him, the trial court. In general the evidence offered by the
testimony of the expert presented by plaintiffs. The big as the amounts were, he would have been able to tell plaintiffs is testimonial and direct evidence, easy of
genuineness of the signatures on the endorsements is not specifically, by dates and in fix amounts, when and how the fabrication; that offered by defendant, documentary and
disputed. How could the defendant have secured these payments were made. The general assertion of alleged circumstantial, not only difficult of fabrication but in most
genuine signatures? Plaintiffs offer no explanation for this, payments, without the concrete days and amounts of cases found in the possession of plaintiffs. There is very
although they do not question them. It follows that the payments, are, according to our experience, positive little room for choice as between the two. The
genuine signatures must have been made on the stock identifications of untruthfulness, for when a witness circumstantial evidence is not only convincing; it is
certificates themselves. testified to a fact that actually occurs, the act is concretely conclusive. The existence of endorsed certificates,
stated and no generalization is made. discovered by the internal revenue agents between 1948
Next in importance among the evidence submitted by the and 1949 in the possession of the Secretary-Treasurer, the
defendant collector to prove his contention that Maria B. With respect to Maria B. Castro's testimony, we find it to be fact that twenty-five certificates were signed by the
Castro is the sole owner of the shares of stock of the Marvel as untruthful as that of Cristobal. She declared that president of the corporation, for no justifiable reason, the
Building Corporation, is the fact that the other stockholders payments of the subscriptions took place between July and fact that two sets of certificates were issued, the
did not have incomes in such amounts, during the time of December, 1946, and that first payments were first undisputed fact that Maria B. Castro had made enormous
the organization of the corporation in 1947, or immediately deposited by her in the National City Bank of New York. A profits and, therefore, had a motive to hide them to evade
thereto, as to enable them to pay in full for their supposed study of her account in said bank (Exhibit 82), however, the payment of taxes, the fact that the other subscribers
subscriptions. This fact is proved by their income tax fails to show the alleged deposit of the subscriptions during had no incomes of sufficient magnitude to justify their big
returns, or the absence thereof. Let us take Amado A. Yatco the year 1946 (See Exhibits 83-112). This fact completely subscriptions, the fact that the subscriptions were not
as an example. Before 1945 his returns were exempt from belies her assertion. As to the testimony of C.S. Gonzales receipted for and deposited by the treasurer in the name of
the tax, in 1945 he had P12,600 and in 1946, P23,000. He that Maria B. Castro advanced his subscription, there is the corporation but were kept by Maria B. Castro herself,
has four children. How could he have paid P100,000 in 1945 nothing in the evidence to corroborate it, and the the fact that the stockholders or the directors never
and 1946? Santiago Tan who also contributed P100,000 had circumstances show otherwise. If he had really been a appeared to have ever met to discuss the business of the
no appreciable income before 1946, and this year an stockholder and Maria B. Castro advanced his subscription, corporation, the fact that Maria B. Castro advanced big
income of only P9,167.95. Jose T. Lopez also did not file the agreement between him and Castro should have been sums of money to the corporation without any previous
any income tax returns before 1940 and 1946 he had an put in writing, the amount advanced being quite arrangement or accounting, and the fact that the books of
income of only P20,785, whereas he is supposed to have considerable (P80,000), and it appearing further that accounts were kept as if they belonged to Maria B. Castro
subscribed P90,000 worth of stock early in 1947. Benita Gonzales is no close relative or confidant of Castro. alone — these facts are of patent and potent significance.
Lamagna had no returns either up to 1945, except in 1942, What are their necessary implications? Maria B. Castro
which was exempt and in 1945 she had an income of P1,550 Lastly, it is significant that the plaintiffs, the supposed would not have asked them to endorse their stock
and in 1946, P6,463.36. In the same situation are all the subscribers, who should have come to court to assert that certificates, or be keeping these in her possession, if they
others, and besides, brothers and sisters and brother-in-law they actually paid for their subscriptions, and are not mere were really the owners. They never would have consented
of Maria B. Castro. On the other hand, Maria B. Castro had dummies, did not do so. They could not have afforded such that Maria B. Castro keep the funds without receipts or
been found to have made enormous gains or profits in her a costly indifference, valued at from P70,000 to P100,000 accounting, nor that she manages the business without
business such that the taxes thereon were assessed at each, if they were not actual dummies. This failure on their their knowledge or concurrence, were they owners of the
around P3,000,000. There was, therefore, a prima facie part to take the witness stand to deny or refute the charge stocks in their own rights. Each and every one of the facts
case out by the defendant collector that Maria B. Castro that they were mere dummies is to us of utmost all set forth above, in the same manner, is inconsistent with
had furnished (& all the money that the Marvel Building significance. What could have been easier to disprove the the claim that the stockholders, other than Maria B. Castro,
Corporation had. charge that they were dummies, than for them to come to own their shares in their own right. On the other hand,
court and show their receipts and testify on the payments each and every one of them, and all of them, can point to
In order the meet the above evidence only three of the they have paid on their subscriptions? This they, however no other conclusion than that Maria B. Castro was the sole
plaintiffs testified, namely, Maximo Cristobal, the refused to do so. They had it in their power to rebut the and exclusive owner of the shares and that they were only
corporation's secretary, who made the general assertion on charges, but they chose to keep silent. The non-production her dummies.
the witness stand that the other stockholders paid for their of evidence that would naturally have been produced by an
shares in full, Maria B. Castro, who stated that payments honest and therefore fearless claimant permits the In our opinion, the facts and circumstances duly set forth
for the subscription were made to her, and C.S. Gonzales, inference that its tenor is unfavorable to the party's cause above, all of which have been proved to our satisfaction,

Page 88 of 160
CORPORATION LAW CASES MAYORDO, M.A
LLB 3

prove conclusively and beyond reasonable doubt (section the period that the plaintiff's (Gregorio Palacio's) child was It appears from Exhibit "A" that Gregorio Palacio, one of the
89, Rule 123 of the Rules of Court and section 42 of the in the hospital and who said child was under treatment for herein plaintiffs, testified that Mario Palacio, the other
Provisional law for the application of the Penal Code) that five months in order to meet the needs of his big family, he plaintiff, is his son; that as a result of the reckless driving
Maria B. Castro is the sole and exclusive owner of all the was forced to sell one air compressor (heavy duty) and one of accused Alfredo Carillo, his child Mario was injured and
shares of stock of the Marvel Building Corporation and that heavy duty electric drill, for a sacrifice sale of P150.00 hospitalized from December 24, 1952, to January 8, 1953;
the other partners are her dummies. which could easily sell at P350.00; that as a consequence of that during all the time that his child was in the hospital,
the negligent and reckless act of the driver Alfredo Carillo he watched him during the night and his wife during the
Wherefore, the judgment appealed from should be, as it of the herein defendant company, the herein plaintiffs day; that during that period of time he could not work as
hereby is, reversed and the action filed by plaintiffs- were forced to litigate this case in Court for an agreed he slept during the day; that before his child was injured,
appellees, dismissed, with costs against plaintiffs- amount of P300.00 for attorney's fee; that the herein he used to earn P10.00 a day on ordinary days and on
appellees. So ordered. plaintiffs have now incurred the amount of P500.00 actual Sundays from P20 to P50 a Sunday; that to meet his
expenses for transportation, representation and similar expenses he had to sell his compressor and electric drill for
G.R. No. L-15121 August 31, 1962 expenses for gathering evidence and witnesses; and that P150 only; and that they could have been sold for P300 at
GREGORIO PALACIO, in his own behalf and in behalf of his because of the nature of the injuries of plaintiff Mario the lowest price.
minor child, Palacio and the fear that the child might become a useless
MARIO PALACIO, plaintiffs-appellants, vs. FELY invalid, the herein plaintiff Gregorio Palacio has suffered During the trial of the criminal case against the driver of
TRANSPORTATION COMPANY, defendant-appellee. moral damages which could be conservatively estimated at the jeep in the Court of First Instance of Quezon City
P1,200.00. (Criminal Case No. Q-1084) an attempt was unsuccessfully
REGALA, J.: made by the prosecution to prove moral damages allegedly
On May 23, 1956, defendant Fely Transportation Co., filed a suffered by herein plaintiff Gregorio Palacio. Likewise an
This is an appeal by the plaintiffs from the decision of the Motion to Dismiss on the grounds (1) that there is no cause attempt was made in vain by the private prosecutor in that
Court of First Instance of Manila which dismissed their of action against the defendant company, and (2) that the case to prove the agreed attorney's fees between him and
complaint. cause of action is barred by prior judgment.. plaintiff Gregorio Palacio and the expenses allegedly
incurred by the herein plaintiffs in connection with that
Originally taken to the Court of Appeals, this appeal was In its Order, dated June 8, 1956, this Court deferred the case. During the trial of this case, plaintiff Gregorio Palacio
certified to this Court on the ground that it raises purely determination of the grounds alleged in the Motion to testified substantially to the same facts.
questions of law. Dismiss until the trial of this case.
The Court of First Instance of Quezon City in its decision in
The parties in this case adopt the following findings of fact On June 20, 1956, defendant filed its answer. By way of Criminal Case No. 1084 (Exhibit "2") determined and
of the lower court: affirmative defenses, it alleges (1) that complaint states no thoroughly discussed the civil liability of the accused in
cause of action against defendant, and (2) that the sale and that case. The dispositive part thereof reads as follows:
In their complaint filed with this Court on May 15, 1954, transfer of the jeep AC-687 by IsabeloCalingasan to the Fely
plaintiffs allege, among other things, "that about Transportation was made on December 24, 1955, long after IN VIEW OF THE FOREGOING, the Court finds the accused
December, 1952, the defendant company hired Alfredo the driver Alfredo Carillo of said jeep had been convicted Alfredo Carillo y Damaso guilty beyond reasonable doubt of
Carillo as driver of AC-787 (687) (a registration for 1952) and had served his sentence in Criminal Case No. Q-1084 of the crime charged in the information and he is hereby
owned and operated by the said defendant company; that the Court of First Instance of Quezon City, in which both sentenced to suffer imprisonment for a period of Two
on December 24, 1952, at about 11:30 a.m., while the the civil and criminal cases were simultaneously tried by Months & One Day of Arresto Mayor; to indemnify the
driver Alfonso (Alfredo) Carillo was driving AC-687 at Halcon agreement of the parties in said case. In the Counterclaim offended party, by way of consequential damages, in the
Street, Quezon City, wilfully, unlawfully and feloniously and of the Answer, defendant alleges that in view of the filing sum of P500.00 which the Court deems reasonable; with
in a negligent, reckless and imprudent manner, run over a of this complaint which is a clearly unfounded civil action subsidiary imprisonment in case of insolvency but not to
child Mario Palacio of the herein plaintiff Gregorio Palacio; merely to harass the defendant, it was compelled to exceed ¹/3 of the principal penalty imposed; and to pay
that on account of the aforesaid injuries, Mario Palacio engage the services of a lawyer for an agreed amount of the costs.
suffered a simple fracture of the right tenor (sic), complete P500.00.
third, thereby hospitalizing him at the Philippine On the basis of these facts, the lower court held action is
Orthopedic Hospital from December 24, 1952, up to During the trial, plaintiffs presented the transcript of the barred by the judgment in the criminal case and, that
January 8, 1953, and continued to be treated for a period stenographic notes of the trial of the case of "People of the under Article 103 of the Revised Penal Code, the person
of five months thereafter; that the plaintiff Gregorio Philippines vs. Alfredo Carillo, Criminal Case No. Q-1084," in subsidiarily liable to pay damages is Isabel Calingasan, the
Palacio herein is a welder by occupation and owner of a the Court of First Instance of Rizal, Quezon City (Branch employer, and not the defendant corporation.
small welding shop and because of the injuries of his child IV), as Exhibit "A".
he has abandoned his shop where he derives income of Against that decision the plaintiffs appealed, contending
P10.00 a day for the support of his big family; that during that:

Page 89 of 160
CORPORATION LAW CASES MAYORDO, M.A
LLB 3

And while it is true that IsabeloCalingasan is not a party in P403,514.28, with legal interest thereon from the date of
THE LOWER COURT ERRED IN NOT SUSTAINING THAT THE this case, yet, is held in the case of Alonso v. Villamor, 16 filing of the action until fully paid, P80,702.26 as liquidated
DEFENDANT-APPELLEE IS SUBSIDIARILY LIABLE FOR DAMAGES Phil. 315, this Court can substitute him in place of the damages, P5,000.00 as attorney's fees, plus costs, but
AS A RESULT OF CRIMINAL CASE NO. Q-1084 OF THE COURT defendant corporation as to the real party in interest. This dismissing the complaint insofar as defendant Francisco
OF FIRST INSTANCE OF QUEZON CITY FOR THE REASON THAT is so in order to avoid multiplicity of suits and thereby save Sycip was concerned, as well as the latter's counterclaim.
THE INCORPORATORS OF THE FELY TRANSPORTATION the parties unnecessary expenses and delay. (Sec. 2, Rule The appeal is only from that portion of the decision
COMPANY, THE DEFENDANT-APPELLEE HEREIN, ARE ISABELO 17, Rules of Court; Cuyugan v. Dizon. 79 Phil. 80; Quison v. dismissing the case as against Francisco Sycip.
CALINGASAN HIMSELF, HIS SON AND DAUGHTERS; Salud, 12 Phil. 109.)
On March 25, 1958, ASSOCIATED, a domestic corporation,
THE LOWER COURT ERRED IN NOT CONSIDERING THAT THE Accordingly, defendants Fely Transportation and Isabelo through its President, appellee Francisco Sycip, entered
INTENTION OF ISABELO CALINGASAN IN INCORPORATING THE Calingasan should be held subsidiarily liable for P500.00 into an agreement to exchange sugar with NAMARCO,
FELY TRANSPORTATION COMPANY, THE DEFENDANT-APPELLEE which Alfredo Carillo was ordered to pay in the criminal represented by its then General Manager, Benjamin
HEREIN, WAS TO EVADE HIS CIVIL LIABILITY AS A RESULT OF case and which amount he could not pay on account of Estrella, whereby the former would deliver to the latter
THE CONVICTION OF HIS DRIVER OF VEHICLE AC-687 THEN insolvency. 22,516 bags (each weighing 100 pounds) of "Victorias" and/
OWNED BY HIM: or "National" refined sugar in exchange for 7,732.71 bags of
We also sustain plaintiffs' third assignment of error and hold "Busilak" and 17,285.08 piculs of "Pasumil" raw sugar
THE LOWER COURT ERRED IN HOLDING THAT THE CAUSE OF that the present action is not barred by the judgment of belonging to NAMARCO, both agreeing to pay liquidated
ACTION OF THE PLAINTIFFS-APPELLANTS IS BARRED BY the Court of First Instance of Quezon City in the criminal damages equivalent to 20% of the contractual value of the
PRIOR JUDGMENT. case. While there seems to be some confusion on part of sugar should either party fail to comply with the terms and
the plaintiffs as to the theory on which the is based — conditions stipulated (Exhibit A). Pursuant thereto, on May
With respect to the first and second assignments of errors, whether ex-delito or quasi ex-delito (culpa aquiliana) — We 19,1958, NAMARCO delivered to ASSOCIATED 7,732.71 bars
plaintiffs contend that the defendant corporate should be are convinced, from the discussion prayer in the brief on of "Busilak" and 17,285.08 piculs of "Pasumil" domestic raw
made subsidiarily liable for damages in the criminal case appeal, that they are insisting the subsidiary civil liability sugar. As ASSOCIATED failed to deliver to NAMARCO the
because the sale to it of the jeep in question, after the of the defendant. As a matter of fact, the record shows 22,516 bags of "Victoria" and/or "National" refined sugar
conviction of Alfred Carillo in Criminal Case No. Q-1084 of that plaintiffs merely presented the transcript of the agreed upon, the latter, on January 12, 1959, demanded in
the Court of First Instance of Quezon City was merely an stenographic notes (Exhibit "A") taken at the hearing of the writing from the ASSOCIATED either (a) immediate delivery
attempt on the part of IsabeloCalingasan its president and criminal case, which Gregorio Palacio corroborated, in thereof before January 20, or (b) payment of its equivalent
general manager, to evade his subsidiary civil liability. support of their claim for damages. This rules out the cash value amounting to P372,639.80.
defense of res judicata, because such liability proceeds
The Court agrees with this contention of the plaintiffs. precisely from the judgment in the criminal action, where On January 19, 1959, ASSOCIATED, through Sycip, offered to
IsabeloCalingasan and defendant Fely Transportation may the accused was found guilty and ordered to pay an pay NAMARCO the value of 22,516 bags of refined sugar at
be regarded as one and the same person. It is evident that indemnity in the sum P500.00. the rate of P15.30 per bag, but the latter rejected the
IsabeloCalingasan's main purpose in forming the corporation offer. Instead, on January 21 of the same year it demanded
was to evade his subsidiary civil liability1 resulting from the WHEREFORE, the decision of the lower court is hereby payment of the 7,732.71 bags of "Busilak" raw sugar at
conviction of his driver, Alfredo Carillo. This conclusion is reversed and defendants Fely Transportation and P15.30 per bag, amounting to P118,310.40. and of the
borne out by the fact that the incorporators of the Fely IsabeloCalingasan are ordered to pay, jointly and severally, 17,285.08 piculs of "Pasumil" raw sugar at P16.50 per picul,
Transportation are IsabeloCalingasan, his wife, his son, Dr. the plaintiffs the amount of P500.00 and the costs. amounting, to P285.203.82, or a total price of P403,514.28
Calingasan, and his two daughters. We believe that this is for both kinds of sugar, based on the sugar quotations (Exh.
one case where the defendant corporation should not be G.R. No. L-20886 April 27, 1967 H) as of March 20, 1958 — the date when the exchange
heard to say that it has a personality separate and distinct NATIONAL MARKETING CORPORATION (NAMARCO), agreement was entered into.
from its members when to allow it to do so would be to plaintiff-appellant, vs. ASSOCIATED FINANCE COMPANY,
sanction the use of the fiction of corporate entity as a INC., and FRANCISCO SYCIP, defendants. As ASSOCIATED refused to deliver the raw sugar or pay for
shield to further an end subversive of justice. (La Campana FRANCISCO SYCIP, defendant-appellee. the refined sugar delivered to it, inspite of repeated
Coffee Factory, et al. v. Kaisahan ng mgaManggagawa, etc., demands therefore, NAMARCO instituted the present action
et al., G.R. No. L-5677, May 25, 1953) Furthermore, the DIZON, J.: in the lower court to recover the sum of P403,514.28 in
failure of the defendant corporation to prove that it has payment of the raw sugar received by defendants from it;
other property than the jeep (AC-687) strengthens the Appeal by the National Marketing Corporation — hereinafter P80,702.86 as liquidated damages; P10,000.00 as attorney's
conviction that its formation was for the purpose above referred to as NAMARCO, from the decision of the Court of fees, expenses of litigation and exemplary damages, with
indicated. First Instance of Manila in Civil Case No. 45770 ordering the legal interest thereon from the filing of the complaint until
Associated Finance Company, Inc. — hereinafter referred to fully paid.
as the ASSOCIATED — to pay the NAMARCO the sum of

Page 90 of 160
CORPORATION LAW CASES MAYORDO, M.A
LLB 3

In their amended answer defendants, by way of affirmative in inducing NAMARCO to enter into the aforesaid exchange executed by the sheriff in favor of the herein petitioner, as
defenses, alleged that the correct value of the sugar agreement, with full knowledge, on his part, on the fact well as the levy on the said property, and ordering the
delivered by NAMARCO to them was P259,451.09 or P13.30 that ASSOCIATED whom he represented and over whose sheriff to return the said machinery to its owner, herein
per bag of 100 lbs. weight (quedan basis) and not business and affairs he had absolute control, was in no private respondent Philippine American Drug Company.
P403,514.38 as claimed by NAMARCO. As counterclaim they position to comply with the obligation it had assumed. Petitioner herein, doing business under the name and style
prayed for the award of P500,000.00 as moral damages, Consequently, he can not now seek refuge behind the of Anchor Supply Co., sold on credit to herein private
P100,000.00 as exemplary damages and P10,000.00 as general principle that a corporation has a personality respondent Graphic Publishing, Inc. (GRAPHIC for short)
attorney's fees. distinct and separate from that of its stockholders and that paper products amounting to P55,214.73. On December 20,
the latter are not personally liable for the corporate 1972, GRAPHIC made partial payment by check to
After due trial court rendered the appealed judgment. The obligations. To the contrary, upon the proven facts, We feel petitioner in the total amount of P24,848.74; and on
appeal was taken to the Court of Appeals, but on January perfectly justified in "piercing the veil of corporate fiction" December 21, 1972, a promissory note was executed to
15, 1963 the latter certified the case to us for final and in holding Sycip personally liable, jointly and severally cover the balance of P30,365.99. In the said promissory
adjudication pursuant to sections 17 and 31 of the Judiciary with his co-defendant, for the sums of money adjudged in note, it was stipulated that the amount will be paid on
Act of 1948, as amended, the amount involved being more favor of appellant. It is settled law in this and other monthly installments and that failure to pay any
than P200,000.00, exclusive of interests and cost. jurisdictions that when the corporation is the mere alter installment would make the amount immediately
ego of a person, the corporate fiction may be disregarded; demandable with an interest of 12% per annum. On
The only issue to be resolved is whether, upon the facts the same being true when the corporation is controlled, September 6, 1973, for failure of GRAPHIC to pay any
found by the trial court, — which, in our opinion, are fully and its affairs are so conducted as to make it merely an installment, petitioner filed with the then Court of First
supported by the evidence — Francisco Sycip may be held instrumentality, agency or conduit of another (Koppel Instance of Manila, Branch XXIII, presided over by herein
liable, jointly and severally with his co-defendant, for the Phils., etc. vs. Yatco, etc., 43 O.G. No. 11. Nov. 1947; respondent judge, Civil Case No. 91857 for a Sum of Money
sums of money adjudged in favor of NAMARCO. Yutivo Sons, etc. vs. Court of Tax Appeals, etc., G.R. No. (Rollo, pp. 36-38). Respondent judge declared GRAPHIC in
L-13203, promulgated on January 28, 1961). default for failure to file its answer within the
The evidence of record shows that, of the capital stock of reglementary period and plaintiff (petitioner herein) was
ASSOCIATED, Sycip owned P60,000.00 worth of shares, Wherefore, the decision appealed from is modified by allowed to present its evidence ex parte. In a Decision
while his wife — the second biggest stockholder — owned sentencing defendant-appellee Francisco Sycip to pay, dated January 18, 1974 (Ibid., pp. 39-40), the trial court
P20,000.00 worth of shares; that the par value of the jointly and severally with the Associated Finance Company, ordered GRAPHIC to pay the petitioner the sum of
subscribed capital stock of ASSOCIATED was only Inc., the sum of money which the trial court sentenced the P30,365.99 with 12% interest from March 30, 1973 until
P105,000.00; that negotiations that lead to the execution latter to pay to the National Marketing Corporation, as fully paid, plus the costs of suit. On motion of petitioner, a
of the exchange agreement in question were conducted follows: the sum of FOUR HUNDRED THREE THOUSAND FIVE writ of execution was issued by respondent judge; but the
exclusively by Sycip on behalf of ASSOCIATED; that, as a HUNDRED FOURTEEN PESOS, and TWENTY-EIGHT CENTAVOS aforestated writ having expired without the sheriff finding
matter of fact, in the course of his testimony, Sycip P403,514.28), with interest at the legal rate from the date any property of GRAPHIC, an alias writ of execution was
referred to himself as the one who contracted or of the filing of the action until fully paid plus an additional issued on July 2, 1974.
transacted the business in his personal capacity, and amount of EIGHTY THOUSAND SEVEN HUNDRED TWO PESOS
asserted that the exchange agreement was his personal and EIGHTY-SIX CENTAVOS (P80,702.86) as liquidated Pursuant to the said issued alias writ of execution, the
contract; that it was Sycip who made personal damages and P5,000.00 as attorney's fees and further to executing sheriff levied upon one (1) unit printing machine
representations and gave assurances that ASSOCIATED was pay the costs. With costs. Identified as "Original Heidelberg Cylinder Press" Type H
in actual possession of the 22,516 bags of "Victorias" and/or 222, NR 78048, found in the premises of GRAPHIC. In a
"National" refined sugar which the latter had agreed to Notice of Sale of Execution of Personal Property dated July
deliver to NAMARCO, and that the same was ready for G.R. No. L-41337 June 30, 1988 29, 1974, said printing machine was scheduled for auction
delivery; that, as a matter of fact, ASSOCIATED was at that TAN BOON BEE & CO., INC., petitioner, vs. THE sale on July 26, 1974 at 10:00 o'clock at 14th St., Cor.
time already insolvent; that when NAMARCO made demands HONORABLE HILARION U. JARENCIO, PRESIDING JUDGE Atlanta St., Port Area, Manila (lbid., p. 45); but in a letter
upon ASSOCIATED to deliver the 22,516 bags of refined OF BRANCH XVIII of the Court of First Instance of Manila, dated July 19, 1974, herein private respondent, Philippine
sugar it was under obligation to deliver to the former, GRAPHIC PUBLISHING, INC., and PHILIPPINE AMERICAN American Drug Company (PADCO for short) had informed
ASSOCIATED and Sycip, instead of making delivery of the CAN DRUG COMPANY, respondents. the sheriff that the printing machine is its property and not
sugar, offered to pay its value at the rate of P15.30 per bag that of GRAPHIC, and accordingly, advised the sheriff to
— a clear indication that they did not have the sugar PARAS, J.: cease and desist from carrying out the scheduled auction
contracted for.1äwphï1.ñët sale on July 26, 1974. Notwithstanding the said letter, the
This is a petition for certiorari, with prayer for preliminary sheriff proceeded with the scheduled auction sale, sold the
The foregoing facts, fully established by the evidence, can injunction, to annul and set aside the March 26, 1975 Order property to the petitioner, it being the highest bidder, and
lead to no other conclusion than that Sycip was guilty of of the then Court of First Instance of Manila, Branch XXIII, issued a Certificate of Sale in favor of petitioner (Rollo, p.
fraud because through false representations he succeeded setting aside the sale of "Heidelberg" cylinder press 48). More than five (5) hours after the auction sale and the

Page 91 of 160
CORPORATION LAW CASES MAYORDO, M.A
LLB 3

issuance of the certificate of sale, PADCO filed an "Affidavit Petitioner contends that respondent judge gravely
of Third Party Claim" with the Office of the City Sheriff exceeded, if not, acted without jurisdiction, in nullifying The plaintiff, however, contends that the controlling
(Ibid., p. 47). Thereafter, on July 30,1974, PADCO filed with the sheriffs sale not only because Section 17, Rule 39 of the stockholders of the Philippine American Drug Co. are also
the Court of First Instance of Manila, Branch XXIII, a Motion Rules of Court was not complied with, but more the same controlling stockholders of the Graphic
to Nullify Sale on Execution (With Injunction) (Ibid., pp, importantly because PADCO could not have litigated its Publishing, Inc. and, therefore, the levy upon the said
49-55), which was opposed by the petitioner (Ibid., pp. claim in the same case, but in an independent civil machinery which was found in the premises occupied by
5668). Respondent judge, in an Order dated March 26, 1975 proceeding. the Graphic Publishing, Inc. should be upheld. This
(Ibid., pp. 64-69), ruled in favor of PADCO. The decretal contention cannot be sustained because the two
portion of the said order, reads: This contention is well-taken. corporations were duly incorporated under the Corporation
Law and each of them has a juridical personality distinct
WHEREFORE, the sale of the 'Heidelberg cylinder press In the case of Bayer Philippines, Inc. vs. Agana (63 SCRA and separate from the other and the properties of one
executed by the Sheriff in favor of the plaintiff as well as 355, 366-367 [1975]), this Court categorically ruled as cannot be levied upon to satisfy the obligation of the other.
the levy on the said property is hereby set aside and follows: This legal preposition is elementary and fundamental.
declared to be without any force and effect. The Sheriff is
ordered to return the said machinery to its owner, the In other words, constitution, Section 17 of Rule 39 of the It is true that a corporation, upon coming into being, is
Philippine American Drug Co. Revised Rules of Court, the rights of third-party claimants invested by law with a personality separate and distinct
over certain properties levied upon by the sheriff to satisfy from that of the persons composing it as well as from any
Petitioner filed a Motion For Reconsideration (Ibid., pp. the judgment should not be decided inthe action where the other legal entity to which it may be related (Yutivo& Sons
7093) and an Addendum to Motion for Reconsideration third-party claims have been presented, but in the separate Hardware Company vs. Court of Tax Appeals, 1 SCRA 160
(Ibid., pp. 94-08), but in an Order dated August 13, 1975, action instituted by the claimants. [1961]; and Emilio Cano Enterprises, Inc. vs. CIR, 13 SCRA
the same was denied for lack of merit (Ibid., p. 109). 290 [1965]). As a matter of fact, the doctrine that a
Hence, the instant petition. ... Otherwise stated, the court issuing a writ of execution is corporation is a legal entity distinct and separate from the
supposed to enforce the authority only over properties of members and stockholders who compose it is recognized
In a Resolution dated September 12, 1975, the Second the judgment debtor, and should a third party appeal- to and respected in all cases which are within reason and the
Division of this Court resolved to require the respondents to claim the property levied upon by the sheriff, the law (Villa Rey Transit, Inc. vs. Ferrer, 25 SCRA 845 [1968]).
comment, and to issue a temporary restraining order procedure laid down by the Rules is that such claim should However, this separate and distinct personality is merely a
(Rollo, p. 111 ). After submission of the parties' be the subject of a separate and independent action. fiction created by law for convenience and to promote
Memoranda, the case was submitted for decision in the justice (Laguna Transportation Company vs. SSS, 107 Phil.
Resolution of November 28, 1975 (Ibid., p. 275). xxx xxx xxx 833 [1960]). Accordingly, this separate personality of the
... This rule is dictated by reasons of convenience, as corporation may be disregarded, or the veil of corporate
Petitioner, to support its stand, raised two (2) issues, to "intervention is more likely to inject confusion into the fiction pierced, in cases where it is used as a cloak or cover
wit: issues between the parties in the case . . . with which the for fraud or illegality, or to work an injustice, or where
third-party claimant has nothing to do and thereby retard necessary to achieve equity or when necessary for the
I instead of facilitate the prompt dispatch of the controversy protection of creditors (Sulo ng Bayan, Inc. vs. Araneta,
which is the underlying objective of the rules of pleading Inc., 72 SCRA 347 [1976]). Corporations are composed of
THE RESPONDENT JUDGE GRAVELY EXCEEDED, IF NOT ACTED and practice." Besides, intervention may not be permitted natural persons and the legal fiction of a separate
WITHOUT JURISDICTION WHEN HE ACTED UPON THE MOTION after trial has been concluded and a final judgment corporate personality is not a shield for the commission of
OF PADCO, NOT ONLY BECAUSE SECTION 17, RULE 39 OF THE rendered in the case. injustice and inequity (Chenplex Philippines, Inc., et al. vs.
RULES OF COURT WAS NOT COMPLIED WITH, BUT ALSO Hon. Pamatian et al., 57 SCRA 408 (19741). Likewise, this is
BECAUSE THE CLAIMS OF PADCO WHICH WAS NOT A PARTY However, the fact that petitioner questioned the true when the corporation is merely an adjunct, business
TO THE CASE COULD NOT BE VENTILATED IN THE CASE jurisdiction of the court during the initial hearing of the conduit or alter ego of another corporation. In such case,
BEFORE HIM BUT IN INDEPENDENT PROCEEDING. case but nevertheless actively participated in the trial, bars the fiction of separate and distinct corporation entities
it from questioning now the court's jurisdiction. A party who should be disregarded (Commissioner of Internal Revenue
II voluntarily participated in the trial, like the herein vs. Norton & Harrison, 11 SCRA 714 [1964]).
petitioner, cannot later on raise the issue of the court's lack
THE RESPONDENT JUDGE GRAVELY ABUSED HIS DISCRETION of jurisdiction (Philippine National Bank vs. Intermediate In the instant case, petitioner's evidence established that
WHEN HE REFUSED TO PIERCE THE PADCO'S (IDENTITY) AND Appellate Court, 143 SCRA [1986]). PADCO was never engaged in the printing business; that the
DESPITE THE ABUNDANCE OF EVIDENCE CLEARLY SHOWING board of directors and the officers of GRAPHIC and PADCO
THAT PADCO WAS CONVENIENTLY SHIELDING UNDER THE As to the second issue (the non-piercing of PADCO's were the same; and that PADCO holds 50% share of stock of
THEORY OF CORPORATE PETITION. corporate Identity) the decision of respondent judge is as GRAPHIC. Petitioner likewise stressed that PADCO's own
follows: evidence shows that the printing machine in question had

Page 92 of 160
CORPORATION LAW CASES MAYORDO, M.A
LLB 3

been in the premises of GRAPHIC since May, 1965, long MAGSAYSAY, and MERCEDES MAGSAYSAY-DIAZ, petitioners, consideration and consequently null and void. She prayed
before PADCO even acquired its alleged title on July 11, vs. THE COURT OF APPEALS and ADELAIDA RODRIGUEZ- that the Deed of Assignment and the Deed of Mortgage be
1966 from Capitol Publishing. That the said machine was MAGSAYSAY, Special Administratrix of the Estate of the annulled and that the Register of Deeds be ordered to
allegedly leased by PADCO to GRAPHIC on January 24, 1966, late Genaro F. Magsaysay respondents. cancel TCT No. 22431 and to issue a new title in her favor.
even before PADCO purchased it from Capital Publishing on
July 11, 1966, only serves to show that PADCO's claim of FERNAN, C.J.: On March 7, 1979, herein petitioners, sisters of the late
ownership over the printing machine is not only farce and senator, filed a motion for intervention on the ground that
sham but also unbelievable. In this petition for review on certiorari, petitioners seek to on June 20, 1978, their brother conveyed to them one-half
reverse and set aside [1] the decision of the Court of (1/2 ) of his shareholdings in SUBIC or a total of 416,566.6
Considering the aforestated principles and the Appeals dated July l3, 1981, 1 affirming that of the Court shares and as assignees of around 41 % of the total
circumstances established in this case, respondent judge of First Instance of Zambales and Olongapo City which outstanding shares of such stocks of SUBIC, they have a
should have pierced PADCO's veil of corporate Identity. denied petitioners' motion to intervene in an annulment substantial and legal interest in the subject matter of
suit filed by herein private respondent, and [2] its litigation and that they have a legal interest in the success
Respondent PADCO argues that if respondent judge erred in resolution dated September 7, 1981, denying their motion of the suit with respect to SUBIC.
not piercing the veil of its corporate fiction, the error is for reconsideration.
merely an error of judgment and not an error of On July 26, 1979, the court denied the motion for
jurisdiction correctable by appeal and not by certiorari. Petitioners are raising a purely legal question; whether or intervention, and ruled that petitioners have no legal
not respondent Court of Appeals correctly denied their interest whatsoever in the matter in litigation and their
To this argument of respondent, suffice it to say that the motion for intervention. being alleged assignees or transferees of certain shares in
same is a mere technicality. In the case of Rubio vs. SUBIC cannot legally entitle them to intervene because
Mariano (52 SCRA 338, 343 [1973]), this Court ruled: The facts are not controverted. SUBIC has a personality separate and distinct from its
stockholders.
While We recognize the fact that these movants — the On February 9, 1979, Adelaida Rodriguez-Magsaysay, widow
MBTC, the Phillips spouses, the Phillips corporation and the and special administratix of the estate of the late Senator On appeal, respondent Court of Appeals found no factual or
Hacienda Benito, Inc.— did raise in their respective answers Genaro Magsaysay, brought before the then Court of First legal justification to disturb the findings of the lower court.
the issue as to the propriety of the instant petition for Instance of Olongapo an action against Artemio Panganiban, The appellate court further stated that whatever claims
certiorari on the ground that the remedy should have been Subic Land Corporation (SUBIC), Filipinas Manufacturer's the petitioners have against the late Senator or against
appeal within the reglementary period, We considered such Bank (FILMANBANK) and the Register of Deeds of Zambales. SUBIC for that matter can be ventilated in a separate
issue as a mere technicality which would have In her complaint, she alleged that in 1958, she and her proceeding, such that with the denial of the motion for
accomplished nothing substantial except to deny to the husband acquired, thru conjugal funds, a parcel of land intervention, they are not left without any remedy or
petitioner the right to litigate the matters he raised ... with improvements, known as "Pequena Island", covered by judicial relief under existing law.
TCT No. 3258; that after the death of her husband, she
Litigations should, as much as possible, be decided on their discovered [a] an annotation at the back of TCT No. 3258 Petitioners' motion for reconsideration was denied. Hence,
merits and not on technicality (De las Alas vs. Court of that "the land was acquired by her husband from his the instant recourse.
Appeals, 83 SCRA 200, 216 [1978]). Every party-litigant separate capital;" [b] the registration of a Deed of
must be afforded the amplest opportunity for the proper Assignment dated June 25, 1976 purportedly executed by Petitioners anchor their right to intervene on the purported
and just determination of his cause, free from the the late Senator in favor of SUBIC, as a result of which TCT assignment made by the late Senator of a certain portion of
unacceptable plea of technicalities (Heirs of Ceferino No. 3258 was cancelled and TCT No. 22431 issued in the his shareholdings to them as evidenced by a Deed of Sale
Morales vs. Court of Appeals, 67 SCRA 304, 310 [1975]). name of SUBIC; and [c] the registration of Deed of Mortgage dated June 20, 1978. 2 Such transfer, petitioners posit,
dated April 28, 1977 in the amount of P 2,700,000.00 clothes them with an interest, protected by law, in the
PREMISES CONSIDERED, the March 26,1975 Order of the executed by SUBIC in favor of FILMANBANK; that the matter of litigation.
then Court of First Instance of Manila, is ANNULLED and SET foregoing acts were void and done in an attempt to defraud
ASIDE, and the Temporary Restraining Order issued is the conjugal partnership considering that the land is Invoking the principle enunciated in the case of PNB v. Phil.
hereby made permanent. conjugal, her marital consent to the annotation on TCT No. Veg. Oil Co., 49 Phil. 857,862 & 853 (1927), 3 petitioners
3258 was not obtained, the change made by the Register of strongly argue that their ownership of 41.66% of the entire
SO ORDERED. Deeds of the titleholders was effected without the approval outstanding capital stock of SUBIC entitles them to a
of the Commissioner of Land Registration and that the late significant vote in the corporate affairs; that they are
G.R. No. 58168 December 19, 1989 Senator did not execute the purported Deed of Assignment affected by the action of the widow of their late brother
CONCEPCION MAGSAYSAY-LABRADOR, SOLEDAD or his consent thereto, if obtained, was secured by for it concerns the only tangible asset of the corporation
MAGSAYSAY-CABRERA, LUISA MAGSAYSAY-CORPUZ, mistake, violence and intimidation. She further alleged and that it appears that they are more vitally interested in
assisted be her husband, Dr. Jose Corpuz, FELICIDAD P. that the assignment in favor of SUBIC was without the outcome of the case than SUBIC.

Page 93 of 160
CORPORATION LAW CASES MAYORDO, M.A
LLB 3

While a share of stock represents a proportionate or aliquot


Viewed in the light of Section 2, Rule 12 of the Revised interest in the property of the corporation, it does not vest The factual findings of the trial court are clear on this
Rules of Court, this Court affirms the respondent court's the owner thereof with any legal right or title to any of the point. The petitioners cannot claim the right to intervene
holding that petitioners herein have no legal interest in the property, his interest in the corporate property being on the strength of the transfer of shares allegedly executed
subject matter in litigation so as to entitle them to equitable or beneficial in nature. Shareholders are in no by the late Senator. The corporation did not keep books and
intervene in the proceedings below. In the case of Batama legal sense the owners of corporate property, which is records. 11 Perforce, no transfer was ever recorded, much
Farmers' Cooperative Marketing Association, Inc. v. Rosal, 4 owned by the corporation as a distinct legal person. 8 less effected as to prejudice third parties. The transfer
we held: "As clearly stated in Section 2 of Rule 12 of the must be registered in the books of the corporation to affect
Rules of Court, to be permitted to intervene in a pending Petitioners further contend that the availability of other third persons. The law on corporations is explicit. Section
action, the party must have a legal interest in the matter in remedies, as declared by the Court of appeals, is totally 63 of the Corporation Code provides, thus: "No transfer,
litigation, or in the success of either of the parties or an immaterial to the availability of the remedy of however, shall be valid, except as between the parties,
interest against both, or he must be so situated as to be intervention. until the transfer is recorded in the books of the
adversely affected by a distribution or other disposition of corporation showing the names of the parties to the
the property in the custody of the court or an officer We cannot give credit to such averment. As earlier stated, transaction, the date of the transfer, the number of the
thereof ." that the movant's interest may be protected in a separate certificate or certificates and the number of shares
proceeding is a factor to be considered in allowing or transferred."
To allow intervention, [a] it must be shown that the movant disallowing a motion for intervention. It is significant to
has legal interest in the matter in litigation, or otherwise note at this juncture that as per records, there are four And even assuming arguendo that there was a valid
qualified; and [b] consideration must be given as to pending cases involving the parties herein, enumerated as transfer, petitioners are nonetheless barred from
whether the adjudication of the rights of the original follows: [1] Special Proceedings No. 122122 before the CFI intervening inasmuch as their rights can be ventilated and
parties may be delayed or prejudiced, or whether the of Manila, Branch XXII, entitled "Concepcion Magsaysay- amply protected in another proceeding.
intervenor's rights may be protected in a separate Labrador, et al. v. Subic Land Corp., et al.", involving the
proceeding or not. Both requirements must concur as the validity of the transfer by the late Genaro Magsaysay of WHEREFORE, the instant petition is hereby DENIED. Costs
first is not more important than the second. 5 one-half of his shareholdings in Subic Land Corporation; [2] against petitioners.
Civil Case No. 2577-0 before the CFI of Zambales, Branch
The interest which entitles a person to intervene in a suit III, "Adelaida Rodriguez-Magsaysay v. Panganiban, etc.; SO ORDERED.
between other parties must be in the matter in litigation Concepcion Labrador, et al. Intervenors", seeking to annul
and of such direct and immediate character that the the purported Deed of Assignment in favor of SUBIC and its G.R. No. 96490 February 3, 1992
intervenor will either gain or lose by the direct legal annotation at the back of TCT No. 3258 in the name of INDOPHIL TEXTILE MILL WORKERS UNION-PTGWO,
operation and effect of the judgment. Otherwise, if respondent's deceased husband; [3] SEC Case No. 001770, petitioner, vs. VOLUNTARY ARBITRATOR TEODORICO P.
persons not parties of the action could be allowed to filed by respondent praying, among other things that she be CALICA and INDOPHIL TEXTILE MILLS, INC., respondents.
intervene, proceedings will become unnecessarily declared in her capacity as the surviving spouse and
complicated, expensive and interminable. And this is not administratrix of the estate of Genaro Magsaysay as the MEDIALDEA, J.:
the policy of the law. 6 sole subscriber and stockholder of SUBIC. There,
petitioners, by motion, sought to intervene. Their motion This is a petition for certiorari seeking the nullification of
The words "an interest in the subject" mean a direct to reconsider the denial of their motion to intervene was the award issued by the respondent Voluntary Arbitrator
interest in the cause of action as pleaded, and which would granted; [4] SP No. Q-26739 before the CFI of Rizal, Branch Teodorico P. Calica dated December 8, 1990 finding that
put the intervenor in a legal position to litigate a fact IV, petitioners herein filing a contingent claim pursuant to Section 1 (c), Article I of the Collective Bargaining
alleged in the complaint, without the establishment of Section 5, Rule 86, Revised Rules of Court. 9 Petitioners' Agreement between Indophil Textile Mills, Inc. and Indophil
which plaintiff could not recover. 7 interests are no doubt amply protected in these cases. Textile Mill Workers Union-PTGWO does not extend to the
employees of Indophil Acrylic Manufacturing Corporation as
Here, the interest, if it exists at all, of petitioners-movants Neither do we lend credence to petitioners' argument that an extension or expansion of Indophil Textile Mills,
is indirect, contingent, remote, conjectural, consequential they are more interested in the outcome of the case than Incorporated.
and collateral. At the very least, their interest is purely the corporation-assignee, owing to the fact that the latter
inchoate, or in sheer expectancy of a right in the is willing to compromise with widow-respondent and since The antecedent facts are as follows:
management of the corporation and to share in the profits a compromise involves the giving of reciprocal concessions,
thereof and in the properties and assets thereof on the only conceivable concession the corporation may give is Petitioner Indophil Textile Mill Workers Union-PTGWO is a
dissolution, after payment of the corporate debts and a total or partial relinquishment of the corporate assets. 10 legitimate labor organization duly registered with the
obligations. Department of Labor and Employment and the exclusive
Such claim all the more bolsters the contingent nature of bargaining agent of all the rank-and-file employees of
petitioners' interest in the subject of litigation. Indophil Textile Mills, Incorporated. Respondent Teodorico

Page 94 of 160
CORPORATION LAW CASES MAYORDO, M.A
LLB 3

P. Calica is impleaded in his official capacity as the After the parties submitted their respective position papers engaged in the same kind of business, which is the
Voluntary Arbitrator of the National Conciliation and and replies, the public respondent Voluntary Arbitrator manufacture and sale of yarns of various counts and kinds
Mediation Board of the Department of Labor and rendered its award on December 8, 1990, the dispositive and of other materials of kindred character or nature.
Employment, while private respondent Indophil Textile portion of which provides as follows:
Mills, Inc. is a corporation engaged in the manufacture, Contrary to petitioner's assertion, the public respondent
sale and export of yarns of various counts and kinds and of PREMISES CONSIDERED, it would be a strained through the Solicitor General argues that the Indophil
materials of kindred character and has its plants at Barrio interpretation and application of the questioned CBA Acrylic Manufacturing Corporation is not an alter ego or an
Lambakin. Marilao, Bulacan. provision if we would extend to the employees of Acrylic adjunct or business conduit of private respondent because
the coverage clause of Indophil Textile Mills CBA. it has a separate legitimate business purpose. In addition,
In April, 1987, petitioner Indophil Textile Mill Workers Wherefore, an award is made to the effect that the proper the Solicitor General alleges that the primary purpose of
Union-PTGWO and private respondent Indophil Textile Mills, interpretation and application of Sec. l, (c), Art. I, of the private respondent is to engage in the business of
Inc. executed a collective bargaining agreement effective 1987 CBA do (sic) not extend to the employees of Acrylic as manufacturing yarns of various counts and kinds and
from April 1, 1987 to March 31, 1990. an extension or expansion of Indophil Textile Mills, Inc. textiles. On the other hand, the primary purpose of
(Rollo, p.21) Indophil Acrylic is to manufacture, buy, sell at wholesale
On November 3, 1967 Indophil Acrylic Manufacturing basis, barter, import, export and otherwise deal in yarns of
Corporation was formed and registered with the Securities Hence, this petition raising four (4) issues, to wit: various counts and kinds. Hence, unlike private respondent,
and Exchange Commission. Subsequently, Acrylic applied for Indophil Acrylic cannot manufacture textiles while private
registration with the Board of Investments for incentives 1. WHETHER OR NOT THE RESPONDENT ARBITRATOR respondent cannot buy or import yarns.
under the 1987 Omnibus Investments Code. The application ERRED IN INTERPRETING SECTION 1(c), ART I OF THE CBA
was approved on a preferred non-pioneer status. BETWEEN PETITIONER UNION AND RESPONDENT COMPANY. Furthermore, petitioner emphasizes that the two
corporations have practically the same incorporators,
In 1988, Acrylic became operational and hired workers 2. WHETHER OR NOT INDOPHIL ACRYLIC IS A directors and officers. In fact, of the total stock
according to its own criteria and standards. Sometime in SEPARATE AND DISTINCT ENTITY FROM RESPONDENT subscription of Indophil Acrylic, P1,749,970.00 which
July, 1989, the workers of Acrylic unionized and a duly COMPANY FOR PURPOSES OF UNION REPRESENTATION. represents seventy percent (70%) of the total subscription
certified collective bargaining agreement was executed. of P2,500,000.00 was subscribed to by respondent
3. WHETHER OR NOT THE RESPONDENT ARBITRATOR Company.
In 1990 or a year after the workers of Acrylic have been GRAVELY ABUSED HIS DISCRETION AMOUNTING TO LACK OR
unionized and a CBA executed, the petitioner union IN EXCESS OF HIS JURISDICTION. On this point, private respondent cited the case of Diatagon
claimed that the plant facilities built and set up by Acrylic Labor Federation v. Ople, G.R. No. L-44493-94, December
should be considered as an extension or expansion of the 4. WHETHER OR NOT THE RESPONDENT ARBITRATOR 3, 1980, 10l SCRA 534, which ruled that two corporations
facilities of private respondent Company pursuant to VIOLATED PETITIONER UNION'S CARDINAL PRIMARY RIGHT TO cannot be treated as a single bargaining unit even if their
Section 1(c), Article I of the CBA, to wit,. DUE PROCESS. (Rollo, pp. 6-7) businesses are related. It submits that the fact that there
are as many bargaining units as there are companies in a
c) This Agreement shall apply to the Company's The central issue submitted for arbitration is whether or conglomeration of companies is a positive proof that a
plant facilities and installations and to any extension and not the operations in Indophil Acrylic Corporation are an corporation is endowed with a legal personality distinctly
expansion thereat. (Rollo, p.4) extension or expansion of private respondent Company. its own, independent and separate from other corporations
Corollary to the aforementioned issue is the question of (see Rollo, pp. 160-161).
In other words, it is the petitioner's contention that Acrylic whether or not the rank-and-file employees working at
is part of the Indophil bargaining unit. Indophil Acrylic should be recognized as part of, and/or Petitioner notes that the foregoing evidence sufficiently
within the scope of the bargaining unit. establish that Acrylic is but an extension or expansion of
The petitioner's contention was opposed by private private respondent, to wit:
respondent which submits that it is a juridical entity Petitioner maintains that public respondent Arbitrator
separate and distinct from Acrylic. gravely erred in interpreting Section l(c), Article I of the (a) the two corporations have their physical plants,
CBA in its literal meaning without taking cognizance of the offices and facilities situated in the same compound, at
The existing impasse led the petitioner and private facts adduced that the creation of the aforesaid Indophil Barrio Lambakin, Marilao, Bulacan;
respondent to enter into a submission agreement on Acrylic is but a devise of respondent Company to evade the
September 6, 1990. The parties jointly requested the public application of the CBA between petitioner Union and (b) many of private respondent's own machineries,
respondent to act as voluntary arbitrator in the resolution respondent Company. such as dyeing machines, reeling, boiler, Kamitsus among
of the pending labor dispute pertaining to the proper others, were transferred to and are now installed and being
interpretation of the CBA provision. Petitioner stresses that the articles of incorporation of the used in the Acrylic plant;
two corporations establish that the two entities are

Page 95 of 160
CORPORATION LAW CASES MAYORDO, M.A
LLB 3

(c) the services of a number of units, departments or be disregarded. In such cases, the corporation will be All premises considered, the Court is convinced that the
sections of private respondent are provided to Acrylic; and considered as a mere association of persons. The members public respondent Voluntary Arbitrator did not commit
or stockholders of the corporation will be considered as the grave abuse of discretion in its interpretation of Section
(d) the employees of private respondent are the corporation, that is liability will attach directly to the l(c), Article I of the CBA that the Acrylic is not an extension
same persons manning and servicing the units of Acrylic. officers and stockholders. The doctrine applies when the or expansion of private respondent.
(see Rollo, pp. 12-13) corporate fiction is used to defeat public convenience,
justify wrong, protect fraud, or defend crime, or when it is ACCORDINGLY, the petition is DENIED and the award of the
Private respondent insists that the existence of a bonafide made as a shield to confuse the legitimate issues, or where respondent Voluntary Arbitrator are hereby AFFIRMED.
business relationship between Acrylic and private a corporation is the mere alter ego or business conduit of a
respondent is not a proof of being a single corporate entity person, or where the corporation is so organized and SO ORDERED.
because the services which are supposedly provided by it to controlled and its affairs are so conducted as to make it
Acrylic are auxiliary services or activities which are not merely an instrumentality, agency, conduit or adjunct of G.R. No. 80043 June 6, 1991
really essential in the actual production of Acrylic. It also another corporation. (Umali et al. v. Court of Appeals, G.R. ROBERTO A. JACINTO, petitioner, vs. HONORABLE COURT
pointed out that the essential services are discharged No. 89561, September 13, 1990, 189 SCRA 529, 542) OF APPEALS and METROPOLITAN BANK AND TRUST
exclusively by Acrylic personnel under the control and COMPANY, respondents.
supervision of Acrylic managers and supervisors. In the case at bar, petitioner seeks to pierce the veil of
corporate entity of Acrylic, alleging that the creation of the DAVIDE, JR., J.:
In sum, petitioner insists that the public respondent corporation is a devise to evade the application of the CBA
committed grave abuse of discretion amounting to lack or between petitioner Union and private respondent Company. This is an appeal by certiorari to partially set aside the
in excess of jurisdiction in erroneously interpreting the CBA While we do not discount the possibility of the similarities Decision of the Court of Appeals in C.A-G.R. CV No.
provision and in failing to disregard the corporate entity of of the businesses of private respondent and Acrylic, neither 081531.promulgated on 19 August 1987, which affirmed in
Acrylic. are we inclined to apply the doctrine invoked by petitioner toto the decision of the Regional Trial Court of Manila,
in granting the relief sought. The fact that the businesses Branch 11, in Civil Case No. 133164 entitled "Metropolitan
We find the petition devoid of merit. of private respondent and Acrylic are related, that some of Bank and Trust Co. vs. Inland Industries Inc. and Roberto
the employees of the private respondent are the same Jacinto," the dispositive portion of which reads:
Time and again, We stress that the decisions of voluntary persons manning and providing for auxilliary services to the
arbitrators are to be given the highest respect and a units of Acrylic, and that the physical plants, offices and WHEREFORE, judgment is hereby rendered ordering
certain measure of finality, but this is not a hard and fast facilities are situated in the same compound, it is our defendants to pay, jointly and severally, the plaintiff, the
rule, it does not preclude judicial review thereof where considered opinion that these facts are not sufficient to principal obligation of P382,015.80 (Annex J-1 to J-3 of
want of jurisdiction, grave abuse of discretion, violation of justify the piercing of the corporate veil of Acrylic. Stipulation), with interest/charges thereon at the rate of
due process, denial of substantial justice, or erroneous 16 % per annum from January 1, 1979 up to the time the
interpretation of the law were brought to our attention. In the same case of Umali, et al. v. Court of Appeals said amount is fully paid, plus the sum of P20,000.00 as
(see Ocampo, et al. v. National Labor Relations (supra), We already emphasized that "the legal corporate attorney's fees. Said defendants are further ordered to pay
Commission, G.R. No. 81677, 25 July 1990, First Division entity is disregarded only if it is sought to hold the officers in solidum the costs of this suit.
Minute Resolution citing Oceanic Bic Division (FFW) v. and stockholders directly liable for a corporate debt or
Romero, G.R. No. L-43890, July 16, 1984, 130 SCRA 392) obligation." In the instant case, petitioner does not seek to SO ORDERED.2
impose a claim against the members of the Acrylic.
It should be emphasized that in rendering the subject Petitioner's co-defendant in the courts below, Inland
arbitral award, the voluntary arbitrator TeodoricoCalica, a Furthermore, We already ruled in the case of Diatagon Industries Inc., just as in the case of petitioner's motion to
professor of the U.P. Asian Labor Education Center, now the Labor Federation Local 110 of the ULGWP v. Ople (supra) reconsider the questioned decision,3 chose not to join him
Institute for Industrial Relations, found that the existing that it is grave abuse of discretion to treat two companies in this appeal.
law and jurisprudence on the matter, supported the private as a single bargaining unit when these companies are
respondent's contentions. Contrary to petitioner's assertion, indubitably distinct entities with separate juridical In Our resolution of 28 August 1988 We required the
public respondent cited facts and the law upon which he personalities. respondent to comment on the petition. Respondent
based the award. Hence, public respondent did not abuse Metropolitan Bank and Trust Co. filed its comment4 on 12
his discretion. Hence, the Acrylic not being an extension or expansion of October 1988. We required the petitioner to file a reply
private respondent, the rank-and-file employees working at thereto,5 which he comment plied with on 20 December
Under the doctrine of piercing the veil of corporate entity, Acrylic should not be recognized as part of, and/or within 1988.6
when valid grounds therefore exist, the legal fiction that a the scope of the petitioner, as the bargaining
corporation is an entity with a juridical personality representative of private respondent. We gave due course to the petition on 8 May 19897 and
separate and distinct from its members or stockholders may required the parties to submit their respective memoranda.

Page 96 of 160
CORPORATION LAW CASES MAYORDO, M.A
LLB 3

Defendant Roberto Jacinto, tried to escape liability and A Yes, sir. Because I am the General Manager of this
Private respondent filed its memorandum on 29 June 19898 shift the entire blame under the trust receipts solely and corporation.
while petitioner asked leave to adopt his petition and reply exclusively on defendant-appellant corporation. He
as his memorandum,9 which We granted on 14 June asserted that he cannot be held solidarily liable with the Q Aside from being the General Manager of the
1989.10 latter (defendant corporation) because he just signed said defendant corporation are you in any other way connected
instruments in his official capacity as president of Inland with the same?
Petitioner submits the following issues: Industries, Inc. and the latter (defendant corporation) has a A I am also a stockholder.
juridical personality distinct and separate from its officers
1. Whether or not the respondent Court of Appeals and stockholders. It is likewise asserted, citing an American Q Does your corporation have a Board of Directors?
can validly pierce the fiction of corporate identity of the case, that the principle of piercing the fiction of corporate A Yes, sir.
defendant corporation Inland Industries, Inc. even if there entity should be applied with great caution and not
is no allegation in the complaint regarding the same, nor is precipitately, because a dual personality by a corporation Q By the way, who are the stockholders of this
there anything in the prayer demanding the piercing of the and its stockholders would defeat the principal purpose for corporation?
corporate veil of the corporation Inland Industries, Inc.; which a corporation is formed. Upon the other hand, A BienvenidaCatabas, Aurora Heresa, Paz Yulo,
plaintiff-appellee reiterated its allegation in the complaint Hedy Y. Jacinto and myself.
2. Whether or not the Court of Appeals can validly that defendant corporation is just a mere alter ego of
pierce the fiction of corporate identity of the defendant defendant Roberto Jacinto who is its President and General Q Who is the President of the defendant
Inland Industries, Inc. even if absolutely no proof was Manager, while the wife of the latter owns a majority of its corporation?
presented in court to serve as legal justification for the shares of stock. A BienvenidaCatabas.
same.
Defendants-appellants' assertion is plainly without legal Q Who is the Chairman of the Board?
We find this petition to be bereft of merit. The issues are basis. This is shown by the undisputed fact that Roberto A Aurora Heresa.
basically factual and a careful scrutiny of the decisions of Jacinto even admitted that he and his wife own 52% of the
both courts below reveals that their findings and stocks of defendant corporation (TSN, April 22, 1985, p. 6). Q Do you have any relation with Hedy Y. Jacinto?
conclusions on the matter of piercing the veil of corporate We cannot accept as true the assertion of defendant A She is my wife.
fiction and on the liability of herein petitioner are Jacinto that he only acted in his official capacity as
overwhelmingly supported by the evidence. President and General Manager of Inland Industries, Inc. Q If you combine the stockholdings of your wife
when he signed the aforesaid trust receipts. To Our mind together with yours and percentage wise, how much is your
Insofar as material and relevant to the issues raised, the the same is just a clever ruse and a convenient ploy to equity?
trial court found and held:11 thwart his personal liability therefor by taking refuge under
the protective mantle of the separate corporate personality Atty. Dizon raised some objections. However, the Court
As to [the] liability of [the] defendant Roberto A. Jacinto, it of defendant corporation. allowed the same.
would appear that he is in factetum (sic), or, in fact, the
corporation itself known as Inland Industries, Inc. Aside As could be expected, Roberto Jacinto in his direct A About 52 % (Ibid., pp. 3-6)
from the fact that he is admittedly the President and testimony presented a different corporate scenario
General Manager of the corporation and a substantial regarding Inland Industries, Inc. and vehemently declared Furthermore, a cursory perusal of the Stipulation of facts
stockholders (sic) thereof, it was defendant Roberto A. that it is BienvenidaCatabas who is its President, while clearly shows that defendant Roberto Jacinto acted in his
Jacinto who dealt entirely with the plaintiff in those Aurora Heresa is its Chairman of the Board. His assertion on capacity as President and General Manager of Inland
transactions. In the Trust Receipts that he signed this point, however, is not convincing in view of his Industries, Inc. when he signed said trust receipts.
supposedly in behalf of Inland Industries, Inc., it is not even admission in the same breath, that his wife, Hedy U. Pertinent portion of his testimony are quoted below:
mentioned that he did so in this official capacity. Jacinto, own (sic) with him 52% of the shares of stock of
said corporation. Indeed, this circumstance –– even if (d) All the goods covered by the three (3) Letters of
x xxxxxxxx standing alone –– cannot but engender in the most Credit (Annexes "A", "B" & "C") and paid for under the Bills
unprejudiced mind doubt and misgiving why Catabas and of Exchange (Annexes "D", "E" & "F") were delivered to and
In this case, the Court is satisfied that Roberto A. Jacinto Heresa would be defendant corporation's President and received by defendant Inland Industries, Inc. through its co-
was practically the corporation itself, the Inland industries, Chairman of the Board, respectively. Pertinent portion of defendant Roberto A. Jacinto, its President and General
Inc. his testimony on this point is quoted hereunder: Manager, who signed for and in behalf of defendant Inland
and agreed to the terms and conditions of three (3)
In a detailed fashion, the respondent Court of Appeals Atty. Carlos Do you know the defendant Inland Industries, separate trust receipts covering the same and herein
brushed aside the posturing of petitioner as follows: Inc.? identified as follows: . . . (p. 3 of Stipulations of Facts and
Formulation of Issues [p. 95, Records]).

Page 97 of 160
CORPORATION LAW CASES MAYORDO, M.A
LLB 3

CONCEPT BUILDERS, INC., petitioner, vs. THE NATIONAL


The conflicting statements by defendant Jacinto place in Petitioner, however, faults the courts below for piercing the LABOR RELATIONS COMMISSION, (First Division); and
extreme doubt his credibility anent his alleged veil of corporate fiction despite the absence of any Norberto Marabe, Rodolfo Raquel, Cristobal Riego,
participation in said transactions and We are thus allegation in the complaint questioning the separate Manuel Gillego, PalcronioGiducos, Pedro Aboigar,
persuaded to agree with the findings of the lower court identity and existence of Inland Industries, Inc. This is not Norberto Comendador, Rogello Salut, Emilio Garcia, Jr.,
that the latter (Roberto Jacinto) was practically the accurate.1âwphi1 While on the face of the complaint there Mariano Rio, Paulina Basea, AifredoAlbera, Paquito Salut,
corporation itself. Indeed, a painstaking examination of the is no specific allegation that the corporation is a mere alter Domingo Guarino, Romeo Galve, Dominador Sabina,
records show that there is no clear-cut delimitation ego of petitioner, subsequent developments, from the Felipe Radiana, GavinoSualibio, Moreno Escares,
between the personality of Roberto Jacinto as an individual stipulation of facts up to the presentation of evidence and Ferdinand Torres, Felipe Basilan, and Ruben Robalos,
and the personality of Inland Industries, Inc. as a the examination of witnesses, unequivocally show that respondents.
corporation. respondent Metropolitan Bank and Trust Company sought to
prove that petitioner and the corporation are one or that DECISION
The circumstances aforestated lead Us to conclude that the he is the corporation. No serious objection was heard from
corporate veil that en-shrouds defendant Inland Industries, petitioner. Section 5 of Rule 10 of the Rules of Court HERMOSISIMA, JR., J.:
Inc. could be validly pierced, and a host of cases decided provides:
by our High Court is supportive of this view. Thus it held The corporate mask may be lifted and the corporate veil
that "when the veil of corporate fiction is made as a shield Sec. 5. Amendment to conform to or authorize may be pierced when a corporation is just but the alter ego
to perpetuate fraud and/or confuse legitimate issues, the presentation of evidence. –– When issues not raised by the of a person or of another corporation. Where badges of
same should be pierced." (Republic vs. Razon, 20 SCRA 234; pleadings are tried by express or implied consent of the fraud exist; where public convenience is defeated; where a
A.D. Santos, Inc. vs. Vasquez, 22 SCRA 1156; Emilio Cano parties, they shall be treated in all respects, as if they had wrong is sought to be justified thereby, the corporate
Enterprises, Inc. vs. Court of Appeals, 13 SCRA 290). Almost been raised in the pleadings. Such amendment of the fiction or the notion of legal entity should come to naught.
in the same vein is the dictum enunciated by the same pleadings as may be necessary to cause them to conform to The law in these instances will regard the corporation as a
court in the case of Commissioner of Internal Revenue vs. the evidence and to raise these issues may be made upon mere association of persons and, in case of two
Norton & Harrison Co., (11 SCRA 714), that "Where a motion of any party at any time, even after judgment; but corporations, merge them into one.
corporation is merely an adjunct, business conduit or alter failure so to amend does not affect the trial of these
ego, the fiction of separate and distinct corporate entity issues. If the evidence is objected to at the time of trial on Thus, where a sister corporation is used as a shield to
should be disregarded." the ground that it is not within the issues made by the evade a corporations subsidiary liability for damages, the
pleadings, the court may allow the pleadings to be corporation may not be heard to say that it has a
In its resolution of 29 September 1987, the respondent amended and shall do so freely when the presentation of personality separate and distinct from the other
Court of Appeals, on the contention again of petitioner that the merits of the action will be subserved thereby and the corporation. The piercing of the corporate veil comes into
the finding that defendant corporation is his mere alter ego objecting party fails to satisfy the court that the admission play.
is not supported by the evidence and has no legal of such evidence would prejudice him in maintaining his
justification, ruled that: action or defense upon the merits. The court may grant This special civil action ostensibly raises the question of
continuance to enable the objecting party to meet such whether the National Labor Relations Commission
The contention . . . is nothing but an empty assertion. A evidence. committed grave abuse of discretion when it issued a
cursory perusal of the decision would at once readily show break-open order to the sheriff to be enforced against
on pages 11-13 of the same that said factual findings of the Pursuant thereto, "when evidence is presented by one personal property found in the premises of petitioners
court is well grounded as the same in fact even include a party, with the express or implied consent of the adverse sister company.
portion of the very testimony of said defendant-appellant party, as to issues not alleged in the pleadings, judgment
admitting that he and his wife own 52% of the stocks of may be rendered validly as regards those issues, which shall Petitioner Concept Builders, Inc., a domestic corporation,
defendant corporation. The stipulation of facts also show be considered as if they have been raised in the pleadings. with principal office at 355 Maysan Road, Valenzuela, Metro
(sic) that appellant Roberto Jacinto acted in his capacity as There is implied consent to the evidence thus presented Manila, is engaged in the construction business. Private
President/General Manager of defendant corporation and when the adverse party fails to object thereto.12 respondents were employed by said company as laborers,
that "all the goods covered by the three (3) Letters of carpenters and riggers.
Credit (Annexes "A", "B" & "C") and paid for under the Bills WHEREFORE, for lack of merit, the Petition is DISMISSED
of Exchange (Annexes "D", "E" & "F") were delivered to and with costs against petitioner. On November, 1981, private respondents were served
received by defendant Inland Industries, Inc. through its co- individual written notices of termination of employment by
defendant Roberto A. Jacinto, its President and General SO ORDERED. petitioner, effective on November 30, 1981. It was stated in
Manager, who signed for and in behalf of defendant Inland the individual notices that their contracts of employment
and agreed to the terms and conditions of three (3) G.R. No. 108734. May 29, 1996] had expired and the project in which they were hired had
separate trust receipts covering the same. been completed.

Page 98 of 160
CORPORATION LAW CASES MAYORDO, M.A
LLB 3

The said writ had not been enforced by the special sheriff Antonio W. Lim 2,900,000.00
Public respondent found it to be, the fact, however, that at because, as stated in his progress report, dated November Dennis S. Cuyegkeng 300.00
the time of the termination of private respondents 2, 1989: Elisa C. Lim 100,000.00
employment, the project in which they were hired had not Teodulo R. Dino 100.00
yet been finished and completed. Petitioner had to engage 1. All the employees inside petitioners premises at 355 Virgilio O. Casino 100.00
the services of sub-contractors whose workers performed Maysan Road, Valenzuela, Metro Manila, claimed that they
the functions of private respondents. were employees of Hydro Pipes Philippines, Inc. (HPPI) and 2. Board of Directors
not by respondent;
Aggrieved, private respondents filed a complaint for illegal Antonio W. Lim Chairman
dismissal, unfair labor practice and non-payment of their 2. Levy was made upon personal properties he found in the Dennis S. Cuyegkeng Member
legal holiday pay, overtime pay and thirteenth-month pay premises; Elisa C. Lim Member
against petitioner. Teodulo R. Dino Member
3. Security guards with high-powered guns prevented him Virgilio O. Casino Member
On December 19, 1984, the Labor Arbiter rendered from removing the properties he had levied upon.4
judgment1 ordering petitioner to reinstate private 3. Corporate Officers
respondents and to pay them back wages equivalent to one The said special sheriff recommended that a break-open
year or three hundred working days. order be issued to enable him to enter petitioners premises Antonio W. Lim President
so that he could proceed with the public auction sale of the Dennis S. Cuyegkeng Assistant to the
On November 27, 1985, the National Labor Relations aforesaid personal properties on November 7, 1989. President
Commission (NLRC) dismissed the motion for Elisa 0. Lim Treasurer
reconsideration filed by petitioner on the ground that the On November 6, 1989, a certain Dennis Cuyegkeng filed a Virgilio O. Casino Corporate Secretary
said decision had already become final and executory.2 third-party claim with the Labor Arbiter alleging that the
properties sought to be levied upon by the sheriff were 4. Principal Office
On October 16, 1986, the NLRC Research and Information owned by Hydro (Phils.), Inc. (HPPI) of which he is the Vice-
Department made the finding that private respondents President. 355 Maysan Road, Valenzuela, Metro Manila.5
backwages amounted to P199,800.00.3
On November 23, 1989, private respondents filed a Motion On the other hand, the General Information Sheet of
On October 29, 1986, the Labor Arbiter issued a writ of for Issuance of a Break-Open Order, alleging that HPPI and HPPI revealed the following:
execution directing the sheriff to execute the Decision, petitioner corporation were owned by the same
dated December 19, 1984. The writ was partially satisfied incorporator! stockholders. They also alleged that 1. Breakdown of Subscribed Capital
through garnishment of sums from petitioners debtor, the petitioner temporarily suspended its business operations in
Metropolitan Waterworks and Sewerage Authority, in the order to evade its legal obligations to them and that Name of Stockholder Amount Subscribed
amount of P81,385.34. Said amount was turned over to the private respondents were willing to post an indemnity bond
cashier of the NLRC. to answer for any damages which petitioner and HPPI may Antonio W. Lim P400,000.00
suffer because of the issuance of the break-open order. Elisa C. Lim 57,700.00
On February 1, 1989, an Alias Writ of Execution was issued AWL Trading 455,000.00
by the Labor Arbiter directing the sheriff to collect from In support of their claim against HPPI, private respondents Dennis S. Cuyegkeng 40,100.00
herein petitioner the sum of P117,414.76, representing the presented duly certified copies of the General Informations Teodulo R. Dino 100.00
balance of the judgment award, and to reinstate private Sheet, dated May 15, 1987, submitted by petitioner to the Virgilio O. Casino 100.00
respondents to their former positions. Securities and Exchange Commission (SEC) and the General
Information Sheet, dated May 15, 1987, submitted by HPPI 2. Board of Directors
On July 13, 1989, the sheriff issued a report stating that he to the Securities and Exchange Commission.
tried to serve the alias writ of execution on petitioner Antonio W. Lim Chairman
through the security guard on duty but the service was The General Information Sheet submitted by the Elisa C. Lim Member
refused on the ground that petitioner no longer occupied petitioner1 revealed the following: Dennis S. Cuyegkeng Member
the premises. Virgilio O. Casino Member
1. Breakdown of Subscribed Capital Teodulo R. Dino Member
On September 26, 1986, upon motion of private
respondents, the Labor Arbiter issued a second alias writ of Name of Stockholder Amount Subscribed 3. Corporate Officers
execution.
HPPI P6,999,500.00 Antonio W. Lim President

Page 99 of 160
CORPORATION LAW CASES MAYORDO, M.A
LLB 3

Dennis S. Cuyegkeng Assistant to the stockholders and from other corporations to which it may 1. Control, not mere majority or complete stock control,
President be connected.8 But, this separate and distinct personality but complete domination, not only of finances but of policy
Elisa O. Lim Treasurer of a corporation is merely a fiction created by law for and business practice in respect to the transaction attacked
Virgilio O. Casino Corporate Secretary convenience and to promote justice.9 So, when the notion so that the corporate entity as to this transaction had at
of separate juridical personality is used to defeat public the time no separate mind, will or existence of its own;
4. Principal Office convenience, justify wrong, protect fraud or defend crime,
or is used as a device to defeat the labor laws,10 this 2. Such control must have been used by the defendant to
355 Maysan Road, Valenzuela, Metro Manila.6 separate personality of the corporation may be disregarded commit fraud or wrong, to perpetuate the violation of a
or the veil of corporate fiction pierced.11 This is true statutory or other positive legal duty, or dishonest and
On February 1, 1990, HPPI filed an Opposition to private likewise when the corporation is merely an adjunct, a unjust act in contravention of plaintiffs legal rights; and
respondents motion for issuance of a break-open order, business conduit or an alter ego of another corporation.12
contending that HPPI is a corporation which is separate and 3. The aforesaid control and breach of duty must
distinct from petitioner. HPPI also alleged that the two The conditions under which the juridical entity may be proximately cause the injury or unjust loss complained of.
corporations are engaged in two different kinds of disregarded vary according to the peculiar facts and
businesses, i.e., HPPI is a manufacturing firm while circumstances of each case. No hard and fast rule can be The absence of any one of these elements prevents piercing
petitioner was then engaged in construction. accurately laid down, but certainly, there are some the corporate veil. in applying the instrumentality or alter
probative factors of identity that will justify the ego doctrine, the courts are concerned with reality and not
On March 2, 1990, the Labor Arbiter issued an Order which application of the doctrine of piercing the corporate veil, form, with how the corporation operated and the individual
denied private respondents motion for break-open order. to wit: defendants relationship to that operation. 14

Private respondents then appealed to the NLRC. On April 1. Stock ownership by one or common ownership of both Thus, the question of whether a corporation is a mere alter
23, 1992, the NLRC set aside the order of the Labor Arbiter, corporations. ego, a mere sheet or paper corporation, a sham or a
issued a break-open order and directed private respondents subterfuge is purely one of fact.15
to file a bond. Thereafter, it directed the sheriff to proceed 2. Identity of directors and officers.
with the auction sale of the properties already levied upon. In this case, the NLRC noted that, while petitioner claimed
It dismissed the third-party claim for lack of merit. 3. The manner of keeping corporate books and records. that it ceased its business operations on April 29, 1986, it
filed an Information Sheet with the Securities and Exchange
Petitioner moved for reconsideration but the motion was 4. Methods of conducting the business.13 Commission on May 15, 1987, stating that its office address
denied by the NLRC in a Resolution, dated December 3, is at 355 Maysan Road, Valenzuela, Metro Manila. On the
1992. The SEC en banc explained the instrumentality rule which other hand, HPPI, the third-party claimant, submitted on
the courts have applied in disregarding the separate the same day, a similar information sheet stating that its
Hence, the resort to the present petition. juridical personality of corporations as follows: office address is at 355 Maysan Road, Valenzuela, Metro
Manila.
Petitioner alleges that the NLRC committed grave abuse of Where one corporation is so organized and controlled and
discretion when it ordered the execution of its decision its affairs are conducted so that it is, in fact, a mere Furthermore, the NLRC stated that:
despite a third-party claim on the levied property. instrumentality or adjunct of the other, the fiction of the
Petitioner further contends, that the doctrine of piercing corporate entity of the instrumentality may be disregarded. Both information sheets were filed by the same Virgilio O.
the corporate veil should not have been applied, in this The control necessary to invoke the rule is not majority or Casino as the corporate secretary of both corporations. It
case, in the absence of any showing that it created HPPI in even complete stock control but such domination of would also not be amiss to note that both corporations had
order to evade its liability to private respondents. It also finances, policies and practices that the controlled the same president, the same board of directors, the same
contends that HPPI is engaged in the manufacture and sale corporation has, so to speak, no separate mind, will or corporate officers, and substantially the same subscribers.
of steel, concrete and iron pipes, a business which is existence of its own, and is but a conduit for its principal.
distinct and separate from petitioners construction It must be kept in mind that the control must be shown to From the foregoing, it appears that, among other things,
business. Hence, it is of no consequence that petitioner and have been exercised at the time the acts complained of the respondent (herein petitioner) and the third-party
HPPI shared the same premises, the same President and the took place. Moreover, the control and breach of duty must claimant shared the same address and/or premises. Under
same set of officers and subscribers.7 proximately cause the injury or unjust loss for which the this circumstances, (sic) it cannot be said that the property
complaint is made. levied upon by the sheriff were not of respondents.16
We find petitioners contention to be unmeritorious.
The test in determining the applicability of the doctrine of Clearly, petitioner ceased its business operations in order to
It is a fundamental principle of corporation law that a piercing the veil of corporate fiction is as follows: evade the payment to private respondents of backwages
corporation is an entity separate and distinct from its and to bar their reinstatement to their former positions.

Page 100 of 160


CORPORATION LAW CASES MAYORDO, M.A
LLB 3

HPPI is obviously a business conduit of petitioner Hence, the NLRC did not commit any grave abuse of up to their actual reinstatement" (p. 12, Decision; p. 27,
corporation and its emergence was skillfully orchestrated discretion when it affirmed the break-open order issued by rec.).
to avoid the financial liability that already attached to the Labor Arbiter.
petitioner corporation. A motion to reconsider the above decision was filed by
Finally, we do not find any reason to disturb the rule that herein petitioners, which respondent Court, sitting en
The facts in this case are analogous to Claparols v. Court of factual findings of quasi-judicial agencies supported by banc, denied in a resolution dated January 27, 1964.
Industrial Relations17 where we had the occasion to rule: substantial evidence are binding on this Court and are
entitled to great respect, in the absence of showing of On March 30, 1964, counsel for herein respondent workers
Respondent courts findings that indeed the Claparols Steel grave abuse of a discretion.18 (complainants in the ULP case) filed a motion for execution
and Nail Plant, which ceased operation of June 30, 1957, of respondent Court's September 16, 1963 decision.
was SUCCEEDED by the Claparols Steel Corporation WHEREFORE, the petition is DISMISSED and the assailed
effective the next day, July 1, 1957, up to December 7, resolutions of the NLRC, dated April 23, 1992 and December On May 14, 1964, respondent Court, in its order of
1962, when the latter finally ceased to operate, were not 3, 1992, are AFFIRMED. September 16, 1963, granted execution and directed herein
disputed by petitioner. it is very clear that the latter petitioners
corporation was a continuation and successor of the first SO ORDERED.
entity x xx. Both predecessors and successor were owned to reinstate the above complainants to their former or
and controlled by petitioner Eduardo Claparols and there G.R. No. L-30822 July 31, 1975 equivalent jobs within five (5) days after receipt of a copy
was no break in the succession and continuity of the same EDUARDO CLAPAROLS, ROMULO AGSAM and/or of this order. In order to implement the award of back
business. This avoiding-the-liability scheme is very patent, CLAPAROLS STEEL AND NAIL PLANT, petitioners, vs. wages, the Chief of the Examining Division or any of his
considering that 90% of the subscribed shares of stock of COURT OF INDUSTRIAL RELATIONS, ALLIED WORKERS' assistants is hereby directed to proceed to the office of the
the Claparols Steel Corporation (the second corporation) ASSOCIATION and/or DEMETRIO GARLITOS, ALFREDO respondents at Matab-ang, Talisay, Negros Occidental, and
was owned by respondent x xxClaparols himself, and all the ONGSUCO, JORGE SEMILLANO, SALVADOR DOROTEO, examine its payrolls and other pertinent records and
assets of the dissolved Claparols Steel and Nail Plant were ROSENDO ESPINOSA, LUDOVICO BALOPENOS, ASER compute the back wages of the complainants in accordance
turned over to the emerging Claparols Steel Corporation. AMANCIO, MAXIMO QUIOYO, GAUDENCIO QUIOYO, and with the decision dated September 16, 1963, and, upon
IGNACIO QUIOYO, respondents. termination, to submit his report as soon as possible for
It is very obvious that the second corporation seeks the further disposition (p. 7, Brief for Respondents, p. 113,
protective shield of a corporate fiction whose veil in the MAKASIAR, J.: rec.).
present case could, and should, be pierced as it was
deliberately and maliciously designed to evade its financial A petition for certiorari to set aside the order of which was reiterated by respondent Court in a subsequent
obligation to its employees. respondent Court of Industrial Relations dated May 30, 1969 order dated November 10, 1964 (pp. 7-8, Brief for
directing petitioners to pay back wages and bonuses to Respondents, p. 113, rec.).
In view of the failure of the sheriff, in the case at bar, to private respondents as well as its resolution of July 5, 1969
effect a levy upon the property subject of the execution, denying the motion for reconsideration of said order in On December 14, 1964, respondent workers were
private respondents had no other recourse but to apply for Case No. 32-ULP-Iloilo entitled "Allied Workers' Association, accompanied by the Chief of Police of Talisay, Negros
a break-open order after the third-party claim of HPPI was et. al., versus Eduardo Claparols, et. al.." Occidental to the compound of herein petitioner company
dismissed for lack of merit by the NLRC. This is in to report for reinstatement per order of the court.
consonance with Section 3, Rule VII of the NLRC Manual of It appears that on August 6, 1957, a complaint for unfair Respondent workers were, however, refused reinstatement
Execution of Judgment which provides that: labor practice was filed by herein private respondent Allied by company accountant Francisco Cusi for he had no order
Workers' Association, respondent Demetrio Garlitos and ten from plant owner Eduardo Claparols nor from his lawyer
Should the losing party, his agent or representative, refuse (10) respondent workers against herein petitioners on Atty. PlaridelKatalbas, to reinstate respondent workers.
or prohibit the Sheriff or his representative entry to the account of the dismissal of respondent workers from
place where the property subject of execution is located or petitioner Claparols Steel and Nail Plant. Again, on December 15, 1964, respondent workers were
kept, the judgment creditor may apply to the Commission accompanied by a police officer to the company compound,
or Labor Arbiter concerned for a break-open order. On September 16, 1963, respondent Court rendered its but then, they were again refused reinstatement by Cusi on
decision finding "Mr. Claparols guilty of union busting and" the same ground.
Furthermore, our perusal of the records shows that the of having "dismissed said complainants because of their
twin requirements of due notice and hearing were complied union activities," and ordering respondents "(1) To cease On January 15, 1965, the CIR Chief Examiner Submitted his
with. Petitioner and the third-party claimant were given and desist from committing unfair labor practices against report containing three computations, to wit:
the opportunity to submit evidence in support of their their employees and laborers; (2) To reinstate said
claim. complainants to their former or equivalent jobs, as soon as The first computation covers the period February 1, 1957 to
possible, with back wages from the date of their dismissal October 31, 1964. The second is up to and including

Page 101 of 160


CORPORATION LAW CASES MAYORDO, M.A
LLB 3

December 7, 1962, when the corporation stopped same end was reiterated by private respondents on June
operations, while the third is only up to June 30, 1957 On December 7, 1966, a motion for reconsideration was 14, 1967.
when the Claparols Steel and Nail Plant ceased to operate filed by petitioner, assailing respondent Court's ruling that
(Annex B, Petition for Review on Certiorari, p. 14, Brief for (1) the ruling in the case of Sta. Cecilia Sawmills Inc. CIR, On July 13, 1967, respondent Court directed a
appellees, p. 113, rec.) with the explanation that: et. al, does not apply in the case at bar; and (2) that bonus recomputation of the back wages of respondent workers in
should be included in the recoverable wages. accordance with its order dated November 28, 1966. The
6. Since the records of the Claparols Steel said order in part reads:
Corporation show that it was established on July 1, 1957 On December 14, 1966, a counter-opposition was filed by
succeeding the Claparols Steel and Nail Plant which ceased private respondents alleging that petitioners' motion for WHEREFORE, the Chief Auditing Examiner of the Court or
operations on June 30, 1957, and that the Claparols Steel reconsideration was pro forma, it not making express any of his assistants, is hereby directed to recompute the
Corporation stopped operations on December 7, 1962, reference to the testimony or documentary evidence or to back wages of the workers involved in this case in
three (3) computations are presented herein for the the provision of law alleged to be contrary to such findings accordance with the Order of November 28, 1966 within 20
consideration of this Honorable Court (p. 2, Report of or conclusions of respondent Court. days from receipt of a copy of this Order (p. 28, Brief for
Examiner, p. 29, rec.). Respondents, p. 113, rec.).
On February 8, 1967, respondent Court of Industrial
On January 23, 1965, petitioners filed an opposition Relations dismissed petitioners' motion for reconsideration Then on March 21, 1968, the Chief Examiner came out with
alleging that under the circumstances presently engulfing for being pro forma. his report, the disputed portion of which (regarding
the company, petitioner Claparols could not personally bonuses) reads:
reinstate respondent workers; that assuming the workers Whereupon, petitioners filed a petition for certiorari with
are entitled to back wages, the same should only be limited this COURT in G.R. No. L-27272 to set aside the November xxx xxx xxx
to three months pursuant to the court ruling in the case of 28, 1966 order of respondent Court, as well as its February
Sta. Cecilia Sawmills vs. CIR (L-19273-74, February 20, 8, 1967 resolution. Petitioners assigned therein as errors of 4. The yearly bonuses of the employees and laborers
1964); and that since Claparols Steel Corporation ceased to law the very same assignment of errors it raises in the of respondent corporation are given on the following basis:
operate on December 7, 1962, re-employment of present case, to wit:
respondent workers cannot go beyond December 7, 1962. Basic Additional:
I
A reply to petitioner's opposition was filed by respondent a. For every dependent 1% of
workers, alleging among others, that Claparols Steel and THE RESPONDENT COURT ERRED AND/OR ACTED WITH monthly salary
Nail Plant and Claparols Steel and Nail Corporation are one GRAVE ABUSE OF DISCRETION, AMOUNTING TO LACK OF
b. For every dependent in elementary grade 2% of
and the same corporation controlled by petitioner JURISDICTION, IN HOLDING IN THE ORDER UNDER REVIEW
monthly salary
Claparols, with the latter corporation succeeding the THAT BONUSES SHOULD BE PAID TO THE RESPONDENT
former. WORKERS DESPITE THE FACT THAT THE SAME WAS NOT c. For every dependent in high school 3% of monthly salary
ADJUDICATED IN ITS ORIGINAL DECISION.
On November 28, 1966, after conducting a series of d. For every dependent in college 5% of
hearings on the report of the examiner, respondent Court II monthly salary
issued an order, the dispositive portion of which reads:
THE RESPONDENT COURT ERRED AND/OR ACTED WITH xxx xxx xxx
WHEREFORE, the Report of the. Examiner filed on January GRAVE ABUSE OF DISCRETION, AMOUNTING TO LACK OF
15, 1965, is hereby approved subject to the foregoing JURISDICTION, IN NOT APPLYING THE DOCTRINE LAID DOWN 7. The computed ... bonuses after deducting the
findings and dispositions. Consequently, the Corporation BY THIS HONORABLE TRIBUNAL IN THE CASE OF "STA. earnings elsewhere of Messrs. Ongsuco, Garlitos and
Auditing Examiner is directed to recompute the back wages CECILIA SAWMILLS, INC. VS. C.I.R., ET. AL.," G.R. No. Semillano are as follows:
of complainants Demetrio Garlitos and Alfredo Ongsuco on L-19273-74, PROMULGATED ON FEBRUARY 29, 1964 (pp.
the basis of P200.00 and P270.00 a month, respectively; to 10-11, rec.). Name x xx Bonuses x xx
compute those of complainant Ignacio Quioyo as aforesaid;
to compute the deductible earnings of complainants On April 27, 1967, the Supreme Court denied petitioners' 1. Alfredo Ongsuco P1,620.00
Ongsuco, Jorge Semillano and Garlitos, as found in the body petition for certiorari (p. 77, rec. of L-27272), which was 2. Demetrio Garlitos 1,200.00
of this order; and to compute the bonuses of each and reiterated on May 19, 1967 (p. 27, Respondent's Brief, p. 3. Ignacio Quioyo 455.23
every complainant, except Honorato Quioyo. Thereafter, as 113, rec.; p. 81, rec. of L-27272). 4. AserAbancio 461.00
soon as possible, the Examiner should submit a report in 5. Ludovico Belopeños 752.05
compliance herewith of the Court's further disposition (p. On May 3, 1967, private respondents moved to have the 6. Salvador Doroteo 714.70
24, Brief for Respondents, p. 113, rec.). workers' back wages properly recomputed. A motion to the 7. Rosendo Espinosa 1,075.40

Page 102 of 160


CORPORATION LAW CASES MAYORDO, M.A
LLB 3

8. Gaudencio Quioyo 1,167.92 On June 7, 1969, petitioners filed a motion for Phil. 381; Ansay, et. al. vs. National Development Co., et.
9. Jorge Semillano 1,212.08 reconsideration on practically the same grounds previously al., 107 Phil. 998, 999; Emphasis supplied).
10. Maximo Quioyo 449.41 raised by them.
Total P9,107.79 In Atok Big Wedge Mining Co. vs. Atok Big Wedge Mutual
On June 30, 1969, respondents filed an opposition to Benefit Association (92 Phil. 754), this Court, thru Justice
(Pp. 30-31, Respondent's Brief, p. 113, rec.) petitioners' motion for reconsideration, with the following Labrador, held:
allegations:
On April 16, 1968, petitioners filed their opposition to the Whether or not bonus forms part of wages depends upon
report of the Examiner dated March 21, 1968 on grounds 1. The issues raised, namely, whether bonuses the condition or circumstance for its payment. If it is an
already rejected by respondent Court in its order dated should be included in the award for back wages had already additional compensation WHICH THE EMPLOYER PROMISED
November 28, 1966, and by the Supreme Court also in its been resolved by respondent court in its orders dated AND AGREED to give without any condition imposed for its
ruling in G.R. No. L-27272. November 28, 1966, and December 7, 1966, and in the payment ... then it is part of the wage. (Emphasis
Resolution of the Honorable Supreme Court in G.R. No. supplied).1äwphï1.ñët
On May 4, 1968, a rejoinder to petitioners' opposition was L-27272 dated April 26, 1967 and May 19, 1967, and the
filed by private respondents, alleging among others "that same is already a settled and final issue. In Altomonte vs. Philippine American Drug Co. (106 Phil.
the grounds of petitioners' opposition were the same 137), the Supreme Court held that an employee is not
grounds raised by them before and passed upon by 2. Petitioners' motion for reconsideration is merely a entitled to bonus where there is no showing that it had
respondent Court and this Honorable Tribunal; that this rehash of previous arguments, effete and unrejuvenated, been granted by the employer to its employees periodically
order of November 28, 1966 which passed upon these issues pro forma, and intended merely to delay the proceedings. or regularly as to become part of their wages or salaries.
became final and executory on June 3, 1967 from the The clear implication is that bonus is recoverable as part of
Honorable Supreme Court. (Order of respondent Court As correctly contended by private respondents, the present the wage or salary where the employer regularly or
dated July 13, 1967). [p. 32, Brief for Respondents, p. 113, petition is barred by Our resolutions of April 26, 1967 and periodically gives it to employees.
rec.]. May 19, 1967 in G.R. No. L-27272 (Eduardo Claparols, et.
al. vs. CIR, et. al.) [pp. 77-83, rec. of L- 27272], dismissing American jurisprudence equally regards bonuses as part of
On July 26, 1968, private respondents filed their motion for said case, wherein said petitioners invoked the applicability compensation or recoverable wages.
approval of the Report of the Examiner submitted on March of the doctrine in Sta. Cecilia Sawmills, Inc. vs. CIR, et. al.
21, 1968, alleging, among others, that petitioners, in their (L-19273-74, Feb. 29, 1964, 10 SCRA 433) and impugned the Thus, it was held that "... it follows that in determining the
opposition, did not actually dispute the data elicited by the illegality of the order of respondent Court dated November regular rate of pay, a bonus which in fact constitutes PART
Chief Examiner but rather harped on grounds which, as 28, 1966 directing the computation and payment of the OF AN EMPLOYEE'S compensation, rather than a true gift or
already stated, had already been turned down by the bonuses, aside from back wages on the ground that these gratuity, has to be taken into consideration." (48 Am. Jur.
Supreme Court. bonuses were not included in the decision of September 16, 2d, Labor and Labor Relations, No. 1555, citing the cases of
1963, which had long become final. Triple "AAA" Co. vs. Wirtz and Haber vs. Americana
On October 19, 1968, herein private respondents filed their Corporation; Emphasis supplied). It was further held that
"Constancia", submitting the case for resolution of The aforesaid resolutions in G.R. No. L-27272 constitute the "... the regular rate includes incentive bonuses paid to the
respondent Court of Industrial Relations. law of the instant case, wherein herein petitioners raised employees in addition to the guaranteed base rates
again practically the same issues invoked in the regardless of any contract provision to the contrary and
On May 30, 1969, respondent Court issued an order, subject abovementioned case. The denial of the petition in G.R. even though such bonuses could not be determined or paid
of the present appeal, the dispositive portion of which No. L-27272 suffices to warrant the denial of the present until such time after the pay day" (48 Am. Jur. 2d, Labor
reads: petition; and We need not go any further. and Labor Relations, No. 1555, citing the case of Walling vs.
Harnischfeger Corp., 325 US 427, 89 L Ed 1711, 65 S Ct.
WHEREFORE, there being no proof offered to substantiate However, without lending a sympathetic ear to the obvious 1246; Emphasis supplied).1äwphï1.ñët
respondent Eduardo Claparols' opposition, the Examiner's desire of herein petitioners of this Court to re-examine —
Report should be, and it is hereby, APPROVED. which would be an exercise in futility — the final ruling in Petitioners in the present case do not dispute that as a
Consequently, pursuant to the decision dated September G.R. No. L-27272, which as above-stated is the law of the matter of tradition, the company has been doling out
16, 1963, respondent ... (petitioners herein) are hereby instant case, but solely to remind herein petitioners, We bonuses to employees. In fact, the company balance sheets
directed to pay the respective back wages and bonuses of reiterate the governing principles. for the years 1956 to 1962 contained bonus and pension
the complainants (respondents herein) ... (p. 35, Brief for computations which were never repudiated or questioned
Respondents; p. 113, rec.; emphasis supplied).1äwphï1.ñët WE uniformly held that "a bonus is not a demandable and by petitioners. As such, bonus for a given year earmarked
enforceable obligation, except when it is a part of the as a matter of tradition for distribution to employees has
wage or salary compensation" (Philippine Education Co. vs. formed part of their recoverable wages from the company.
CIR and the Union of Philippine Co. Employees [NLU], 92 Moreover, with greater reason, should recovery of bonuses

Page 103 of 160


CORPORATION LAW CASES MAYORDO, M.A
LLB 3

as part of back wages be observed in the present case since deliberately and maliciously designed to evade its financial
the company, in the light of the very admission of company obligation to its employees. This is a tri-party appeal from the decision of the Court of
accountant Francisco Cusi, distributes bonuses to its First Instance of Manila, Civil Case No. 41845, declaring null
employees even if the company has suffered losses. It is well remembering that in Yutivo& Sons Hardware and void the sheriff's sale of two certificates of public
Specifically, petitioner company has done this in 1962 Company vs. Court of Tax Appeals (L-13203, Jan. 28, 1961, convenience in favor of defendant Eusebio E. Ferrer and
(t.s.n., p. 149, Sept. 20, 1965). 1 SCRA 160), We held that when the notion of legal entity is the subsequent sale thereof by the latter to defendant
used to defeat public convenience, justify wrong, protect Pangasinan Transportation Co., Inc.; declaring the plaintiff
Since bonuses are part of back wages of private fraud, or defend crime, the law will regard the corporation Villa Rey Transit, Inc., to be the lawful owner of the said
respondents, the order of May 30, 1969, directing the as an association or persons, or, in the case of two certificates of public convenience; and ordering the private
payment of their bonuses, did not amend the decision of corporations, will merge them into one. defendants, jointly and severally, to pay to the plaintiff,
September 16, 1963 of respondent Court directing payment the sum of P5,000.00 as and for attorney's fees. The case
of their wages, which has long become final and executory, In Liddel& Company, Inc. vs. Collector of Internal Revenue against the PSC was dismissed.
in the same way that the previous order of May 14, 1964 (L-9687, June 30, 1961, 2 SCRA 632), this Court likewise
granting execution of said decision of September 16, 1963 held that where a corporation is a dummy and serves no The rather ramified circumstances of the instant case can
also directed the computation of the wages to be paid to business purpose and is intended only as a blind, the best be understood by a chronological narration of the
private respondents as decreed by the decision of corporate fiction may be ignored. essential facts, to wit:
September 16, 1963. All the orders of May 30, 1969,
November 28, 1966 and May 14, 1964 merely implement the In Commissioner of Internal Revenue vs. Norton and Prior to 1959, Jose M. Villarama was an operator of a bus
already final and executory decision of September 16, Harrison Company (L-17618, Aug. 31, 1964, 11 SCRA 714), transportation, under the business name of Villa Rey
1963. We ruled that where a corporation is merely an adjunct, Transit, pursuant to certificates of public convenience
business conduit or alter ego of another corporation, the granted him by the Public Service Commission (PSC, for
Petitioners insist that We adopt the ruling in the Sta. fiction of separate and distinct corporate entities should be short) in Cases Nos. 44213 and 104651, which authorized
Cecilia Sawmills case wherein the recoverable back wages disregarded. him to operate a total of thirty-two (32) units on various
were limited to only three (3) months; because as in the routes or lines from Pangasinan to Manila, and vice-versa.
Sta. Cecilia Sawmills case, the Claparols Steel and Nail To the same uniform effect are the decisions in the cases of On January 8, 1959, he sold the aforementioned two
Plant ceased operations due to enormous business reverses. Republic vs. Razon (L-17462, May 29, 1967, 20 SCRA 234) certificates of public convenience to the Pangasinan
and A.D. Santos, Inc. vs. Vasquez (L-23586, March 20, 1968, Transportation Company, Inc. (otherwise known as
Respondent Court's findings that indeed the Claparols Steel 22 SCRA 1156). Pantranco), for P350,000.00 with the condition, among
and Nail Plant, which ceased operation of June 30, 1957, others, that the seller (Villarama) "shall not for a period of
was SUCCEEDED by the Claparols Steel Corporation WE agree with respondent Court of Industrial Relations, 10 years from the date of this sale, apply for any TPU
effective the next day, July 1, 1957 up to December 7, therefore, that the amount of back wages recoverable by service identical or competing with the buyer."
1962, when the latter finally ceased to operate, were not respondent workers from petitioners should be the amount
disputed by petitioners. It is very clear that the latter accruing up to December 7, 1962 when the Claparols Steel Barely three months thereafter, or on March 6, 1959: a
corporation was a continuation and successor of the first Corporation ceased operations. corporation called Villa Rey Transit, Inc. (which shall be
entity, and its emergence was skillfully timed to avoid the referred to hereafter as the Corporation) was organized
financial liability that already attached to its predecessor, WHEREFORE, PETITION IS HEREBY DENIED WITH TREBLE with a capital stock of P500,000.00 divided into 5,000
the Claparols Steel and Nail Plant. Both predecessors and COSTS AGAINST PETITIONERS TO BE PAID BY THEIR shares of the par value of P100.00 each; P200,000.00 was
successor were owned and controlled by the petitioner COUNSEL. the subscribed stock; Natividad R. Villarama (wife of Jose
Eduardo Claparols and there was no break in the succession M. Villarama) was one of the incorporators, and she
and continuity of the same business. This "avoiding-the- G.R. No. L-23893 October 29, 1968 subscribed for P1,000.00; the balance of P199,000.00 was
liability" scheme is very patent, considering that 90% of the VILLA REY TRANSIT, INC., plaintiff-appellant, vs. EUSEBIO subscribed by the brother and sister-in-law of Jose M.
subscribed shares of stocks of the Claparols Steel E. FERRER, PANGASINAN TRANSPORTATION CO., INC. and Villarama; of the subscribed capital stock, P105,000.00 was
Corporation (the second corporation) was owned by PUBLIC SERVICE COMMISSION, defendants. paid to the treasurer of the corporation, who was Natividad
respondent (herein petitioner) Claparols himself, and all EUSEBIO E. FERRER and PANGASINAN TRANSPORTATION R. Villarama.
the assets of the dissolved Claparols Steel and Nail Plant CO., INC., defendants-appellants.
were turned over to the emerging Claparols Steel In less than a month after its registration with the
Corporation. PANGASINAN TRANSPORTATION CO., INC., third-party Securities and Exchange Commission (March 10, 1959), the
plaintiff-appellant, vs. JOSE M. VILLARAMA, third-party Corporation, on April 7, 1959, bought five certificates of
It is very obvious that the second corporation seeks the defendant-appellee. public convenience, forty-nine buses, tools and equipment
protective shield of a corporate fiction whose veil in the from one Valentin Fernando, for the sum of P249,000.00, of
present case could, and should, be pierced as it was ANGELES, J.: which P100,000.00 was paid upon the signing of the

Page 104 of 160


CORPORATION LAW CASES MAYORDO, M.A
LLB 3

contract; P50,000.00 was payable upon the final approval Corporation should be the one to operate the lines Ferrer, for his part, challenges the decision insofar as it
of the sale by the PSC; P49,500.00 one year after the final provisionally. holds that the sheriff's sale is null and void; and the sale of
approval of the sale; and the balance of P50,000.00 "shall the two certificates in question by Valentin Fernando to the
be paid by the BUYER to the different suppliers of the On November 4, 1959, the Corporation filed in the Court of Corporation, is valid. He also assails the award of P5,000.00
SELLER." First Instance of Manila, a complaint for the annulment of as attorney's fees in favor of the Corporation, and the
the sheriff's sale of the aforesaid two certificates of public failure to award moral damages to him as prayed for in his
The very same day that the aforementioned contract of convenience (PSC Cases Nos. 59494 and 63780) in favor of counterclaim.
sale was executed, the parties thereto immediately applied the defendant Ferrer, and the subsequent sale thereof by
with the PSC for its approval, with a prayer for the issuance the latter to Pantranco, against Ferrer, Pantranco and the The Corporation, on the other hand, prays for a review of
of a provisional authority in favor of the vendee PSC. The plaintiff Corporation prayed therein that all the that portion of the decision awarding only P5,000.00 as
Corporation to operate the service therein involved.1 On orders of the PSC relative to the parties' dispute over the attorney's fees, and insisting that it is entitled to an award
May 19, 1959, the PSC granted the provisional permit said certificates be annulled. of P100,000.00 by way of exemplary damages.
prayed for, upon the condition that "it may be modified or
revoked by the Commission at any time, shall be subject to In separate answers, the defendants Ferrer and Pantranco After a careful study of the facts obtaining in the case, the
whatever action that may be taken on the basic application averred that the plaintiff Corporation had no valid title to vital issues to be resolved are: (1) Does the stipulation
and shall be valid only during the pendency of said the certificates in question because the contract pursuant between Villarama and Pantranco, as contained in the deed
application." Before the PSC could take final action on said to which it acquired them from Fernando was subject to a of sale, that the former "SHALL NOT FOR A PERIOD OF 10
application for approval of sale, however, the Sheriff of suspensive condition — the approval of the PSC — which has YEARS FROM THE DATE OF THIS SALE, APPLY FOR ANY TPU
Manila, on July 7, 1959, levied on two of the five not yet been fulfilled, and, therefore, the Sheriff's levy and SERVICE IDENTICAL OR COMPETING WITH THE BUYER," apply
certificates of public convenience involved therein, namely, the consequent sale at public auction of the certificates to new lines only or does it include existing lines?; (2)
those issued under PSC cases Nos. 59494 and 63780, referred to, as well as the sale of the same by Ferrer to Assuming that said stipulation covers all kinds of lines, is
pursuant to a writ of execution issued by the Court of First Pantranco, were valid and regular, and vested unto such stipulation valid and enforceable?; (3) In the
Instance of Pangasinan in Civil Case No. 13798, in favor of Pantranco, a superior right thereto. affirmative, that said stipulation is valid, did it bind the
Eusebio Ferrer, plaintiff, judgment creditor, against Corporation?
Valentin Fernando, defendant, judgment debtor. The Pantranco, on its part, filed a third-party complaint against
Sheriff made and entered the levy in the records of the Jose M. Villarama, alleging that Villarama and the For convenience, We propose to discuss the foregoing issues
PSC. On July 16, 1959, a public sale was conducted by the Corporation, are one and the same; that Villarama and/or by starting with the last proposition.
Sheriff of the said two certificates of public convenience. the Corporation was disqualified from operating the two
Ferrer was the highest bidder, and a certificate of sale was certificates in question by virtue of the aforementioned The evidence has disclosed that Villarama, albeit was not
issued in his name. agreement between said Villarama and Pantranco, which an incorporator or stockholder of the Corporation, alleging
stipulated that Villarama "shall not for a period of 10 years that he did not become such, because he did not have
Thereafter, Ferrer sold the two certificates of public from the date of this sale, apply for any TPU service sufficient funds to invest, his wife, however, was an
convenience to Pantranco, and jointly submitted for identical or competing with the buyer." incorporator with the least subscribed number of shares,
approval their corresponding contract of sale to the PSC.2 and was elected treasurer of the Corporation. The finances
Pantranco therein prayed that it be authorized provisionally Upon the joinder of the issues in both the complaint and of the Corporation which, under all concepts in the law, are
to operate the service involved in the said two certificates. third-party complaint, the case was tried, and thereafter supposed to be under the control and administration of the
decision was rendered in the terms, as above stated. treasurer keeping them as trust fund for the Corporation,
The applications for approval of sale, filed before the PSC, were, nonetheless, manipulated and disbursed as if they
by Fernando and the Corporation, Case No. 124057, and As stated at the beginning, all the parties involved have were the private funds of Villarama, in such a way and
that of Ferrer and Pantranco, Case No. 126278, were appealed from the decision. They submitted a joint record extent that Villarama appeared to be the actual owner-
scheduled for a joint hearing. In the meantime, to wit, on on appeal. treasurer of the business without regard to the rights of the
July 22, 1959, the PSC issued an order disposing that during stockholders. The following testimony of Villarama,4
the pendency of the cases and before a final resolution on Pantranco disputes the correctness of the decision insofar together with the other evidence on record, attests to that
the aforesaid applications, the Pantranco shall be the one as it holds that Villa Rey Transit, Inc. (Corporation) is a effect:
to operate provisionally the service under the two distinct and separate entity from Jose M. Villarama; that
certificates embraced in the contract between Ferrer and the restriction clause in the contract of January 8, 1959 Q. Doctor, I want to go back again to the incorporation
Pantranco. The Corporation took issue with this particular between Pantranco and Villarama is null and void; that the of the Villa Rey Transit, Inc. You heard the testimony
ruling of the PSC and elevated the matter to the Supreme Sheriff's sale of July 16, 1959, is likewise null and void; and presented here by the bank regarding the initial opening
Court,3 which decreed, after deliberation, that until the the failure to award damages in its favor and against deposit of ONE HUNDRED FIVE THOUSAND PESOS, of which
issue on the ownership of the disputed certificates shall Villarama. amount Eighty-Five Thousand Pesos was a check drawn by
have been finally settled by the proper court, the yourself personally. In the direct examination you told the

Page 105 of 160


CORPORATION LAW CASES MAYORDO, M.A
LLB 3

Court that the reason you drew a check for Eighty-Five Q. The subscription of your brother-in-law, Mr. Reyes, is
Thousand Pesos was because you and your wife, or your Fifty-Two Thousand Pesos, did your wife give you Fifty-two Exhibits 6 to 19 and Exh. 22, which are photostatic copies
wife, had spent the money of the stockholders given to her Thousand Pesos? of ledger entries and vouchers showing that Villarama had
for incorporation. Will you please tell the Honorable Court co-mingled his personal funds and transactions with those
if you knew at the time your wife was spending the money A. I have testified before that sometimes my wife gives made in the name of the Corporation, are very illuminating
to pay debts, you personally knew she was spending the me money and I do not know exactly for what. evidence. Villarama has assailed the admissibility of these
money of the incorporators? exhibits, contending that no evidentiary value whatsoever
The evidence further shows that the initial cash should be given to them since "they were merely
A. You know my money and my wife's money are one. capitalization of the corporation of P105,000.00 was mostly photostatic copies of the originals, the best evidence being
We never talk about those things. financed by Villarama. Of the P105,000.00 deposited in the the originals themselves." According to him, at the time
First National City Bank of New York, representing the Pantranco offered the said exhibits, it was the most likely
Q. Doctor, your answer then is that since your money initial paid-up capital of the Corporation, P85,000.00 was possessor of the originals thereof because they were stolen
and your wife's money are one money and you did not know covered by Villarama's personal check. The deposit slip for from the files of the Corporation and only Pantranco was
when your wife was paying debts with the incorporator's the said amount of P105,000.00 was admitted in evidence able to produce the alleged photostat copies thereof.
money? as Exh. 23, which shows on its face that P20,000.00 was
paid in cash and P85,000.00 thereof was covered by Check Section 5 of Rule 130 of the Rules of Court provides for the
A. Because sometimes she uses my money, and No. F-50271 of the First National City Bank of New York. requisites for the admissibility of secondary evidence when
sometimes the money given to her she gives to me and I The testimonies of Alfonso Sancho5 and Joaquin Amansec,6 the original is in the custody of the adverse party, thus: (1)
deposit the money. both employees of said bank, have proved that the drawer opponent's possession of the original; (2) reasonable notice
of the check was Jose Villarama himself. to opponent to produce the original; (3) satisfactory proof
Q. Actually, aside from your wife, you were also the of its existence; and (4) failure or refusal of opponent to
custodian of some of the incorporators here, in the Another witness, Celso Rivera, accountant of the produce the original in court.11 Villarama has practically
beginning? Corporation, testified that while in the books of the admitted the second and fourth requisites.12 As to the
corporation there appears an entry that the treasurer third, he admitted their previous existence in the files of
A. Not necessarily, they give to my wife and when my received P95,000.00 as second installment of the paid-in the Corporation and also that he had seen some of them.13
wife hands to me I did not know it belonged to the subscriptions, and, subsequently, also P100,000.00 as the Regarding the first element, Villarama's theory is that since
incorporators. first installment of the offer for second subscriptions worth even at the time of the issuance of the subpoena duces
P200,000.00 from the original subscribers, yet Villarama tecum, the originals were already missing, therefore, the
Q. It supposes then your wife gives you some of the directed him (Rivera) to make vouchers liquidating the Corporation was no longer in possession of the same.
money received by her in her capacity as treasurer of the sums.7 Thus, it was made to appear that the P95,000.00 However, it is not necessary for a party seeking to
corporation? was delivered to Villarama in payment for equipment introduce secondary evidence to show that the original is in
purchased from him, and the P100,000.00 was loaned as the actual possession of his adversary. It is enough that the
A. Maybe. advances to the stockholders. The said accountant, circumstances are such as to indicate that the writing is in
however, testified that he was not aware of any amount of his possession or under his control. Neither is it required
Q. What did you do with the money, deposit in a regular money that had actually passed hands among the parties that the party entitled to the custody of the instrument
account? involved,8 and actually the only money of the corporation should, on being notified to produce it, admit having it in
was the P105,000.00 covered by the deposit slip Exh. 23, of his possession.14 Hence, secondary evidence is admissible
A. Deposit in my account. which as mentioned above, P85,000.00 was paid by where he denies having it in his possession. The party
Villarama's personal check. calling for such evidence may introduce a copy thereof as
Q. Of all the money given to your wife, she did not in the case of loss. For, among the exceptions to the best
receive any check? Further, the evidence shows that when the Corporation was evidence rule is "when the original has been lost,
in its initial months of operation, Villarama purchased and destroyed, or cannot be produced in court."15 The originals
A. I do not remember. paid with his personal checks Ford trucks for the of the vouchers in question must be deemed to have been
Corporation. Exhibits 20 and 21 disclose that the said lost, as even the Corporation admits such loss. Viewed upon
Q. Is it usual for you, Doctor, to be given Fifty Thousand purchases were paid by Philippine Bank of Commerce this light, there can be no doubt as to the admissibility in
Pesos without even asking what is this? Checks Nos. 992618-B and 993621-B, respectively. These evidence of Exhibits 6 to 19 and 22.
checks have been sufficiently established by Fausto Abad,
xxx xxxxxx Assistant Accountant of Manila Trading & Supply Co., from Taking account of the foregoing evidence, together with
which the trucks were purchased9 and Aristedes Solano, an Celso Rivera's testimony,16 it would appear that: Villarama
JUDGE: Reform the question. employee of the Philippine Bank of Commerce,10 as having supplied the organization expenses and the assets of the
been drawn by Villarama. Corporation, such as trucks and equipment;17 there was no

Page 106 of 160


CORPORATION LAW CASES MAYORDO, M.A
LLB 3

actual payment by the original subscribers of the amounts with the Corporation his credit with some companies. And that a seller or promisor may not make use of a corporate
of P95,000.00 and P100,000.00 as appearing in the books;18 his main reason for mingling his funds with that of the entity as a means of evading the obligation of his covenant.
Villarama made use of the money of the Corporation and Corporation and for the latter's paying his private bills is 31 Where the Corporation is substantially the alter ego of
deposited them to his private accounts;19 and the that it would be more convenient that he kept the money the covenantor to the restrictive agreement, it can be
Corporation paid his personal accounts.20 to be used in paying the registration fees on time, and enjoined from competing with the covenantee.32
since he had loaned money to the Corporation, this would
Villarama himself admitted that he mingled the corporate be set off by the latter's paying his bills. Villarama The Corporation contends that even on the supposition that
funds with his own money.21 He also admitted that gasoline admitted, however, that the corporate funds in his Villa Rey Transit, Inc. and Villarama are one and the same,
purchases of the Corporation were made in his name22 possession were not only for registration fees but for other the restrictive clause in the contract between Villarama
because "he had existing account with Stanvac which was important obligations which were not specified.26 and Pantranco does not include the purchase of existing
properly secured and he wanted the Corporation to benefit lines but it only applies to application for the new lines.
from the rebates that he received."23 Indeed, while Villarama was not the Treasurer of the The clause in dispute reads thus:
Corporation but was, allegedly, only a part-time manager,
The foregoing circumstances are strong persuasive evidence 27 he admitted not only having held the corporate money (4) The SELLER shall not, for a period of ten (10)
showing that Villarama has been too much involved in the but that he advanced and lent funds for the Corporation, years from the date of this sale apply for any TPU service
affairs of the Corporation to altogether negative the claim and yet there was no Board Resolution allowing it.28 identical or competing with the BUYER. (Emphasis supplied)
that he was only a part-time general manager. They show
beyond doubt that the Corporation is his alter ego. Villarama's explanation on the matter of his involvement As We read the disputed clause, it is evident from the
with the corporate affairs of the Corporation only renders context thereof that the intention of the parties was to
It is significant that not a single one of the acts enumerated more credible Pantranco's claim that his control over the eliminate the seller as a competitor of the buyer for ten
above as proof of Villarama's oneness with the Corporation corporation, especially in the management and disposition years along the lines of operation covered by the
has been denied by him. On the contrary, he has admitted of its funds, was so extensive and intimate that it is certificates of public convenience subject of their
them with offered excuses. impossible to segregate and identify which money belonged transaction. The word "apply" as broadly used has for frame
to whom. The interference of Villarama in the complex of reference, a service by the seller on lines or routes that
Villarama has admitted, for instance, having paid affairs of the corporation, and particularly its finances, are would compete with the buyer along the routes acquired by
P85,000.00 of the initial capital of the Corporation with the much too inconsistent with the ends and purposes of the the latter. In this jurisdiction, prior authorization is needed
lame excuse that "his wife had requested him to reimburse Corporation law, which, precisely, seeks to separate before anyone can operate a TPU service,33whether the
the amount entrusted to her by the incorporators and personal responsibilities from corporate undertakings. It is service consists in a new line or an old one acquired from a
which she had used to pay the obligations of Dr. Villarama the very essence of incorporation that the acts and conduct previous operator. The clear intention of the parties was to
(her husband) incurred while he was still the owner of Villa of the corporation be carried out in its own corporate name prevent the seller from conducting any competitive line for
Rey Transit, a single proprietorship." But with his admission because it has its own personality. 10 years since, anyway, he has bound himself not to apply
that he had received P350,000.00 from Pantranco for the for authorization to operate along such lines for the
sale of the two certificates and one unit,24 it becomes The doctrine that a corporation is a legal entity distinct duration of such period.34
difficult to accept Villarama's explanation that he and his and separate from the members and stockholders who
wife, after consultation,25 spent the money of their compose it is recognized and respected in all cases which If the prohibition is to be applied only to the acquisition of
relatives (the stockholders) when they were supposed to are within reason and the law.29 When the fiction is urged new certificates of public convenience thru an application
have their own money. Even if Pantranco paid the as a means of perpetrating a fraud or an illegal act or as a with the Public Service Commission, this would, in effect,
P350,000.00 in check to him, as claimed, it could have vehicle for the evasion of an existing obligation, the allow the seller just the same to compete with the buyer as
been easy for Villarama to have deposited said check in his circumvention of statutes, the achievement or perfection long as his authority to operate is only acquired thru
account and issued his own check to pay his obligations. of a monopoly or generally the perpetration of knavery or transfer or sale from a previous operator, thus defeating
And there is no evidence adduced that the said amount of crime,30 the veil with which the law covers and isolates the intention of the parties. For what would prevent the
P350,000.00 was all spent or was insufficient to settle his the corporation from the members or stockholders who seller, under the circumstances, from having a
prior obligations in his business, and in the light of the compose it will be lifted to allow for its consideration representative or dummy apply in the latter's name and
stipulation in the deed of sale between Villarama and merely as an aggregation of individuals. then later on transferring the same by sale to the seller?
Pantranco that P50,000.00 of the selling price was Since stipulations in a contract is the law between the
earmarked for the payments of accounts due to his Upon the foregoing considerations, We are of the opinion, contracting parties,
creditors, the excuse appears unbelievable. and so hold, that the preponderance of evidence have
shown that the Villa Rey Transit, Inc. is an alter ego of Jose Every person must, in the exercise of his rights and in the
On his having paid for purchases by the Corporation of M. Villarama, and that the restrictive clause in the contract performance of his duties, act with justice, give everyone
trucks from the Manila Trading & Supply Co. with his entered into by the latter and Pantranco is also enforceable his due, and observe honesty and good faith. (Art. 19, New
personal checks, his reason was that he was only sharing and binding against the said Corporation. For the rule is Civil Code.)

Page 107 of 160


CORPORATION LAW CASES MAYORDO, M.A
LLB 3

however, which restrains a man from entering into business ... Numerous authorities hold that a covenant which is
We are not impressed of Villarama's contention that the re- or trade without either a limitation as to time or place, will incidental to the sale and transfer of a trade or business,
wording of the two previous drafts of the contract of sale be held invalid. and which purports to bind the seller not to engage in the
between Villarama and Pantranco is significant in that as it same business in competition with the purchaser, is lawful
now appears, the parties intended to effect the least The public welfare of course must always be considered and enforceable. While such covenants are designed to
restriction. We are persuaded, after an examination of the and if it be not involved and the restraint upon one party is prevent competition on the part of the seller, it is
supposed drafts, that the scope of the final stipulation, not greater than protection to the other requires, contracts ordinarily neither their purpose nor effect to stifle
while not as long and prolix as those in the drafts, is just as like the one we are discussing will be sustained. The competition generally in the locality, nor to prevent it at all
broad and comprehensive. At most, it can be said that the general tendency, we believe, of modern authority, is to in a way or to an extent injurious to the public. The
re-wording was done merely for brevity and simplicity. make the test whether the restraint is reasonably necessary business in the hands of the purchaser is carried on just as
for the protection of the contracting parties. If the it was in the hands of the seller; the former merely takes
The evident intention behind the restriction was to contract is reasonably necessary to protect the interest of the place of the latter; the commodities of the trade are as
eliminate the sellers as a competitor, and this must be, the parties, it will be upheld. (Emphasis supplied.) open to the public as they were before; the same
considering such factors as the good will35 that the seller competition exists as existed before; there is the same
had already gained from the riding public and his adeptness Analyzing the characteristics of the questioned stipulation, employment furnished to others after as before; the profits
and proficiency in the trade. On this matter, Corbin, an We find that although it is in the nature of an agreement of the business go as they did before to swell the sum of
authority on Contracts has this to say.36 suppressing competition, it is, however, merely ancillary or public wealth; the public has the same opportunities of
incidental to the main agreement which is that of sale. The purchasing, if it is a mercantile business; and production is
When one buys the business of another as a going concern, suppression or restraint is only partial or limited: first, in not lessened if it is a manufacturing plant.
he usually wishes to keep it going; he wishes to get the scope, it refers only to application for TPU by the seller in
location, the building, the stock in trade, and the competition with the lines sold to the buyer; second, in The reliance by the lower court on tile case of Red Line
customers. He wishes to step into the seller's shoes and to duration, it is only for ten (10) years; and third, with Transportation Co. v. Bachrach41 and finding that the
enjoy the same business relations with other men. He is respect to situs or territory, the restraint is only along the stipulation is illegal and void seems misplaced. In the said
willing to pay much more if he can get the "good will" of lines covered by the certificates sold. In view of these Red Line case, the agreement therein sought to be
the business, meaning by this the good will of the limitations, coupled with the consideration of P350,000.00 enforced was virtually a division of territory between two
customers, that they may continue to tread the old for just two certificates of public convenience, and operators, each company imposing upon itself an obligation
footpath to his door and maintain with him the business considering, furthermore, that the disputed stipulation is not to operate in any territory covered by the routes of the
relations enjoyed by the seller. only incidental to a main agreement, the same is other. Restraints of this type, among common carriers have
reasonable and it is not harmful nor obnoxious to public always been covered by the general rule invalidating
... In order to be well assured of this, he obtains and pays service.38 It does not appear that the ultimate result of the agreements in restraint of trade. 42
for the seller's promise not to reopen business in clause or stipulation would be to leave solely to Pantranco
competition with the business sold. the right to operate along the lines in question, thereby Neither are the other cases relied upon by the plaintiff-
establishing monopoly or predominance approximating appellee applicable to the instant case. In Pampanga Bus
As to whether or not such a stipulation in restraint of trade thereto. We believe the main purpose of the restraint was Co., Inc. v. Enriquez,43the undertaking of the applicant
is valid, our jurisprudence on the matter37says: to protect for a limited time the business of the buyer. therein not to apply for the lifting of restrictions imposed
on his certificates of public convenience was not an
The law concerning contracts which tend to restrain Indeed, the evils of monopoly are farfetched here. There ancillary or incidental agreement. The restraint was the
business or trade has gone through a long series of changes can be no danger of price controls or deterioration of the principal objective. On the other hand, in Red Line
from time to time with the changing condition of trade and service because of the close supervision of the Public Transportation Co., Inc. v. Gonzaga,44 the restraint there in
commerce. With trifling exceptions, said changes have Service Commission.39 This Court had stated long ago,40 question not to ask for extension of the line, or trips, or
been a continuous development of a general rule. The early that "when one devotes his property to a use in which the increase of equipment — was not an agreement between
cases show plainly a disposition to avoid and annul all public has an interest, he virtually grants to the public an the parties but a condition imposed in the certificate of
contract which prohibited or restrained any one from using interest in that use and submits it to such public use under public convenience itself.
a lawful trade "at any time or at any place," as being reasonable rules and regulations to be fixed by the Public
against the benefit of the state. Later, however, the rule Utility Commission." Upon the foregoing considerations, Our conclusion is that
became well established that if the restraint was limited to the stipulation prohibiting Villarama for a period of 10
"a certain time" and within "a certain place," such contracts Regarding that aspect of the clause that it is merely years to "apply" for TPU service along the lines covered by
were valid and not "against the benefit of the state." Later ancillary or incidental to a lawful agreement, the the certificates of public convenience sold by him to
cases, and we think the rule is now well established, have underlying reason sustaining its validity is well explained in Pantranco is valid and reasonable. Having arrived at this
held that a contract in restraint of trade is valid providing 36 Am. Jur. 537-539, to wit: conclusion, and considering that the preponderance of the
there is a limitation upon either time or place. A contract, evidence have shown that Villa Rey Transit, Inc. is itself the

Page 108 of 160


CORPORATION LAW CASES MAYORDO, M.A
LLB 3

alter ego of Villarama, We hold, as prayed for in alienate, mortgage, encumber or lease its property, Villa Rey Transit, Inc. It is further claimed by Pantranco
Pantranco's third party complaint, that the said Corporation franchise, certificates, privileges, or rights or any part that the underhanded manner in which Villarama violated
should, until the expiration of the 1-year period thereof, ...," the same section also provides: the contract is pertinent in establishing punitive or moral
abovementioned, be enjoined from operating the line damages. Its contention as to the proper measure of
subject of the prohibition. ... Provided, however, That nothing herein contained shall damages is that it should be the purchase price of
be construed to prevent the transaction from being P350,000.00 that it paid to Villarama. While We are fully in
To avoid any misunderstanding, it is here to be emphasized negotiated or completed before its approval or to prevent accord with Pantranco's claim of entitlement to damages it
that the 10-year prohibition upon Villarama is not against the sale, alienation, or lease by any public service of any of suffered as a result of Villarama's breach of his contract
his application for, or purchase of, certificates of public its property in the ordinary course of its business. with it, the record does not sufficiently supply the
convenience, but merely the operation of TPU along the necessary evidentiary materials upon which to base the
lines covered by the certificates sold by him to Pantranco. It is clear, therefore, that the requisite approval of the PSC award and there is need for further proceedings in the
Consequently, the sale between Fernando and the is not a condition precedent for the validity and lower court to ascertain the proper amount.
Corporation is valid, such that the rightful ownership of the consummation of the sale.
disputed certificates still belongs to the plaintiff being the PREMISES CONSIDERED, the judgment appealed from is
prior purchaser in good faith and for value thereof. In view Anent the question of damages allegedly suffered by the hereby modified as follows:
of the ancient rule of caveat emptor prevailing in this parties, each of the appellants has its or his own version to
jurisdiction, what was acquired by Ferrer in the sheriff's allege. 1. The sale of the two certificates of public
sale was only the right which Fernando, judgment debtor, convenience in question by Valentin Fernando to Villa Rey
had in the certificates of public convenience on the day of Villa Rey Transit, Inc. claims that by virtue of the "tortious Transit, Inc. is declared preferred over that made by the
the sale.45 acts" of defendants (Pantranco and Ferrer) in acquiring the Sheriff at public auction of the aforesaid certificate of
certificates of public convenience in question, despite public convenience in favor of Eusebio Ferrer;
Accordingly, by the "Notice of Levy Upon Personalty" the constructive and actual knowledge on their part of a prior
Commissioner of Public Service was notified that "by virtue sale executed by Fernando in favor of the said corporation, 2. Reversed, insofar as it dismisses the third-party
of an Order of Execution issued by the Court of First which necessitated the latter to file the action to annul the complaint filed by Pangasinan Transportation Co. against
Instance of Pangasinan, the rights, interests, or sheriff's sale to Ferrer and the subsequent transfer to Jose M. Villarama, holding that Villa Rey Transit, Inc. is an
participation which the defendant, VALENTIN A. FERNANDO Pantranco, it is entitled to collect actual and compensatory entity distinct and separate from the personality of Jose M.
— in the above entitled case may have in the following damages, and attorney's fees in the amount of P25,000.00. Villarama, and insofar as it awards the sum of P5,000.00 as
realty/personalty is attached or levied upon, to wit: The The evidence on record, however, does not clearly show attorney's fees in favor of Villa Rey Transit, Inc.;
rights, interests and participation on the Certificates of that said defendants acted in bad faith in their acquisition
Public Convenience issued to Valentin A. Fernando, in Cases of the certificates in question. They believed that because 3. The case is remanded to the trial court for the
Nos. 59494, etc. ... Lines — Manila to Lingayen, Dagupan, the bill of sale has yet to be approved by the Public Service reception of evidence in consonance with the above
etc. vice versa." Such notice of levy only shows that Ferrer, Commission, the transaction was not a consummated sale, findings as regards the amount of damages suffered by
the vendee at auction of said certificates, merely stepped and, therefore, the title to or ownership of the certificates Pantranco; and
into the shoes of the judgment debtor. Of the same was still with the seller. The award by the lower court of
principle is the provision of Article 1544 of the Civil Code, attorney's fees of P5,000.00 in favor of Villa Rey Transit, 4. On equitable considerations, without costs. So
that "If the same thing should have been sold to different Inc. is, therefore, without basis and should be set aside. ordered.
vendees, the ownership shall be transferred to the person
who may have first taken possession thereof in good faith, Eusebio Ferrer's charge that by reason of the filing of the [G.R. No. 160039. June 29, 2004]
if it should be movable property." action to annul the sheriff's sale, he had suffered and RAYMUNDO ODANI SECOSA, EL BUENASENSO SY and
should be awarded moral, exemplary damages and DASSAD WAREHOUSING and PORT SERVICES,
There is no merit in Pantranco and Ferrer's theory that the attorney's fees, cannot be entertained, in view of the INCORPORATED, petitioners, vs. HEIRS OF ERWIN SUAREZ
sale of the certificates of public convenience in question, conclusion herein reached that the sale by Fernando to the FRANCISCO, respondents.
between the Corporation and Fernando, was not Corporation was valid.
consummated, it being only a conditional sale subject to DECISION
the suspensive condition of its approval by the Public Pantranco, on the other hand, justifies its claim for YNARES-SANTIAGO, J.:
Service Commission. While section 20(g) of the Public damages with the allegation that when it purchased
Service Act provides that "subject to established limitation ViIlarama's business for P350,000.00, it intended to build up This is a petition for review under Rule 45 of the Rules of
and exceptions and saving provisions to the contrary, it the traffic along the lines covered by the certificates but it Court seeking the reversal of the decision[1] of the Court of
shall be unlawful for any public service or for the owner, was rot afforded an opportunity to do so since barely three Appeals dated February 27, 2003 in CA-G.R. CV No. 61868,
lessee or operator thereof, without the approval and months had elapsed when the contract was violated by which affirmed in toto the June 19, 1998 decision[2] of
authorization of the Commission previously had ... to sell, Villarama operating along the same lines in the name of

Page 109 of 160


CORPORATION LAW CASES MAYORDO, M.A
LLB 3

Branch 20 of the Regional Trial Court of Manila in Civil Case 6. The sum of P50,000.00 as attorneys fees plus cost of a quasi-delict and is governed by the provisions of this
No. 96-79554. suit. Chapter.

The facts are as follows: SO ORDERED. On the other hand, Article 2180, in pertinent part, states:

On June 27, 1996, at around 4:00 p.m., Erwin Suarez Petitioners appealed the decision to the Court of Appeals, The obligation imposed by article 2176 is demandable not
Francisco, an eighteen year old third year physical therapy which affirmed the appealed decision in toto.[4] only for ones own acts or omissions, but also for those of
student of the Manila Central University, was riding a persons for whom one is responsible x xx.
motorcycle along Radial 10 Avenue, near the Veteran Hence the present petition, based on the following
Shipyard Gate in the City of Manila. At the same time, arguments: Employers shall be liable for the damages caused by their
petitioner, Raymundo OdaniSecosa, was driving an Isuzu employees and household helpers acting within the scope
cargo truck with plate number PCU-253 on the same road. I. of their assigned tasks, even though the former are not
The truck was owned by petitioner, Dassad Warehousing engaged in any business or industry x xx.
and Port Services, Inc. THE COURT OF APPEALS SERIOUSLY ERRED WHEN IT
AFFIRMED THE DECISION OF THE TRIAL COURT THAT The responsibility treated of in this article shall cease when
Traveling behind the motorcycle driven by Francisco was a PETITIONER DASSAD DID NOT EXERCISE THE DILIGENCE OF A the persons herein mentioned prove that they observed all
sand and gravel truck, which in turn was being tailed by the GOOD FATHER OF A FAMILY IN THE SELECTION AND the diligence of a good father of a family to prevent
Isuzu truck driven by Secosa. The three vehicles were SUPERVISION OF ITS EMPLOYEES WHICH IS NOT IN damage.
traversing the southbound lane at a fairly high speed. When ACCORDANCE WITH ARTICLE 2180 OF THE NEW CIVIL CODE
Secosa overtook the sand and gravel truck, he bumped the AND RELATED JURISPRUDENCE ON THE MATTER. Based on the foregoing provisions, when an injury is caused
motorcycle causing Francisco to fall. The rear wheels of the by the negligence of an employee, there instantly arises a
Isuzu truck then ran over Francisco, which resulted in his II. presumption that there was negligence on the part of the
instantaneous death. Fearing for his life, petitioner Secosa employer either in the selection of his employee or in the
left his truck and fled the scene of the collision.[3] THE COURT OF APPEALS SERIOUSLY ERRED WHEN IT supervision over him after such selection. The presumption,
AFFIRMED THE DECISION OF THE TRIAL COURT IN HOLDING however, may be rebutted by a clear showing on the part of
Respondents, the parents of Erwin Francisco, thus filed an PETITIONER EL BUENASENSO SY SOLIDARILY LIABLE WITH the employer that it exercised the care and diligence of a
action for damages against Raymond OdaniSecosa, Dassad PETITIONERS DASSAD AND SECOSA IN VIOLATION OF THE good father of a family in the selection and supervision of
Warehousing and Port Services, Inc. and Dassads president, CORPORATION LAW AND RELATED JURISPRUDENCE ON THE his employee. Hence, to evade solidary liability for quasi-
El Buenasucenso Sy. The complaint was docketed as Civil MATTER. delict committed by an employee, the employer must
Case No. 96-79554 of the RTC of Manila, Branch 20. adduce sufficient proof that it exercised such degree of
III. care.[6]
On June 19, 1998, after a full-blown trial, the court a quo
rendered a decision in favor of herein respondents, the THE JUDGMENT OF THE TRIAL COURT AS AFFIRMED BY THE How does an employer prove that he indeed exercised the
dispositive portion of which states: COURT OF APPEALS AWARDING P500,000.00 AS MORAL diligence of a good father of a family in the selection and
DAMAGES IS MANIFESTLY ABSURD, MISTAKEN AND UNJUST.[5] supervision of his employee? The case of Metro Manila
WHEREFORE, premised on the foregoing, judgment is Transit Corporation v. Court of Appeals[7] is instructive:
hereby rendered in favor of the plaintiffs ordering the The petition is partly impressed with merit.
defendants to pay plaintiffs jointly and severally: In fine, the party, whether plaintiff or defendant, who
On the issue of whether petitioner Dassad Warehousing and asserts the affirmative of the issue has the burden of
1. The sum of P55,000.00 as actual and compensatory Port Services, Inc. exercised the diligence of a good father presenting at the trial such amount of evidence required by
damages; of a family in the selection and supervision of its law to obtain a favorable judgment[8] . . . In making proof
employees, we find the assailed decision to be in full in its or his case, it is paramount that the best and most
2. The sum of P20,000.00 for the repair of the motorcycle; accord with pertinent provisions of law and established complete evidence is formally entered.[9]
jurisprudence.
3. The sum of P100,000.00 for the loss of earning capacity; Coming now to the case at bar, while there is no rule which
Article 2176 of the Civil Code provides: requires that testimonial evidence, to hold sway, must be
4. The sum of P500,000.00 as moral damages; corroborated by documentary evidence, inasmuch as the
Whoever by act or omission causes damage to another, witnesses testimonies dwelt on mere generalities, we
5. The sum of P50,000.00 as exemplary damages; there being fault or negligence, is obliged to pay for the cannot consider the same as sufficiently persuasive proof
damage done. Such fault or negligence, if there is no pre- that there was observance of due diligence in the selection
existing contractual relation between the parties, is called and supervision of employees. Petitioners attempt to prove

Page 110 of 160


CORPORATION LAW CASES MAYORDO, M.A
LLB 3

its deligentissimipatrisfamilias in the selection and be physically and mentally fit for he had undergone rigid
supervision of employees through oral evidence must fail as In this case, MMTC sought to prove that it exercised the training and attended the PPA safety seminar.[15]
it was unable to buttress the same with any other diligence of a good father of a family with respect to the
evidence, object or documentary, which might obviate the selection of employees by presenting mainly testimonial Petitioner Dassad Warehousing and Port Services, Inc. failed
apparent biased nature of the testimony.[10] evidence on its hiring procedure. According to MMTC, to support the testimony of its lone witness with
applicants are required to submit professional driving documentary evidence which would have strengthened its
Our view that the evidence for petitioner MMTC falls short licenses, certifications of work experience, and clearances claim of due diligence in the selection and supervision of its
of the required evidentiary quantum as would convincingly from the National Bureau of Investigation; to undergo tests employees. Such an omission is fatal to its position, on
and undoubtedly prove its observance of the diligence of a of their driving skills, concentration, reflexes, and vision; account of which, Dassad can be rightfully held solidarily
good father of a family has its precursor in the underlying and, to complete training programs on traffic rules, vehicle liable with its co-petitioner Raymundo Secosa for the
rationale pronounced in the earlier case of Central Taxicab maintenance, and standard operating procedures during damages suffered by the heirs of Erwin Francisco.
Corp. vs. Ex-Meralco Employees Transportation Co., et al., emergency cases.
[11] set amidst an almost identical factual setting, where However, we find that petitioner El Buenasenso Sy cannot
we held that: x xxxxxxxx be held solidarily liable with his co-petitioners. While it
may be true that Sy is the president of petitioner Dassad
The failure of the defendant company to produce in court Although testimonies were offered that in the case of Pedro Warehousing and Port Services, Inc., such fact is not by
any record or other documentary proof tending to establish Musa all these precautions were followed, the records of itself sufficient to hold him solidarily liable for the
that it had exercised all the diligence of a good father of a his interview, of the results of his examinations, and of his liabilities adjudged against his co-petitioners.
family in the selection and supervision of its drivers and service were not presented. . . [T]here is no record that
buses, notwithstanding the calls therefor by both the trial Musa attended such training programs and passed the said It is a settled precept in this jurisdiction that a corporation
court and the opposing counsel, argues strongly against its examinations before he was employed. No proof was is invested by law with a personality separate from that of
pretensions. presented that Musa did not have any record of traffic its stockholders or members.[16] It has a personality
violations. Nor were records of daily inspections, allegedly separate and distinct from those of the persons composing
We are fully aware that there is no hard-and-fast rule on conducted by supervisors, ever presented. . . The failure of it as well as from that of any other entity to which it may
the quantum of evidence needed to prove due observance MMTC to present such documentary proof puts in doubt the be related. Mere ownership by a single stockholder or by
of all the diligence of a good father of a family as would credibility of its witnesses. another corporation of all or nearly all of the capital stock
constitute a valid defense to the legal presumption of of a corporation is not in itself sufficient ground for
negligence on the part of an employer or master whose Jurisprudentially, therefore, the employer must not merely disregarding the separate corporate personality.[17] A
employee has by his negligence, caused damage to another. present testimonial evidence to prove that he observed the corporations authority to act and its liability for its actions
x xx (R)educing the testimony of Albert to its proper diligence of a good father of a family in the selection and are separate and apart from the individuals who own it.[18]
proportion, we do not have enough trustworthy evidence supervision of his employee, but he must also support such
left to go by. We are of the considered opinion, therefore, testimonial evidence with concrete or documentary The so-called veil of corporation fiction treats as separate
that the believable evidence on the degree of care and evidence. The reason for this is to obviate the biased and distinct the affairs of a corporation and its officers and
diligence that has been exercised in the selection and nature of the employers testimony or that of his witnesses. stockholders. As a general rule, a corporation will be
supervision of Roberto Leon y Salazar, is not legally [14] looked upon as a legal entity, unless and until sufficient
sufficient to overcome the presumption of negligence reason to the contrary appears. When the notion of legal
against the defendant company. Applying the foregoing doctrines to the present case, we entity is used to defeat public convenience, justify wrong,
hold that petitioner Dassad Warehousing and Port Services, protect fraud, or defend crime, the law will regard the
The above-quoted ruling was reiterated in a recent case Inc. failed to conclusively prove that it had exercised the corporation as an association of persons.[19] Also, the
again involving the Metro Manila Transit Corporation,[12] requisite diligence of a good father of a family in the corporate entity may be disregarded in the interest of
thus: selection and supervision of its employees. justice in such cases as fraud that may work inequities
among members of the corporation internally, involving no
In the selection of prospective employees, employers are EdilbertoDuerme, the lone witness presented by Dassad rights of the public or third persons. In both instances,
required to examine them as to their qualifications, Warehousing and Port Services, Inc. to support its position there must have been fraud and proof of it. For the
experience, and service records.[13] On the other hand, that it had exercised the diligence of a good father of a separate juridical personality of a corporation to be
with respect to the supervision of employees, employers family in the selection and supervision of its employees, disregarded, the wrongdoing must be clearly and
should formulate standard operating procedures, monitor testified that he was the one who recommended petitioner convincingly established.[20] It cannot be presumed.[21]
their implementation, and impose disciplinary measures for Raymundo Secosa as a driver to Dassad Warehousing and
breaches thereof. To establish these factors in a trial Port Services, Inc.; that it was his duty to scrutinize the The records of this case are bereft of any evidence tending
involving the issue of vicarious liability, employers must capabilities of drivers; and that he believed petitioner to to show the presence of any grounds enumerated above
submit concrete proof, including documentary evidence. that will justify the piercing of the veil of corporate fiction

Page 111 of 160


CORPORATION LAW CASES MAYORDO, M.A
LLB 3

such as to hold the president of Dassad Warehousing and SO ORDERED.


Port Services, Inc. solidarily liable with it. Q: Mr. Witness, how did you feel when you learned of the
untimely death of your son, Erwin Suares (sic)? G.R. No. 97212 June 30, 1993
The Isuzu cargo truck which ran over Erwin Francisco was BENJAMIN YU, petitioner, vs. NATIONAL LABOR
registered in the name of Dassad Warehousing and Port A: Masakit po ang mawalan ng anak. Its really hard for me, RELATIONS COMMISSION and JADE MOUNTAIN PRODUCTS
Services, Inc., and not in the name of El Buenasenso Sy. the thought that my son is dead. COMPANY LIMITED, WILLY CO, RHODORA D. BENDAL, LEA
Raymundo Secosa is an employee of Dassad Warehousing BENDAL, CHIU SHIAN JENG and CHEN HO-
and Port Services, Inc. and not of El Buenasenso Sy. All x xxxxxxxx FU, respondents.
these things, when taken collectively, point toward El
Buenasenso Sys exclusion from liability for damages arising Q: How did your family react to the death of Erwin Suarez FELICIANO, J.:
from the death of Erwin Francisco. Francisco? Petitioner Benjamin Yu was formerly the Assistant General
Manager of the marble quarrying and export business
Having both found Raymundo Secosa and Dassad A: All of my family and relatives were felt (sic) sorrow operated by a registered partnership with the firm name of
Warehousing and Port Services, Inc. liable for negligence because they knew that my son is (sic) good. "Jade Mountain Products Company Limited" ("Jade
for the death of Erwin Francisco on June 27, 1996, we now Mountain"). The partnership was originally organized on 28
consider the question of moral damages which his parents, Q: We know that it is impossible to put money terms(s) [on] June 1984 with Lea Bendal and Rhodora Bendal as general
herein respondents, are entitled to recover. Petitioners the life of [a] human, but since you are now in court and if partners and Chin Shian Jeng, Chen Ho-Fu and Yu Chang, all
assail the award of moral damages of P500,000.00 for being you were to ask this court how much would you and your citizens of the Republic of China (Taiwan), as limited
manifestly absurd, mistaken and unjust. We are not family compensate? (sic) partners. The partnership business consisted of exploiting a
persuaded. marble deposit found on land owned by the Sps. Ricardo
A: Even if they pay me millions, they cannot remove the and Guillerma Cruz, situated in Bulacan Province, under a
Under Article 2206, the spouse, legitimate and illegitimate anguish of my son (sic).[23] Memorandum Agreement dated 26 June 1984 with the Cruz
descendants and ascendants of the deceased may demand spouses.  1  The partnership had its main office in Makati,
moral damages for mental anguish for the death of the Moral damages are emphatically not intended to enrich a Metropolitan Manila.
deceased. The reason for the grant of moral damages has plaintiff at the expense of the defendant. They are
been explained in this wise: awarded to allow the former to obtain means, diversion or Benjamin Yu was hired by virtue of a Partnership Resolution
amusements that will serve to alleviate the moral suffering dated 14 March 1985, as Assistant General Manager with a
. . . the award of moral damages is aimed at a restoration, he has undergone due to the defendants culpable action monthly salary of P4,000.00. According to petitioner Yu,
within the limits possible, of the spiritual status quo ante; and must, perforce, be proportional to the suffering however, he actually received only half of his stipulated
and therefore, it must be proportionate to the suffering inflicted.[24] We have previously held as proper an award monthly salary, since he had accepted the promise of the
inflicted. The intensity of the pain experienced by the of P500,000.00 as moral damages to the heirs of a deceased partners that the balance would be paid when the firm
relatives of the victim is proportionate to the intensity of family member who died in a vehicular accident. In our shall have secured additional operating funds from abroad.
affection for him and bears no relation whatsoever with the 2002 decision in Metro Manila Transit Corporation v. Court Benjamin Yu actually managed the operations and finances
wealth or means of the offender.[22] of Appeals, et al.,[25] we affirmed the award of moral of the business; he had overall supervision of the workers
damages of P500,000.00 to the heirs of the victim, a at the marble quarry in Bulacan and took charge of the
In the instant case, the spouses Francisco presented mother, who died from injuries she sustained when a bus preparation of papers relating to the exportation of the
evidence of the searing pain that they felt when the driven by an employee of the petitioner hit her. In the case firm's products.
premature loss of their son was relayed to them. That pain at bar, we likewise affirm the portion of the assailed Sometime in 1988, without the knowledge of Benjamin Yu,
was highly evident in the testimony of the father who was decision awarding the moral damages. the general partners Lea Bendal and Rhodora Bendal sold
forever deprived of a son, a son whose untimely death and transferred their interests in the partnership to private
came at that point when the latter was nearing the Since the petitioners did not question the other damages respondent Willy Co and to one Emmanuel Zapanta. Mr. Yu
culmination of every parents wish to educate their adjudged against them by the court a quo, we affirm the Chang, a limited partner, also sold and transferred his
children. The death of Francis has indeed left a void in the award of these damages to the respondents. interest in the partnership to Willy Co. Between Mr.
lives of the respondents. Antonio Francisco testified on the Emmanuel Zapanta and himself, private respondent Willy
effect of the death of his son, Francis, in this manner: WHEREFORE, the petition is DENIED. The assailed decision Co acquired the great bulk of the partnership interest. The
is AFFIRMED with the MODIFICATION that petitioner El partnership now constituted solely by Willy Co and
Q: (Atty. Balanag): What did you do when you learned that Buenasenso Sy is ABSOLVED from any liability adjudged Emmanuel Zapanta continued to use the old firm name of
your son was killed on June 27, 1996? against his co-petitioners in this case. Jade Mountain, though they moved the firm's main office
from Makati to Mandaluyong, Metropolitan Manila. A
A: (ANTONIO FRANCISCO): I boxed the door and pushed the Costs against petitioners. Supplement to the Memorandum Agreement relating to the
image of St. Nio telling why this happened to us. operation of the marble quarry was entered into with the

Page 112 of 160


CORPORATION LAW CASES MAYORDO, M.A
LLB 3

Cruz spouses in February of 1988.2 The actual operations of Petitioner Benjamin Yu is now before the Court on a
the business enterprise continued as before. All the Petition for Certiorari, asking us to set aside and annul the (2) in contravention of the agreement between the
employees of the partnership continued working in the Resolution of the NLRC as a product of grave abuse of partners, where the circumstances do not permit a
business, all, save petitioner Benjamin Yu as it turned out. discretion amounting to lack or excess of jurisdiction. dissolution under any other provision of this article, by the
express will of any partner at any time;
On 16 November 1987, having learned of the transfer of the The basic contention of petitioner is that the NLRC has x x x           x x x          x x x
firm's main office from Makati to Mandaluyong, petitioner overlooked the principle that a partnership has a juridical (Emphasis supplied)
Benjamin Yu reported to the Mandaluyong office for work personality separate and distinct from that of each of its
and there met private respondent Willy Co for the first members. Such independent legal personality subsists, In the case at bar, just about all of the partners had sold
time. Petitioner was informed by Willy Co that the latter petitioner claims, notwithstanding changes in the identities their partnership interests (amounting to 82% of the total
had bought the business from the original partners and that of the partners. Consequently, the employment contract partnership interest) to Mr. Willy Co and Emmanuel
it was for him to decide whether or not he was responsible between Benjamin Yu and the partnership Jade Mountain Zapanta. The record does not show what happened to the
for the obligations of the old partnership, including could not have been affected by changes in the latter's remaining 18% of the original partnership interest. The
petitioner's unpaid salaries. Petitioner was in fact not membership.7 acquisition of 82% of the partnership interest by new
allowed to work anymore in the Jade Mountain business partners, coupled with the retirement or withdrawal of the
enterprise. His unpaid salaries remained unpaid.3 Two (2) main issues are thus posed for our consideration in partners who had originally owned such 82% interest, was
the case at bar: (1) whether the partnership which had enough to constitute a new partnership.
On 21 December 1988. Benjamin Yu filed a complaint for hired petitioner Yu as Assistant General Manager had been
illegal dismissal and recovery of unpaid salaries accruing extinguished and replaced by a new partnerships composed The occurrence of events which precipitate the legal
from November 1984 to October 1988, moral and exemplary of Willy Co and Emmanuel Zapanta; and (2) if indeed a new consequence of dissolution of a partnership do not,
damages and attorney's fees, against Jade Mountain, Mr. partnership had come into existence, whether petitioner Yu however, automatically result in the termination of the
Willy Co and the other private respondents. The could nonetheless assert his rights under his employment legal personality of the old partnership. Article 1829 of the
partnership and Willy Co denied petitioner's charges, contract as against the new partnership. Civil Code states that:
contending in the main that Benjamin Yu was never hired as
an employee by the present or new partnership.4 In respect of the first issue, we agree with the result [o]n dissolution the partnership is not terminated, but
reached by the NLRC, that is, that the legal effect of the continues until the winding up of partnership affairs is
In due time, Labor Arbiter Nieves Vivar-De Castro rendered changes in the membership of the partnership was the completed.
a decision holding that petitioner had been illegally dissolution of the old partnership which had hired
dismissed. The Labor Arbiter decreed his reinstatement and petitioner in 1984 and the emergence of a new firm In the ordinary course of events, the legal personality of
awarded him his claim for unpaid salaries, backwages and composed of Willy Co and Emmanuel Zapanta in 1987. the expiring partnership persists for the limited purpose of
attorney's fees.5 winding up and closing of the affairs of the partnership. In
On appeal, the National Labor Relations Commission The applicable law in this connection — of which the NLRC the case at bar, it is important to underscore the fact that
("NLRC") reversed the decision of the Labor Arbiter and seemed quite unaware — is found in the Civil Code the business of the old partnership was simply continued by
dismissed petitioner's complaint in a Resolution dated 29 provisions relating to partnerships. Article 1828 of the Civil the new partners,  without  the old partnership undergoing
November 1990. The NLRC held that a new partnership Code provides as follows: the procedures relating to dissolution and winding up of its
consisting of Mr. Willy Co and Mr. Emmanuel Zapanta had business affairs. In other words, the new partnership simply
bought the Jade Mountain business, that the new Art. 1828. The dissolution of a partnership is the change in took over the business enterprise owned by the preceeding
partnership had not retained petitioner Yu in his original the relation of the partners caused by any partner ceasing partnership, and continued using the old name of Jade
position as Assistant General Manager, and that there was to be associated in the carrying on  as distinguished from Mountain Products Company Limited, without winding up
no law requiring the new partnership to absorb the the winding up of the business. (Emphasis supplied) the business affairs of the old partnership, paying off its
employees of the old partnership. Benjamin Yu, therefore, debts, liquidating and distributing its net assets, and then
had not been illegally dismissed by the new partnership Article 1830 of the same Code must also be noted: re-assembling the said assets or most of them and opening
which had simply declined to retain him in his former a new business enterprise. There were, no doubt, powerful
managerial position or any other position. Finally, the NLRC Art. 1830. Dissolution is caused: tax considerations which underlay such an informal
held that Benjamin Yu's claim for unpaid wages should be (1) without violation of the agreement between the approach to business on the part of the retiring and the
asserted against the original members of the preceding partners; incoming partners. It is not, however, necessary to inquire
partnership, but these though impleaded had, apparently, x x x           x x x          x x x into such matters.
not been served with summons in the proceedings before (b) by the express will of any partner, who must act in good What is important for present purposes is that, under the
the Labor Arbiter.6 faith, when no definite term or particular undertaking is above described situation,  not only the retiring partners
specified; (Rhodora Bendal, et al.) but also the new partnership itself
x x x           x x x          x x x which continued the business of the old, dissolved, one, are

Page 113 of 160


CORPORATION LAW CASES MAYORDO, M.A
LLB 3

liable for the debts of the preceding partnership. out of the partnership property only, unless there is a Mountain. Benjamin Yu's old position as Assistant General
In Singson, et al. v. Isabela Saw Mill, et al,8 the Court held stipulation to the contrary. Manager thus became superfluous or redundant.  11It
that under facts very similar to those in the case at bar, a follows that petitioner Benjamin Yu is entitled to
withdrawing partner remains liable to a third party creditor When the business of a partnership after dissolution is separation pay at the rate of one month's pay for each year
of the old partnership.9  The liability of the new continued under any conditions set forth in this article the of service that he had rendered to the old partnership, a
partnership, upon the other hand, in the set of creditors of the retiring or deceased partner or the fraction of at least six (6) months being considered as a
circumstances obtaining in the case at bar, is established in representative of the deceased partner, have a prior right whole year.
Article 1840 of the Civil Code which reads as follows: to any claim of the retired partner or the representative of
the deceased partner against the person or partnership While the new Jade Mountain was entitled to decline to
Art. 1840. In the following cases creditors of the dissolved continuing the business on account of the retired or retain petitioner Benjamin Yu in its employ, we consider
partnership are  also  creditors of the person or partnership deceased partner's interest in the dissolved partnership or that Benjamin Yu was very shabbily treated by the new
continuing the business: on account of any consideration promised for such interest partnership. The old partnership certainly benefitted from
or for his right in partnership property. the services of Benjamin Yu who, as noted, previously ran
(1) When any new partner is admitted into an existing the whole marble quarrying, processing and exporting
partnership, or when any partner retires and assigns (or the Nothing in this article shall be held to modify any right of enterprise. His work constituted value-added to the
representative of the deceased partner assigns) his rights in creditors to set assignment on the ground of fraud. business itself and therefore, the new partnership similarly
partnership property to two or more of the partners, or to xxx xxx xxx benefitted from the labors of Benjamin Yu. It is worthy of
one or more of the partners and one or more third note that the new partnership did not try to suggest that
persons, if the business is continued without liquidation of (Emphasis supplied) there was any cause consisting of some blameworthy act or
the partnership affairs; Under Article 1840 above, creditors of the old Jade omission on the part of Mr. Yu which compelled the new
Mountain are also creditors of the new Jade Mountain partnership to terminate his services. Nonetheless, the new
(2) When all but one partner retire and assign (or the which continued the business of the old one without Jade Mountain did not notify him of the change in
representative of a deceased partner assigns) their rights in liquidation of the partnership affairs. Indeed, a creditor of ownership of the business, the relocation of the main office
partnership property to the remaining partner, the old Jade Mountain, like petitioner Benjamin Yu in of Jade Mountain from Makati to Mandaluyong and the
who  continues the business without liquidation of respect of his claim for unpaid wages, is entitled to assumption by Mr. Willy Co of control of operations. The
partnership affairs, either alone or with others; priority  vis-a-vis  any claim of any retired or previous treatment (including the refusal to honor his claim for
partner insofar as such retired partner's interest in the unpaid wages) accorded to Assistant General Manager
(3) When any Partner retires or dies and the business of the dissolved partnership is concerned. It is not necessary for Benjamin Yu was so summary and cavalier as to amount to
dissolved partnership is continued as set forth in Nos. 1 and the Court to determine under which one or mare of the arbitrary, bad faith treatment, for which the new Jade
2 of this Article, with the consent of the retired partners or above six (6) paragraphs, the case at bar would fall, if only Mountain may legitimately be required to respond by
the representative of the deceased partner, but without because the facts on record are not detailed with sufficient paying moral damages. This Court, exercising its discretion
any assignment of his right in partnership property; precision to permit such determination. It is, however, and in view of all the circumstances of this case, believes
clear to the Court that under Article 1840 above, Benjamin that an indemnity for moral damages in the amount of
(4) When all the partners or their representatives assign Yu is entitled to enforce his claim for unpaid salaries, as P20,000.00 is proper and reasonable.
their rights in partnership property to one or more third well as other claims relating to his employment with the
persons  who promise to pay the debts and  who continue previous partnership, against the new Jade Mountain. In addition, we consider that petitioner Benjamin Yu is
the business of the dissolved partnership; entitled to interest at the legal rate of six percent (6%) per
It is at the same time also evident to the Court that the annum  on the amount of unpaid wages, and of his
(5) When any partner wrongfully causes a dissolution and new partnership was entitled to appoint and hire a new separation pay, computed from the date of promulgation of
remaining partners continue the business  under the general or assistant general manager to run the affairs of the award of the Labor Arbiter. Finally, because the new
provisions of article 1837, second paragraph, No. 2, either the business enterprise take over. An assistant general Jade Mountain compelled Benjamin Yu to resort to
alone or with others,  and  without liquidation of the manager belongs to the most senior ranks of management litigation to protect his rights in the premises, he is
partnership affairs; and a new partnership is entitled to appoint a top manager entitled to attorney's fees in the amount of ten percent
of its own choice and confidence. The non-retention of (10%) of the total amount due from private respondent
(6) When a partner is expelled and the remaining Benjamin Yu as Assistant General Manager did not therefore Jade Mountain.
partners  continue the business either alone or with others constitute unlawful termination, or termination without
without liquidation of the partnership affairs; just or authorized cause. We think that the precise WHEREFORE, for all the foregoing, the Petition
The liability of a third person becoming a partner in the authorized cause for termination in the case at bar for Certiorari is GRANTED DUE COURSE, the Comment filed
partnership continuing the business, under this article, to was  redundancy.  10  The new partnership had its own new by private respondents is treated as their Answer to the
the creditors of the dissolved partnership shall be satisfied General Manager, apparently Mr. Willy Co, the principal new Petition for Certiorari, and the Decision of the NLRC dated
owner himself, who personally ran the business of Jade 29 November 1990 is hereby NULLIFIED and SET ASIDE. A

Page 114 of 160


CORPORATION LAW CASES MAYORDO, M.A
LLB 3

new Decision is hereby ENTERED requiring private Milling and Plantation Company and in the course of its Florence, the decision containing the following dispositive
respondent Jade Mountain Products Company Limited to corporate existence the company acquired various part:
pay to petitioner Benjamin Yu the following amounts: properties but at the same time all the other original
incorporators were bought out by Forrest L. Cease together VIEWED IN THE LIGHT OF ALL THE FOREGOING, judgment is
(a) for unpaid wages which, as found by the Labor Arbiter, with his children namely Ernest, Cecilia, Teresita, hereby rendered in favor of plaintiffs and against the
shall be computed at the rate of P2,000.00 per month Benjamin, Florence and one Bonifacia Tirante also defendants declaring that:
multiplied by thirty-six (36) months (November 1984 to considered a member of the family; the charter of the
December 1987) in the total amount of P72,000.00; company lapsed in June 1958; but whether there were 1) The assets or properties of the defunct Tiaong Milling
steps to liquidate it, the record is silent; on 13 August and Plantation Company now appearing under the name of
(b) separation pay computed at the rate of P4,000.00 1959, Forrest L. Cease died and by extrajudicial partition of F.L. Cease Plantation Company as Trustee, is the estate also
monthly pay multiplied by three (3) years of service or a his shares, among the children, this was disposed of on 19 of the deceased Forrest L. Cease and ordered divided,
total of P12,000.00; October 1959; it was here where the trouble among them share and share alike, among his six children the plaintiffs
came to arise because it would appear that Benjamin and and the defendants in accordance with Rule 69, Rules of
(c) indemnity for moral damages in the amount of Florence wanted an actual division while the other children Court;
P20,000.00; wanted reincorporation; and proceeding on that, these
other children Ernesto, Teresita and Cecilia and 2) The Resolution to Sell dated October 12, 1959 and the
(d) six percent (6%) per annum legal interest computed on aforementioned other stockholder Bonifacia Tirante Transfer and Conveyance with Trust Agreement is hereby
items (a) and (b) above, commencing on 26 December 1989 proceeded to incorporate themselves into the F.L. Cease set aside as improper and illegal for the purposes and
and until fully paid; and Plantation Company and registered it with the Securities effect that it was intended and, therefore, null and void;
and Exchange Commission on 9 December, 1959; apparently
(e) ten percent (10%) attorney's fees on the total amount in view of that, Benjamin and Florence for their part 3) That F.L. Cease Plantation Company is removed as
due from private respondent Jade Mountain. initiated a Special Proceeding No. 3893 of the Court of First 'Trustee for interest against the estate and essential to the
Instance of Tayabas for the settlement of the estate of protection of plaintiffs' rights and is hereby ordered to
Costs against private respondents. Forest L. Cease on 21 April, 1960 and one month afterwards deliver and convey all the properties and assets of the
SO ORDERED. on 19 May 1960 they filed Civil Case No. 6326 against defunct Tiaong Milling now under its name, custody and
Ernesto, Teresita and Cecilia Cease together with Bonifacia control to whomsoever be appointed as Receiver -
G.R. No. L-33172 October 18, 1979 Tirante asking that the Tiaong Milling and Plantation disqualifying and of the parties herein - the latter to act
ERNESTO CEASE, CECILIA CEASE, MARION CEASE, TERESA Corporation be declared Identical to F.L. Cease and that its accordingly upon proper assumption of office; and
CEASE-LACEBAL and the F.L. CEASE PLANTATION CO., properties be divided among his children as his intestate
INC. as Trustee of properties of the defunct TIAONG heirs; this Civil Case was resisted by aforestated defendants 4) Special Proceedings No. 3893 for administration is
MILLING & PLANTATION CO., petitioners,  vs. HONORABLE and notwithstanding efforts of the plaintiffs to have the terminated and dismissed; the instant case to proceed but
COURT OF APPEALS, (Special Seventh Division), HON. properties placed under receivership, they were not able to on issues of damages only and for such action inherently
MANOLO L. MADDELA, Presiding Judge, Court of First succeed because defendants filed a bond to remain as they essential for partition.
Instance of Quezon, BENJAMIN CEASE and FLORENCE have remained in possession; after that and already, during
CEASE, respondents. the pendency of Civil Case No. 6326 specifically on 21 May, SO ORDERED.
1961 apparently on the eve of the expiry of the three (3)
GUERRERO, J: year period provided by the law for the liquidation of Lucena City, December 27, 1969., pp. 122-a-123, rollo.
Appeal by certiorari from the decision of the Court of corporations, the board of liquidators of Tiaong Milling
Appeals in CA-G.R. No. 45474, entitled "Ernesto Cease, et executed an assignment and conveyance of properties and upon receipt of that, defendants there filled a notice of
al. vs. Hon. Manolo L. Maddela, Judge of the Court of First trust agreement in favor of F.L. Cease Plantation Co. Inc. as appeal p. 129, rollo together with an appeal bond and a
Instance of Quezon, et al."  1  which dismissed the petition trustee of the Tiaong Milling and Plantation Co. so Chat record on appeal but the plaintiffs moved to dismiss the
for certiorari, mandamus, and prohibition instituted by the upon motion of the plaintiffs trial Judge ordered that this appeal on the ground that the judgment was in fact
petitioners against the respondent judge and the private alleged trustee be also included as party defendant; now interlocutory and not appealable p. 168 rollo and this
respondents. this being the situation, it will be remembered that there position of defendants was sustained by trial Judge, His
The antecedents of the case, as found by the appellate were thus two (2) proceedings pending in the Court of First Honor ruling that
court, are as follows: Instance of Quezon namely Civil Case No. 6326 and Special
Proceeding No. 3893 but both of these were assigned to the IN VIEW OF THE FOREGOING, the appeal interposed by
IT RESULTING: That the antecedents are not difficult to Honorable Respondent Judge Manolo L. Maddela p. 43 and plaintiffs is hereby dismissed as premature and the Record
understand; sometime in June 1908, one Forrest L. Cease the case was finally heard and submitted upon stipulation on Appeal is necessarily disapproved as improper at this
common predecessor in interest of the parties together of facts pp, 34-110, rollo; and trial Judge by decision dated stage of the proceedings.
with five (5) other American citizens organized the Tiaong 27 December 1969 held for the plaintiffs Benjamin and

Page 115 of 160


CORPORATION LAW CASES MAYORDO, M.A
LLB 3

SO ORDERED. II. IN AFFIRMING - UNSUPPORTED BY ANY EVIDENCE the matter until further orders from this Court (pp. 99-100,
WHATSOEVER NOR CITATION OF ANY LAW TO JUSTIFY - THE r o l l o ) . Pr i v a t e r e s p o n d e n t s f i l e d a m o t i o n f o r
Lucena City, April 27, 1970. UNWARRANTED CONCLUSION THAT SUBJECT PROPERTIES, reconsideration of Our resolution of January 29, 1975. After
FOUND BY THE LOWER COURT AND THE COURT OF APPEALS weighing the arguments of the parties and taking note of
and so it was said defendants brought the matter first to AS ACTUALLY REGISTERED IN THE NAME OF PETITIONER Our resolution in G.R. No. L-35629 which upheld the
the Supreme Court, on mandamus on 20 May, 1970 to CORPORATION AND/OR ITS PREDECESSOR IN INTEREST, THE appointment of a receiver, We issued another resolution
compel the appeal and certiorari and prohibition to annul TIAONG MILLING AND PLANTATION COMPANY, DURING ALL dated April 11, 1975 lifting effective immediately Our
the order of 27 April, 1970 on the ground that the decision THE 50 YEARS OF ITS CORPORATE EXISTENCE "ARE ALSO previous temporary restraining order which enforced the
was "patently erroneous" p. 16, rollo; but the Supreme PROPERTIES OF THE ESTATE OF FOREST L. CEASE." earlier resolution of January 29, 1975 (pp. 140-141, rollo).
Court remanded the case to this Court of Appeals by
resolution of 27 May 1970, p. 173, and this Court of Appeals III. IN AFFIRMING THE ARBITRARY CONCLUSION OF THE 3. On February 6, 1976, private respondents filed an urgent
on 1 July 1970 p. 175 dismissed the petition so far as the LOWER COURT THAT ITS DECISION OF DECEMBER 27,1969 IS petition to restrain proceedings below in view of the
mandamus was concerned taking the view that the decision AN "INTERLUCUTORY DECISION." IN DISMISSED NG THE precipitate replacement of the court appointed receiver
sought to be appealed dated 27 December, 1969 was PETITION FOR WRIT OF MANDAMUS, AND IN AFFIRMING THE Mayor Francisco Escueta (vice Mr. Eleno M. Joyas) and the
interlocutory and not appealable but on motion for MANIFESTLY UNJUST JUDGMENT RENDERED WHICH appointment of Mr. Guillermo Lagrosa on the eve of
reconsideration of petitioners and since there was possible CONTRADICTS THE FINDINGS OF ULTIMATE FACTS THEREIN respondent Judge Maddela's retirement (p. 166, rollo). The
merit so far as its prayer for certiorari and prohibition was CONTAINED. urgent petition was denied in Our resolution of February
concerned, by resolution of the Court on 19 August, 1970, 18, 1976 (p. 176, rollo).
p. 232, the petition was permitted to go ahead in that During the period that ensued after the filing in this Court
capacity; and it is the position of petitioners that the of the respective briefs and the subsequent submission of 4. Several attempts at a compromise agreement failed to
decision of 27 December, 1969 as well as the order of 27 the case for decision, some incidents had transpired, the materialize. A Tentative Compromise Agreement dated July
April, 1970 suffered of certain fatal defects, which summary of which may be stated as follows: 30, 1975 was presented to the Court on August 6, 1976 for
respondents deny and on their part raise the preliminary the signature of the parties, but respondents
point that this Court of Appeals has no authority to give 1. Separate from this present appeal, petitioners filed a "unceremoniously" repudiated the same by leaving the
relief to petitioners because not in aid of its appellate petition for certiorari and prohibition in this Court, courtroom without the permission of the court (Court of
jurisdiction, and that the questions presented cannot be docketed as G.R. No. L-35629 (Ernesto Cease, et al. vs. First Instance of Quezon, Branch 11) as a result of which
raised for the first time before this Court of Appeals; Hon. Manolo L. Maddela, et al.) which challenged the order respondents and their counsel were cited for contempt (p.
Respondent Court of Appeals in its decision promulgated of respondent judge dated September 27, 1972 appointing 195, 197, rollo) that respondents' reason for the
December 9, 1970 dismissed the petition with costs against his Branch Clerk of Court, Mr. Eleno M. Joyas, as receiver of repudiation appears to be petitioners' failure to render an
petitioners, hence the present petition to this Court on the the properties subject of the appealed civil case, which audited account of their administration covering the period
following assignment of errors: order, petitioners saw as a virtual execution of the lower from May 31, 1961 up to January 29, 1974, plus the
court's judgment (p. 92, rollo). In Our resolution of inclusion of a provision on waiver and relinquishment by
THE COURT OF APPEALS ERRED - November 13, 1972, issued in G.R. No. L-35629, the respondents of whatever rights that may have accrued to
petition was denied since respondent judge merely their favor by virtue of the lower court's decision and the
I. IN SANCTIONING THE WRONGFUL EXERCISE OF appointed an auxilliary receiver for the preservation of the affirmative decision of the appellate court.
JURISDICTION BEYOND THE LIMITS OF AUTHORITY properties as well as for the protection of the interests of
CONFERRED BY LAW UPON THE LOWER COURT, WHEN IT all parties in Civil Case No. 6326; but at the same time, We We go now to the alleged errors committed by the
PROCEEDED TO HEAR, ADJUDGE AND ADJUDICATE - expressed Our displeasure in the appointment of the branch respondent Court of Appeals.
clerk of court or any other court personnel for that matter
(a) Special Proceedings No. 3893 for the settlement of as receiver. (p. 102, rollo). As can be gleaned from petitioners' brief and the petition
the Estate of Forrest L. Cease, simultaneously and itself, two contentions underlie the first assigned error.
concurrently with - 2. Meanwhile, sensing that the appointed receiver was First, petitioners argue that there was an irregular and
making some attempts to take possession of the properties, arbitrarte termination and dismissal of the special
(b) Civil Case No. 6326, wherein the lower Court petitioners filed in this present appeal an urgent petition to proceedings for judicial administration simultaneously
ordered Partition under Rule 69, Rules of Court - restrain proceedings in the lower court. We resolved the ordered in the lower court . s decision in Civil Case No.
THE ISSUE OF LEGAL OWNERSHIP OF THE PROPERTIES petition on January 29, 1975 by issuing a corresponding 6326 adjudicating the partition of the estate, without
COMMONLY INVOLVED IN BOTH ACTIONS HAVING BEEN temporary restraining order enjoining the court a quo from categorically, reasoning the opposition to the petition for
RAISED AT THE OUTSET BY THE TIAONG MILLING AND implementing its decision of December 27, 1969, more administration Second, that the issue of ownership had
PLANTATION COMPANY, AS THE REGISTERED OWNER OF particularly, the taking over by a receiver of the properties been raised in the lower court when Tiaong Milling asserted
SUCH PROPERTIES UNDER ACT 496. subject of the litigation, and private respondents Benjamin title over the properties registered in its corporate name
and Florence Cease from proceeding or taking any action on adverse to Forrest L. Cease or his estate, and that the said

Page 116 of 160


CORPORATION LAW CASES MAYORDO, M.A
LLB 3

issue was erroneously disposed of by the trial court in the charge the estate within the two-year period after the whose partition is sought, and claims instead, exclusive
partition proceedings when it concluded that the assets or death of Forrest L. Cease, hence, the presumption under title thereto the action becomes one for recovery of
properties of the defunct company is also the estate of the Section 1, Rule 74 that the estate is free from creditors property cognizable in the courts of ordinary jurisdiction. 2
deceased proprietor. must apply. Neither has the status of the parties as legal
heirs, much less that of respondents, been raised as an Petitioners' argument has only theoretical persuasion, to
The propriety of the dismissal and termination of the issue. Besides, extant in the records is the stipulation of say the least, rather apparent than real. It must be
special proceedings for judicial administration must be the parties to submit the pleadings and contents of the remembered that when Tiaong Milling adduced its defense
affirmed in spite of its rendition in another related case in administration proceedings for the cognizance of the trial and raised the issue of ownership, its corporate existence
view of the established jurisprudence which favors judge in adjudicating the civil case for partition already terminated through the expiration of its charter. It
partition when judicial administration become, (Respondents' Brief, p, 20, rollo). As respondents observe, is clear in Section 77 of Act No. 1459 (Corporation Law)
unnecessary. As observed by the Court of Appeals, the the parties in both cases are the same, so are the that upon the expiration of the charter period, the
dismissal at first glance is wrong, for the reason that what properties involved; that actual division is the primary corporation ceases to exist and is dissolved  ipso
was actually heard was Civil Case No. 6326. The technical objective in both actions; the theory and defense of the facto  except for purposes connected with the winding up
consistency, however, it is far less importance than the respective parties are likewise common; and that both and liquidation. The provision allows a three year, period
reason behind the doctrinal rule against placing an estate cases have been assigned to the same respondent judge. from expiration of the charter within which the entity
under administration. Judicial rulings consistently hold the We feel that the unifying effect of the foregoing gradually settles and closes its affairs, disposes and convey
view that where partition is possible, either judicial or circumstances invites the wholesome exception to the its property and to divide its capital stock, but not for the
extrajudicial, the estate should not be burdened with an structures of procedural rule, thus allowing, instead, room purpose of continuing the business for which it was
administration proceeding without good and compelling for judicial flexibility. Respondent judge's dismissal of the established. At this terminal stage of its existence, Tiaong
reason. When the estate has no creditors or pending administration proceedings then, is a judicious move, Milling may no longer persist to maintain adverse title and
obligations to be paid, the beneficiaries in interest are not appreciable in today's need for effective and speedy ownership of the corporate assets as against the
bound to submit the property to judicial administration administration of justice. There being ample reason to prospective distributees when at this time it merely holds
which is always long and costly, or to apply for the support the dismissal of the special proceedings in this the property in trust, its assertion of ownership is not only
appointment of an administrator by the court, especially appealed case, We cannot see in the records any a legal contradiction, but more so, to allow it to maintain
when judicial administration is unnecessary and compelling reason why it may not be dismissed just the adverse interest would certainly thwart the very purpose of
superfluous. Thus – same even if considered in a separate action. This is liquidation and the final distribute loll of the assets to the
inevitably certain specially when the subject property has proper, parties.
When a person dies without leaving pending obligations to already been found appropriate for partition, thus reducing
be paid, his heirs, whether of age or not, are bound to the petition for administration to a mere unnecessary We agree with the Court of Appeals in its reasoning that
submit the property to a judicial administration, which is solicitation. substance is more important than form when it sustained
always long and costly, or to apply for the appointment of the dismissal of Special Proceedings No. 3893, thus –
an administrator by the court. It has been uniformly held The second point raised by petitioners in their first assigned
that in such case the judicial administration and the error is equally untenable. In effect, petitioners argue that a) As to the dismissal of Special Proceedings No. 3893, of
appointment of an administrator are superfluous and the action for partition should not have prospered in view course, at first glance, this was wrong, for the reason that
unnecessary proceedings (Ilustre vs. Alaras Frondosa, 17 of the repudiation of the co-ownership by Tiaong Milling the case trial had been heard was Civil Case No. 6326; but
Phil., 321; Malahacan vs. Ignacio, 19 Phil, 434; Bondad vs. and Plantation Company when, as early in the trial court, it what should not be overlooked either is Chat respondent
Bondad, 34 Phil., 232; Baldemor vs. Malangyaon, 34 Phil., already asserted ownership and corporate title over the Judge was the same Judge that had before him in his own
367; Fule vs. Fule, 46 Phil., 317). Syllabus, Intestate estate properties adverse to the right of ownership of Forrest L. sala, said Special Proceedings No. 3893, p. 43 rollo, and the
of the deceased Luz Garcia. Pablo G. Utulo vs. Leona Pasion Cease or his estate. We are not unmindful of the doctrine parties to the present Civil Case No. 6326 had themselves
Viuda de Garcia, 66 Phil. 302. relied upon by petitioners in Rodriguez vs. Ravilan, 17 Phil. asked respondent Judge to take judicial notice of the same
63 wherein this Court held that in an action for partition, it and its contents page 34, rollo; it is not difficult to see that
Where the estate has no debts, recourse may be had to an is assumed that the parties by whom it is prosecuted are all when respondent Judge in par. 4 of the dispositive part of
administration proceeding only if the heirs have good co-owners or co-proprietors of the property to be divided, his decision complained of, ordered that,
reasons for not resorting to an action for partition. Where and that the question of common ownership is not to be
partition is possible, either in or out of court, the estate argued, not the fact as to whether the intended parties are 4) Special Proceedings No. 3893 for administration is
should not be burdened with an administration proceeding or are not the owners of the property in question, but only terminated and dismissed; the instant case to proceed but
without good and compelling reasons. (Intestate Estate of as to how and in what manner and proportion the said on issues of damages only and for such action inherently
Mercado vs. Magtibay, 96 Phil. 383) property of common ownership shall be distributed among essential or partition. p. 123, rollo, in truth and in fact, His
the interested parties by order of the Court. Consistent Honor was issuing that order also within Civil Case No. 632
In the records of this case, We find no indication of any with this dictum, it has been field that if any party to a suit but in connection with Special Proceedings No. 389:3: for
indebtedness of the estate. No creditor has come up to for partition denies the pro-indivisocharacter of the estate substance is more important Chan form, the contending par

Page 117 of 160


CORPORATION LAW CASES MAYORDO, M.A
LLB 3

ties in both proceedings being exactly the same, but not While the records showed that originally its incorporators this reason, it may not be used or invoked for ends
only this, let it not be forgotten that when His Honor were aliens, friends or third-parties in relation of one to subversive of the policy and purpose behind its creation
dismissed Special Proceedings No. 3893, that dismissal another, in the course of its existence, it developed into a (Emiliano Cano Enterprises, Inc. vs. CIR, L-20502, Feb. 26,
precisely was a dismissal that petitioners herein had close family corporation. The Board of Directors and 1965) or which could not have been intended by law to
themselves sought and solicited from respondent Judge as stockholders belong to one family the head of which Forrest which it owes its being McConnel vs. Court of Appeals, L-
petitioners themselves are in their present petition pp. 5-6, L. Cease always retained the majority stocks and hence the 10510, March 17, 1961, 1 SCRA 722). This is particularly
rollo; this Court must find difficulty in reconciling control and management of its affairs. In fact, during the true where the fiction is used to defeat public
petitioners' attack with the fact that it was they reconstruction of its records in 1947 before the Security convenience, justify wrong, protect fraud, defend crime
themselves that had insisted on that dismissal; on the and Exchange Commission only 9 nominal shares out of 300 (Yutivo Sons Hardware Company vs. Court of Tax Appeals,
principle that not he who is favored but he who is hurt by a appears in the name of his 3 eldest children then and L-13203, Jan. 28, 1961, 1 SCRA 160), confuse legitimate
judicial order is he only who should be heard to complain another person close to them. It is likewise noteworthy to legal or judicial issues (R. F. Sugay & Co. vs. Reyes,
and especially since extraordinary legal remedies are observe that as his children increase or perhaps become of L-20451, Dec. 28, 1964), perpetrate deception or otherwise
remedies in extermies granted to parties ' who have been age, he continued distributing his shares among them circumvent the law (Gregorio Araneta, Inc. vs. reason de
the victims not merely of errors but of grave wrongs, and it adding Florence, Teresa and Marion until at the time of his Paterno, L-2886, Aug. 22, 1952, 49 O.G. 721). This is
cannot be seen how one who got what he had asked could death only 190 were left to his name. Definitely, only the likewise true where the corporate entity is being used as an
be heard to claim that he had been the victim of a wrong, members of his family benefited from the Corporation. alter ego, adjunct, or business conduit for the sole benefit
petitioners should not now complain of an order they had of the stockholders or of another corporate entity
themselves asked in order to attack such an order The accounts of the corporation and therefore its (McConnel vs. Court of Appeals,  supra; Commissioner of
afterwards; if at all, perhaps, third parties, creditors, the operation, as well as that of the family appears to be Internal Revenue vs. Norton Harrison Co., L-7618, Aug. 31,
Bureau of Internal Revenue, might have been prejudiced, indistinguishable and apparently joined together. As 1964).
and could have had the personality to attack that dismissal admitted by the defendants (Manifestation of Compliance
of Special Proceedings No. 3893, but not petitioners herein, with Order of March 7, 1963 [Exhibit "21"] the corporation In any of these cases, the notion of corporate entity will be
and it is not now for this Court of Appeals to protect said 'never' had any account with any banking institution or if pierced or disregarded, and the corporation will be treated
third persons who have not come to the Court below or any account was carried in a bank on its behalf, it was in merely as an association of persons or, where there are two
sought to intervene herein; On the second assigned error, the name of Mr. Forrest L. Cease. In brief, the operation of corporations, they will be merged as one, the one being
petitioners argue that no evidence has been found to the Corporation is merged with those of the majority merely regarded as part or the instrumentality of the otter
support the conclusion that the registered properties of stockholders, the latter using the former as his (Koppel [Phil.] Inc. vs. Yatco, 77 Phil. 496, Yutivo Sons
Tiaong Milling are also properties of the estate of Forrest L. instrumentality and for the exclusive benefits of all his Hardware Company vs. Court of Tax Appeals, supra).
Cease; that on the contrary, said properties are registered family. From the foregoing indication, therefore, there is
under Act No. 496 in the name of Tiaong Milling as lawful truth in plaintiff's allegation that the corporation is only a So must the case at bar add to this jurisprudence. An
owner and possessor for the last 50 years of its corporate business conduit of his father and an extension of his indubitable deduction from the findings of the trial court
existence. personality, they are one and the same thing. Thus, the cannot but lead to the conclusion that the business of the
assets of the corporation are also the estate of Forrest L. corporation is largely, if not wholly, the personal venture of
We do not agree. In reposing ownership to the estate of Cease, the father of the parties herein who are all Forrest L. Cease. There is not even a shadow of a showing
Forrest L. Cease, the trial court indeed found strong legitimate children of full blood. that his children were subscribers or purchasers of the
support, one that is based on a well-entrenched principle stocks they own. Their participation as nominal
of law. In sustaining respondents' theory of "merger of A rich store of jurisprudence has established the rule known shareholders emanated solely from Forrest L. Cease's
Forrest L. Cease and The Tiaong Milling as one personality", as the doctrine of disregarding or piercing the veil of gratuitous dole out of his own shares to the benefit of his
or that "the company is only the business conduit and alter corporate fiction. Generally, a corporation is invested by children and ultimately his family.
ego of the deceased Forrest L. Cease and the registered law with a personality separate and distinct from that of Were we sustain the theory of petitioners that the trial
properties of Tiaong Milling are actually properties of the persons composing it as well as from that of any other court acted in excess of jurisdiction or abuse of discretion
Forrest L. Cease and should be divided equally, share and legal entity to which it may be related. By virtue of this amounting to lack of jurisdiction in deciding Civil Case No.
share alike among his six children, ... ", the trial court did attribute, a corporation may not, generally, be made to 6326 as a case for partition when the defendant therein,
aptly apply the familiar exception to the general rule by answer for acts or liabilities of its stockholders or those of Tiaong Milling and Plantation Company, Inc. as registered
disregarding the legal fiction of distinct and separate the legal entities to which it may be connected, and  vice owner asserted ownership of the assets and properties
corporate personality and regarding the corporation and versa.  This separate and distinct personality is, however, involved in the litigation, which theory must necessarily be
the individual member one and the same. In shredding the merely a fiction created by law for convenience and to based on the assumption that said assets and properties of
fictitious corporate veil, the trial judge narrated the promote the ends of justice (Laguna Transportation Tiaong Milling and Plantation Company, Inc. now appearing
undisputed factual premise, thus: Company vs. Social Security System, L-14606, April 28, under the name of F. L. Cease Plantation Company as
1960; La Campana Coffee Factory, Inc. vs. Kaisahan ng mga Trustee are distinct and separate from the estate of Forrest
Manggagawa sa La Campana, L-5677, May 25, 1953). For L. Cease to which petitioners and respondents as legal heirs

Page 118 of 160


CORPORATION LAW CASES MAYORDO, M.A
LLB 3

of said Forrest L. Cease are equally entitled share and abandoned by the Court in Miranda vs. Court of Appeals, 71 expressly provided and spelled out in Rule 69 of the Rules
share alike, then that legal fiction of separate corporate SCRA 295; 331-333 (June 18, 1976) wherein Mr. Justice of Court, with special reference to Sections 1, 2, 3, 6, 7
personality shall have been used to delay and ultimately Teehankee, speaking for the Court, laid down the following and 11, to wit, that there must first be a preliminar, order
deprive and defraud the respondents of their successional doctrine: for partition of the real estate (section 2) and where the
rights to the estate of their deceased father. For Tiaong parties-co-owners cannot agree, the court appointed
Milling and Plantation Company shall have been able to The Court, however, deems it proper for the guidance of commissioners make a plan of actual partition which must
extend its corporate existence beyond the period of its the bench and bar to now declare as is clearly indicated first be passed upon and accepted by the trial court and
charter which lapsed in June, 1958 under the guise and from the compelling reasons and considerations embodied in a judgment to be rendered by it (sections 6
cover of F. L, Cease Plantation Company, Inc. as Trustee hereinabove stated: and 11). In partition cases, it must be further borne in mind
which would be against the law, and as Trustee shall have that Rule 69, section 1 refers to "a person having
been able to use the assets and properties for the benefit - that the Court considers the better rule to be that stated the right to compel the partition of real estate," so that the
of the petitioners, to the great prejudice and defraudation. in H. E. Heacock Co. vs. American Trading Co., to wit, that general rule of partition that an appeal will not lie until the
of private respondents. Hence, it becomes necessary and where the primary purpose of a case is to ascertain and partition or distribution proceedings are terminated will
imperative to pierce that corporate veil. determine who between plaintiff and defendant is the true not apply where appellant claims  exclusive ownership  of
owner and entitled to the exclusive use of the disputed the whole property and denies the adverse party's right to
Under the third assigned error, petitioners claim that the property, "the judgment . . . rendered by the lower court any partition, as was the ruling in  Villanueva vs.
decision of the lower court in the partition case is not [is] a judgment on the  merits  as to those questions, and Capistrano  and  Africa vs .Africa, supra, Fuentebellas
interlocutory but rather final for it consists of final and [that] the order of the court for an  accounting  was based express rehearsal of these cases must likewise be deemed
determinative dispositions of the contentions of the upon, and is incidental to the judgment on the merits. That now also abandoned in view of the Court's expressed
parties. We find no merit in petitioners' stand. is to say, that the judgment . . . [is] a final judgment ... preference for the rationale of the Heacock case.
that in this kind of a case an accounting is a mere incident
Under the 1961 pronouncement and ruling of the Supreme to the judgment; that an appeal lies from the rendition of The Court's considered opinion is that imperative
Court in Vda. de Zaldarriaga vs. Enriquez, 1 SCRA 1188 (and the judgment as rendered ... "(as is widely held by a great considerations of public policy and of sound practice in the
the sequel case of  Vda. de Zaldarriaga vs. Zaldarriaga, 2 number of judges and members of the bar, as shown by the courts and adherence to the  constitutional mandate  of
SCRA 356), the lower court's dismissal of petitioners' cases so decided and filed and still pending with the Court) simplified, just, speedy and inexpensive determination of
proposed appeal from its December 27, 1969 judgment as for the fundamental reasons therein stated that "this is every action call for considering such judgments for
affirmed by the Court of Appeals on the ground of more in harmony with the administration of justice and the recovery of property with  accounting as final  judgments
prematurity in that the judgment was not final but spirit and intent of the [Rules]. If on appeal the judgment which are duly  appealable  (and would therefore become
interlocutory was in order. As was said in said case: of the lower court is affirmed, it would not in the least final and executory if not appealed within the reglementary
work an injustice to any of the legal rights of [appellee]. period) with the  accounting  as a mere incident of the
It is true that in  Africa vs. Africa, 42 Phil. 934 and other On the other hand, if for any reason this court should judgment to be rendered during the course of the appeal as
cases it was held - contrary to the rule laid down in Ron vs. reverse the judgment of the lower court, the accounting provided in Rule 39, section 4 or to be implemented at the
Mojica, 8 Phil. 328; Rodriguez vs. Ravilan, 17 Phil. 63 - that would be a waste of time and money, and might work a execution stage upon final affirmance on appeal of the
in a partition case where defendant relies on the defense material injury to the [appellant]; and - that accordingly, judgment (as in Court of Industrial Relations unfair labor
of exclusive ownership, the action becomes one for title the contrary ruling in  Fuentebella vs. Carrascoso  which practice cases ordering the reinstatement of the worker
and the decision or order directing partition is final, but expressly reversed the Heacock case and a line of similar with accounting, computation and payment of his
the ruling to this effect has been expressly reversed in the decisions and ruled that such a decision for recovery of backwages less earnings elsewhere during his layoff) and
Fuentebella case which, in our opinion, expresses the property with accounting "is not final but merely that the only reason given in Fuentebelia for the contrary
correct view, considering that a decision or order directing interlocutory and therefore not appealable" and subsequent ruling, viz, "the general harm that would follow from
partition is not final because it leaves something more to cases adhering to the same must be now in turn abandoned throwing the door open to multiplicity of appeals in a single
be done in the trial court for the complete disposition of and set aside. case" of lesser import and consequence. (Emphasis copied).
the case, namely, the appointment of commissioners, the
proceedings to be had before them, the submission of their Fuentebella adopted instead the opposite line of conflicting The miranda ruling has since then been applied as the new
report which, according to law, must be set for hearing. In decisions mostly in partition proceedings and exemplified rule by a unanimous Court in  Valdez vs. Bagasao, 82 SCRA
fact, it is only after said hearing that the court may render by  Ron vs. Mojica  8 Phil. 928 (under the old Code of Civil 22 (March 8, 1978).
a final judgment finally disposing of the action (Rule 71, Procedure) that an order for partition of real property is
section 7, Rules of Court). (1 SCRA at page 1193). not final and appealable until after the actual partition of If there were a valid genuine claim of Exclusive ownership
the property as reported by the court appointed of the inherited properties on the part of petitioners to
It should be noted, however, that the said ruling in commissioners and approved by the court in respondents' action for partition, then under the Miranda
Zaldarriaga as based on  Fuentebella vs. Carrascoso, XIV its  judgment  accepting the report. lt must be especially ruling, petitioners would be sustained, for as expressly held
Lawyers Journal 305 (May 27, 1942), has been expressly noted that such rule governing partitions is now so therein " the general rule of partition that an appeal will

Page 119 of 160


CORPORATION LAW CASES MAYORDO, M.A
LLB 3

not lie until the partition or distribution proceedings are WHEREFORE, IN VIEW OF THE FOREGOING, the judgment Valenzuela, Metro Manila (TCT No. 4273) for 2,500 shares of
terminated will not apply where appellant claims exclusive appealed from is hereby AFFIRMED with costs against the stock of defendant corporation with a total value of
ownership of the whole property and denies the adverse petitioners. P1,500,000.00 (Exhs. C to C-5, inclusive) (pp. 44-45, Rollo)
party's right to any partition." SO ORDERED. On the ground that it was not given the first option to buy
the leased property pursuant to the proviso in the lease
But this question has now been rendered moot and G.R. No. L-69259 January 26, 1988 agreement, respondent Hydro Pipes Philippines, Inc., filed
academic for the very issue of exclusive ownership claimed DELPHER TRADES CORPORATION, and DELPHIN an amended complaint for reconveyance of Lot. No. 1095 in
by petitioners to deny and defeat respondents' right to PACHECO, petitioners,  vs. INTERMEDIATE APPELLATE its favor under conditions similar to those whereby Delpher
partition - which is the very core of their rejected appeal - COURT and HYDRO PIPES PHILIPPINES, INC., respondents. Trades Corporation acquired the property from Pelagia
has been squarely resolved herein against them, as if the Pacheco and Delphin Pacheco.
appeal had been given due course. The Court has herein GUTIERREZ, JR., J.:
expressly sustained the trial court's findings, as affirmed by The petitioners question the decision of the Intermediate After trial, the Court of First Instance of Bulacan ruled in
the Court of Appeals, that the assets or properties of the Appellate Court which sustained the private respondent's favor of the plaintiff. The dispositive portion of the
defunct company constitute the estate of the deceased contention that the deed of exchange whereby Delfin decision reads:
proprietor (supra at page 7) and the defunct company's Pacheco and Pelagia Pacheco conveyed a parcel of land to ACCORDINGLY, the judgment is hereby rendered declaring
assertion of ownership of the properties is a legal Delpher Trades Corporation in exchange for 2,500 shares of the valid existence of the plaintiffs preferential right to
contradiction and would but thwart the liquidation and stock was actually a deed of sale which violated a right of acquire the subject property (right of first refusal) and
final distribution and partition of the properties among the first refusal under a lease contract. ordering the defendants and all persons deriving rights
parties hereof as children of their deceased father Forrest therefrom to convey the said property to plaintiff who may
L. Cease. There is therefore no further hindrance to effect Briefly, the facts of the case are summarized as follows: offer to acquire the same at the rate of P14.00 per square
the partition of the properties among the parties in In 1974, Delfin Pacheco and his sister, Pelagia Pacheco, meter, more or less, for Lot 1095 whose area is 27,169
implementation of the appealed judgment. were the owners of 27,169 square meters of real estate square meters only. Without pronouncement as to
Identified as Lot. No. 1095, Malinta Estate, in the attorney's fees and costs. (Appendix I; Rec., pp. 246- 247).
One last consideration. Parties are brothers and sisters, Municipality of Polo (now Valenzuela), Province of Bulacan (Appellant's Brief, pp. 1-2; p. 134, Rollo)
legal heirs of their deceased father, Forrest L. Cease. By all (now Metro Manila) which is covered by Transfer Certificate
rights in law and jurisprudence, each is entitled to share of Title No. T-4240 of the Bulacan land registry. The lower court's decision was affirmed on appeal by the
and share alike in the estate, which the trial court Intermediate Appellate Court.
correctly ordained and sustained by the appellate court. On April 3, 1974, the said co-owners leased to Construction
Almost 20 years have lapsed since the filing of Special Components International Inc. the same property and The defendants-appellants, now the petitioners, filed a
Proceedings No. 3893 for the administration of the Estate providing that during the existence or after the term of this petition for certiorari to review the appellate court's
of Forrest L. Cease and Civil Case No. 6326 for liquidation lease the lessor should he decide to sell the property decision.
and partition of the assets of the defunct Tiaong Milling and leased shall first offer the same to the lessee and the letter We initially denied the petition but upon motion for
Plantation Co., Inc. A succession of receivers were has the priority to buy under similar conditions (Exhibits A reconsideration, we set aside the resolution denying the
appointed by the court to take, keep in possession, to A-5) petition and gave it due course.
preserve and manage properties of the corporation which
at one time showed an income of P386,152.90 and expenses On August 3, 1974, lessee Construction Components The petitioners allege that:
of P308,405.01 for the period covering January 1, 1960 to International, Inc. assigned its rights and obligations under
August 31, 1967 as per Summary of Operations of the contract of lease in favor of Hydro Pipes Philippines, The denial of the petition will work great injustice to the
Commissioner for Finance appointed by the Court (Brief for Inc. with the signed conformity and consent of lessors petitioners, in that:
Respondents, p. 38). In the meantime, ejectment cases Delfin Pacheco and Pelagia Pacheco (Exhs. B to B-6
were filed by and against the heirs in connection with the inclusive) 1. Respondent Hydro Pipes Philippines, Inc, ("private
properties involved, aggravating the already strained respondent") will acquire from petitioners a parcel
relations of the parties. A prudent and practical realization The contract of lease, as well as the assignment of lease of industrial land consisting of 27,169 square meters or 2.7
of these circumstances ought and must constrain the were annotated at he back of the title, as per stipulation of hectares (located right after the Valenzuela, Bulacan exit
parties to give each one his due in law and with fairness the parties (Exhs. A to D-3 inclusive) of the toll expressway) for only  P14/sq. meter, or a total
and dispatch that their basic rights be enjoyed. And by of  P380,366, although the prevailing value thereof is
remanding this case to the court  a quo  for the actual On January 3, 1976, a deed of exchange was executed approximately P300/sq. meter or P8.1 Million;
partition of the properties, the substantial rights of between lessors Delfin and Pelagia Pacheco and defendant
everyone of the heirs have not been impaired, for in fact, Delpher Trades Corporation whereby the former conveyed 2. Private respondent is allowed to exercise its right of first
they have been preserved and maintained. to the latter the leased property (TCT No.T-4240) together refusal even if there is no "sale" or transfer of actual
with another parcel of land also located in Malinta Estate, ownership interests by petitioners to third parties; and

Page 120 of 160


CORPORATION LAW CASES MAYORDO, M.A
LLB 3

corporation and the co-owners should be deemed to be the aliquot sharer in the assets of the corporation. But this
3. Assuming  arguendo  that there has been a transfer of same, there being in substance and in effect an Identity of character of proportionate interest is not hidden beneath a
actual ownership interests, private respondent will acquire interest." (p. 254, Rollo) false appearance of a given sum in money, as in the case of
the land  not  under "similar conditions" by which it was par value shares. The capital stock of a corporation issuing
transferred to petitioner Delpher Trades Corporation, as The petitioners maintain that the Pachecos did not sell the only no-par value shares is not set forth by a stated amount
provided in the same contractual provision invoked by property. They argue that there was no sale and that they of money, but instead is expressed to be divided into a
private respondent. (pp. 251-252, Rollo) exchanged the land for shares of stocks in their own stated number of shares, such as, 1,000 shares. This
corporation. "Hence, such transfer is not within the letter, indicates that a shareholder of 100 such shares is an aliquot
The resolution of the case hinges on whether or not the or even spirit of the contract. There is a sale when sharer in the assets of the corporation, no matter what
"Deed of Exchange" of the properties executed by the ownership is transferred for a price certain in money or its value they may have, to the extent of 100/1,000 or 1/10.
Pachecos on the one hand and the Delpher Trades equivalent (Art. 1468, Civil Code) while there is a barter or Thus, by removing the par value of shares, the attention of
Corporation on the other was meant to be a contract of exchange when one thing is given in consideration of persons interested in the financial condition of a
sale which, in effect, prejudiced the private respondent's another thing (Art. 1638, Civil Code)." (pp. 254-255, Rollo) corporation is focused upon the value of assets and the
right of first refusal over the leased property included in amount of its debts. (Agbayani, Commentaries and
the "deed of exchange." On the other hand, the private respondent argues that Jurisprudence on the Commercial Laws of the Philippines,
Delpher Trades Corporation is a corporate entity separate Vol. III, 1980 Edition, p. 107).
Eduardo Neria, a certified public accountant and son-in-law and distinct from the Pachecos. Thus, it contends that it
of the late Pelagia Pacheco testified that Delpher Trades cannot be said that Delpher Trades Corporation is the Moreover, there was no attempt to state the true or current
Corporation is a family corporation; that the corporation Pacheco's same alter ego or conduit; that petitioner Delfin market value of the real estate. Land valued at P300.00 a
was organized by the children of the two spouses (spouses Pacheco, having treated Delpher Trades Corporation as such square meter was turned over to the family's corporation
Pelagia Pacheco and Benjamin Hernandez and spouses a separate and distinct corporate entity, is not a party who for only P14.00 a square meter.
Delfin Pacheco and Pilar Angeles) who owned in common may allege that this separate corporate existence should be
the parcel of land leased to Hydro Pipes Philippines in order disregarded. It maintains that there was actual transfer of It is to be stressed that by their ownership of the 2,500 no
to perpetuate their control over the property through the ownership interests over the leased property when the par shares of stock, the Pachecos have control of the
corporation and to avoid taxes; that in order to accomplish same was transferred to Delpher Trades Corporation in corporation. Their equity capital is 55% as against 45% of
this end, two pieces of real estate, including Lot No. 1095 exchange for the latter's shares of stock. the other stockholders, who also belong to the same family
which had been leased to Hydro Pipes Philippines, were group.
transferred to the corporation; that the leased property We rule for the petitioners.
was transferred to the corporation by virtue of a deed of In effect, the Delpher Trades Corporation is a business
exchange of property; that in exchange for these After incorporation, one becomes a stockholder of a conduit of the Pachecos. What they really did was to invest
properties, Pelagia and Delfin acquired 2,500 unissued no corporation by subscription or by purchasing stock directly their properties and change the nature of their ownership
par value shares of stock which are equivalent to a 55% from the corporation or from individual owners thereof from unincorporated to incorporated form by organizing
majority in the corporation because the other owners only (Salmon, Dexter & Co. v. Unson, 47 Phil, 649, citing Bole v. Delpher Trades Corporation to take control of their
owned 2,000 shares; and that at the time of incorporation, Fulton [1912], 233 Pa., 609). In the case at bar, in exchange properties and at the same time save on inheritance taxes.
he knew all about the contract of lease of Lot. No. 1095 to for their properties, the Pachecos acquired 2,500 original
Hydro Pipes Philippines. In the petitioners' motion for unissued no par value shares of stocks of the Delpher Trades As explained by Eduardo Neria:
reconsideration, they refer to this scheme as "estate Corporation. Consequently, the Pachecos became xxx xxx xxx
planning." (p. 252, Rollo) stockholders of the corporation by subscription "The ATTY. LINSANGAN:
essence of the stock subscription is an agreement to take Q Mr. Neria, from the point of view of taxation, is there any
Under this factual backdrop, the petitioners contend that and pay for original unissued shares of a corporation, benefit to the spouses Hernandez and Pacheco in
there was actually no transfer of ownership of the subject formed or to be formed." (Rohrlich 243, cited in Agbayani, connection with their execution of a deed of exchange on
parcel of land since the Pachecos remained in control of Commentaries and Jurisprudence on the Commercial Laws the properties for no par value shares of the defendant
the property. Thus, the petitioners allege: "Considering that of the Philippines, Vol. III, 1980 Edition, p. 430) It is corporation?
the beneficial ownership and control of petitioner significant that the Pachecos took no par value shares in A Yes, sir.
corporation remained in the hands of the original co- exchange for their properties.
owners, there was no transfer of actual ownership interests COURT:
over the land when the same was transferred to petitioner A no-par value share does not purport to represent any Q What do you mean by "point of view"?
corporation in exchange for the latter's shares of stock. The stated proportionate interest in the capital stock measured A To take advantage for both spouses and corporation in
transfer of ownership, if anything, was merely in form but by value, but only an aliquot part of the whole number of entering in the deed of exchange.
not in substance. In reality, petitioner corporation is a mere such shares of the issuing corporation. The holder of no-par
alter ego or conduit of the Pacheco co-owners; hence the shares may see from the certificate itself that he is only an ATTY. LINSANGAN:

Page 121 of 160


CORPORATION LAW CASES MAYORDO, M.A
LLB 3

Q (What do you mean by "point of view"?) What are these A The property is not subjected to taxes on succession as The sole question here is whether defendant, Southern
benefits to the spouses of this deed of exchange? the corporation does not die. Railway Company (hereinafter referred to as Southern),
A Continuous control of the property, tax exemption which acquired the entire capital stock of Lenoir Car Works
benefits, and other inherent benefits in a corporation. Q So the benefit you are talking about are inheritance (referred to hereinafter as Lenoir) in 1904, the year of the
taxes? latter's organization, so completely dominated Lenoir that
Q What are these advantages to the said spouses from the A Yes, sir. (pp. 3-5, tsn., December 15, 1981) the latter was but an adjunct, or instrumentality, of
point of view of taxation in entering in the deed of Southern. If it did, then the complainant would, for the
exchange? The records do not point to anything wrong or purpose of this case, be an employee of Southern and
A Having fulfilled the conditions in the income tax law, objectionable about this "estate planning" scheme resorted would be entitled to recover under the Federal Employers'
providing for tax free exchange of property, they were able to by the Pachecos. "The legal right of a taxpayer to Liability Act, 45 U.S.C.A. § 51 et seq. Because there is a
to execute the deed of exchange free from income tax and decrease the amount of what otherwise could be his taxes real question that Lenoir is but an instrumentality of
acquire a corporation. or altogether avoid them, by means which the law permits, Southern, the plaintiff in order to protect his rights, has
cannot be doubted." (Liddell & Co., Inc. v.The collector of also brought suit in the State courts against Lenoir for
Q What provision in the income tax law are you referring Internal Revenue, 2 SCRA 632 citing Gregory v. Helvering, Workmen's Compensation.
to? 293 U.S. 465, 7 L. ed. 596).
A I refer to Section 35 of the National Internal Revenue The general rule is that stock ownership alone by one
Code under par. C-sub-par. (2) Exceptions regarding the The "Deed of Exchange" of property between the Pachecos corporation of the stock of another does not thereby render
provision which I quote: "No gain or loss shall also be and Delpher Trades Corporation cannot be considered a the dominant corporation liable for the torts of the
recognized if a person exchanges his property for stock in a contract of sale. There was no transfer of actual ownership subsidiary unless the separate corporate existence of the
corporation of which as a result of such exchange said interests by the Pachecos to a third party. The Pacheco subsidiary is a mere sham, or unless the control of the
person alone or together with others not exceeding four family merely changed their ownership from one form to subsidiary is such that it is but an instrumentality or
persons gains control of said corporation." another. The ownership remained in the same hands. adjunct of the dominant corporation. Sheridan v. Pan-
Hence, the private respondent has no basis for its claim of American Refining Co., D.C.N.Y. 123 F. Supp. 81. The
Q Did you explain to the spouses this benefit at the time a light of first refusal under the lease contract. general rule is also stated in an Annotation appearing in 50
you executed the deed of exchange? A.L.R. 611. See also Kentucky Electric Power Co. v. Norton
A Yes, sir WHEREFORE, the instant petition is hereby GRANTED, The Coal Mining Co., 6 Cir., 93 F.2d 923, 926. Whether the
questioned decision and resolution of the then subsidiary is an instrumentality of the owner corporation is
Q You also, testified during the last hearing that the Intermediate Appellate Court are REVERSED and SET ASIDE. a question to be determined from all the surrounding
decision to have no par value share in the defendant The amended complaint in Civil Case No. 885-V-79 of the circumstances. Fletcher Cyclopedia Corporations, Sec. 43
corporation was for the purpose of flexibility. Can you then Court of First Instance of Bulacan is DISMISSED. No (p. 157), Sec. 6222; Berkey v. Third Ave. R. Co., 244 N.Y.
explain flexibility in connection with the ownership of the costs. 84, 155 N.E. 58, 50 A.L.R. 599 (opinion by Cardozo); Costan
property in question? SO ORDERED. v. Manila Electric Co., 2 Cir., 24 F.2d 383, 384. Items to be
A There is flexibility in using no par value shares as the considered in reaching such a determination are common
value is determined by the board of directors in increasing 173 F. Supp. 915 (1959) directorships, common officers, common fiscal set up, etc.
capitalization. The board can fix the value of the shares W. H. GARRETT v. SOUTHERN RAILWAY COMPANY. The plaintiff relies upon the following circumstances:
equivalent to the capital requirements of the corporation.
Civ. A. No. 3465. That all directors and officers of Lenoir are employees of
Q Now also from the point of taxation, is there any United States District Court E. D. Tennessee, N. D. Southern and live in Washington; that Southern owns all
flexibility in the holding by the corporation of the property May 8, 1959. the *917 stock of Lenoir except five qualifying shares; that
in question? *916 Hodges & Doughty, Knoxville, Tenn., for plaintiff. between 1942-1957 Lenoir sold to Southern, or its
A Yes, since a corporation does not die it can continue to Key & Lee, Knoxville, Tenn., for defendant. affiliates, over 30 million dollars worth of its products
hold on to the property indefinitely for a period of at least while selling approximately four and a half million to
50 years. On the other hand, if the property is held by the ROBERT L. TAYLOR, District Judge. outside purchasers; that all profits of Lenoir went to
spouse the property will be tied up in succession Southern; that all claims of Lenoir employees for accidents
proceedings and the consequential payments of estate and The plaintiff in this case was employed as a wheel moulder are handled by the claim's office of Southern; that all
inheritance taxes when an owner dies. by Lenoir Car Works, a Tennessee corporation. He claims litigation against Lenoir is handled by Southern's attorneys;
injuries from silicosis contracted from silica dust that general accounting of Lenoir is handled in Washington
Q Now what advantage is this continuity in relation to permeating the foundry. But the questions whether he had by personnel of Southern; that the Railroad Retirement
ownership by a particular person of certain properties in silicosis, and the amount of injury, are not reached in this Board decided that employees of Lenoir were entitled to
respect to taxation? phase of the case. benefits under the Railroad Retirement Act, 45 U.S.C.A. §
215 et seq., because of the relationship between Lenoir

Page 122 of 160


CORPORATION LAW CASES MAYORDO, M.A
LLB 3

and Southern; that Lenoir has the power of eminent maintains a separate bank account and has never duties as a wheel moulder served to further interstate
domain; and that on May 13, 1904, the Board of Directors intermingled its funds with those of Southern; that the commerce. The Supreme Court decided he was entitled to
of Southern authorized the purchase of the entire capital companies keep separate books and Lenoir pays its own the benefit of the Federal Employers' Liability Act.
stock of Lenoir. taxes; that Southern issues no passes to the manager or any
other employee of Lenoir; that Lenoir owns no track or But before the Gileo decision applies here we must first
Our question is whether these factors would establish rolling stock, publishes no tariffs, files no reports with the determine whether Lenoir was operated so tightly by
Lenoir as the mere creature of Southern. The defendant Interstate Commerce Commission and uses no facilities or Southern that it was an agency or instrumentality of
railroad says that these matters are not controlling and to property jointly with Southern; that claims of employees Southern. In our opinion it was not. Policy decisions and
sustain its position emphasizes the following facts: are investigated and settled by the Claim Department of pricings remained in the hands of Marius. Certain
the Southern Railway System, which likewise defends accounting and claims work was done in Washington to
That although Lenoir sells the majority of its products to claims litigation; that the general accounting of Lenoir is eliminate duplication. But there is no evidence that policy
Southern or its affiliates, it does not sell to them handled *918by the Accounting Department of the Southern decisions of Lenoir were made or dictated by Southern.
exclusively and in the fifteen-year period prior to the suit Railway System, but local accounting is handled by Lenoir Marius operated the business as a going concern in the
sold twelve percent to other customers; that Lenoir employees at Lenoir City; that salaries of employees of the fields it was equipped to handle. Where Southern, or the
maintains its offices and business in Lenoir City, Tennessee, System doing this general work are paid by Southern, but Southern Railway System, did work for Lenoir there was
although it concedes that the corporate and accounting reimbursement of its fair proportionate part thereof is careful accounting and adjustment of costs at the end of
offices are in Washington in a building owned by Southern; made by Lenoir and other members of the System at each month. The facts do not reveal the intimacy and
that the management of Lenoir is vested in a manager, monthly intervals, and that the share of Lenoir is $2,000 inseparability of control which would lead the Court to hold
Henry Marius, who has held the position continuously since monthly; that Lenoir makes separate collective bargaining that Southern and Lenoir were one and the same. There is
July 1, 1945, is paid by Lenoir, and although he holds and agreements with its employees and there is no interchange no such control here as in Costan v. Manila Electric Co.,
votes the proxy of Southern at the annual stockholders of seniority between operations of Lenoir and the railroad, supra.
meeting, he has no other connection with Southern; that or vice versa; that Lenoir has its own legal counsel in Lenoir
never has any individual served at the same time as a City, selected by its manager in addition to the use it makes Lenoir was not in the words of the Act a "common carrier by
Director of Lenoir and Southern; that the work of Lenoir is of the Legal Department of the System. railroad." It was not performing what have been called non-
a specialty and in the last 20 years no official or supervising delegable duties of the railroad. It was not an operator of a
employee of Southern has by training, or experience, been Counsel have been commendably frank and candid in the terminal, performed no switching or transportation
in a position to direct or supervise the operation of such a pre-trial conference, and in answering interrogatories and functions at all. It was a manufacturer and plaintiff was
business; that Southern has not purchased all its wheels, filing exhibits. The factual data before the Court is not in one of its employees. It was hence not an "agent" of
steel and brass castings from Lenoir, but has bought conflict. The Court's responsibility is to determine whether Southern in the sense used in some of the cases cited by
substantial amounts from others; that Southern has Lenoir is operated as a sham for Southern, or as the the plaintiff, since it performed no common carrier
followed the custom in the trade as to price quotations and instrumentality, or as an adjunct of its operation. operations. Chicago, M. & St. P. Ry. Co. v. Minnesota Civic &
supplying scrap material in its dealings with Lenoir, and all Commerce Ass'n,  247 U.S. 490, 38 S. Ct. 553, 62 L. Ed.
sales to Southern have been upon the basis of a bid price or The facts set forth above outline with singular sharpness 1229; Davis v. Alexander,  269 U.S. 114, 46 S. Ct. 34, 70 L.
negotiated price; that in every case Southern obtains bids the relation between the two companies. The Court finds Ed. 186; Sinkler v. Missouri Pacific R. Co., 356 U.S. 326, 78
from several manufacturers and purchases from the lowest the existence of two distinct operations. There is no S. Ct. 758, 2 L. Ed. 2d 799.
bidder; that the manager of Lenoir establishes the prices of evidence that Southern dictated the management of Lenoir.
all prospective purchases, including that of Southern; that In fact, the evidence indicates that Henry Marius was in full Defendant manufactured car wheels, bearings of various
the manager expects to make a profit and is not informed control of the operation. He established prices. He handled kinds and items of a related nature. Southern could
by Southern of bids of its competitors even when the prices all negotiations in collective bargaining agreements. Lenoir produce such items for its own use and if it did  *919  so
are negotiated; that bids are revised sometimes, but never paid local taxes, had local counsel, maintained Workmen's would be liable in tort under the F.E. L.A. for accidents
with knowledge of the bids of competitors; that all sales Compensation. There is no evidence that Lenoir was run occurring in the activity. But Southern did not manufacture
are the result of the business judgment of the Manager and solely for the benefit of Southern. In fact a substantial part these things in its own plants. Although necessary to keep
not the result of threats, persuasion or intimidation by of its requirements in the field of operation of Lenior were its rolling stock in good repair, the items could be
Southern; that at no time has any individual occupied the bought elsewhere. Lenoir sold substantial quantities to purchased anywhere. In fact the evidence indicates that
same official position in both companies with the exception other companies. It operated no rolling stock and had substantial purchases of these products were made
of corporate and financial officials; that Lenior has since nothing to do with the transportation business. elsewhere. If purchased of a completely independent
1919 been a duly qualified employer under the Tennessee company, no claim could be made that such company was
Workmen's Compensation Law, T.C.A. § 50-901 et seq., and The situation here is not like that in Southern Pacific Co. v. an instrumentality of Southern.
with the exception of this and several similar suits has Gileo, 351 U.S. 493, 76 S. Ct. 952, 100 L. Ed. 1357, where
settled all claims made under and covered by the the railroad itself operated a wheel foundry. The only The question only arises because of the ownership of Lenoir
Tennessee Workmen's Compensation Law; that Lenoir question there as to employee Aranda was whether his by Southern and because of the other relationships listed

Page 123 of 160


CORPORATION LAW CASES MAYORDO, M.A
LLB 3

above. We come back to the question whether the control of fraud or wrongdoing on the part of the parent  
was such as to make Lenoir a mere adjunct of Southern. We corporation to the detriment of the susbidiary and third "`(h) In the papers of the parent corporation or in the
repeat again that it was not. Friedman v. Vandalia R. Co., 8 persons in their relations with the subsidiary. statements of its officers, the subsidiary is described as a
Cir., 254 F. 292, 294; Atlantic Coast Line R. Co. v. Shields, 5   department or division of the parent corporation, or its
Cir., 220 F.2d 242, 246. "Sections 5 and 6 of Powell on Parent and Subsidiary business or financial responsibility is referred to as the
Corporations, in part, read: parent corporation's own.
In a case not involving a tort the Court of Appeals for the    
Sixth Circuit has set down the following guide lines. "`The Instrumentality Rule, in its shortest form, may now "`(i) The parent corporation uses the property of the
Kentucky Electric Power Co. v. Norton Coal Mining Co., be stated: subsidiary as its own.
supra:    
  "`So far as the question of control alone is concerned, the "`(j) The directors or executives of the subsidiary do not
"On the other hand, it is likewise well settled that a parent corporation will be responsible for the obligations of act independently in the interest of the subsidiary but take
corporation is ordinarily an entity, separate and apart from its subsidiary when its control has been exercised to their orders from the parent corporation in the latter's
its stockholders, and mere ownership of all the stock of one such *920 a degree that the subsidiary has become its mere interest.
corporation by another, and the identity of officers of one instrumentality.  
with officers of another, are not alone sufficient to create   "`(k) The formal legal requirements of the subsidiary are
identity of corporate interest between the two companies "`The Instrumentality Rule is recognized in all jurisdictions not observed.'"
or to create the relation of principal and agent or to create in this country and our problem therefore is to determine
a representative or fiduciary relationship between the two. the circumstances which render the subsidiary an In the case at bar only two of the eleven listed indicia
If such stock ownership and potential control be resorted to "instrumentality" within the meaning of the decisions. This occur, namely, the ownership of most of the capital stock of
only for the purpose of normally participating in the affairs is primarily a question of fact and of degree.' Lenoir by Southern, and possibly subscription by Southern
of the subsidiary corporation in a manner usual to   to the capital stock of Lenoir. In our opinion the principles
stockholders and not for the purpose of taking some unfair "`The circumstances rendering the subsidiary an of the Kentucky Electric Power case apply here, and when
advantage of the subsidiary or using it as a mere adjunct to instrumentality. It is manifestly impossible to catalogue the applied we conclude that the control of Southern over
the main corporation or as a subterfuge to justify infinite variations of fact that can arise but there are Lenoir was not such as to constitute the latter an adjunct
wrongdoing, their identity as separate corporations will not certain common circumstances which are important and of Southern. The complaint must be dismissed.
be disregarded but their respective rights when dealing which, if present in the proper combination, are
with each other in respect to their separate property will controlling. G.R. No. 151438 July 15, 2005
be recognized and maintained. The extent of stock   JARDINE DAVIES, INC., Petitioners, vs. JRB REALTY, INC.,
ownership and mere potential control of one company over "These are as follows: Respondent.
another has never been regarded as the determining factor   DECISION
in the consideration of such cases. Something must be "`(a) The parent corporation owns all or most of the capital CALLEJO, SR., J.:
disclosed to indicate the exercise of undue domination or stock of the subsidiary.  
influence resulting in an infringement upon the rights of   Before us is a petition for review of the Decision[1] of the
the subservient corporation for the benefit of the dominant "`(b) The parent and subsidiary corporations have common Court of Appeals (CA) in CA-G.R. CV No. 54201 affirming in
one. Otherwise, the rights of the separate corporations in directors or officers. toto that of the Regional Trial Court (RTC) in Civil Case No.
respect to their corporate property must be governed by   90-237 for specific performance; and the Resolution dated
the rules applicable in ordinary cases." [93 F.2d 926.] "`(c) The parent corporation finances the subsidiary. January 11, 2002 denying the motion for reconsideration
  thereof.
This excerpt from the Kentucky Electric Power case was "`(d) The parent corporation subscribes to all the capital 

quoted with approval in a footnote to the case of Taylor v. stock of the subsidiary or otherwise causes its The facts are as follows:
Standard Gas & Electric Co., 10 Cir., 96 F.2d 693, 704. In incorporation.  
that case the Court in considering the indicia of control   In 1979-1980, respondent JRB Realty, Inc. built a nine-
quoted at length from Powell on Parent and Subsidiary "`(e) The subsidiary has grossly inadequate capital. storey building, named Blanco Center, on its parcel of land
Corporations:   located at 119 Alfaro St., Salcedo Village, Makati City. An
  "`(f) The parent corporation pays the salaries and other air conditioning system was needed for the Blanco Law Firm
"There is respectable authority for the proposition that to expenses or losses of the subsidiary. housed at the second floor of the building. On March 13,
justify the application of the instrumentality rule between   1980, the respondents Executive Vice-President, Jose R.
parent and subsidiary corporation, there must be present in "`(g) The subsidiary has substantially no business except Blanco, accepted the contract quotation of Mr. A.G.
addition to the elements of control through stock with the parent corporation or no assets except those Morrison, President of Aircon and Refrigeration Industries,
ownership and common directorates and officers, elements conveyed to it by the parent corporation. Inc. (Aircon), for two (2) sets of Fedders Adaptomatic

Page 124 of 160


CORPORATION LAW CASES MAYORDO, M.A
LLB 3

30,000 kcal (Code: 10-TR) air conditioning equipment with from 7th  March 1987 to 11th  January 1990, with legal 2000, the CA affirmed the trial courts ruling in toto; hence,
a net total selling price of  P99,586.00.[2]  Thereafter, two interest thereon from the filing of this Complaint until fully this petition.
(2) brand new packaged air conditioners of 10 tons capacity reimbursed, but also like unsaved electricity costs and like  
each to deliver 30,000 kcal or 120,000 BTUH[3]  were repair costs therefrom until Prayer No. 1 above shall have The petitioner raises the following assignment of errors:
installed by Aircon. When the units with rotary compressors been complied with;
were installed, they could not deliver the desired cooling   I. THE COURT OF APPEALS ERRED IN HOLDING
temperature. Despite several adjustments and corrective 3. Ordering defendants to jointly and severally pay JARDINE LIABLE FOR THE ALLEGED CONTRACTUAL BREACH
measures, the respondent conceded that Fedders Air plaintiffs  P150,000.00 attorneys fees and other costs of OF AIRCON SOLELY BECAUSE THE LATTER WAS FORMERLY
Conditioning USAs technology for rotary compressors for big litigation, as well as exemplary damages in an amount not JARDINES SUBSIDIARY.
capacity conditioners like those installed at the Blanco less than or equal to Prayer 2 above; and  
Center had not yet been perfected. The parties thereby   II. ASSUMING  ARGUENDO  THAT AIRCON MAY BE
agreed to replace the units with reciprocating/semi- 4. Granting plaintiff such other and further relief as shall CONSIDERED AS JARDINES MERE ALTER EGO, THE COURT OF
hermetic compressors instead. In a Letter dated March 26, be just and equitable in the premises.[7] APPEALS ERRED IN NOT DECLARING AIRCONS OBLIGATION TO
1981,[4] Aircon stated that it would be replacing the units   DELIVER THE TWO (2) AIRCONDITIONING UNITS TO JRB AS
currently installed with new ones using rotary compressors, Of the four defendants, only the petitioner filed its Answer. HAVING BEEN SUBSTANTIALLY COMPLIED WITH IN GOOD
at the earliest possible time. Regrettably, however, it could The court did not acquire jurisdiction over Aircon because FAITH.
not specify a date when delivery could be effected. the latter ceased operations, as its corporate life ended on
December 31, 1986.[8]  Upon motion, defendants Fedders  III. ASSUMING  ARGUENDO  THAT AIRCON MAY BE
TempControl Systems, Inc. (a subsidiary of Aircon until Air Conditioning USA and Maxim were declared in default. CONSIDERED AS JARDINES MERE ALTER EGO, THE COURT OF
1987) undertook the maintenance of the units, inclusive of [9] APPEALS ERRED IN NOT DECLARING JRBS CAUSES OF ACTION
parts and services. In October 1987, the respondent AS HAVING BEEN BARRED BY LACHES.
learned, through newspaper ads,[5]  that Maxim Industrial On May 17, 1996, the RTC rendered its Decision, the
and Merchandising Corporation (Maxim, for short) was the dispositive portion of which reads: IV. ASSUMING  ARGUENDO  THAT AIRCON MAY BE
new and exclusive licensee of Fedders Air Conditioning USA   CONSIDERED AS JARDINES MERE ALTER EGO, THE COURT OF
in the Philippines for the manufacture, distribution, sale, WHEREFORE, judgment is hereby rendered ordering APPEALS ERRED IN FINDING JRB ENTITLED TO RECOVER
installation and maintenance of Fedders air conditioners. defendants Jardine Davies, Inc., Fedders Air Conditioning ALLEGED UNSAVED ELECTRICITY EXPENSES.
The respondent requested that Maxim honor the obligation USA, Inc. and Maxim Industrial and Merchandising  
of Aircon, but the latter refused. Considering that the ten- Corporation, jointly and severally: V. THE COURT OF APPEALS ERRED IN HOLDING
year period of prescription was fast approaching, to expire   JARDINE LIABLE TO PAY ATTORNEYS FEES.
on March 13, 1990, the respondent then instituted, on 1.                  To deliver, install and place into operation
January 29, 1990, an action for specific performance with the two (2) brand new units of Fedders unitary packaged VI. THE COURT OF APPEALS ERRED IN NOT HOLDING
damages against Aircon & Refrigeration Industries, Inc., airconditioning units each of 10 tons capacity with rotary JRB LIABLE TO JARDINE FOR DAMAGES.[11]
Fedders Air Conditioning USA, Inc., Maxim Industrial & compressors to deliver 30,000 kcal or 120,000 BTUH to the  
Merchandising Corporation and petitioner Jardine Davies, second floor of the Blanco Center building, or to pay It is the well-settled rule that factual findings of the trial
Inc.[6] The latter was impleaded as defendant, considering plaintiff the current price for two such units; court, as affirmed by the CA, are accorded high respect,
that Aircon was a subsidiary of the petitioner. The   even finality at times. However, considering that the
respondent prayed that judgment be rendered, as follows: 2.                                    To reimburse plaintiff the amount factual findings of the CA and the RTC were based on
  of P556,551.55 as and for the unsaved electricity bills from speculation and conjectures, unsupported by substantial
 1. Ordering the defendants to jointly and severally at their October 21, 1981 up to April 30, 1995; and another amount evidence, the Court finds that the instant case falls under
account and expense deliver, install and place in operation of P185,951.67 as and for repair costs; one of the excepted instances. There is, thus, a need to
two  brand new units of each 10-tons capacity Fedders   correct the error.
unitary packaged air conditioners with Fedders USAs 3.                                   To pay plaintiff P50,000.00 as and for  
technology perfected rotary compressors to always deliver attorneys fees; and The trial court ruled that Aircon was a subsidiary of the
30,000 kcal or 120,000 BTUH to the second floor of the   petitioner, and concluded, thus:
Blanco Center building at 119 Alfaro St., Salcedo Village, 4.                  Cost of suit.[10]  
Makati, Metro Manila;   Plaintiffs documentary evidence shows that at the time it
  The petitioner filed its notice of appeal with the CA, contracted with Aircon on March 13, 1980 (Exhibit D) and
2. Ordering defendants to jointly and severally reimburse alleging that the trial court erred in holding it liable on the date the revised agreement was reached on March
plaintiff not only the sums of  P415,118.95 for unsaved because it was not a party to the contract between JRB 26, 1981, Aircon was a subsidiary of Jardine. The phrase A
electricity from 21st  October 1981 to 7th  January 1990 Realty, Inc. and Aircon, and that it had a personality subsidiary of Jardine Davies, Inc. was printed on Aircons
and  P99,287.77 for repair costs of the two service units separate and distinct from that  of Aircon. On March 23, letterhead of its March 13, 1980 contract with plaintiff

Page 125 of 160


CORPORATION LAW CASES MAYORDO, M.A
LLB 3

(Exhibit D-1), as well as the Aircons letterhead of Jardines (f) Financial Statement of Aircon as of December 31, 1982  
Director and Senior Vice-President A.G. Morrison and and 1981 (Exh. S); The records bear out that Aircon is a subsidiary of the
Aircons President in his March 26, 1981 letter to plaintiff   petitioner only because the latter acquired Aircons majority
(Exhibit J-2) confirming the revised agreement. Aircons (g) Financial Statement of Aircon as of December 31, 1981 of capital stock. It, however, does not exercise complete
newspaper ads of April 12 and 26, 1981 and a press release (Exh. S-1).[13] control over Aircon; nowhere can it be gathered that the
on August 30, 1982 (Exhibits E, F and L) also show that   petitioner manages the business affairs of Aircon. Indeed,
defendant Jardine publicly represented Aircon to be its   Applying the doctrine of piercing the veil of corporate no management agreement exists between the petitioner
subsidiary. fiction, both the respondent and trial courts conveniently and Aircon, and the latter is an entirely different entity
  held the petitioner liable for the alleged omissions of from the petitioner.[19]
Records from the Securities and Exchange Commission (SEC) Aircon, considering that the latter was its instrumentality  
also reveal that as per Jardines December 31, 1986 and or corporate alter ego. The petitioner is now before us, Jardine Davies, Inc., incorporated as early as June 28,
1985 Financial Statements that The company acts as reiterating its defense of separateness, and the fact that it 1946,[20]  is primarily a financial and trading company. Its
general manager of its subsidiaries (Exhibit P). Jardines is not a party to the contract. Articles of Incorporation states among many others that the
Consolidated Balance Sheet as of December 31, 1979 filed   purposes for which the said corporation was formed, are as
with the SEC listed Aircon as its subsidiary by owning We find merit in the petition. follows:
94.35% of Aircon (Exhibit P-1). Also, Aircons reportorial    
General Information Sheet as of April 1980 and April 1981   (a) To carry on the business of merchants, commission
filed with the SEC show that Jardine was 94.34% owner of It is an elementary and fundamental principle of merchants, brokers, factors, manufacturers, and agents;
Aircon (Exhibits Q and R) and that out of seven members of corporation law that a corporation is an artificial being  
the Board of Directors of Aircon, four (4) are also of invested by law with a personality separate and distinct (b) Upon complying with the requirements of law
Jardine. from its stockholders and from other corporations to which applicable thereto, to act as agents of companies and
  it may be connected. While a corporation is allowed to underwriters doing and engaging in any and all kinds of
Defendant Jardines witness, Atty. Fe delos Santos-Quiaoit exist solely for a lawful purpose, the law will regard it as insurance business.[21]
admitted that defendant Aircon, renamed Aircon & an association of persons or in case of two corporations,  
Refrigeration Industries, Inc. is one of the subsidiaries of merge them into one, when this corporate legal entity is On the other hand, Aircon, incorporated on December 27,
Jardine Davies (TSN, September 22, 1995, p. 12). She also used as a cloak for fraud or illegality.[14]  This is the 1952,[22]  is a manufacturing firm. Its Articles of
testified that Jardine nominated, elected, and appointed doctrine of piercing the veil of corporate  fiction which Incorporation states that its purpose is mainly -
the controlling majority of the Board of Directors and the applies only when such corporate fiction is used to defeat  
highest officers of Aircon (Ibid, pp. 10,13-14). public convenience, justify wrong, protect fraud or defend To carry on the business of manufacturers of commercial
  crime.[15]  The rationale behind piercing a corporations and household appliances and accessories of any form,
The foregoing circumstances provide justifiable basis for identity is to remove the barrier between the corporation particularly to manufacture, purchase, sell or deal in air
this Court to disregard the fiction of corporate entity and from the persons comprising it to thwart the fraudulent and conditioning and refrigeration products of every class and
treat defendant Aircon as part of the instrumentality of co- illegal schemes of those who use the corporate personality description as well as accessories and parts thereof,  or
defendant Jardine.[12] as a shield for undertaking certain proscribed activities.[16] other kindred articles; and to erect, or buy, lease, manage,
    or otherwise acquire manufactories, warehouses, and
  The respondent court arrived at the same conclusion While it is true that Aircon is a subsidiary of the petitioner, depots for manufacturing, assemblage, repair and storing,
basing its ruling on the following documents, to wit: it does not necessarily follow that Aircons corporate legal buying, selling, and dealing in the aforesaid appliances,
  existence can just be disregarded. In Velarde v. Lopez, Inc., accessories and products. [23]
(a)    Contract/Quotation #78-No. 80-1639 dated March 03, [17]  the Court categorically held that a subsidiary has an  
1980 (Exh. D-1); independent and separate juridical personality, distinct The existence of interlocking directors, corporate officers
  from that of its parent company; hence, any claim or suit and shareholders, which the respondent court considered,
(b)   Newspaper Advertisements (Exhs. E-1 and F-1); against the latter does not bind the former, and vice versa. is not enough justification to pierce the veil of corporate
  In applying the doctrine, the following requisites must be fiction, in the absence of fraud or other public policy
(c) Letter dated March 26, 1981 of A.G. Morrison, President established: (1) control, not merely majority or complete considerations.[24] But even when there is dominance over
of Aircon, to Atty. J.R. Blanco (Exh. J); stock control; (2) such control must have been used by the the affairs of the subsidiary, the doctrine of piercing the
  defendant to commit fraud or wrong, to perpetuate the veil of corporate fiction applies only when such fiction is
(d)      News items of Bulletin Today dated August 30, 1982 violation of a statutory or other positive legal duty, or used to defeat public convenience, justify wrong, protect
(Exh. L); dishonest acts in contravention of plaintiffs legal rights; fraud or defend crime.[25]  To warrant resort to this
  and (3) the aforesaid control and breach of duty must extraordinary remedy, there must be proof that the
(e)    Balance Sheet of Jardine Davies, Inc. as of December proximately cause the injury or unjust loss complained of. corporation is being used as a cloak or cover for fraud or
31, 1979 listing Aircon as one of its subsidiaries (Exh. P); [18] illegality, or to work injustice.[26]  Any piercing of the

Page 126 of 160


CORPORATION LAW CASES MAYORDO, M.A
LLB 3

corporate veil has to be done with caution.[27]  The Second. After such print advertisements, the respondent 

wrongdoing must be clearly and convincingly established. It informed Aircon that it was going to install an electric HILADO, J.:
cannot just be presumed.[28] meter to register its electric consumption so as to This is an appeal by Koppel (Philippines), Inc., from the
  determine the electric costs not saved by the presently judgment of the Court of First Instance of Manila in civil
In the instant case, there is no evidence that Aircon was installed units with semi-hermetic compressors. Contrary to case No. 51218 of said court dismissing said corporation's
formed or utilized with the intention of defrauding its the allegations of the respondent that this was in pursuance complaint for the recovery of the sum of P64,122.51 which
creditors or evading its contracts and obligations. There to their Revised Agreement, no proof was adduced that it had paid under protest to the Collector of Internal
was nothing fraudulent in the acts of Aircon in this case. Aircon agreed to the respondents proposition. It was a Revenue on October 30, 1936, as merchant sales tax. The
Aircon, as a manufacturing firm of air  conditioners, unilateral act on the part of the respondent, which Aircon main facts of the case were stipulated in the court below
complied with its obligation of providing two air did not oblige or commit itself to pay. as follows:
conditioning units for the second floor of the Blanco Center  
in good faith, pursuant to its contract with the respondent. Third. Needless to state, the amounts computed are mere AGREED STATEMENT OF FACTS
Unfortunately, the performance of the air conditioning estimates representing the respondents self-serving claim
units did not satisfy the respondent despite several of unsaved electricity cost, which is too speculative and Now come the plaintiff by attorney Eulogio P. Revilla and
adjustments and corrective measures. In a Letter[29] dated conjectural to merit consideration. No other proofs, reports the defendant by the Solicitor General and undersigned
October 22, 1980, the respondent even conceded that or bases of comparison showing that Fedders Air Assistant Attorney of the Bureau of Justice and, with leave
Fedders Air Conditioning USA has not yet perhaps perfected Conditioning USA could indeed cut down electricity cost by of this Honorable Court, hereby respectfully stipulated and
its technology of rotary compressors, and agreed to change 30% were adduced. agree to the following facts, to wit:
the compressors with the semi-hermetic type. Thus, Aircon  
substituted the units with serviceable ones which delivered Likewise, there is no basis for the award of  P185,951.67 I. That plaintiff is a corporation duly organized and existing
the cooling temperature needed for the law office. After representing maintenance cost. The respondent merely under and by virtue of the laws of the Philippines, with
enjoying ten (10) years of its cooling power, respondent submitted a schedule[31]  prepared by the respondents principal office therein at the City of Manila, the capital
cannot now complain about the performance of these units, accountant, listing the alleged repair costs from March stock of which is divided into thousand (1,000) shares of
nor can it demand a replacement thereof. 1987 up to June 1994. Such evidence is self-serving and can P100 each. The Koppel Industrial Car and Equipment
  not also be given probative weight, considering that there company, a corporation organized and existing under the
Moreover, it was reversible error to award the respondent are no proofs of receipts, vouchers,  etc., which would laws of the State of Pennsylvania, United States of America,
the amount of  P556,551.55 representing the alleged 30% substantiate the amounts paid for such services. Absent any and not licensed to do business in the Philippines, owned
unsaved electricity costs and  P185,951.67 as maintenance more convincing proof, the Court finds that the nine hundred and ninety-five (995) shares out of the total
cost without showing any basis for such award. To justify a respondents claims are without basis, and cannot, capital stock of the plaintiff from the year 1928 up to and
grant of actual or compensatory damages, it is necessary to therefore, be awarded. including the year 1936, and the remaining five (5) shares
prove with a reasonable degree of certainty, premised upon   only were and are owned one each by officers of the
competent proof and on the best evidence obtainable by We sustain the petitioners separateness from that of Aircon plaintiff corporation.
the injured party, the actual amount of loss.[30]  The in this case. It bears stressing that the petitioner was never
respondent merely based its cause of action on Aircons a party to the contract. Privity of contracts take effect only II. That plaintiff, at all times material to this case, was and
alleged representation that Fedders air conditioners with between parties, their successors-in-interest, heirs and now is duly licensed to engage in business as a merchant
rotary compressors can save as much as 30% on electricity assigns.[32]  The petitioner, which has a  separate and and commercial broker in the Philippines; and was and is
compared to other brands. Offered in evidence were distinct legal personality from that of Aircon, cannot, the holder of the corresponding merchant's and commercial
newspaper advertisements published on April 12 and 26, therefore, be held liable. broker's privilege tax receipts.
1981. The respondent then recorded its electricity  
consumption from October 21, 1981 up to April 3, 1995 and IN VIEW OF THE FOREGOING, the petition is GRANTED. The III. That the defendant Collector of Internal revenue is now
computed 30% thereof, which amounted to  P556,551.55. assailed decision of the Court of Appeals, affirming the Mr. Bibiano L. Meer in lieu of Mr. Alfredo L. Yatco.
The Court rules that this amount is highly speculative and decision of the Regional Trial Court is  REVERSED  and  SET
merely hypothetical, and for which the petitioner can not ASIDE. The complaint of the respondent is DISMISSED. Costs IV. That during the period from January 1, 1929, up to and
be held accountable. against the respondent. including December 31, 1932, plaintiff transacted business
    in the Philippines in the following manner, with the
First. The respondent merely relied on the newspaper SO ORDERED. exception of the transactions which are described in
advertisements showing the Fedders window-type air paragraphs V and VI of this stipulation:
conditioners, which are far different from the big capacity G.R. No. L-47673             October 10, 1946
air conditioning units installed at Blanco Center. KOPPEL (PHILIPPINES), INC., plaintiff-appellant,  vs. When a local buyer was interested in the purchase of
  ALFREDO L. YATCO, Collector of Internal railway materials, machinery, and supplies, it asked for
Revenue, defendant-appellee. price quotations from plaintiff. Atypical form of such

Page 127 of 160


CORPORATION LAW CASES MAYORDO, M.A
LLB 3

request is attached hereto and made a part hereof as so advanced, were in every case reimbursed to plaintiff by account. The engines were left stored at Stockholm,
Exhibit A. (Exhibit A represents typical transactions arising Koppel Industrial Car and Equipment Company. The price Sweden. On April 1, 1930, a new local buyer, Mr. Cesar
from written requests for quotations, while Exhibits B to G, were payable by drafts agreed upon in each case and drawn Barrios, of Iloilo, Philippines, was found and the same
inclusive, are typical transactions arising from verbal by Koppel Industrial Car and Equipment Company on engines were sold to him for $21,000 (P42,000) C. I. F.
requests for quotation.) Plaintiff then cabled for the respective purchasers through local banks, and payments Hongkong. The engines were shipped to Hongkong and a
quotation desired for Koppel Industrial Car and Equipment were made to the banks by the purchasers on presentation draft for $21,000 was drawn by Koppel Industrial Car and
Company. A sample of the pertinent cable is hereto and delivery to them of the above-mentioned shipping Equipment Company on Mr. Cesar Barrios. After the draft
attached and made a part hereof as Exhibit B. Koppel documents or copies thereof. A sample of said drafts is was fully paid by Mr. Barrios, Koppel Industrial Car and
Industrial Car and Equipment Company answered by cable hereto attached as Exhibit G. Plaintiff received by way of Equipment Company reimbursed plaintiff with cost price of
quoting its cost price, usually A. C. I. F. Manila cost price, compensation a percentage of the profits realized on the $16,508.32 and credited it with $1,152.95 as its share of
which was later followed by a letter of confirmation. A above transactions as fixed in paragraph 6 of the plaintiff's the profit on the transaction. Exhibits J and J-1 are
sample of the said cable quotation and of the letter of contract with Koppel Industrial Car and Equipment herewith attached and made integral parts of this
confirmation are hereto attached and made a part hereof Company, which contract is hereto attached as Exhibit H, stipulation with particular reference to paragraph VI
as Exhibits C and C-1. Plaintiff, however, quoted by Koppel and suffered its corresponding share in the losses resulting hereof.
Industrial Car and Equipment Company. Copy of the from some of the transactions.
plaintiff's letter to purchaser is hereto attached and made VII. That plaintiff's share in the profits realized out of these
a part hereof as Exhibit D. On the basis of these quotations, That the total gross sales from January 1, 1929, up to and transactions described in paragraphs IV, V and VI hereof
orders were placed by the local purchasers, copies of which including December 31, 1932, effected in the foregoing totaling P3,772,403.82, amounts to P132,201.30; and that
orders are hereto attached as Exhibits E and E-1. manner and under the above specified conditions, amount plaintiff within the time provided by law returned the
to P3, 596,438.84. aforesaid amount P132,201.30 for the purpose of the
A cable was then sent to Koppel Industrial Car and commercial broker's 4 per cent tax and paid thereon the
Equipment company giving instructions to ship the V. That when a local sugar central was interested in the sum P5,288.05 as such tax.
merchandise to Manila forwarding the customer's order. purchase of railway materials, machinery and supplies, it
Sample of said cable is hereto attached as Exhibit F. The secured quotations from, and placed the corresponding VIII. That defendant demanded of the plaintiff the sum of
bills of lading were usually made to "order" and indorsed in orders with, the plaintiff in substantially the same manner P64,122.51 as the merchants' sales tax of 1% per cent on
blank with notation to the effect that the buyer be notified as outlined in paragraph IV of this stipulation, with the only the amount of P3,772,403.82, representing the total gross
of the shipment of the goods covered in the bills of lading; difference that the purchase orders which were agreed to value of the sales mentioned in paragraphs IV, V and VI
commercial invoices were issued by Koppel Industrial Car by the central and the plaintiff are similar to the sample hereof, including the 25 per cent surcharge for the late
and Equipment Company in the names of the purchasers hereto attached and made a part hereof as Exhibit I. payment of the said tax, which tax and surcharge were
and certificates of insurance were likewise issued in their Typical samples of the bills of lading covering the herein determined after the amount of P5,288.05 mentioned in
names, or in the name of Koppel Industrial Car and transaction are hereto attached and made a part hereto as paragraph VI hereof was deducted.
Equipment Company but indorsed in blank and attached to Exhibits I-1, I-2 and I-3. The value of the sales carried out IX. That plaintiff, on October 30, 1936, paid under protest
drafts drawn by Koppel Industrial Car and Equipment in the manner mentioned in this paragraph is P133,964.98. said sum of P64,122.51 in order to avoid further penalties,
Company on the purchasers, which were forwarded through levy and distraint proceedings.
foreign banks to local banks. Samples of the bills of lading VI. That sometime in February, 1929, Miguel J. Ossorio, of
are hereto attached as Exhibits F-1, I-1, I-2 and I-3. Bills of Manila, Philippines, placed an option with Koppel Industrial X. That defendant, on November 10, 1936, overruled
ladings, Exhibits I-1, I-2 and I-3, may equally have been Car and Equipment Company, through plaintiff, to purchase plaintiff's protest, and defendant has failed and refused
employed, but said Exhibits I-1, I-2 and I-3 have no within three months a pair of Atlas-Diesel Marine Engines. and still fails and refuses, notwithstanding demands by
connection with the transaction covered by Exhibits B to G, Koppel Industrial Car and Equipment Company purchased plaintiff, to return to the plaintiff said sum of P64,122.51
inclusive. The purchasers secured the shipping papers by said Diesel Engines in Stockholm, Sweden, for $16,508.32. or any part thereof.
arrangement with the banks, and thereupon received and The suppliers drew a draft for the amount of $16,508.32 on xxx     xxx     xxx
cleared the shipments. If the merchandise were of the Koppel Industrial Car and Equipment Company, which That the parties hereby reserve the right to present
European origin, and if there was not sufficient time to paid the amount covered by the draft. Later, Miguel J. additional
forward the documents necessary for clearance, through Ossorio definitely called the deal off, and as Koppel evidence in support of their respective contentions.
foreign banks to local banks, to the purchasers, the Koppel Industrial Car and Equipment Company could not ship to or
Industrial Car and Equipment company did, in many cases, draw on said Mr. Miguel J. Ossorio, it in turn drew another Manila, Philippines, December 26, 1939
send the documents directly from Europe to plaintiff with draft on plaintiff for the same amount at six months sight,
instructions to turn these documents over to the with the understanding that Koppel Industrial Car and (Sgd.) ROMAN OZAETA           

purchasers. In many cases, where sales was effected on the Equipment Company would reimburse plaintiff when said Solicitor General          
basis of C. I. F. Manila, duty paid, plaintiff advanced the engines were disposed of. Plaintiff honored the draft and   
sums required for the payment of the duty, and these sums, debited the said sum of $16,508.32 to merchandise

Page 128 of 160


CORPORATION LAW CASES MAYORDO, M.A
LLB 3

      (Sgd.) ANTONIO CAÑIZARES
 as set forth above, and shall not take possession of any of
       Assistant Attorney (f) Where the goods were European origin, consular the materials or equipment applying to said orders or
invoices, bill of lading, and, in general, the documents perform any acts or duties outside the scope of a broker;
(Sgd.) E. P. REVILLA           
 necessary for clearance were sent directly to plaintiff and in no sense shall this contract be construed as granting
Attorney for the Plaintiff           
 (t.s.n., p. 14); to the broker the power to represent the principal as its
3rd Floor, Perez Samanillo Bldg., Manila           agent or to make commitments on its behalf.
(g) If the plaintiff had in stock the merchandise desired by
Both parties adduced some oral evidence in clarification of local buyers, it immediately filled the orders of such local The Court of First Instance held for the defendant and
or addition to their agreed statement of facts. A buyers and made delivery in the Philippines without the dismissed plaintiff's complaint with costs to it.
preponderance of evidence has established, besides the necessity of cabling its principal in America either for price
facts thus stipulated, the following: quotations or confirmation or rejection of that agreed upon Upon this appeal, seven errors are assigned to said
between it and the buyer (t.s.n., pp. 39-43); judgment as follows:.
(a) The shares of stock of plaintiff corporation were and are
all owned by Koppel Industries Car and Equipment Company (h) Whenever the deliveries made by Koppel Industrial Car 1. That the court a quo erred in not holding that appellant
of Pennsylvania, U. S. A., exceptive which were necessary and Equipment Company were incomplete or insufficient to is a domestic corporation distinct and separate from, and
to qualify the Board of Directors of said plaintiff fill the local buyer's orders, plaintiff used to make good the not a mere branch of Koppel Industrial Car and Equipment
corporation; deficiencies by deliveries from its own local stock, but in Co.;
such cases it charged its principal only the actual cost of
(b) In the transactions involved herein the plaintiff the merchandise thus delivered by it from its stock and in 2. the court  a quo  erred in ignoring the ruling of the
corporation acted as the representative of Koppel Industrial such transactions plaintiff did not realize any profit (t.s.n., Secretary of Finance, dated January 31, 1931, Exhibit M;
Car and Equipment Company only, and not as the agent of pp. 53-54);
both the latter company and the respective local 3. the court a quo erred in not holding that a character of a
purchasers — plaintiff's principal witness, A.H. Bishop, its (i) The contract of sale involved herein were all perfected broker is determined by the nature of the transaction and
resident Vice-President, in his testimony invariably referred in the Philippines. not by the basis or measure of his compensation;
to Koppel Industrial Car and Equipment Co. as "our
principal" 9 t. s. n., pp. 10, 11, 12, 19, 75), except that at Those described in paragraph IV of the agreed statement of 4. The court a quo erred in not holding that appellant acted
the bottom of page 10 to the top of page 11, the witness facts went through the following process: (1) "When a local as a commercial broker in the transactions covered under
stated that they had "several principal" abroad but that "our buyer was interested in the purchase of railway materials, paragraph VI of the agreed statement of facts;
principal abroad was, for the years in question, Koppel machinery, and supplies, it asked for price quotations from
Industrial Car and Equipment Company," and on page 68, he plaintiff"; (2) "Plaintiff then cabled for the quotation 5. The court a quo erred in not holding that appellant acted
testified that what he actually said was ". . . but desired from Koppel Industrial Car and Equipment as a commercial broker in the transactions covered under
our principal abroad" and not "our principal abroad" — as to Company"; (3) "Plaintiff, however, quoted to the purchaser paragraph v of the agreed statement of facts;
which it is very significant that neither this witness nor any a selling price above the figures quoted by Koppel Industrial
other gave the name of even a single other principal abroad Car and Equipment Company"; (4) "On the basis of these 6. The court a quo erred in not holding that appellant acted
of the plaintiff corporation; quotations, orders were placed by the local purchasers . . ." as a commercial broker in the sole transaction covered
under paragraph VI of the agreed statement of facts;
(c) The plaintiff corporation bore alone incidental expenses Those described in paragraph V of said agreed statement of
— as, for instance, cable expenses-not only those of its own facts were transacted "in substantially the same manner as 7. the court a quo erred in dismissing appellant's complaint.
cables but also those of its "principal" (t.s.n., pp. 52, 53); outlined in paragraph IV."
The lower court found and held that Koppel (Philippines),
(d) the plaintiff's "share in the profits" realized from the As to the single transaction described in paragraph VI of the Inc. is a mere dummy or brach ("hechura") of Koppel
transactions in which it intervened was left virtually in the same agreed statement of facts, discarding the Ossorio industrial Car and Equipment Company. The lower court did
hands of Koppel Industrial Car and Equipment Company option which anyway was called off, "On April 1, 1930, a not deny legal personality to Koppel (Philippines), Inc. for
(t.s.n., p. 51); new local buyer, Mr. Cesar Barrios, of Iloilo, Philippines, any and all purposes, but in effect its conclusion was that,
was found and the same engines were sold to him for in the transactions involved herein, the public interest and
(e) Where drafts were not paid by the purchasers, the local $21,000(P42,000) C.I.F. Hongkong." (Emphasis supplied.). convenience would be defeated and what would amount to
banks were instructed not to protest them but to refer a tax evasion perpetrated, unless resort is had to the
them to plaintiff which was fully empowered by Koppel (j) Exhibit H contains the following paragraph: doctrine of "disregard of the corporate fiction."
Industrial Car and Equipment company to instruct the banks
with regards to disposition of the drafts and documents It is clearly understood that the intent of this contract is I. In its first assignment of error appellant submits that the
(t.s.n., p. 50; Exhibit G); that the broker shall perform only the functions of a broker trial court erred in not holding that it is a domestic

Page 129 of 160


CORPORATION LAW CASES MAYORDO, M.A
LLB 3

corporation distinct and separate from and not a mere being held by the officers of the plaintiff herein in order to corporation. (Hanter vs. Baker Motor Vehicle Co., 190 Fed.,
branch of Koppel Industrial Car and Equipment Company. It permit the incorporation thereof and to enable its 665.)
contends that its corporate existence as Philippine aforesaid officers to act as directors of the plaintiff
corporation can not be collaterally attacked and that the corporation; and that plaintiff was organized as a Philippine In United States vs. Lehigh Valley R. Co. 9220 U.S., 257; 55
Government is estopped from so doing. As stated above, corporation for the purpose of evading the payment by its Law. ed., 458, 464), the Supreme Court of the United
the lower court did not deny legal personality to appellant parent foreign corporation of merchants' sales tax on the States disregarded the artificial personality of the
for any and all purposes, but held in effect that in the transactions involved in this case and others of similar subsidiary coal company in order to avoid that the parent
transaction involved in this case the public interest and nature." corporation, the Lehigh Valley R. Co., should be able,
convenience would be defeated and what would amount to through the fiction of that personality, to evade the
a tax evasion perpetrated, unless resort is had to the By most courts the entity is normally regarded but is prohibition of the Hepburn Act against the transportation
doctrine of "disregard of the corporate fiction." In other disregarded to prevent injustice, or the distortion or hiding by railroad companies of the articles and commodities
words, in looking through the corporate form to the of the truth, or  to let in a just defense. (1 Fletcher, described therein.
ultimate person or corporation behind that form, in the Cyclopedia of Corporation, Permanent Edition, pp. 139,140;
particular transactions which were involved in the case emphasis supplied.) Chief Justice White, speaking for the court, said:
submitted to its determination and judgment, the court did
so in order to prevent the contravention of the local Another rule is that, when the corporation is the mere alter . . . Coming to discharge this duty it follows, in view of the
internal revenue laws, and the perpetration of what would ego, or business conduit of a person, it may de express prohibitions of the commodities clause, it must be
amount to a tax evasion, inasmuch as it considered — and disregarded." (1 Fletcher, Cyclopedia of Corporation, held that while the right of a railroad company as a
in our opinion, correctly — that appellant Koppel Permanent Edition, p. 136.) stockholder to use its stock ownership for the purpose of
(Philippines), Inc. was a mere branch or agency or dummy a  bona fide  separate administration of the affairs of a
("hechura") of Koppel Industrial Car and Equipment Co. The Manifestly, the principle is the same whether the "person" corporation in which it has a stock interest may not be
court did not hold that the corporate personality of Koppel be natural or artificial. denied, the use of such stock ownership in substance for
(Philippines), Inc., would also be disregarded in other cases the purpose of destroying the entity of a producing, etc.,
or for other purposes. It would have had no power to so A very numerous and growing class of cases wherein the corporation, and commingling its affairs in administration
hold. The courts' action in this regard must be confined to corporate entity is disregarded is that (it is so organized with the affairs of the railroad company, so as to make the
the transactions involved in the case at bar "for the purpose and controlled, and its affairs are so conducted, as to make two corporations virtually one, brings the railroad company
of adjudging the rights and liabilities of the parties in the it merely an instrumentality, agency, conduit or adjunct of so voluntarily acting as to such producing, etc., corporation
case. They have no jurisdiction to do more." (1 Flethcer, another corporation)." (1 Fletcher, Cyclopedia of within the prohibitions of the commodities clause. In other
Cyclopedia of Corporation, Permanent ed., p. 124, section Corporation, Permanent ed., pp. 154, 155.) words, that by operation and effect of the commodities
41.) clause there is duty cast upon a railroad company proposing
While we recognize the legal principle that a corporation to carry in interstate commerce the product of a producing,
A leading and much cited case puts it as follows: does not lose its entity by the ownership of the bulk or etc., corporation in which it has a stock interest, not to
even the whole of its stock, by another corporation abuse such power so as virtually to do  by indirection  that
If any general rule can be laid down, in the present state of (Monongahela Co. vs. Pittsburg Co., 196 Pa., 25; 46 Atl., 99; which the commodities clause prohibits, — a duty which
authority, it is that a corporation will be looked upon as a 79 Am. St. Rep., 685) yet it is equally well settled and plainly would be violated by the unnecessary commingling
legal entity as a general rule, and until sufficient reason to ignore corporate forms." (Colonial Trust Co. vs. Montello of the affairs of the producing company with its own, so as
the contrary appears; but, when the notion of legal entity Brick Works, 172 Fed., 310.) to cause them to be one and inseparable.
is used to defeat public convinience, justify wrong, protect
fraud, or defend crime, the law will regard the corporation Where it appears that two business enterprises are owned, Corrobarative authorities can be cited in support of the
as an association of persons. (1 Fletcher Cyclopedia of conducted and controlled by the same parties, both law same proposition, which we deem unnecessary to mention
Corporation [Permanent Edition], pp. 135, 136; United and equity will, when necessary to protect the rights of here.
States vs. Milwaukee Refrigeration Transit Co., 142 Fed., third persons, disregard the legal fiction that two
247, 255, per Sanborn, J.) corporations are distinct entities, and treat them as From the facts hereinabove stated, as established by a
identical. (Abney vs. Belmont Country Club Properties, Inc., preponderance of the evidence , particularly those
In his second special defense appellee alleges "that the 279 Pac., 829.) narrated in paragraph (a), (b), (c), (d), (e),(f), (h), (i), and
plaintiff was and is in fact a branch or subsidiary of Koppel (j) after the agreed statement of facts, we find that, in so
Industrial Car and Equipment Co., a Pennsylvania . . . the legal fiction of distinct corporate existence will be far as the sales involved herein are concerned, Koppel
corporation not licensed to do business in the Philippines disregarded in a case where a corporation is so organized (Philippines), Inc., and Koppel Industrial Car and Equipment
but actually doing business here through the plaintiff; that and controlled and its affairs are so conducted, as to make company are to all intents and purposes one and the same;
the said foreign corporation holds 995 of the 1,000 shares it merely an instrumentality or adjunct of another or, to use another mode of expression, that, as regards
of the plaintiff's capital stock, the remaining five shares those transactions, the former corporation is a mere

Page 130 of 160


CORPORATION LAW CASES MAYORDO, M.A
LLB 3

branch, subsidiary or agency of the latter. To our mind, this in the Philippines, where it was formed, the record and the accounting operations between them, except upon the
is conclusively borne out by the fact, among others, that evidence do not disclose any reason why all its officers basis that the Philippine corporation was to all intents and
the amount of he so-called "share in the profits" of Koppel should not reside and perform their functions in the purposes a mere subsidiary, branch, or agency of the
(Philippines), Inc., was ultimately left to the sole, Philippines. American parent entity. Only upon this basis can it be
unbridled control of Koppel Industrial Car and Equipment comprehended why it seems not to matter at all how much
Company. If, in their relations with each other, Koppel Other facts appearing from the evidence, and presently to profit would be allocated to plaintiff, or even that no profit
(Philippines), Inc., was considered and intended to function be stated, strengthen our conclusion, because they can at all be so allocated to it, at any given time or after any
as a  bona fide  separate corporation, we can not conceive only be explained if the local entity is considered as a mere given period.
how this arrangement could have been adopted, for if subsidiary, branch or agency of the parent organization.
there was any factor in its business as to which it would in Plaintiff charged the parent corporation no more than As already stated above, under the evidence the sales in
that case naturally have been opposed to being thus actual cost — without profit whatsoever — for merchandise the Philippines of the railway materials, machinery and
controlled, it must have been precisely the amount allegedly of its own to complete deficiencies of shipments supplies imported here by Koppel Industrial Car and
of  profit  which it could endeavor and hope to earn. No made by said parent corporation (t.s.n., pp. 53, 54) — a Equipment Company could have been as conviniently and
group of businessmen could be expected to organize a fact which could not conceivably have been the case if efficiently transacted and handled — if not more so — had
mercantile corporation — the ultimate end of which could plaintiff had acted in such transactions as an entirely said corporation merely established a branch or agency in
only be profit — if the amount of that profit were to be independent entity doing business — for profit, of course — the Philippines and obtained license to do business locally;
subjected to such a unilateral control of another with the American concern. There has been no attempt and if it had done so and said sales had been effected by
corporation, unless indeed the former has previously been even to explain, if the latter situation really obtained, why such branch or agency, there seems to be no dispute that
designed by the incorporators to serve as a mere subsidiary, these two corporations should have thus departed from the the 1½ per cent merchants' sales tax then in force would
branch or agency of the latter. Evidently, Koppel Industrial ordinary course of business. Plaintiff was charged by the have been collectible. So far as we can discover, there
Car and Equipment Company made us of its ownership of American corporation with the cost even of the latter's would be only one, but very important, difference between
the overwhelming majority — 99.5% — of the capital stock cable quotations — from ought that appears from the the two schemes — a difference in  tax liability  on the
of the local corporation to control the operations of the evidence, this can only be comprehended by considering ground that the sales were made through another and
latter to such an extent that it had the final say even as to plaintiff as such a subsidiary, branch or agency of the distinct corporation, as alleged broker, when we have seen
how much should be allotted to said local entity in the so- parent entity, in which case it would be perfectly that this latter corporation is  virtually owned  by the
called sharing in the profits. We can not overlook the fact understandable that for convenient accounting purposes former, or that they practically one and the same, is to
that in the practical working of corporate organizations of and the easy determination of the profits or losses of the sanction a circumvention of our tax laws, and permit a tax
the class to which these two entities belong, the holder or parent corporation's Philippines should be charged against evasion of no mean proportions and the consequent
holders of the controlling part of the capital stock of the the Philippine office and set off against its receipts, thus commission of a grave injustice to the Government. Not
corporation, particularly where the control is determined separating the accounts of said branch from those which only this; it would allow the taxpayer to do  by
by the virtual ownership of the totality of the shares, the central organization might have in other countries. The indirection what the tax laws prohibited to be done directly
dominate not only the selection of the Board of Directors reference to plaintiff by local banks, under a standing (non-payment of legitimate taxes), paraphrasing the United
but, more often than not, also the action of that Board. instruction of the parent corporation, of unpaid drafts States Supreme Court in United States vs. Lehigh Valley R.
Applying this to the instant case, we can not conceive how drawn on Philippine customers by said parent corporation, Co., supra.
the Philippine corporation could effectively go against the whenever said customers dishonored the drafts, and the
policies, decisions, and desires of the American corporation fact that the American corporation had previously advised The act of one corporation crediting or debiting the other
with regards to the scheme which was devised through the said banks that plaintiff in those cases was "fully for certain items, expenses or even merchandise sold or
instrumentality of the contract Exhibit H, as well as all the empowered to instruct (the banks) with regard to the disposed of, is perfectly compatible with the idea of the
other details of the system which was adopted in order to disposition of the drafts and documents" (t.s.n., p. 50), in domestic entity being or acting as a mere branch, agency
avoid paying the 1½ per cent merchants sales tax. Neither the absence of any other satisfactory explanation naturally or subsidiary of the parent organization. Such operations
can we conceive how the Philippine corporation could avoid give rise to the inference that plaintiff was a subsidiary, were called for any way by the exigencies or convenience
following the directions of the American corporation held branch or agency of the American concern, rather than an of the entire business. Indeed, accounting operation such
99.5 per cent of the capital stock of the Philippine independent corporation acting as a broker. For, without as these are invitable, and have to be effected in the
corporation. In the present instance, we note that Koppel such positive explanation, this delegation of power is ordinary course of business enterprise extends its trade to
(Philippines), Inc., was represented in the Philippines by its indicative of the relations between central and branch another land through a branch office, or through another
"resident Vice-President." This fact necessarily leads to the offices of the same business enterprise, with the latter scheme amounting to the same thing.
inference that the corporation had at least a Vice- acting under instructions already given by the former. Far
President, and presumably also a President, who were not from disclosing a real separation between the two entities, If plaintiff were to act as broker in the Philippines for any
resident in the Philippines but in America, where the parent particularly in regard to the transactions in question, the other corporation, entity or person, distinct from Koppel
corporation is domiciled. If Koppel (Philippines), Inc., had evidence reveals such commongling and interlacing of their Industrial Car and Equipment company, an entirely
been intended to operate as a regular domestic corporation activities as to render even incomprehensible certain

Page 131 of 160


CORPORATION LAW CASES MAYORDO, M.A
LLB 3

different question will arise, which, however, we are not because the contracting parties were here and the consent Code of Commerce, the respective contracts were
called upon, nor in a position, to decide. of each was given here. While it is true that when the completed from the time of the acceptance by the local
contract was thus perfected in the Philippines the pair of buyers, which happened in the Philippines.
As stated above, Exhibit H contains to the following Atlas-Diesel Marine Engines were in Sweden and the
paragraph: agreement was to deliver them C.I.F. Hongkong, the Contracts executed through correspondence shall be
contract of sale being consensual — perfected by mere completed from the time an answer is made accepting the
It is clearly understood that the intent of this contract is consent — (Civil Code, article 1445; 10 Manresa, 4th ed., p. proposition  or the conditions by which the latter may be
that the broker shall perform only the functions of a broker 11), the location of the property and the place of delivery modified." (Code of Commerce, article 54; emphasis
as set forth above, and shall not take possession of any of did not matter in the question of where the agreement was supplied.)
the materials or equipment applying to said orders or perfected.
perform any acts or duties outside the scope of a broker; A contract is as a rule considered as entered into at the
and in no sense shall this contract be construed as granting In said paragraph VI, we read the following, as indicating place where the place it is performed. So where delivery is
to the broker the power to represent the principal as its where the contract was perfected, considering beforehand regarded as made at the place of delivery." (13 C. J.,
agent or to make commitments on its behalf. that one party, Koppel (Philippines),Inc., which in 580-81, section 581.)
contemplation of law, as to that transaction, was the same
The foregoing paragraph, construed in the light of other Koppel Industrial Car Equipment Co., was in the Philippines: (In the consensual contract of sale delivery is not needed
facts noted elsewhere in this decision, betrays, we think a for its perfection.)
deliberate intent, through the medium of a scheme devised . . . on April 1, 1930, a new local buyer Mr. Cesar Barrios,
with great care, to avoid the payment of precisely the 1½ of Iloilo, Philippines, was found and the same engines were II. Appellant's second assignment of error can be summarily
per cent merchants' sales tax in force in the Philippines sold to him  for $21,000 (P42,000) C.I.F. Hongkong . . . disposed of. It is clear that the ruling of the Secretary of
before, at the time of, and after, the making of the said (Emphasis supplied.) Finance, Exhibit M, was not binding upon the trial court,
contract Exhibit H. If this were to be allowed, the payment much less upon this tribunal, since the duty and power of
of a tax, which directly could not have been avoided, could Under the revenue law in force when the sales in question interpreting the laws is primarily a function of the
be evaded  by indirection, consideration being had of the took place, the merchants' sales tax attached upon the judiciary. (Ortua vs. Singson Encarnacion, 59 Phil., 440,
aforementioned peculiar relations between the said happening of the respective sales of the "commodities, 444.) Plaintiff cannot be excused from abiding by this legal
American and local corporations. Such evasion, involving as goods, wares, and merchandise" involved, and we are principle, nor can it properly be heard to say that it relied
it would, a violation of the former Internal Revenue Law, clearly of opinion that such "sales" took place upon the on the Secretary's ruling and that, therefore, the courts
would even fall within the penal sanction of section 2741 of perfection of the corresponding contracts. If such should not now apply an interpretation at variance
the Revised Administrative Code. Which only goes to show perfection took place in the Philippines, the merchants' therewith. The rule of stare decisis is undoubtedly entitled
the illegality of the whole scheme. We are not here sales tax then in force here attached to the transactions. to more respect in the construction of statutes than the
concerned with the impossibility of collecting the interpretations given by officers of the administrative
merchants' sales tax, as a mere incidental consequence of Even if we should consider that the Philippine buyers in the branches of the government, even those entrusted with the
transactions legal in themselves and innocent in their cases covered by paragraph IV and V of the agreed administration of particular laws. But this court, in
purpose. We are dealing with a scheme the primary, not to statement of facts, contracted with Koppel Industrial Car Philippine Trust Company and Smith, Bell and Co. vs.
say the sole, object of which the evasion of the payment of and Equipment company, we will arrive at the same final Mitchell(59 Phil., 30, 36), said:
such tax. It is this aim of the scheme that makes it illegal. result. It can not be denied in that case that said American
corporation contracted through Koppel (Philippines), Inc., . . . The rule of stare decisis is entitled to respect. Stability
We have said above that the contracts of sale involved which was in the Philippines. The real transaction in each in the law, particularly in the business field, is desirable.
herein were all perfected in the Philippines. From the facts case of sale, in final effect, began with an offer of sale But idolatrous reverence for precedent, simply as
stipulated in paragraph IV of the agreed statement of facts, from the seller, said American corporation, through its precedent, no longer rules. More important than anything
it clearly appears that the Philippine purchasers had to wait agent, the local corporation, of the railway materials, else is that court should be right. . . .
for Koppel Industrial Car and Equipment Company to machinery, and supplies at the prices quoted, and
communicate its cost prices to Koppel (Philippines), Inc., perfected or completed by the acceptance of that offer by III. In the view we take of the case, and after the
were perfected in the Philippines. In those cases where no the local buyers when the latter, accepting those prices, disposition made above of the first assignment of error, it
such price quotations from the American corporation were placed their orders. The offer could not correctly be said to becomes unnecessary to make any specific ruling on the
needed, of course, the sales effected in those cases have been made by the local buyers when they asked for third, fourth, fifth, sixth, and seventh assignments of error,
described in paragraph V of the agreed statement of facts price quotations, for they could not rationally be taken to all of which are necessarily disposed of adversely to
were, as expressed therein, transacted "in substantially the have bound themselves to buy  before knowing the prices. appellant's contention.
same manner as outlined in paragraph VI." Even the single And even if we should take into consideration the fact that
transaction described in paragraph VI of the agreed the american corporation contracted, at least partly, Wherefore, he judgment appealed from is affirmed, with
statement of facts was also perfected in the Philippines, through correspondence, according to article 54 of the costs of both instances against appellant. So ordered.

Page 132 of 160


CORPORATION LAW CASES MAYORDO, M.A
LLB 3

Securities & Exchange Commission approved on March 3,          


G.R. No. L-9687             June 30, 1961 1947. Upon such approval, Frank Liddell subscribed to G. W. Kernot 500 50,000 2.64%
LIDDELL & CO., INC., petitioner-appellant,  vs. THE 3,000 additional shares, for which he paid into the
COLLECTOR OF INTERNAL REVENUE, respondent- corporation P300,000 so that he had in his own name 4,960 P1,900,0 100.00
appellee. shares. 19,000 00 %

BENGZON, C.J.: On May 24, 1957, Frank Liddell, on one hand and Messrs.
Statement. This is an appeal from the decision of the Court Kurz, Darras, Manzano and Serrano on the other, executed On November 15, 1948, in accordance with a resolution of a
of Tax Appeals imposing a tax deficiency liability of an agreement (Exhibit A) which was further supplemented special meeting of the Board of Directors of Liddell & Co.,
P1,317,629.61 on Liddell & Co., Inc. by two other agreements (Exhibits B and C) dated May 24, stock dividends were again declared. As a result of said
1947 and June 3, 1948, wherein Frank Liddell transferred declaration and in accordance with the agreements,
Said Company lists down several issues which may be boiled (On June 7, 1948) to various employees of Liddell & Co. Exhibits, A, B, and C, the stockholdings in the company
to the following: shares of stock. appeared to be:

(a) Whether or not Judge Umali of the Tax Court below At the annual meeting of stockholders of Liddell & Co. held No. of P e r
could validly participate in the making of the decision; on March 9, 1948, a 100% stock dividend was declared, Name Amount
Shares Cent
thereby increasing the issued capital stock of aid
(b) Whether or not Liddell & Co. Inc., and the Liddell corporation from P1,000.000 to P 3,000,000 which increase P1,973,8 6 5 . 7 9 1
Motors, Inc. are (practically) identical corporations, the was duly approved by the Securities and Exchange Frank Liddell 19,738 00 %
latter being merely .the alter ego of the former; Commission on June 7, 1948. Frank Liddell subscribed to
and paid 20% of the increase of P400,000. He paid 25% Irene Liddell 1 100 .003%
(c) Whether or not, granting the identical nature of the thereof in the amount of P100,000 and the balance of
Mercedes Vecin 1 100 .003%
corporations, the assessment of tax liability, including the P3,000,000 was merely debited to Frank Liddell-Drawing
surcharge thereon by the Court of Tax Appeals, is correct. Account and credited to Subscribed Capital Stock on Charles Kurz 2,215 221,500 7.381%
December 11, 1948.
Undisputed Facts. The parties submitted a partial E.J. Darras 2,215 221,500 7.381%
stipulation of facts, each reserving the right to present On March 8, 1949, stock dividends were again issued by
additional evidence. Liddell & Co. and in accordance with the agreements, Angel Manzano 1,810 181,000 6.031%
Exhibits A, B, and C, the stocks of said company stood as
Said undisputed facts are substantially as follows: follows: Julian Serrano 1,700 170,000 5.670%

E. Hasim 830 83,000 2.770%


The petitioner, Liddell & Co. Inc., (Liddell & Co. for short)
No. of P e r
is a domestic corporation establish in the Philippines on Name Amount
Shares Cent          
February 1, 1946, with an authorized capital of P100,000 G. W. Kernot 1,490 149,000 4.970%
divided into 1000 share at P100 each. Of this authorized P1,368,8
capital, 196 shares valued at P19,600 were subscribed and Frank Liddell 13,688 00 72.00% P3,000,0 100.000
paid by Frank Liddell while the other four shares were in 30,000 00 %
the name of Charles Kurz, E.J. Darras, Angel Manzano and Irene Liddell 1 100 .01%
Julian Serrano at one shares each. Its purpose was to
engage in the business of importing and retailing Mercedes Vecin 1 100 .01% On the basis of the agreement Exhibit A, (May, 1947) "40%"
Oldsmobile and Chevrolet passenger cars and GMC and of the earnings available for dividends accrued to Frank
Chevrolet trucks.. Charles Kurz 1,225 122,500 6.45% Liddell although at the time of the execution of aid
instrument, Frank Liddell owned all of the shares in said
E.J. Darras 1,225 122,500 6.45% corporation. 45% accrued to the employees, parties
On January 31, 1947, with the limited paid-in capital of
P20,000, Liddell & Co. was able to declare a 90% stock thereto; Kurz 12-1/2%; Darras 12-1/2%; A. Manzano 12-1/2%
Angel Manzano 1,150 115,000 6.06%
dividend after which declaration on, Frank Liddells holding and Julian Serrano 7-1/2%. The agreement Exhibit A was
in the Company increased to 1,960 shares and the also made retroactive to 1946. Frank Liddell reserved the
Julian Serrano 710 71,000 3.74%
employees, Charles Kurz E.J. Darras, Angel Manzano and right to reapportion the 45% dividends pertaining to the
Julian Serrano at 10 share each. The declaration of stock E. Hasim 500 50,000 2.64% employees in the future for the purpose of including such
dividend was followed by a resolution increasing the other faithful and efficient employees as he may
authorized capital of the company to P1,000.000 which the subsequently designate. (As a matter of fact, Frank Liddell

Page 133 of 160


CORPORATION LAW CASES MAYORDO, M.A
LLB 3

did so designate two additional employees namely: E. surcharges, in the amount of P1,317,629.61. In the law governing the procedure in the court of Industrial
Hasim and G. W. Kernot). It was for such inclusion of future computation, the gross selling price of Liddell Motors, Inc. Relations, there is no provision invalidating decisions
faithful employees that Exhibits B-1 and C were executed. to the general public from January 1, 1949 to September rendered after the lapse of 30 days, the requirement of
As per Exhibit C, dated May 13, 1948, the 45% given by 15, 1950, was made the basis without deducting from the Section 13, Republic Act No. 1125 should be construed as
Frank Liddell to his employees was reapportioned as selling price, the taxes already paid by Liddell & Co. in its directory.4
follows: C. Kurz — 12,%; E. J. Darras — 12%; A. Manzano — sales to the Liddell Motors Inc.
l2%; J. Serrano — 3-1/2%; G. W. Kernot — 2%. Besides as pointed out by appellee, the third paragraph of
The Court of Tax Appeals upheld the position taken by the Section 13 of Republic Act No. 1125 (quoted in the
Exhibit B contains the employees' definition in detail of the Collector of Internal Revenue. margin)5  confirms this view; because in providing for two
manner by which they sought to prevent their share- thirty-day periods, the law means that decision may still be
holdings from being transferred to others who may be A.  Judge Umali: Appellant urges the disqualification on of rendered within the second period of thirty days (Judge
complete strangers to the business on Liddell & Co. Judge Roman M. Umali to participate in the decision of the Umali signed his decision within that period).
instant case because he was Chief of the Law Division, then
From 1946 until November 22, 1948 when the purpose Acting Deputy Collector and later Chief Counsel of the B.  Identity of the two corporations: On the question
clause of the Articles of Incorporation of Liddell & Co. Inc., Bureau of Internal Revenue during the time when the whether or not Liddell Motors, Inc. is the  alter ego  of
was amended so as to limit its business activities to assessment in question was made.1 In refusing to disqualify Liddell & Co. Inc., we are fully convinced that Liddell & Co.
importations of automobiles and trucks, Liddell & Co. was himself despite admission that had held the is wholly owned by Frank Liddell. As of the time of its
engaged in business as an importer and at the same time aforementioned offices, Judge Umali stated that he had not organization, 98% of the capital stock belonged to Frank
retailer of Oldsmobile and Chevrolet passenger cars and in any way participated, nor expressed any definite Liddell. The 20% paid-up subscription with which the
GMC and Chevrolet trucks. opinion, on any question raised by the parties when this company began its business was paid by him. The
case was presented for resolution before the said bureau. subsequent subscriptions to the capital stock were made by
On December 20, 1948, the Liddell Motors, Inc. was Furthermore, after careful inspection of the records of the him and paid with his own money.
organized and registered with the Securities and Exchange Bureau, he (Judge Umali as well as the other members of
Commission with an authorized capital stock of P100,000 of the court below), had not found any indication that he had These stipulations and conditions appear in Exhibit A: (1)
which P20,000 was subscribed and paid for as follows: Irene expressed any opinion or made any decision that would that Frank Liddell had the authority to designate in the
Liddell wife of Frank Liddell 19,996 shares and Messrs. tend to disqualify him from participating in the future the employee who could receive earnings of the
Marcial P. Lichauco, E. K. Bromwell, V. E. del Rosario and consideration of the case in the Tax Court. corporation; to apportion among the stock holders the
Esmenia Silva, 1 share each. share in the profits; (2) that all certificates of stock in the
At this juncture, it is well to consider that petitioner did names of the employees should be deposited with Frank
At about the end of the year 1948, Messrs. Manzano, Kurz not question the truth of Judge Umali's statements. In view Liddell duly indorsed in blank by the employees concerned;
and Kernot resigned from their respective positions in the thereof, this Tribunal is not inclined to disqualify said (3) that each employee was required to sign an agreement
Retail Dept. of Liddell & Co. and they were taken in and judge. Moreover, in furtherance of the presumption of the with the corporation to the effect that, upon his death or
employed by Liddell Motors, Inc.: Kurz as Manager- judge's moral sense of responsibility this Court has adopted, upon his retirement or separation for any cause whatsoever
Treasurer, Manzano as General Sales Manager for cars and and now here repeats, the ruling that the mere from the corporation, the said corporation should, within a
Kernot as General Sales Manager for trucks. participation of a judge in prior proceedings relating to the period of sixty days therefor, have the absolute and
subject in the capacity of an administrative official does exclusive option to purchase and acquire the whole of the
Beginning January, 1949, Liddell & Co. stopped retailing not necessarily disqualify him from acting as judge.2 stock interest of the employees so dying, resigning, retiring
cars and trucks; it conveyed them instead to Liddell Motors, or separating.
Inc. which in turn sold the vehicles to the public with a Appellant also contends that Judge Umali signed the said
steep mark-up. Since then, Liddell & Co. paid sales taxes decision contrary to the provision of Section 13, Republic These stipulations in our opinion attest to the fact that
on the basis of its sales to Liddell Motors Inc. considering Act No. 1125;3  that whereas the case was submitted for Frank Liddell also owned it. He supplied the original his
said sales as its original sales. decision of the Court of Tax Appeals on July 12, 1955, and complete control over the corporation.
the decision of Associate Judge Luciano and Judge Nable
Upon review of the transactions between Liddell & Co. and were both signed on August 11, 1955 (that is, on the last As to Liddell Motors, Inc. we are fully persuaded that Frank
Liddell Motors, Inc. the Collector of Internal Revenue day of the 30-day period provided for in Section 13, Liddell also owned it. He supplied the original capital
determined that the latter was but an alter ego of Liddell & Republic Act No. 1125), Judge Umali signed the decision funds.6  It is not proven that his wife Irene, ostensibly the
Co. Wherefore, he concluded, that for sales tax purposes, August 31, 1955 or 20 days after the lapse of the 30-day sole incorporator of Liddell Motors, Inc. had money of her
those sales made by Liddell Motors, Inc. to the public were period allotted by law. own to pay for her P20,000 initial subscription.7  Her
considered as the original sales of Liddell & Co. income in the United States in the years 1943 and 1944 and
Accordingly, the Collector of Internal Revenue assessed By analogy it may be said that inasmuch as in Republic Act the savings therefrom could not be enough to cover the
against Liddell & Co. a sales tax deficiency, including No. 1125 (law creating the Court of Tax Appeals) like the amount of subscription, much less to operate an expensive

Page 134 of 160


CORPORATION LAW CASES MAYORDO, M.A
LLB 3

trade like the retail of motor vehicles. The alleged sale of conducted lawfully and without fraud on another, its Consistently with this view, the United States Supreme
her property in Oregon might have been true, but the separate identity is to be respected. Court14 held that "a taxpayer may gain advantage of doing
money received therefrom was never shown to have been business thru a corporation if he pleases, but the revenue
saved or deposited so as to be still available at the time of Accordingly, the mere fact that Liddell & Co. and Liddell officers in proper cases, may disregard the separate
the organization of the Liddell Motors, Inc. Motors, Inc. are corporations owned and controlled by corporate entity where it serves but as a shield for tax
Frank Liddell directly or indirectly is not by itself sufficient evasion and treat the person who actually may take the
The evidence at hand also shows that Irene Liddell had to justify the disregard of the separate corporate identity benefits of the transactions as the person accordingly
scant participation in the affairs of Liddell Motors, Inc. She of one from the other. There is, however, in this instant taxable."
could hardly be said to possess business experience. The case, a peculiar consequence of the organization and
income tax forms record no independent income of her activities of Liddell Motors, Inc. Thus, we repeat: to allow a taxpayer to deny tax liability
own. As a matter of fact, the checks that represented her on the ground that the sales were made through an other
salary and bonus from Liddell Motors, Inc. found their way Under the law in force at the time of its incorporation the and distinct corporation when it is proved that the latter is
into the personal account of Frank Liddell. Her frequent sales tax on original sales of cars (sections 184, 185 and 186 virtually owned by the former or that they are practically
absences from the country negate any active participation of the National Internal Revenue Code), was progressive, one and the same is to sanction a circumvention of our tax
in the affairs of the Motors company. i.e. 10% of the selling price of the car if it did not exceed laws.15
P5000, and 15% of the price if more than P5000 but not
There are quite a series of conspicuous circumstances that more than P7000, etc. This progressive rate of the sales tax C. Tax liability computation: In the Yutivo case16 the same
militate against the separate and distinct personality of naturally would tempt the taxpayer to employ a way of question involving the computation of the alleged
Liddell Motors, Inc. from Liddell & Co.8 We notice that the reducing the price of the first sale. And Liddell Motors, Inc. deficiency sales tax has been raised. In accordance with
bulk of the business of Liddell & Co. was channeled through was the medium created by Liddell & Co. to reduce the our ruling in said case we hold as correctly stated by Judge
Liddell Motors, Inc. On the other hand, Liddell Motors, Inc. price and the tax liability. Nable in his concurring and dissenting opinion on this case,
pursued no activities except to secure cars, trucks, and that the deficiency sales tax should be based on the selling
spare parts from Liddell & Co. Inc. and then sell them to Let us illustrate: a car with engine motor No. 212381 was price obtained by Liddell Motors, Inc. to the public AFTER
the general public. These sales of vehicles by Liddell & Co. sold by Liddell & Co. Inc. to Liddell Motors, Inc. on January DEDUCTING THE TAX ALREADY PAID BY LIDDELL & CO., INC.
to Liddell Motors, Inc. for the most part were shown to 17, 1948 for P4,546,000.00 including tax; the price of the in its sales to Liddell Motors, Inc.
have taken place on the same day that Liddell Motors, Inc. car was P4,133,000.23, the tax paid being P413.22, at 10%.
sold such vehicles to the public. We may even say that the And when this car was later sold (on the same day) by On the imposition of the 50% surcharge by reason of fraud,
cars and trucks merely touched the hands of Liddell Motors, Liddell Motors, Inc. to P.V. Luistro for P5500, no more sales we see that the transactions between Liddell Motors Inc.
Inc. as a matter of formality. tax was paid.11  In this price of P5500 was included the and Liddell & Co., Inc. have always been embodied in
P413.32 representing taxes paid by Liddell & Co. Inc. in the proper documents, constantly subject to inspection by the
During the first six months of 1949, Liddell & Co. issued ten sale to Liddell Motors, Inc. Deducting P413.32 representing tax authorities. Liddell & Co., Inc. have always made a full
(10) checks payable to Frank Liddell which were deposited taxes paid by Liddell & Co., Inc. the price of P5500, the report of its income and receipts in its income tax returns.
by Frank Liddell in his personal account with the Philippine balance of P5,087.68 would have been the net selling price
National Bank. During this time also, he issued in favor of of Liddell & Co., Inc. to the general public (had Liddell Paraphrasing our decision in the Yutivo case, we may now
Liddell Motors, Inc. six (6) checks drawn against his Motors, Inc. not participated and intervened in the sale), say, in filing its return on the basis of its sales to Liddell
personal account with the same bank. The checks issued by and 15% sales tax would have been due. In this transaction, Motors, Inc. and not on those by the latter to the public, it
Frank Liddell to the Liddell Motors, Inc. were significantly P349.68 in the form of taxes was evaded. All the other cannot be held that the Liddell & Co., Inc. deliberately
for the most part issued on the same day when Liddell & transactions (numerous) examined in this light will made a false return for the purpose of defrauding the
Co. Inc. issued the checks for Frank Liddell9  and for the inevitably reveal that the Government coffers had been government of its revenue, and should suffer a 50%
same amounts. deprived of a sizeable amount of taxes. surcharge. But penalty for late payment (25%) should be
imposed.
It is of course accepted that the mere fact that one or As opined in the case of Gregory v. Helvering,12 "the legal
more corporations are owned and controlled by a single right of a taxpayer to decrease the amount of what In view of the foregoing, the decision appealed from is
stockholder is not of itself sufficient ground for otherwise would be his taxes, or altogether avoid them by hereby modified: Liddell & Co., Inc. is declared liable only
disregarding separate corporate entities. means which the law permits, cannot be doubted." But, as for the amount of P426,811.67 with 25% surcharge for late
Authorities10  support the rule that it is lawful to obtain a held in another case,13 "where a corporation is a dummy, is payment and 6% interest thereon from the time the
corporation charter, even with a single substantial unreal or a sham and serves no business purpose and is judgment becomes final.
stockholder, to engage in a specific activity, and such intended only as a blind, the corporate form may be
activity may co-exist with other private activities of the ignored for the law cannot countenance a form that is bald As it appears that, during the pendency of this litigation
stockholder. If the corporation is a substantial one, and a mischievous fiction." appellant paid under protest to the Government the total

Page 135 of 160


CORPORATION LAW CASES MAYORDO, M.A
LLB 3

amount assessed by the Collector, the latter is hereby With the case already pending in the industrial court, the housing the gaugau factory to his son Tan Keng Lim,
required to return the excess to the petitioner. No costs. Secretary of Labor, on September 5, 1951, revoked manager of the La Campana Coffee Factory. But the lease
the  Kalipunan Ng Mga Kaisahang Manggagawa's  permit as a was executed only on September 1, 1951, while the dispute
G.R. No. L-5677             May 25, 1953 labor union on the strength of information received that it between the parties was pending before the Court.
LA CAMPANA FACTORY, INC., and TAN TONG doing was dominated by subversive elements, and, in
business under the trial name "LA CAMPANA GAUGAU consequence, on the 20th of the same month, also C. There is only one entity La Campana Starch and Coffee
PACKING", petitioners,  vs. KAISAHAN NG MGA suspended the permit of its affiliate, the Factory, as shown by the signboard (Exhibit 1), the
MANGGAGAWA SA LA CAMPANA (KKM) and THE COURT OF respondent Kaisahan. advertisement in the delivery trucks (Exhibit I-1), the
INDUSTRIAL RELATIONS, respondents. packages of gaugau(Exhibit K), and delivery forms (Exhibits
We have it from the court's order of January 15, 1952, J, J-1, and J-2).
REYES, J.: which forms one of the annexes to the present petition,
Tan Tong, one of the herein petitioners, has since 1932 that following the revocation of the Kaisahan's permit, "La D. All the laborers working in the gaugau or in the coffee
been engaged in the business of buying and selling gaugau Campana Gaugau and Coffee Factory" (obviously the factory receive their pay from the same person, the
under the trade name La Campana Gaugau Packing with an combined name of La Campana Gaugau Packing and La cashier, Miss Natividad Garcia, secretary of Mr. Tan Tong;
establishment in Binondo, Manila, which was later Campana Coffee Factory Co., Inc,) and the PLOW, which and they are transferred from the gaugau to the coffee and
transferred to España Extension, Quezon City. But on July had been allowed to intervene as a party having an interest vice-versa as the management so requires.
6, 1950, Tan Tong, with himself and members of his family in the dispute, filed separate motions for the dismissal of
corporation known as La Campana Factory Co., Inc., with the case on the following grounds: E. There has been only one payroll for the entire La
its principal office located in the same place as that of La Campana personnel and only one person preparing the
Campana Gaugau Packing. 1. That the action is directed against two different entities same — Miss Natividad Garcia, secretary of Mr. Tan Tong.
with distinct personalities, with "La Campana Starch But after the case at bar was certified to this Court on July
About a year before the formation of the corporation, or on Factory" and the "La Campana Coffee Factory, Inc."; 17, 1951, the company began making separate payrolls for
July 11, 1949, Tan Tong had entered into a collective the coffee factory (Exhibits M-2 and M-3, and for the
bargaining agreement with the Philippine Legion of 2. That the workers of the "La Campana Coffee Factory, gaugau factory (Exhibits O-2, O-3 and O-4). It is to be noted
Organized Workers, known as PLOW for short, to which the Inc." are less than thirty-one; that before July 21, 1951, the coffee payrolls all began
union of Tan Tong's employees headed by Manuel E. Sadde with number "41-Maria Villanueva" with 24 or more laborers
was then affiliated. Seceding, however, from the PLOW, Tan 3. That the petitioning union has no legal capacity to sue, (Exhibits M and M-1), whereas beginning July 21, 1951, the
Tong's employees later formed their own organization because its registration as an organized union has been payrolls for the coffee factory began with No. 1-Loreta
known as  Kaisahan Ng Mga Manggagawa Sa La Campana, revoked by the Department of Labor on September 5, 1951; Bernabe with only 14 laborers (Exhibits M-2 and M-3).
one of the herein respondents, and applied for registration and
in the Department of Labor as an independent entity. F. During the ocular inspection made in the factory on
Pending consideration of this application, the Department 4. That there is an existing valid contract between the August 26, 1951 the Court has found the following:
gave the new organization legal standing by issuing it a respondent "La Campana Gaugau Packing" and the
permit as an affiliate to the Kalipunan Ng Mga Manggagawa. intervenor PLOW, where-in the petitioner's members are In the ground floor and second floor of the gaugau factory
contracting parties bound by said contract. there were hundreds of bags of raw coffee behind the pile
On July 19, 1951, the  Kaisahan Ng Mga Manggagawa Sa La of gaugau sacks. There were also women employees
Campana, hereinafter to be referred to as the respondent Several hearings were held on the above motions, in the working paper wrappers for gaugau, and, in the same place
Kaisahan, which, as of that date, counted with 66 members course of which ocular inspections were also made, and on there were about 3,000 cans to be used as containers for
— workers all of them of both La Campana Gaugau Packing the basis of the evidence received and the facts observed coffee.
and La Campana Coffee Factory Co., Inc. — presented a in the ocular inspections, the Court of Industrial Relations
demand for higher wages and more privileges, the demand denied the said motions in its order of January 14, 1952, The Court found out also that there were 16 trucks used
being addressed to La Campana Starch and Coffee Factory, because if found as a fact that: both for the delivery of coffee and gaugau. To show that
by which name they sought to designate, so it appears, the those trucks carried both coffee and gaugau, the union
La Campana Gaugau Packing and the La Campana Coffee A. While the coffee corporation is a family corporation with president invited the Court to examine the contents of
Factory Co., Inc. As the demand was not granted and an Mr. Tan Tong, his wife, and children as the incorporations delivery truck No. T-582 parked in a garage between the
attempt at settlement through the mediation of the and stockhelders (Exhibit 1), the La Campana Gaugau gaugau building and the coffee factory, and upon
Conciliation Service of the Department of Labor had given Packing is merely a business name (Exhibit 4). examination, there were found inside the said truck boxes
no result, the said Department certified the dispute to the of gaugau and cans of coffee, and held that:
Court of Industrial Relations on July 17, 1951, the case B. According to the contract of lease (Exhibit 23), Mr. Tan
being there docketed as Case No. 584-V. Tong., propriety and manager of the Ka Campana Gaugau . . . there is only one management for the business of
Factory, leased a space of 200 square meters in the bodega gaugau and coffee with whom the laborers are dealing

Page 136 of 160


CORPORATION LAW CASES MAYORDO, M.A
LLB 3

regarding their work. Hence, the filing of action against the ends of justice cannot be invoked to further an end This jurisdiction was not when the Department of Labor
Ka Campana Starch and Coffee Factory is proper and subversive of that purpose. suspended the permit of the respondent Kaisahan as a labor
justified. organization. For once jurisdiction is acquired by the Court
Disregarding Corporate Entity. — The doctrine that a of Industrial Relations it is retained until the case is
With regards to the alleged lack of personality, it is to be corporation is a legal entity existing separate and apart completely decided. (Manila Hotel Employees
noted that before the certification of the case to this Court from the person composing it is a legal theory introduced Association vs. Manila Hotel Co. et al., 73 Phil., 374.)
on July 17, 1951, the petitioner  Kaisahan Ng Mga for purposes of convenience and to subserve the ends of
Manggagawa Sa La Campana, had a separate permit from justice. The concept cannot, therefore, be extended to a In view of the foregoing, the petition is denied, with costs
the Department of Labor. This permit was suspended on point beyond its reason and policy, and when invoked in against the petitioner.
September 30, 1951. (Exhibit M-Intervenor, page 55, of the support of an end subversive of this policy, will be
record). It is not true that, on July 17, 1951, when this case disregarded by the courts. Thus, in an appropriate case and G.R. No. 182770               September 17, 2014
forwarded to this Court, the petitioner's permit, as an in furtherance of the ends of justice, a corporation and the WPM INTERNATIONAL TRADING, INC. and WARLITO P.
independent union, had not yet been issued, for the very individual or individuals owning all its stocks and assets will MANLAPAZ, Petitioners,  vs. FE CORAZON,
Exhibit MM-Intervenor regarding the permit, conclusively be treated as identical, the corporate entity being LABAYEN, Respondent.
shows the preexistence of said permit. (Annex G.) disregarded where used as a cloak or cover for fraud or
illegality. (13 Am. Jur., 160-161.) DECISION
Their motion for reconsideration of the above order having BRION, J.:
been denied, Tan Tong and La Campana Coffee Factory, Inc. . . . A subsidiary or auxiliary corporation which is created
(same as La Campana Coffee Factory Co., Inc.), later by a parent corporation merely as an agency for the latter We review in this petition for review on certiorari1  the
joined by the PLOW, filed the present petition for certiorari may sometimes be regarded as identical with the parent d e c i s i o n 2  d a t e d S e p t e m b e r 2 8 , 2 0 0 7 a n d t h e
on the grounds that the Court of Industrial Relations had no corporation, especially if the stockholders or officers of the resolution3  dated April 28, 2008 of the Court of Appeals
jurisdiction to take cognizance of the case, for the reason, two corporations are substantially the same or their system (CA) in CA-G.R. CV No. 68289 that affirmed with
according to them, "(1) that the petitioner La Campana of operation unified. (Ibid. 162; see Annotation 1 A. L. R. modification the decision4  of the Regional Trial Court
Coffee Factory, Inc. has only 14 employees, only 5 of whom 612, s. 34 A. L. R. 599.) (RTC), Branch 77, Quezon City.
are members of the respondent union and therefore the
absence of the jurisdictional number (30) as provided by In the present case Tan Tong appears to be the owner of The Factual Background
sections 1 and 4 of Commonwealth Act No. 103; and, (2) the  gaugau  factory. And the coffee factory, though an
that the suspension of respondent union's permit by the incorporated business, is in reality owned exclusively by The respondent, Fe Corazon Labayen, is the owner of
Secretary of Labor has the effect of taking away the union's Tan Tong and his family. As found by the Court of industrial H.B.O. Systems Consultants, a management and consultant
right to collective bargaining under section 2 of Relations, the two factories have but one office, one firm. The petitioner, WPM International Trading, Inc.
Commonwealth Act No. 213 and consequently, its management and one payroll, except after July 17, the day (WPM), is a domestic corporation engaged in the restaurant
personality to sue for ad in behalf of its members." the case was certified to the Court of Industrial Relations, business, while Warlito P. Manlapaz (Manlapaz) is its
when the person who was discharging the office of cashier president.
As to the first ground, petitioners obviously do not question for both branches of the business began preparing separate
the fact that the number of employees of the La Campana payrolls for the two. And above all, it should not be Sometime in 1990, WPM entered into a management
Gaugau Packing involved in the case is more than the overlooked that, as also found by the industrial court, the agreement with the respondent, by virtue of which the
jurisdictional number (31) required bylaw, but they do laborers of the gaugau factory and the coffee factory were respondent was authorized to operate, manage and
contend that the industrial court has no jurisdiction to try interchangeable, that is, the laborers from the gaugau rehabilitate Quickbite, a restaurant owned and operated by
the case as against La Campana Coffee Factory, Inc. factory were sometimes transferred to the coffee factory WPM. As part of her tasks, the respondent looked for a
because the latter has allegedly only 14 laborers and only and vice-versa. In view of all these, the attempt to make contractor who would renovate the two existing Quickbite
of these are members of the respondent Kaisahan. This the two factories appears as two separate businesses, when outlets in Divisoria, Manila and Lepanto St., University Belt,
contention loses force when it is noted that, as found by in reality they are but one, is but a device to defeat the Manila. Pursuant to the agreement, the respondent
the industrial court — and this finding is conclusive upon us ends of the law (the Act governing capital and labor engaged the services of CLN Engineering Services (CLN) to
— La Campana Gaugau Packing and La Campana Coffee relations) and should not be permitted to prevail. renovate Quickbite-Divisoria at the cost of ₱432,876.02.
Factory Co. Inc., are operating under one single
management, that is, as one business though with two The second point raised by petitioners is likewise with-out On June 13, 1990, Quickbite-Divisoria’s renovation was
trade names. True, the coffee factory is a corporation and, merit. In the first place, there being more than 30 laborers finally completed, and its possession was delivered to the
by legal fiction, an entity existing separate and apart fro involved and the Secretary of Labor having certified the respondent. However, out of the ₱432,876.02 renovation
the persons composing it, that is, Tan Tong and his family. dispute to the Court of Industrial Relations, that court duly cost, only the amount of ₱320,000.00 was paid to CLN,
But it is settled that this fiction of law, which has been acquired jurisdiction over the case (International Oil leaving a balance of ₱112,876.02.
introduced as a matter of convenience and to subserve the Factory  vs.  NLU, Inc. 73 Phil., 401; section 4, C. A. 103).

Page 137 of 160


CORPORATION LAW CASES MAYORDO, M.A
LLB 3

Complaint for Sum of Money (Civil Case No. Q-90-7013) separate and distinct personality, Manlapaz cannot be made The CA applied the principle of piercing the veil of
liable for the respondent’s claim. corporate fiction and agreed with the RTC that Manlapaz
On October 19, 1990, CLN filed a complaint for sum of cannot evade his liability by simply invoking WPM’s
money and damages before the RTC against the respondent Manlapaz prayed for the dismissal of the complaint for lack separate and distinct personality.
and Manlapaz, which was docketed as Civil Case No. of cause of action, and by way of counterclaim, for the After the CA's denial of their motion for reconsideration,
Q-90-7013. CLN later amended the complaint to exclude award of ₱350,000.00 as moral and exemplary damages and the petitioners filed the present petition for review on
Manlapaz as defendant. The respondent was declared in ₱50,000.00 attorney’s fees. certiorari under Rule 45 of the Rules of Court.
default for her failure to file a responsive pleading.
The RTC, through an order dated March 2, 1993 declared The Petition
The RTC, in its January 28, 1991 decision, found the WPM in default for its failure to file a responsive pleading.
respondent liable to pay CLN actual damages inthe amount The petitioners submit that the CA gravely erred in
of ₱112,876.02 with 12% interest per annum from June The Decision of the RTC sustaining the RTC’s application of the principle of piercing
18,1990 (the date of first demand) and 20% of the amount the veil of corporate fiction. They argue that the legal
recoverable as attorney’s fees. In its decision, the RTC held that the respondent is entitled fiction of corporate personality could only be discarded
to indemnity from Manlapaz. The RTC found that based on upon clear and convincing proof that the corporation is
Complaint for Damages (Civil Case No. Q-92-13446) the records, there is a clear indication that WPM is a mere being used as a shield to avoid liability or to commit a
instrumentality or business conduit of Manlapaz and as fraud. Since the respondent failed to establish that any of
Thereafter, the respondent instituted a complaint for such, WPM and Manlapaz are considered one and the same. the circumstances that would warrant the piercing is
damages against the petitioners, WPM and Manlapaz. The The RTC also found that Manlapaz had complete control present, Manlapaz claims that he cannot be made solidarily
respondent alleged that in Civil Case No. Q-90-7013, she over WPM considering that he is its chairman, president and liable with WPM to answerfor damages allegedly incurred
was adjudged liable for a contract that she entered into for treasurer at the same time. The RTC thus concluded that by the respondent.
and in behalf of the petitioners, to which she should be Manlapaz is liable in his personal capacity to reimburse the
entitled to reimbursement; that her participation in the respondent the amount she paid to CLN inconnection with The petitioners further argue that, assuming they may be
management agreement was limited only to introducing the renovation agreement. held liable to reimburse to the respondentthe amount she
Manlapaz to Engineer Carmelo Neri (Neri), CLN’s general The petitioners appealed the RTC decision with the CA. paid in Civil Case No. Q-90-7013, such liability is only
manager; that it was actually Manlapaz and Neri who There, they argued that in view of the respondent’s act of limited to the amount of ₱112,876.02, representing the
agreed on the terms and conditions of the agreement; that entering into a renovation agreement with CLN in excess of balance of the obligation to CLN, and should not include
when the complaint for damages was filed against her, she her authority as WPM’s agent, she is not entitled to the twelve 12% percent interest, damages and attorney’s
was abroad; and that she did not know of the case until she indemnity for the amount she paid. Manlapaz also fees.
returned to the Philippines and received a copy of the contended that by virtue ofWPM’s separate and distinct
decision of the RTC. personality, he cannot be madesolidarily liable with WPM. The Issues

In her prayer, the respondent sought indemnification in the The Ruling of the Court of Appeals The core issues are: (1) whether WPM is a mere
amount of ₱112,876.60 plus interest at 12%per annum from instrumentality, alter-ego, and business conduit of
June 18, 1990 until fully paid; and 20% of the award as On September 28, 2007, the CA affirmed, with modification Manlapaz; and (2) whether Manlapaz is jointly and severally
attorney’s fees. She likewise prayed that an award of on the award of attorney’s fees, the decision of the liable with WPM to the respondent for reimbursement,
₱100,000.00 as moral damages and ₱20,000.00 as RTC.The CA held that the petitioners are barred from damages and interest.
attorney’s fees be paid to her. raising as a defense the respondent’s alleged lack of
authority to enter into the renovation agreement in view of Our Ruling
In his defense, Manlapaz claims that it was his fellow their tacit ratification of the contract.
incorporator/director Edgar Alcansajewho was in-charge We find merit in the petition.
with the daily operations of the Quickbite outlets; that The CA likewise affirmed the RTC ruling that WPM and
when Alcansaje left WPM, the remaining directors were Manlapaz are one and the same based on the following: (1) We note, at the outset, that the question of whether a
compelled to hire the respondent as manager; that the Manlapaz is the principal stockholder of WPM; (2) Manlapaz corporation is a mere instrumentality or alter-ego of
respondent had entered intothe renovation agreement with had complete control over WPM because he concurrently another is purely one of fact.5 This is also true with respect
CLN in her own personal capacity; that when he found the held the positions of president, chairman of the board and to the question of whether the totality of the evidence
amount quoted by CLN too high, he instructed the treasurer, in violation of the Corporation Code; (3) two of adduced by the respondentwarrants the application of the
respondent to either renegotiate for a lower price or to the four other stockholders of WPM are employed by piercing the veil of corporate fiction doctrine.6
look for another contractor; that since the respondent had Manlapaz either directly or indirectly; (4) Manlapaz’s
exceeded her authority as agent of WPM, the renovation residence is the registered principal office of WPM; and (5) Generally, factual findings of the lower courts are accorded
agreement should only bind her; and that since WPM has a the acronym "WPM" was derived from Manlapaz’s initials. the highest degree of respect, if not finality. When adopted

Page 138 of 160


CORPORATION LAW CASES MAYORDO, M.A
LLB 3

and confirmed by the CA, these findings are final and On the contrary, the evidence establishes that CLN and the
conclusive and may not be reviewed on appeal,7save in (3) The aforesaid control and breach of duty must have respondent knew and acted on the knowledgethat they
some recognized exceptions8  among others, when the proximately caused the injury or unjust loss complained of. were dealing with WPM for the renovation of the latter’s
judgment is based on misapprehension of facts. The absence of any ofthese elements prevents piercing the restaurant, and not with Manlapaz. That WPM later reneged
corporate veil.12 on its monetary obligation to CLN, resulting to the filing of
We have reviewed the records and found that the a civil case for sum of money against the respondent, does
application of the principle of piercing the veil of corporate In the present case, the attendantcircumstances do not not automatically indicate fraud, in the absence of any
fiction is unwarranted in the present case. establish that WPM is a mere alter ego of Manlapaz. proof to support it.

On the Application ofthe Principle of Piercing the Veil of Aside from the fact that Manlapaz was the principal This Court also observed that the CA failed to demonstrate
Corporate Fiction stockholder of WPM, records do not show that WPM was how the separate and distinct personalityof WPM was used
organized and controlled, and its affairs conducted in a by Manlapaz to defeat the respondent’s right for
The rule is settled that a corporation has a personality manner that made it merely an instrumentality, agency, reimbursement. Neither was there any showing that WPM
separate and distinct from the persons acting for and in its conduit or adjunct ofManlapaz. As held in Martinez v. Court attempted to avoid liability or had no property against
behalf and, in general, from the people comprising it. of Appeals,13 the mere ownership by a singlestockholder of which to proceed.
9  Following this principle, the obligations incurred by the even all or nearly all of the capital stocks ofa corporation is
corporate officers, orother persons acting as corporate not by itself a sufficient ground to disregard the separate Since no harm could be said to have been proximately
agents, are the direct accountabilities ofthe corporation corporate personality. To disregard the separate juridical caused by Manlapaz for which the latter could be held
they represent, and not theirs. Thus, a director, officer or personality of a corporation, the wrongdoing must be solidarily liable with WPM, and considering that there was
employee of a corporation is generally not held personally clearly and convincingly established.14 no proof that WPM had insufficient funds, there was no
liable for obligations incurred by the corporation;10  it is sufficient justification for the RTC and the CA to have ruled
only in exceptional circumstances that solidary liability will Likewise, the records of the case do not support the lower that Manlapaz should be held jointly and severally liable to
attach to them. courts’ finding that Manlapaz had control or domination the respondent for the amount she paid to CLN. Hence,
over WPM or its finances. That Manlapaz concurrentlyheld only WPM is liable to indemnify the respondent.
Incidentally, the doctrine of piercing the corporate veil the positions of president, chairman and treasurer, or that
applies only in three (3) basic instances, namely: a) when the Manlapaz’s residence is the registered principal office Finally, we emphasize that the piercing of the veil of
the separate and distinct corporate personality defeats of WPM, are insufficient considerations to prove that he corporate fiction is frowned upon and thus, must be done
public convenience, as when the corporate fiction is used had exercised absolutecontrol over WPM. with caution.15  It can only be done if it has been clearly
as a vehicle for the evasion of an existing obligation; b) in established that the separate and distinct personality of
fraud cases, or when the corporate entity is used to justify In this connection, we stress thatthe control necessary to the corporation is used to justify a wrong, protect fraud, or
a wrong, protect a fraud, or defend a crime; or c) is used in invoke the instrumentality or alter ego rule is not majority perpetrate a deception. The court must be certain that the
alter ego cases, i.e., where a corporation is essentially a or even complete stock control but such domination of corporate fiction was misused to such an extent that
farce, since it is a mere alter ego or business conduit of a finances, policies and practices that the controlled injustice, fraud, or crime was committed against another,
person, or where the corporation is so organized and corporation has, so tospeak, no separate mind, will or in disregard of its rights; it cannot be presumed.
controlled and its affairs so conducted as to make it merely existence of its own, and is but a conduit for its principal.
aninstrumentality, agency, conduit or adjunct of another The control must be shown to have been exercised at the On the Award of Moral Damages
corporation.11 time the acts complained of took place. Moreover, the
control and breach of duty must proximately cause the On the award of moral damages, we find the same in order
Piercing the corporate veil based on the alter ego theory injury or unjust loss for which the complaint is made. in view of WPM's unjustified refusal to pay a just debt.
requires the concurrence of three elements, namely: Under Article 2220 of the New Civil Code,16 moral damages
Here, the respondent failed to prove that Manlapaz, acting may be awarded in cases of a breach of contract where the
(1) Control, not mere majority or complete stock control, as president, had absolute control over WPM.1âwphi1 Even defendant acted fraudulently or in bad faith or was guilty
but complete domination, not only of finances but of policy granting that he exercised a certain degree of control over of gross negligence amounting to bad faith.
and business practice in respect to the transaction attacked the finances, policies and practices of WPM, in view of his
so that the corporate entity as to this transaction had at position as president, chairman and treasurer of the In the present case, when payment for the balance of the
the time no separate mind, will or existence of its own; corporation, such control does not necessarily warrant renovation cost was demanded, WPM, instead of complying
piercing the veil of corporate fiction since there was not a with its obligation, denied having authorized the
(2) Such control must have beenused by the defendant to single proof that WPM was formed to defraud CLN or the respondent to contract in its behalf and accordingly refused
commit fraud or wrong, to perpetuate the violation of a respondent, or that Manlapaz was guilty of bad faith or to pay. Such cold refusal to pay a just debt amounts to a
statutory or other positive legal duty, or dishonest and fraud. breach of contract in bad faith, as contemplated by Article
unjust act in contravention of plaintiff’s legal right; and

Page 139 of 160


CORPORATION LAW CASES MAYORDO, M.A
LLB 3

2220. Hence, the CA's order to pay moral damages was in 
 the Court of Industrial Relations on July 17, 1951, where it
order. DECISION was docketed as Case No. 584-V. On the theory that the
MONTEMAYOR, J.: laborers presenting the demands were only the ones
WHEREFORE, in light of the foregoing, the decision dated 
 working in the coffee factory, said company filed through
September 28, 2007 of the Court of Appeals in CA-G.R. CV This is a petition for certiorari and prohibition with prayer the management a motion to dismiss claiming that
No. 68289 is MODIFIED and that petitioner Warlito P. for issuance of a writ of preliminary injunction to prohibit inasmuch as there were only 14 of them in said factory, the
Manlapaz is ABSOLVED from any liability under the respondent Court of Industrial Relations from proceeding Court of Industrial Relations had no jurisdiction to entertain
renovation agreement. with the hearing of the contempt proceedings for which and decide the case. The motion was denied by the Court
petitioner Ricardo Tantongco was cited to appear and of Industrial Relations, which said:
SO ORDERED. present his evidence. The contempt proceedings which 

petitioner seeks to stop are based on the order of the Court ". . . There was only one management for the business of
of Industrial Relations, dated September 30, 1957, which gawgaw and coffee with whom the laborers are dealing
reads as follows: regarding their work. Hence, the filing of action against the
La Campana Starch and Coffee Factory is proper and
"It appearing that the Order of this Court, in the above- justified."
entitled case, dated February 18, 1957 (folios 134-166), has 

become final and executory and the respondents have The order of denial was appealed to this Tribunal through
failed to comply with the same, the said respondents, certiorari under G. R. No. L-5677. In disposing of the case,
namely, the La Campana Starch and Coffee Factory or its we held: 

manager or the person who has charge of the management, 

and the administrator of the Estate of Ramon Tantongco are "As to the first ground, petitioners obviously do not
hereby ordered to comply with said Order, within five days question the fact that the number of employees La
from receipt hereof, particularly the following, to wit: 
 Campana Gaugau Packing involved in the case is more than

 the jurisdictional number (31) required by law, but they
G.R. No. L-13119. September 22, 1959

"(a) To reinstate the persons named in the said Order of contend that the industrial court has no jurisdiction to try
RICARDO TANTONGCO, Petitioner, v. KAISAHAN NG MGA
February 18, 1957; the case against La Campana Coffee Factory, Inc. because
MANGGAGAWA SA LA CAMPANA (KKM) AND THE

 the latter has allegedly only 14 laborers and only five of
HONORABLE COURT OF INDUSTRIAL RELATIONS,
"(b) To deposit the amount of P65,534.01 with this Court. these are members of respondent Kaisahan. This contention
Respondents.

 loses force when it is noted that, as found by the industrial
"With respect to possible back wages from August 28, 1957 court — and this finding is conclusive upon us — La
SYLLABUS
as mentioned in the petition for contempt of August 30, Campana Gaugau Packing and La Campana Coffee Factory
1957, the same shall first be determined. Co. Inc., are operating under one single management, that
1. CORPORATIONS; DISTINCT PERSONALITY OF A
is, one business though with two trade names. True, the
CORPORATION; JURISDICTION OF COURT OF INDUSTRIAL
"Failure to comply with this Order shall be directly dealt coffee factory is a corporation, and, by legal fiction, an
RELATIONS NOT AFFECTED BY DEATH OF OWNER. — The
with accordingly." 
 entity existing separate and apart from the persons
death of an owner and manager of a corporation, against

 composing it, that is, Tan Tong and his family. But it is
which cases are pending in the Court of Industrial relations,
It would appear that petitioner Ricardo Tantongco failed to settled this fiction of law, which has been introduced as a
does not deprive the latter of its jurisdiction over the
comply with said order and so, as already stated, he was matter of convenience and to subserve the ends of justice
same. The party in those cases being the corporation and
cited to appear and to adduce evidence on his behalf to cannot be invoked to further an end subversive of that
not the owner or manager personally, the claims of the
show why he should not be punished for indirect contempt. purpose.
laborers therein, which are merely incidental to their

 

demands for reinstatement for having been unjustly
The facts in this case may be briefly narrated thus: ". . . The attempt to make the two factories appear as two
dismissed, and for better working conditions, are not the
Sometime in June, 1951, members of the Kaisahan ng mga separate businesses, when in reality they are but one is but
claims contemplated by law to be submitted before the
Manggagawa sa La Campana, a labor union to which were a device to defeat the ends of the law (the Act governing
administrator of the estate of a deceased person.
affiliated workers in the La Campana Starch Factory and La capital and labor relations) and should not be permitted to

Campana Coffee Factory, two separate entities but under prevail." (La Campana Coffee Factory, Et. Al. v. Kaisahan ng
2. ID.; EXISTENCE AND OPERATION; ESTOPPEL. — Having
one management, presented demands for higher wages, mga Manggagawa, etc. Et. Al., 93 Phil., 160; 49 Off. Gaz.,
admitted the existence and operation of the two entities in
and more privileges and benefits in connection with their [6] 2300.)
a complaint filed for injunction by the said entities and on
work. When the management failed and refused to grant 

two other occasions, Petitioner, who is now estopped from
the demands, the Department of Labor intervened; but Upon the return of the case to the Court of Industrial
denying the existence of the same.
failing to settle the controversy, it certified the dispute to Relations, the latter proceeded with the hearing. In the

Page 140 of 160


CORPORATION LAW CASES MAYORDO, M.A
LLB 3

meantime incidental cases involving the same parties came "management of the respondent company and or the same question was raised by petitioner in G. R. No.
up and were filed before the Court of Industrial Relations in administrator of the Estate of Ramon Tantongco", to L-12355, entitled "La Campana Starch and Coffee Factory
the following cases: reinstate the dismissed laborers mentioned therein with and Ricardo Tantongco, etc. v. Kaisahan ng mga

 back wages. This order of February 18, 1957, as well as the Manggagawa sa La Campana (KKM)," which, as already
Case No. 584-V(1) — petition for contempt against the La order directing the inclusion of the administrator of the stated, was summarily dismissed by this Court in a
Campana Starch and Coffee Factory for having employed 21 estate of Ramon Tantongco as additional respondent in the resolution dated June 12, 1957. Consequently, said question
new laborers in violation of the order of July 21, 1951, filed incidental cases, and the order denying the petition of the may not again be raised in the present case. Furthermore,
on July 25, 1951;
 administrator to dismiss said incidental cases were it may be recalled that both in the main case and in the

 appealed to this tribunal through certiorari. The appeal, incidental cases No. 584-V to 584-V(6), Ramon Tantongco
Case No. 584-V(2) — petition of La Campana for authority however, was summarily dismissed by this Court in its was never a party. The party there was the La Campana
to dismiss Loreto Bernabe, filed on July 25, 1951; resolution of June 12, 1957, as follows: Starch and Coffee Factory by which name it was sought to
designate the two entities La Campana Starch Packing and
Case No. 584-V(3) — petition of Union to reinstate Bonifacio "This Court, deliberating upon the allegations of the the La Campana Coffee Factory. Naturally, the claims
Calderon with backpay, filed on August 3, 1951; petition filed in case L-12355 (La Campana Starch Coffee contained in said cases were not the claims contemplated
Factory Et. Al. v. Kaisahan ng Mga Manggagawa sa La by law to be submitted before the administrator. In other
Case No. 584-V(5) — petition of Union to reinstate Marcelo Campana, KKM, et al) for review, on certiorari of the words the death of Ramon Tantongco did not deprive the
Estrada and Exequiel Rapiz with back pay and to punish decision of the Court of Industrial Relations referred to CIR of its jurisdiction over the cases aforementioned.
officials of the company for contempt, filed on February therein, and finding that there is no merit in the petition, Moreover, the money claims of the laborers were merely
13, 1952; and RESOLVES TO DISMISS the same."cralaw virtua1aw library
 incidental to their demands for reinstatement for having

 been unjustly dismissed, and for better working conditions. 

Case No. 584-V(6) — petition of union for reinstatement of The CIR order of February 18, 1957, in the incidental cases 

Ibardolaza and seven other member-laborers and to punish Nos. 584-V to V(6), having become final and executory, the Petitioner, however, contends that in G.R. No. L-5677, we
the officers of the company for contempt, filed on July 15, laborers involved reported for work on August 28, 1957, but "pierced the veil of corporate existence", and held that the
1953. 
 they were not admitted by the management. Consequently, La Campana Starch and Coffee Factory and its owner,

 the union filed a petition dated August 30, 1957, to hold Ramon Tantongco, were one; so that with the death of
These five cases were heard jointly. In the meantime respondents in said cases for contempt. After hearing the Ramon, the La Campana entities ceased to exist, resulting
Ramon Tantongco supposed to be the owner and manager of CIR issued the order of September 30, 1957, subject of this in the loss of jurisdiction of the CIR to enforce its order
the La Campana Starch Factory and the person in charge of petition, ordering "the La Campana Starch and Coffee against said entities. The reason we applied the so-called
the La Campana Coffee Factory died on May 16, 1956. On Factory or its manager or the person who has charge of its "piercing the veil of corporate existence" in G. R. No.
motion of the labor union, the Court of Industrial Relations management and the administrator of the estate of Ramon L-5677 was to avoid the technicality therein advanced in
ordered the inclusion as party respondent of the Tantongco" to "reinstate the persons named in the order of order to defeat the jurisdiction of the CIR. We there found
administrator of the estate Ramon Tantongco who was February 18, 1957" and "to deposit the amount of that although there were ostensibly two separate
Ricardo Tantongco. P65,534.01." For refusal or failure to comply with said companies or entities, they were managed by the same
order, petitioner Ricardo Tantongco was required to appear person or persons and the workers in both were used
Ricardo Tantongco, as administrator, under a special before the attorney of the CIR in contempt proceedings. interchangeably so that in order to determine whether or
appearance filed a motion to dismiss all the cases including Petitioner now seeks to prohibit the CIR from proceeding not the CIR had jurisdiction, the number of workers in both
the main case, that is to say, Cases No. 584-(V) to 584-V(6), with the trial for contempt and to enjoin respondent CIR entities, not in only one, was to be considered. However,
on the ground that said cases involved claims for sums of from enforcing its order of September 30, 1957. 
 we still believe that although the family of Ramon
money and consequently should be filed before the probate 
 Tantongco was practically the owner of both the coffee
court having jurisdiction over the estate, pursuant to the Petitioner contends that upon the death of Ramon factory and the starch factory, nevertheless these entities
provisions of Rule 3, Section 21, and Rule 88, Section 1 of Tantongco, the claims of the laborers should have been are separate from the personality of Ramon. The coffee
the Rules of Court. On August 23, 1956, the Court of dismissed and that said claims should have been filed with factory is a stock corporation and the shares are owned not
Industrial Relations denied the motion to dismiss and the probate court having jurisdiction over the only by Ramon but also by others, such as petitioner
proceeded to hear the incidental cases against the La administration proceedings of the estate of Ramon Ricardo who not only is a stockholder and director and
Campana entities. 
 Tantongco, pursuant to the provisions of Rule 3, Section 21 treasurer but also the manager of the same. Furthermore,

 of the Rules of Court and that the failure to file said claims petitioner is now estopped from claiming that the two
On June 12, 1956, a partial decision was rendered in the with the administrator forever barred said claims as entities in question and Ramon are one. Thus in Annex 3-
main case No. 584-V, which partial decision was elevated to provided in Rule 87, Section 5 of the Rules of Court, CIR (par. 1 thereof) which is a complaint for injunction filed
us and is still pending appeal. On February 18, 1957, the especially after the assets of the estate had been by La Campana Food Products, et al and La Campana Starch
Court of Industrial Relations issued an order in incidental distributed among the heirs, and petitioner had ceased to Packing against the Consolidated Labor Organization of the
Cases No. 584-V(1), V(2), V(3), V(5) and V(6), directing the be the administrator of the estate. As already stated this Philippines, in Civil Case No. P-2482 in the Court of First

Page 141 of 160


CORPORATION LAW CASES MAYORDO, M.A
LLB 3

Instance of Rizal, petitioner admitted the existence and bound to comply with the same. We further find that the against respondent Dalisay and that it was just a
operation of said entities; in Annex 4-CIR where petitioner CIR acted within its jurisdiction in issuing its order of "misunderstanding" between them. Upon respondent's
appeared as General Manager representing the two entities September 30, 1957 and in requiring petitioner to appear to motion, the Executive Judge issued an order dated May 29,
in its agreement with the La Campana Workers Union to give his evidence if any in relation with the contempt 1986 recommending the dismissal of the case.
resolve the dispute between the two entities and the proceedings instituted against him. 
 It has been held that the desistance of complainant does
laborers in cases Nos. 1072-V and 1371-ULP, the existence 
 not preclude the taking of disciplinary action against
of the two entities appears to have been admitted; and in In view of the foregoing, the petition for certiorari is respondent. Neither does it dissuade the Court from
Annex 5-A-CIR, an answer to the complaint of La Campana hereby denied and the writ of preliminary injunction imposing the appropriate corrective sanction. One who
Workers Union in case No. 1471-ULP (Annex 5-CIR), dissolved, with costs. holds a public position, especially an office directly
petitioner admitted the allegation that said two factories connected with the administration of justice and the
were in existence and doing business with petitioner as Adm. Matter No. R-181-P               July 31, 1987 execution of judgments, must at all times be free from the
manager of the same. 
 ADELIO C. CRUZ, complainant, vs.QUITERIO L. DALISAY, appearance of impropriety.1

 Deputy Sheriff, RTC, Manila, respondents.
In relation to the order of the CIR requiring petitioner to We hold that respondent's actuation in enforcing a
appear in the contempt proceedings instituted against him, RESOLUTION judgment against complainant who is not the judgment
petitioner contends that after he ceased to be the FERNAN, J.: debtor in the case calls for disciplinary action. Considering
administrator of the estate of Ramon Tantongco, he may In a sworn complaint dated July 23, 1984, Adelio C. Cruz the ministerial nature of his duty in enforcing writs of
not now be compelled to comply with the order of the charged Quiterio L. Dalisay, Senior Deputy Sheriff of Manila, execution, what is incumbent upon him is to ensure that
court. In answer, it is enough to bear in mind the with "malfeasance in office, corrupt practices and serious only that portion of a decision ordained or decreed in the
jurisdiction and authority of the CIR as to compliance with irregularities" allegedly committed as follows: dispositive part should be the subject of execution.2 No
and violations of its orders under section 6, Commonwealth more, no less. That the title of the case specifically names
Act No. 143, which we quote below: 1. Respondent sheriff attached and/or levied the money complainant as one of the respondents is of no moment as

 belonging to complainant Cruz when he was not himself the execution must conform to that directed in the dispositive
". . . The Court or any Judge thereof shall have judgment debtor in the final judgment of NLRC NCR Case portion and not in the title of the case.
furthermore, all the inherent powers of a court of justice No. 8-12389-91 sought to be enforced but rather the
provided in paragraph 5 of Rule 124 of the Supreme Court, company known as "Qualitrans Limousine Service, Inc.," a The tenor of the NLRC judgment and the implementing writ
as well as the power to punish direct and indirect contempt duly registered corporation; and, is clear enough. It directed Qualitrans Limousine Service,
as provided in Rule 64 of the same Court, under the same Inc. to reinstate the discharged employees and pay them
procedure and penalties provided therein. 
 2. Respondent likewise caused the service of the alias writ full backwages. Respondent, however, chose to "pierce the

 of execution upon complainant who is a resident of Pasay veil of corporate entity" usurping a power belonging to the
"Any violation of any order, award, or decision of the Court City, despite knowledge that his territorial jurisdiction court and assumed improvidently that since the
of Industrial Relations shall, after such order, award or covers Manila only and does not extend to Pasay City. complainant is the owner/president of Qualitrans Limousine
decision has become final, conclusive, and executory, Service, Inc., they are one and the same. It is a well-
constitute contempt of court: . . . In his Comments, respondent Dalisay explained that when settled doctrine both in law and in equity that as a legal
he garnished complainant's cash deposit at the Philtrust entity, a corporation has a personality distinct and separate
"In case the employer (or landlord) committing any such bank, he was merely performing a ministerial duty. While it from its individual stockholders or members. The mere fact
violation or contempt is an association or corporation, the is true that said writ was addressed to Qualitrans Limousine that one is president of a corporation does not render the
manager or the person who has the charge of the Service, Inc., yet it is also a fact that complainant had property he owns or possesses the property of the
management of the business of the association or executed an affidavit before the Pasay City assistant fiscal corporation, since the president, as individual, and the
corporation and the officers or directors thereof who have stating that he is the owner/president of said corporation corporation are separate entities.3
ordered or authorized the violation of contempt shall be and, because of that declaration, the counsel for the
liable. . . ." plaintiff in the labor case advised him to serve notice of Anent the charge that respondent exceeded his territorial
garnishment on the Philtrust bank. jurisdiction, suffice it to say that the writ of execution
In conclusion, we find and hold that the La Campana sought to be implemented was dated July 9, 1984, or prior
Starch and Food Products Company which stands for the La On November 12, 1984, this case was referred to the to the issuance of Administrative Circular No. 12 which
Campana Starch and Coffee Factory are entities distinct Executive Judge of the Regional Trial Court of Manila for restrains a sheriff from enforcing a court writ outside his
from the personality of Ramon Tantongco; that after the investigation, report and recommendation. territorial jurisdiction without first notifying in writing and
death of Ramon these two entities continued to exist and seeking the assistance of the sheriff of the place where
to operate under the management of petitioner and that Prior to the termination of the proceedings, however, execution shall take place.
consequently he is the proper person and official to which complainant executed an affidavit of desistance stating
the orders of the CIR are addressed and who is in duty that he is no longer interested in prosecuting the case

Page 142 of 160


CORPORATION LAW CASES MAYORDO, M.A
LLB 3

ACCORDINGLY, we find Respondent Deputy Sheriff Quiterio bargaining agreement (CBA) with the respondent and
L. Dalisay NEGLIGENT in the enforcement of the writ of submitted its proposals thereto. By Decision15 dated March 19, 2001 (first CA Decision), the
execution in NLRC Case-No. 8-12389-91, and a fine CA partly granted the petition and ruled that a
equivalent to three [3] months salary is hereby imposed On June 8, 1995, petitioner and respondent agreed to sign recomputation and reevaluation of the benefits awarded
with a stern warning that the commission of the same or a CBA on non-economic terms.8 was in order.
similar offense in the future will merit a heavier penalty.
Let a copy of this Resolution be filed in the personal record On September 24, 1996, petitioner filed a notice of strike WHEREFORE, the instant petition is partly GRANTED in that
of the respondent. because of respondent’s refusal to bargain for economic the case is remanded to the Secretary of Labor for purposes
benefits in the CBA. Following conciliation hearings, the of recomputation and reevaluation of the CBA benefits.
SO ORDERED. parties again commenced CBA negotiations and started to
resolve the issues on wage increase, productivity bonus, SO ORDERED.16
G.R. No. 165442               August 25, 2010 incentive bonus, allowances, and other benefits but failed
NASECO GUARDS ASSOCIATION-PEMA (NAGA-PEMA), to reach an agreement. In compliance with the CA directive, then DOLE Secretary
Petitioner, vs. NATIONAL SERVICE CORPORATION Patricia A. Sto. Tomas conducted several clarificatory
(NASECO), Respondent. Meanwhile, respondent and NEMU-PEMA entered into a CBA hearings. On January 15, 2003, Secretary Sto. Tomas issued
on non-economic terms.9 Unfortunately, a dispute among an Order which provides:
DECISION the leaders of NEMU-PEMA arose and at a certain point, From the above, it is indubitable that the total cost to
VILLARAMA, JR., J.: leadership of the organization was unclear. Hence, the NASECO of our questioned award would amount to only
negotiations concerning the economic terms of the CBA ₱322,725,000, not ₱531,446,666.67 as claimed by the
This petition for review on certiorari under Rule 45 assails were put on hold until the internal dispute could be company. Thus, our November 19, 1999 Order is hereby
the Decision1 dated May 27, 2004 of the Court of Appeals resolved. affirmed en toto.
(CA) in CA-G.R. SP No. 76667. The appellate court set aside
the January 15, 20032 and March 11, 20033 Orders of the On April 29, 1997, petitioner filed a notice of strike before WHEREFORE, judgment is hereby rendered:
Department of Labor and Employment (DOLE) and ordered the National Conciliation and Mediation Board (NCMB)
the latter to allow the parties to adduce evidence in against respondent and PNB due to a bargaining deadlock. 1. [D]irecting NAGA-PEMA and NASECO to execute
support of their respective positions. The following day, NEMU-PEMA likewise filed a notice of a new collective bargaining agreement effective
strike against respondent and PNB on the ground of unfair November 1, 1993, incorporating therein the
The facts follow. labor practices.10 Efforts by the NCMB to conciliate failed dispositions contained in our November 19, 1999
and pursuant to Article 263(g) of the Labor Code,11 as Order as well as all other items agreed upon by
Respondent National Service Corporation (NASECO) is a amended, then DOLE Secretary Cresenciano B. Trajano the parties.
wholly-owned subsidiary of the Philippine National Bank assumed jurisdiction over the strike notices on June 25, 2. Ordering NASECO to negotiate with NEMA-PEMA
(PNB) organized under the Corporation Code in 1975. It 1998.12 for a new collective bargaining agreement.
supplies security and manpower services to different clients
such as the Securities and Exchange Commission, the On November 19, 1999, then DOLE Secretary Bienvenido E. The charges of unfair labor practice against NASECO and
Philippine Deposit Insurance Corporation, Food Terminal Laguesma issued a Resolution13 directing petitioner and PNB are dismissed for lack of merit.
Incorporated, Forex Corporation and PNB. Petitioner respondent to execute a new CBA incorporating therein his
NASECO Guards Association-PEMA (NAGA-PEMA) is the dispositions regarding benefits of the employees as to wage SO ORDERED.17
collective bargaining representative of the regular rank and increase, productivity bonus, vacation and sick leave,
file security guards of respondent. NASECO Employees medical allowances and signing bonus. Respondent was Respondent filed a motion for reconsideration with the
Union-PEMA (NEMU-PEMA) is the collective bargaining further ordered to negotiate, for purposes of collective DOLE Secretary which was denied on March 11, 2003.
representative of the regular rank and file (non-security) bargaining agreement, with NEMU-PEMA led by its
employees of respondent such as messengers, janitors, president, Ligaya Valencia. The charge of unfair labor Respondent thus filed a petition for certiorari with the CA
typists, clerks and radio-telephone operators.4 practice against respondent and PNB was dismissed.14 arguing that the DOLE Secretary, in issuing the January 15,
Respondent promptly filed a petition for certiorari before 2003 Order deprived respondent of due process of law for
On December 2, 1993, respondent entered into a the CA questioning the DOLE Secretary’s order and arguing there was no reevaluation that took place in the DOLE. It
memorandum of agreement5 with petitioner. The terms of that the ruling of the DOLE Secretary in favor of the unions also argued that the order merely recomputed the DOLE
the agreement covered the monetary claims of the and awarding them monetary benefits totaling five hundred Secretary’s initial award of ₱531,446,666.67 and reduced it
petitioner such as salary adjustments, conversion of salary thirty-one million four hundred forty-six thousand six to ₱322,725,000.00, contrary to the ruling of the CA to
scheme under Republic Act (R.A.) No. 67586 to R.A. No. hundred sixty-six and 67/100 (₱531,446,666.67) was recompute and reevaluate. Respondent claimed that what
6727,7 signing bonus, leaves and other benefits. A year inimical and deleterious to its financial standing and will the DOLE Secretary should have done was to let the parties
after, petitioner demanded full negotiation for a collective result in closure and cessation of business for the company.

Page 143 of 160


CORPORATION LAW CASES MAYORDO, M.A
LLB 3

introduce evidence to show the proper computation of the original proceeding are to be visited and reviewed again. In
monetary awards under the approved CBA. Respondent on the other hand maintains that the DOLE this light, the respondent has been given the opportunity to
Secretary violated its right to due process when she merely be heard by the DOLE Secretary.
In its second Decision dated May 27, 2004, the CA granted recomputed the CBA award instead of reevaluating the
the petition, thus: entire case and allowing it to present supporting documents Also, contrary to the claim of the respondent that it was
in accordance with the first CA decision.22 It claims that the barred by the DOLE Secretary to introduce supporting
WHEREFORE, the orders dated 15 January 2003 and 11 order of the CA to reevaluate included and required a full documents during the recomputation and reevaluation, the
March 2003 are hereby SET ASIDE and the case remanded to assessment of the case together with reception of evidence records show that an Order by then Secretary of Labor
the public respondent to allow the parties to adduce such as financial statements, and the omission of such is a Patricia A. Sto. Tomas dated July 11, 2002 specifically
evidence in support of their respective positions. violation of its right to due process. allowed both parties to submit their respective
computations as regards the awarded benefits. To wit:
SO ORDERED.18 As to the petitioner’s argument that respondent and PNB
are essentially the same when it comes to financial WHEREFORE, the Bureau of Working Conditions is hereby
A motion for reconsideration was filed by herein petitioner condition, respondent contends that although a subsidiary, directed to submit to this Office a detailed computation of
but the same was denied by the CA on September 22, it has a separate and distinct personality from PNB with its the CBA benefits indicated in the resolution of November
200419 finding no reason to reverse and set aside its earlier own charter. Hence, the issue of PNB’s financial well-being 19, 2001 within twenty (20) days from receipt of this Order.
decision. is immaterial in this case. The parties may submit their own computations to the
Petitioner now comes to this Court for relief by way of a The petition is partly meritorious. Bureau for validation.
petition for review on certiorari seeking to set aside and SO ORDERED.24 (Italics supplied.)
reverse the May 27, 2004 Decision and the September 22, In simple terms, the constitutional guarantee of due
2004 Resolution of the CA. process requires that a litigant be given "a day in court." It It is thus inaccurate for the respondent to claim that it was
is the availability of the opportunity to be heard that denied due process because it had all the opportunity to
The main issue in this case is whether or not the determines whether or not due process was violated. A introduce any supporting document in the course of the
respondent’s right to due process was violated. A side issue litigant may or may not avail of the opportunity to be heard recomputation and reevaluation of the DOLE Secretary.
raised by the petitioner is whether or not PNB, being the but as long as such was made available to him/her, there is Respondent admits that it did attach the financial
undisputed owner of and exercising control over no violation of the due process clause. In the case of statements and other documents in support of its alleged
respondent, should be made liable to pay the CBA benefits Lumiqued v. Exevea,23 this Court declared that "[a]s long as financial incapacity to pay the CBA awarded benefits, the
awarded to the petitioner. a party was given the opportunity to defend his interests in same evidence it had earlier submitted before the CA
due course, he cannot be said to have been denied due (Memorandum in the first CA decision) in the motion for
Petitioner argues first that there was no violation of due process of law, for this opportunity to be heard is the very reconsideration of the DOLE Secretary’s January 15, 2003
process because respondent was never prohibited by the essence of due process. Moreover, this constitutional Order.25 There is thus no showing that the DOLE Secretary
DOLE Secretary to submit supporting documents when the mandate is deemed satisfied if a person is granted an denied respondent this basic constitutional right.
instant case was pending on remand. Petitioner contends opportunity to seek reconsideration of the action or ruling
that due process is properly observed when there is an complained of." On the issue of liability, petitioner contends that PNB
opportunity to be heard, to present evidence and to file The respondent’s right to due process in this case has not should be held liable to shoulder the CBA benefits awarded
pleadings, which was never denied to respondent. been denied. The order in the first CA decision to to them by virtue of it being a company having full
recompute and reevaluate was satisfied when the DOLE financial, managerial and functional control over
Second, petitioner argues that the CA erred in stating that Secretary reexamined their initial findings and adjusted the respondent as its subsidiary, and by reason of the unique
respondent was a company operating at a loss and awarded benefits. A reevaluation, contrary to what the "no loss, no profit" scheme implemented between
therefore cannot be expected to act generously and confer respondent claims, is a process by which a person or office respondent and PNB.
upon its employees additional benefits exceeding what is (in this case the DOLE secretary) revisits its own initial
mandated by law. It is the petitioner’s position that based pronouncement and makes another assessment of its We are not persuaded.
on the "no loss, no profit" policy of respondent with PNB, findings. In simple terms, to reevaluate is to take another
respondent in truth has no "pocket" of its own and is, in look at a previous matter in issue. A reevaluation does not Verily, what the petitioner is asking this Court to do is to
effect, one (1) and the same with PNB with regard to necessitate the introduction of new materials for review pierce the veil of corporate fiction of respondent and hold
financial gains and/or liabilities. Thus, petitioners contend nor does it require a full hearing for new arguments. PNB (being the mother company) liable for the CBA
that the CBA benefits should be shouldered by PNB benefits.
considering the poor financial condition of respondent. To From a procedural standpoint, a reevaluation is a
support such claim, petitioner submitted evidence20 to continuation of the original case and not a new proceeding. In Concept Builders, Inc. v. NLRC,26 we explained the
show that PNB is in superb financial condition and is very Hence, the evidence, financial reports and other doctrine of piercing the corporate veil, as follows:
much capable of shouldering the CBA award.21 documents submitted by the parties in the course of the

Page 144 of 160


CORPORATION LAW CASES MAYORDO, M.A
LLB 3

It is a fundamental principle of corporation law that a defeat the labor laws, nor does the scheme show that On the scales of justice precariously lie the right of a
corporation is an entity separate and distinct from its respondent is a mere business conduit or alter ego of PNB. prevailing party to his victor's cup, no more, no less; and
stockholders and from other corporations to which it may Absent proof of these circumstances, respondent’s the right of a separate entity from being dragged by the
be connected. But, this separate and distinct personality of corporate personality cannot be pierced. ball and chain of the vanquished party.
a corporation is merely a fiction created by law for
convenience and to promote justice. So, when the notion It is apparent that petitioner wants the Court to disregard The facts of this case as garnered from the Decision1 dated
of separate juridical personality is used to defeat public the corporate personality of respondent and directly go April 26, 2012 of the Court of Appeals (CA) in CA-G.R. SP
convenience, justify wrong, protect fraud or defend crime, after PNB in order for it to collect the CBA benefits. On the No. 120979 are as follows:
or is used as a device to defeat the labor laws, this same breath, however, petitioner argues that ultimately it
separate personality of the corporation may be disregarded is PNB, by virtue of the "no loss, no profit" scheme, which We trace the roots of this case to a complaint instituted
or the veil of corporate fiction pierced. This is true likewise shoulders and provides the funds for financial liabilities of with the Makati City Regional Trial Court (RTC), Branch 66,
when the corporation is merely an adjunct, a business respondent including wages and benefits of employees. If against EIB Securities Inc. (E-Securities) for unauthorized
conduit or an alter ego of another corporation. such scheme was indeed true as the petitioner presents it, sale of 32,180,000 DMCI shares of private respondents
then there was absolutely no need to pierce the veil of Pacific Rehouse Corporation, Pacific Concorde Corporation,
Also in Pantranco Employees Association (PEA-PTGWO) v. corporate fiction of respondent. Moreover, the Court notes Mizpah Holdings, Inc., Forum Holdings Corporation, and
National Labor Relations Commission,27 this Court ruled: the pendency of a separate suit for absorption or East Asia Oil Company, Inc. In its October 18, 2005
regularization of NASECO employees filed by petitioner and Resolution, the RTC rendered judgment on the pleadings.
Whether the separate personality of the corporation should NEMU-PEMA against PNB and respondent, docketed as NLRC The fallo reads:
be pierced hinges on obtaining facts appropriately pleaded NCR Case No. 06-03944-96), which is still on appeal with
or proved. However, any piercing of the corporate veil has the National Labor Relations Commission (NLRC), as per WHEREFORE, premises considered, judgment is hereby
to be done with caution, albeit the Court will not hesitate manifestation by respondent. In the said case, petitioner rendered directing the defendant [E-Securities] to return
to disregard the corporate veil when it is misused or when submitted for resolution by the labor tribunal the issues of the plaintiffs’ [private respondents herein] 32,180,000
necessary in the interest of justice. After all, the concept whether PNB is the employer of NASECO’s work force and DMCI shares, as of judicial demand.
of corporate entity was not meant to promote unfair whether NASECO is a labor-only contractor.29
objectives. On the other hand, plaintiffs are directed to reimburse the
WHEREFORE, the petition is PARTLY GRANTED. The Decision defendant the amount of [P]10,942,200.00, representing
Applying the doctrine to the case at bar, we find no reason dated May 27, 2004 and Resolution dated September 22, the buy back price of the 60,790,000 KPP shares of stocks
to pierce the corporate veil of respondent and go beyond 2004 in CA-G.R. SP No. 76667 are hereby REVERSED and SET at [P]0.18 per share.
its legal personality. Control, by itself, does not mean that ASIDE as to the order to remand the case to the Secretary
the controlled corporation is a mere instrumentality or a of Labor for introduction of supporting evidence. SO ORDERED. x x x
business conduit of the mother company. Even control over Accordingly, the Orders of the Secretary of Labor dated
the financial and operational concerns of a subsidiary January 15, 2003 and March 11, 2003 are REINSTATED and The Resolution was ultimately affirmed by the Supreme
company does not by itself call for disregarding its UPHELD. Court and attained finality.
corporate fiction. There must be a perpetuation of fraud
behind the control or at least a fraudulent or illegal No costs. SO ORDERED. When the Writ of Execution was returned unsatisfied,
purpose behind the control in order to justify piercing the private respondents moved for the issuance of an alias writ
veil of corporate fiction. Such fraudulent intent is lacking G.R. No. 199687 of execution to hold Export and Industry Bank, Inc. liable
in this case. PACIFIC REHOUSE CORPORATION, Petitioners, vs. COURT for the judgment obligation as E- Securities is "a wholly-
OF APPEALS and EXPORT AND INDUSTRY BANK, INC., owned controlled and dominated subsidiary of Export and
Petitioner argues that the appreciation, analysis and inquiry Respondents. Industry Bank, Inc., and is[,] thus[,] a mere alter ego and
of this case may go beyond the presentation of respondent, business conduit of the latter. E-Securities opposed the
and therefore must include the PNB, the bank being the G.R. No. 201537 motion[,] arguing that it has a corporate personality that is
undisputed whole owner of respondent and the sole PACIFIC REHOUSE CORPORATION, PACIFIC CONCORDE separate and distinct from petitioner. On July 27, 2011,
provider of funds for the company’s operations and for the CORPORATION, MIZPAH HOLDINGS, INC., FORUM private respondents filed their (1) Reply attaching for the
payment of wages and benefits of the employees, under HOLDINGS CORPORATION and EAST ASIA OIL COMPANY, first time a sworn statement executed by Atty. Ramon F.
the "no loss, no profit" scheme.28 INC., Petitioners, vs. EXPORT AND INDUSTRY BANK, INC., Aviado, Jr., the former corporate secretary of petitioner
Respondent. and E-Securities, to support their alter ego theory; and (2)
We disagree. There is no showing that such "no loss, no Ex-Parte Manifestation alleging service of copies of the Writ
profit" scheme between respondent and PNB was DECISION of Execution and Motion for Alias Writ of Execution on
implemented to defeat public convenience, justify wrong, REYES, J.: petitioner.
protect fraud or defend crime, or is used as a device to

Page 145 of 160


CORPORATION LAW CASES MAYORDO, M.A
LLB 3

On July 29, 2011, the RTC concluded that E-Securities is a Pursuant to Rule 39, Section 10 (a) of the Rules of Court, (P50,000,000.00) within ten (10) days from notice, to
mere business conduit or alter ego of petitioner, the the Branch Clerk of Court or the Branch Sheriff of this Court answer for any damage which private respondents may
dominant parent corporation, which justifies piercing of is hereby directed to acquire 32,180,000 DMCI shares of suffer by reason of this Temporary Restraining Order;
the veil of corporate fiction. The trial court brushed aside stock from the Philippine Stock Exchange at the cost of EIB otherwise, the same shall automatically become
E-Securities’ claim of denial of due process on petitioner as Securities, Inc. and Export and Industry Bank[,] Inc. and to ineffective.
"xxx case records show that notices regarding these deliver the same to the plaintiffs pursuant to this Court’s
proceedings had been tendered to the latter, which Resolution dated October 18, 2005. Let the HEARING be set on September 27, 2011 at 2:00 in
refused to even receive them. Clearly, [petitioner] had the afternoon at the Paras Hall, Main Building, Court of
been sufficiently put on notice and afforded the chance to To implement this Order, let GARNISHMENT issue against Appeals, to determine the necessity of issuing a writ of
give its side[,] yet[,] it chose not to." Thus, the RTC ALL THOSE HOLDING MONEYS, PROPERTIES OF ANY AND ALL preliminary injunction. The Division Clerk of Court is
disposed as follows: KINDS, REAL OR PERSONAL BELONGING TO OR OWNED BY DIRECTED to notify the parties and their counsel with
WHEREFORE, xxx, DEFENDANT EIB SECURITIES, INC. AND/OR EXPORT AND dispatch.
INDUSTRY BANK[,] INC., [sic] in such amount as may be
Let an Alias Writ of Execution be issued relative to the sufficient to acquire 32,180,000 DMCI shares of stock to the SO ORDERED.11
above-entitled case and pursuant to the RESOLUTION dated Philippine Stock Exchange, based on the closing price of
October 18, 2005 and to this Order directing defendant EIB Php45.55 per share of DMCI shares as of August 1, 2011, the Pacific Rehouse Corporation (Pacific Rehouse), Pacific
Securities, Inc., and/or Export and Industry Bank, Inc., to date of the issuance of the Alias Writ of Execution, or the Concorde Corporation, Mizpah Holdings, Inc., Forum
fully comply therewith. total amount of PhP1,465,799,000.00. Holdings Corporation and East Asia Oil Company, Inc.
The Branch Sheriff of this Court is directed to cause the SO ORDERED.7 (petitioners) filed their Comment12 to Export Bank’s
immediate implementation of the given alias writ in petition and proffered that the cases mentioned by Export
accordance with the Order of Execution to be issued anew CA-G.R. SP No. 120979 Bank are inapplicable owing to their clearly different
by the Branch Clerk of Court. factual antecedents. The petitioners alleged that unlike the
Export and Industry Bank, Inc. (Export Bank) filed before other cases, there are circumstances peculiar only to E-
SO ORDERED. x x x the CA a petition for certiorari with prayer for the issuance Securities and Export Bank such as: 499,995 out of 500,000
of a temporary restraining order (TRO)8 seeking the outstanding shares of stocks of E-Securities are owned by
With this development, petitioner filed an Omnibus Motion nullification of the RTC Order dated August 26, 2011 for Export Bank;13 Export Bank had actual knowledge of the
(Ex Abundanti Cautela) questioning the alias writ because having been made with grave abuse of discretion amounting subject matter of litigation as the lawyers who represented
it was not impleaded as a party to the case. The RTC to lack or excess of jurisdiction. In its petition, Export Bank E-Securities are also lawyers of Export Bank.14 As an alter
denied the motion in its Order dated August 26, 2011 and made reference to several rulings9 of the Court upholding ego, there is no need for a finding of fraud or illegality
directed the garnishment of P1,465,799,000.00, the total the separate and distinct personality of a corporation. before the doctrine of piercing the veil of corporate fiction
amount of the 32,180,000 DMCI shares at P45.55 per share, can be applied.15
against petitioner and/or E-Securities.2 x x x. (Citations In a Resolution10 dated September 2, 2011, the CA issued a
omitted) 60-day TRO enjoining the execution of the Orders of the After oral arguments before the CA, the parties were
RTC dated July 29, 2011 and August 26, 2011, which directed to file their respective memoranda.16
The Regional Trial Court (RTC) ratiocinated that being one granted the issuance of an alias writ of execution and
and the same entity in the eyes of the law, the service of ordered the garnishment of the properties of E-Securities On October 25, 2011, the CA issued a Resolution,17 granting
summons upon EIB Securities, Inc. (E-Securities) has and/or Export Bank. The CA also set a hearing to determine Export Bank’s application for the issuance of a writ of
bestowed jurisdiction over both the parent and wholly- the necessity of issuing a writ of injunction, viz: preliminary injunction, viz:
owned subsidiary.3 The RTC cited the cases of Sps. Violago
v. BA Finance Corp. et al.4 and Arcilla v. Court of Appeals5 Considering the amount ordered to be garnished from WHEREFORE, finding [Export Bank’s] application for the
where the doctrine of piercing the veil of corporate fiction petitioner Export and Industry Bank, Inc. and the fiduciary ancillary injunctive relief to be meritorious, and it further
was applied notwithstanding that the affected corporation duty of the banking institution to the public, there is grave appearing that there is urgency and necessity in restraining
was not brought to the court as a party. Thus, the RTC held and irreparable injury that may be caused to [Export Bank] the same, a Writ of Preliminary Injunction is hereby
in its Order6 dated August 26, 2011: if the assailed Orders are immediately implemented. We GRANTED and ISSUED against the Sheriff of the Regional
thus resolve to GRANT the Temporary Restraining Order Trial Court of Makati City, Branch 66, or his deputies,
WHEREFORE, premises considered, the Motion for effective for a period of sixty (60) days from notice, agents, representatives or any person acting in their behalf
Reconsideration with Motion to Inhibit filed by defendant restraining/enjoining the Sheriff of the Regional Trial Court from executing the July 29, 2011 and August 26, 2011
EIB Securities, Inc. is denied for lack of merit. The Omnibus of Makati City or his deputies, agents, representatives or Orders. Public respondents are ordered to CEASE and
Motion Ex Abundanti C[au]tela is likewise denied for lack of any person acting in their behalf from executing the July DESIST from enforcing and implementing the subject orders
merit. 29, 2011 and August 26, 2011 Orders. [Export Bank] is until further notice from this Court.18
DIRECTED to POST a bond in the sum of fifty million pesos

Page 146 of 160


CORPORATION LAW CASES MAYORDO, M.A
LLB 3

The petitioners filed a Manifestation19 and Supplemental per Resolution dated September 26, 2012, for a thorough created only when the three members of a division cannot
Manifestation20 challenging the above-quoted CA resolution discussion of the merits of the case. reach a unanimous vote in deciding a case on the merits.35
for lack of concurrence of Associate Justice Socorro B. Furthermore, for petitioner Pacific Rehouse, this Resolution
Inting (Justice Inting), who was then on official leave. Issues is likewise infirm because the purpose of the formation of
the Special Division of Five is to decide the case on the
On December 22, 2011, the CA, through a Special Division In précis, the issues for resolution of this Court are the merits and not to grant Export Bank’s application for a writ
of Five, issued another Resolution,21 which reiterated the following: of preliminary injunction.36
Resolution dated October 25, 2011 granting the issuance of
a writ of preliminary injunction. In G.R. No. 199687, We hold that the opposition to the CA resolutions is already
nugatory because the CA has already rendered its Decision
On January 2, 2012, one of the petitioners herein, Pacific WHETHER THE CA COMMITTED GRAVE ABUSE OF DISCRETION on April 16, 2012, which disposed of the substantial merits
Rehouse filed before the Court a petition for certiorari22 IN GRANTING EXPORT BANK’S APPLICATION FOR THE of the case. Consequently, the petitioners’ concern that the
under Rule 65, docketed as G.R. No. 199687, ISSUANCE OF A WRIT OF PRELIMINARY INJUNCTION. Special Division of Five should have been created to resolve
demonstrating its objection to the Resolutions dated cases on the merits has already been addressed by the
October 25, 2011 and December 22, 2011 of the CA. In G.R. No. 201537, rendition of the CA Decision dated April 16, 2012.
I.
On April 26, 2012, the CA rendered the assailed Decision23 WHETHER THE CA COMMITTED A REVERSIBLE ERROR IN "It is well-settled that courts will not determine questions
on the merits of the case, granting Export Bank’s petition. RULING THAT EXPORT BANK MAY NOT BE HELD LIABLE FOR A that have become moot and academic because there is no
The CA disposed of the case in this wise: FINAL AND EXECUTORY JUDGMENT AGAINST E-SECURITIES IN longer any justiciable controversy to speak of. The
AN ALIAS WRIT OF EXECUTION BY PIERCING ITS VEIL OF judgment will not serve any useful purpose or have any
We GRANT the petition. The Orders dated July 29, 2011 and CORPORATE FICTION; and practical legal effect because, in the nature of things, it
August 26, 2011 of the Makati City Regional Trial Court, II. cannot be enforced."37 In such cases, there is no actual
Branch 66, insofar as [Export Bank] is concerned, are WHETHER THE CA COMMITTED A REVERSIBLE ERROR IN substantial relief to which the petitioners would be entitled
NULLIFIED. The Writ of Preliminary Injunction (WPI) is RULING THAT THE ALTER EGO DOCTRINE IS NOT APPLICABLE. to and which would be negated by the dismissal of the
rendered PERMANENT. Ruling of the Court petition.38 Thus, it would be futile and pointless to address
the issue in G.R. No. 199687 as this has become moot and
SO ORDERED.24 G.R. No. 199687 academic.

The CA explained that the alter ego theory cannot be The Resolution dated October 25, 2011 was initially G.R. No. 201537
sustained because ownership of a subsidiary by the parent challenged by the petitioners in its Manifestation29 and
company is not enough justification to pierce the veil of Supplemental Manifestation 30 due to the lack of The petitioners bewail that the certified true copy of the
corporate fiction. There must be proof, apart from mere concurrence of Justice Inting, which according to the CA Decision dated April 26, 2012 along with its Certification
ownership, that Export Bank exploited or misused the petitioners rendered the aforesaid resolution null and void. at the bottom portion were not signed by the Chairperson39
corporate fiction of E-Securities. The existence of of the Special Division of Five; thus, it is not binding upon
interlocking incorporators, directors and officers between To the petitioners’ mind, Section 5, Rule VI of the Internal the parties. 40 The petitioners quoted this Court’s
the two corporations is not a conclusive indication that Rules of the CA (IRCA)31 requires the submission of the pronouncement in Limkaichong v. Commission on Elections,
they are one and the same.25 The records also do not show resolution granting an application for TRO or preliminary 41 that a decision must not only be signed by the Justices

that Export Bank has complete control over the business injunction to the absent Justice/s when they report back to who took part in the deliberation, but must also be
policies, affairs and/or transactions of E-Securities. It was work for ratification, modification or recall, such that when promulgated to be considered a Decision.42
solely E-Securities that contracted the obligation in the absent Justice/s do not agree with the issuance of the
furtherance of its legitimate corporate purpose; thus, any TRO or preliminary injunction, the resolution is recalled A cursory glance on a copy of the signature page43 of the
fall out must be confined within its limited liability.26 and without force and effect.32 Since the resolution which decision attached to the records would show that, indeed,
granted the application for preliminary injunction appears the same was not signed by CA Associate Justice Magdangal
The petitioners, without filing a motion for short of the required number of consensus, owing to the M. De Leon. However, it must be noted that the CA, on May
reconsideration, filed a Petition for Review27 under Rule 45 absence of Justice Inting’s signature, the petitioners 7, 2012, issued a Resolution44 explaining that due to
docketed as G.R. No. 201537,28 impugning the Decision contest the validity of said resolution. inadvertence, copies of the decision not bearing the
dated April 26, 2012 of the CA. signature of the Chairperson were sent to the parties on
Considering that G.R. Nos. 199687 and 201537 originated The petitioners also impugn the CA Resolution dated the same day of promulgation. The CA directed the Division
from the same set of facts, involved the same parties and December 22, 2011 rendered by the Special Division of Clerk of Court to furnish the parties with copies of the
raised intertwined issues, the cases were then consolidated Five. The petitioners maintain that pursuant to Batas signature page with the Chairperson’s signature.
Pambansa Bilang 12933 and the IRCA,34 such division is Consequently, as the mistake was immediately clarified and

Page 147 of 160


CORPORATION LAW CASES MAYORDO, M.A
LLB 3

remedied by the CA, the lack of the Chairperson’s signature From the preceding, it is therefore correct to say that the already sold to someone else. VMSC was not impleaded as a
on the copies sent to the parties has already become a non- court must first and foremost acquire jurisdiction over the third party defendant. Avelino contended that he was not a
issue. parties; and only then would the parties be allowed to party to the transaction personally, but VMSC. The Court
present evidence for and/or against piercing the veil of ruled that "[t]he fact that VMSC was not included as
It must be emphasized that the instant cases sprang from corporate fiction. If the court has no jurisdiction over the defendant in [Spouses Violago’s] third party complaint does
Pacific Rehouse Corporation v. EIB Securities, Inc.45 which corporation, it follows that the court has no business in not preclude recovery by Spouses Violago from Avelino;
was decided by this Court last October 13, 2010. piercing its veil of corporate fiction because such action neither would such non-inclusion constitute a bar to the
Significantly, Export Bank was not impleaded in said case offends the corporation’s right to due process. application of the piercing-of-the-corporate-veil
but was unexpectedly included during the execution stage, doctrine."57 It should be pointed out that although VMSC
in addition to E-Securities, against whom the writ of "Jurisdiction over the defendant is acquired either upon a was not made a third party defendant, the person who was
execution may be enforced in the Order46 dated July 29, valid service of summons or the defendant’s voluntary found liable in Violago, Avelino, was properly made a third
2011 of the RTC. In including Export Bank, the RTC appearance in court. When the defendant does not party defendant in the first instance. The present case
considered E-Securities as a mere business conduit of voluntarily submit to the court’s jurisdiction or when there could not be any more poles apart from Violago, because
Export Bank.47 Thus, one of the arguments interposed by is no valid service of summons, ‘any judgment of the court Export Bank, the parent company which was sought to be
the latter in its Opposition48 that it was never impleaded as which has no jurisdiction over the person of the defendant accountable for the judgment against E-Securities, is not a
a defendant was simply set aside. is null and void.’"51 "The defendant must be properly party to the main case.
This action by the RTC begs the question: may the RTC apprised of a pending action against him and assured of the
enforce the alias writ of execution against Export Bank? opportunity to present his defenses to the suit. Proper In Arcilla, meanwhile, Calvin Arcilla (Arcilla) obtained a
service of summons is used to protect one’s right to due loan in the name of Csar Marine Resources, Inc. (CMRI) from
The question posed before us is not novel. process."52 Emilio Rodulfo. A complaint was then filed against Arcilla
for non-payment of the loan. CMRI was not impleaded as a
The Court already ruled in Kukan International Corporation As Export Bank was neither served with summons, nor has it defendant. The trial court eventually ordered Arcilla to pay
v. Reyes49 that compliance with the recognized modes of voluntarily appeared before the court, the judgment sought the judgment creditor for such loan. Arcilla argued that he
acquisition of jurisdiction cannot be dispensed with even in to be enforced against E-Securities cannot be made against is not personally liable for the adjudged award because the
piercing the veil of corporate fiction, to wit: its parent company, Export Bank. Export Bank has same constitutes a corporate liability which cannot even
consistently disputed the RTC jurisdiction, commencing bind the corporation as the latter is not a party to the
The principle of piercing the veil of corporate fiction, and from its filing of an Omnibus Motion53 by way of special collection suit. The Court made the succeeding
the resulting treatment of two related corporations as one appearance during the execution stage until the filing of its observations:
and the same juridical person with respect to a given Comment54 before the Court wherein it was pleaded that
transaction, is basically applied only to determine "RTC [of] Makati[, Branch] 66 never acquired jurisdiction [B]y no stretch of even the most fertile imagination may
established liability; it is not available to confer on the over Export [B]ank. Export [B]ank was not pleaded as a one be able to conclude that the challenged Amended
court a jurisdiction it has not acquired, in the first place, party in this case. It was never served with summons by nor Decision directed Csar Marine Resources, Inc. to pay the
over a party not impleaded in a case. Elsewise put, a did it voluntarily appear before RTC [of] Makati[, Branch] amounts adjudged. By its clear and unequivocal language,
corporation not impleaded in a suit cannot be subject to 66 so as to be subjected to the latter’s jurisdiction."55 it is the petitioner who was declared liable therefor and
the court’s process of piercing the veil of its corporate consequently made to pay. x x x, even if We are to assume
fiction. In that situation, the court has not acquired In dispensing with the requirement of service of summons arguendo that the obligation was incurred in the name of
jurisdiction over the corporation and, hence, any or voluntary appearance of Export Bank, the RTC applied the corporation, the petitioner would still be personally
proceedings taken against that corporation and its property the cases of Violago and Arcilla. The RTC concluded that in liable therefor because for all legal intents and purposes,
would infringe on its right to due process. Aguedo Agbayani, these cases, the Court decided that the doctrine of piercing he and the corporation are one and the same. Csar Marine
a recognized authority on Commercial Law, stated as much: the veil of corporate personality can be applied even when Resources, Inc. is nothing more than his business conduit
one of the affected parties has not been brought to the and alter ego. The fiction of a separate juridical
"23. Piercing the veil of corporate entity applies to Court as a party.56 personality conferred upon such corporation by law should
determination of liability not of jurisdiction. x x x be disregarded. x x x.58 (Citation omitted)
A closer perusal on the rulings of this Court in Violago and
This is so because the doctrine of piercing the veil of Arcilla, however, reveals that the RTC misinterpreted the It is important to bear in mind that although CMRI was not a
corporate fiction comes to play only during the trial of doctrines on these cases. We agree with the CA that these party to the suit, it was Arcilla, the defendant himself who
the case after the court has already acquired jurisdiction cases are not congruent to the case at bar. In Violago, was found ultimately liable for the judgment award. CMRI
over the corporation. Hence, before this doctrine can be Spouses Pedro and Florencia Violago (Spouses Violago) filed and its properties were left untouched from the main case,
applied, based on the evidence presented, it is imperative a third party complaint against their cousin Avelino Violago not only because of the application of the alter ego
that the court must first have jurisdiction over the (Avelino), who is also the president of Violago Motor Sales doctrine, but also because it was never made a party to
corporation. x x x"50 (Citations omitted) Corporation (VMSC), for selling them a vehicle which was that case.

Page 148 of 160


CORPORATION LAW CASES MAYORDO, M.A
LLB 3

domination of finances, policies and practices that the corporations, Atty. Ramon Aviado, Jr., in his submitted
The disparity between the instant case and those of Violago controlled corporation has, so to speak, no separate mind, Sworn Statement which is deemed allowable "evidence
and Arcilla is that in said cases, although the corporations will or existence of its own, and is but a conduit for its on motion", under Sec. 7, Rule 133, Rules on Evidence;
were not impleaded as defendant, the persons made liable principal. It must be kept in mind that the control must be Bravo vs. Borja, 134 SCRA 438];
in the end were already parties thereto since the inception shown to have been exercised at the time the acts
of the main case. Consequently, it cannot be said that the complained of took place. Moreover, the control and breach 3. For effective control purposes, defendant EIB
Court had, in the absence of fraud and/or bad faith, of duty must proximately cause the injury or unjust loss for Securities and its operating office and staff are all
applied the doctrine of piercing the veil of corporate which the complaint is made."62 housed in Exportbank Plaza located at Chino Roces cor.
fiction to make a non-party liable. In short, liabilities Sen. Gil Puyat Avenue, Makati City which is the same
attached only to those who are parties. None of the non- The Court has laid down a three-pronged control test to building w[h]ere the bank parent corporation has its
party corporations (VMSC and CMRI) were made liable for establish when the alter ego doctrine should be operative: headquarters;
the judgment award against Avelino and Arcilla.
(1) Control, not mere majority or complete stock 4. As shown in the General Information Sheets annually
The Alter Ego Doctrine is not applicable control, but complete domination, not only of finances filed with the S.E.C. from 2002 to 2011, both defendant
but of policy and business practice in respect to the EIB Securities and the bank parent corporation share
"The question of whether one corporation is merely an alter transaction attacked so that the corporate entity as to common key Directors and corporate officers. Three of
ego of another is purely one of fact. So is the question of this transaction had at the time no separate mind, will the 5-man Board of Directors of defendant EIB
whether a corporation is a paper company, a sham or or existence of its own; Securities are Directors of the bank parent corporation,
subterfuge or whether petitioner adduced the requisite (2) Such control must have been used by the defendant namely: Jaime C. Gonzales, Pauline C. Tan and Dionisio
quantum of evidence warranting the piercing of the veil of to commit fraud or wrong, to perpetuate the violation E. Carpio, Jr. In addition, Mr. Gonzales is Chairman of
respondent’s corporate entity."59 of a statutory or other positive legal duty, or dishonest the Board of both corporations, whereas Pauline C. Tan
and unjust act in contravention of plaintiff’s legal right; is concurrently President/General Manager of EIB
As a rule, the parties may raise only questions of law under and Securities, and Dionisio Carpio Jr., is not only director
Rule 45, because the Supreme Court is not a trier of facts. (3) The aforesaid control and breach of duty must of the bank, but also Director Treasurer of defendant
Generally, we are not duty-bound to analyze again and [have] proximately caused the injury or unjust loss EIB Securities;
weigh the evidence introduced in and considered by the complained of.63
tribunals below.60 However, justice for all is of primordial 5. As admitted by the bank parent corporation in its
importance that the Court will not think twice of reviewing The absence of any one of these elements prevents consolidated audited financial statements[,] EIB
the facts, more so because the RTC and the CA arrived in ‘piercing the corporate veil’ in applying the Securities is a CONTROLLED SUBSIDIARY, and for which
contradicting conclusions. ‘instrumentality’ or ‘alter ego’ doctrine, the courts are reason its financial condition and results of operations
concerned with reality and not form, with how the are included and integrated as part of the group’s
"It is a fundamental principle of corporation law that a corporation operated and the individual defendant’s consolidated financial statements, examined and
corporation is an entity separate and distinct from its relationship to that operation.64 Hence, all three elements audited by the same auditing firm;
stockholders and from other corporations to which it may should concur for the alter ego doctrine to be applicable.
be connected. But, this separate and distinct personality of 6. The lawyers handling the suits and legal matters of
a corporation is merely a fiction created by law for In its decision, the RTC maintained that the subsequently defendant EIB Securities are the same lawyers in the
convenience and to promote justice. So, when the notion enumerated factors betray the true nature of E-Securities Legal Department of the bank parent corporation.
of separate juridical personality is used to defeat public as a mere alter ego of Export Bank: 1âwphi1 The Court notes that in [the] above-entitled
convenience, justify wrong, protect fraud or defend crime, suit, the lawyers who at the start represented said
or is used as a device to defeat the labor laws, this 1. Defendant EIB Securities, a subsidiary corporation defendant EIB Securities and filed all the pleadings and
separate personality of the corporation may be disregarded 100% totally owned by Export and Industry Bank, Inc., filings in its behalf are also the lawyers in the Legal
or the veil of corporate fiction pierced. This is true likewise was only re-activated by the latter in 2002-2003 and Services Division of the bank parent corporation. They
when the corporation is merely an adjunct, a business the continuance of its operations was geared for no are Attys. Emmanuel A. Silva, Leonardo C. Bool, Riva
conduit or an alter ego of another corporation."61 other reason tha[n] to serve as the securities Khristine E. Maala and Ma. Esmeralda R. Cunanan, all of
brokerage arm of said parent corporation bank; whom worked at the Legal Services Division of Export
"Where one corporation is so organized and controlled and Industry Bank located at 36/F, Exportbank Plaza, Don
its affairs are conducted so that it is, in fact, a mere 2. It was the parent corporation bank that provided and Chino Roces Avenue, cor. Sen. Gil Puyat Avenue, Makati
instrumentality or adjunct of the other, the fiction of the infused the fresh working cash capital needed by City.
corporate entity of the "instrumentality" may be defendant EIB Securities which prior thereto was non-
disregarded. The control necessary to invoke the rule is not operating and severely cash-strapped. [This was so 7. Finally[,] and this is very significant, the control and
majority or even complete stock control but such attested by the then Corporate Secretary of both sway that the bank parent corporation held over

Page 149 of 160


CORPORATION LAW CASES MAYORDO, M.A
LLB 3

defendant EIB Securities was prevailing in June 2004 of these DMCI shares ballooned to P1,465,799,000.00.74 It
when the very act complained of in plaintiff’s must be taken into account that this unexpected turnabout SO ORDERED.
Complaint took place, namely the unauthorized did not inure to the benefit of E-Securities, much less
disposal of the 32,180,000 DMCI shares of stock. Being Export Bank. G.R. No. 182729               September 29, 2010
then under the direction and control of the bank parent KUKAN INTERNATIONAL CORPORATION, Petitioner, 

corporation, the unauthorized disposal of those shares Furthermore, ownership by Export Bank of a great majority vs. HON. AMOR REYES, in her capacity as Presiding Judge
by defendant EIB Securities is attributable to, and the or all of stocks of E-Securities and the existence of of the Regional Trial Court of Manila, Branch 21, and
responsibility of the former.65 interlocking directorates may serve as badges of control, ROMEO M. MORALES, doing business under the name and
but ownership of another corporation, per se, without style "RM Morales Trophies and Plaques," Respondents.
All the foregoing circumstances, with the exception of the proof of actuality of the other conditions are insufficient to
admitted stock ownership, were however not properly establish an alter ego relationship or connection between DECISION
pleaded and proved in accordance with the Rules of Court. the two corporations, which will justify the setting aside of VELASCO, JR., J.:
66 These were merely raised by the petitioners for the first the cover of corporate fiction. The Court has declared that
time in their Motion for Issuance of an Alias Writ of "mere ownership by a single stockholder or by another The Case
Execution67 and Reply,68 which the Court cannot consider. corporation of all or nearly all of the capital stock of a
"Whether the separate personality of the corporation corporation is not of itself sufficient ground for This Petition for Review on Certiorari under Rule 45 seeks
should be pierced hinges on obtaining facts appropriately disregarding the separate corporate personality." The Court to nullify and reverse the January 23, 2008 Decision1 and
pleaded or proved."69 has likewise ruled that the "existence of interlocking the April 16, 2008 Resolution2 rendered by the Court of
directors, corporate officers and shareholders is not enough Appeals (CA) in CA-G.R. SP No. 100152.
Albeit the RTC bore emphasis on the alleged control justification to pierce the veil of corporate fiction in the
exercised by Export Bank upon its subsidiary E-Securities, absence of fraud or other public policy considerations."75 The assailed CA decision affirmed the March 12, 20073 and
"[c]ontrol, by itself, does not mean that the controlled June 7, 20074 Orders of the Regional Trial Court (RTC) of
corporation is a mere instrumentality or a business conduit While the courts have been granted the colossal authority Manila, Branch 21, in Civil Case No. 99-93173, entitled
of the mother company. Even control over the financial and to wield the sword which pierces through the veil of Romeo M. Morales, doing business under the name and
operational concerns of a subsidiary company does not by corporate fiction, concomitant to the exercise of this style RM Morales Trophies and Plaques v. Kukan, Inc. In the
itself call for disregarding its corporate fiction. There must power, is the responsibility to uphold the doctrine of said orders, the RTC disregarded the separate corporate
be a perpetuation of fraud behind the control or at least a separate entity, when rightly so; as it has for so long identities of Kukan, Inc. and Kukan International
fraudulent or illegal purpose behind the control in order to encouraged businessmen to enter into economic endeavors Corporation and declared them to be one and the same
justify piercing the veil of corporate fiction. Such fraught with risks and where only a few dared to venture. entity. Accordingly, the RTC held Kukan International
fraudulent intent is lacking in this case."70 Corporation, albeit not impleaded in the underlying
Hence, any application of the doctrine of piercing the complaint of Romeo M. Morales, liable for the judgment
Moreover, there was nothing on record demonstrative of corporate veil should be done with caution. A court should award decreed in a Decision dated November 28, 20025 in
Export Bank’s wrongful intent in setting up a subsidiary, E- be mindful of the milieu where it is to be applied. It must favor of Morales and against Kukan, Inc.
Securities. If used to perform legitimate functions, a be certain that the corporate fiction was misused to such
subsidiary’s separate existence shall be respected, and the an extent that injustice, fraud, or crime was committed The Facts
liability of the parent corporation as well as the subsidiary against another, in disregard of its rights. The wrongdoing
will be confined to those arising in their respective must be clearly and convincingly established; it cannot be Sometime in March 1998, Kukan, Inc. conducted a bidding
business.71 To justify treating the sole stockholder or presumed. Otherwise, an injustice that was never for the supply and installation of signages in a building
holding company as responsible, it is not enough that the unintended may result from an erroneous application.76 being constructed in Makati City. Morales tendered the
subsidiary is so organized and controlled as to make it winning bid and was awarded the PhP 5 million contract.
"merely an instrumentality, conduit or adjunct" of its In closing, we understand that the petitioners are Some of the items in the project award were later excluded
stockholders. It must further appear that to recognize their disgruntled at the turnout of this case-that they cannot resulting in the corresponding reduction of the contract
separate entities would aid in the consummation of a enforce the award due them on its entirety; however, the price to PhP 3,388,502. Despite his compliance with his
wrong.72 Court cannot supplant a remedy which is not sanctioned by contractual undertakings, Morales was only paid the
As established in the main case73 and reiterated by the CA, our laws and prescribed rules. amount of PhP 1,976,371.07, leaving a balance of PhP
the subject 32,180,000 DMCI shares which E-Securities is WHEREFORE, the petition in G.R. No. 199687 is hereby 1,412,130.93, which Kukan, Inc. refused to pay despite
obliged to return to the petitioners were originally bought DISMISSED for having been rendered moot and academic. demands. Shortchanged, Morales filed a Complaint6 with
at an average price of P0.38 per share and were sold for an The petition in G.R. No. 201537, meanwhile, is hereby the RTC against Kukan, Inc. for a sum of money, the case
average price of P0.24 per share. The proceeds were then DENIED for lack of merit. Consequently, the Decision dated docketed as Civil Case No. 99-93173 and eventually raffled
used to buy back 61,100,000 KPP shares earlier sold by E- April 26, 2012 of the Court of Appeals in CA-G.R. SP No. to Branch 17 of the court.
Securities. Quite unexpectedly however, the total amount 120979 is AFFIRMED.

Page 150 of 160


CORPORATION LAW CASES MAYORDO, M.A
LLB 3

Following the joinder of issues after Kukan, Inc. filed an Morales’ motion. By Order of May 29, 20039 as reiterated in
answer with counterclaim, trial ensued. However, starting a subsequent order, the court denied the omnibus motion. The CA later denied KIC’s motion for reconsideration in the
November 2000, Kukan, Inc. no longer appeared and assailed resolution.
participated in the proceedings before the trial court, In a bid to establish the link between KIC and Kukan, Inc.,
prompting the RTC to declare Kukan, Inc. in default and and thus determine the true relationship between the two, Hence, the instant petition for review, with the following
paving the way for Morales to present his evidence ex Morales filed a Motion for Examination of Judgment Debtors issues KIC raises for the Court’s consideration:
parte. dated May 4, 2005. In this motion Morales sought that
subponae be issued against the primary stockholders of 1. There is no legal basis for the [CA] to resolve and
On November 28, 2002, the RTC rendered a Decision finding Kukan, Inc., among them Michael Chan, a.k.a. Chan Kai Kit. declare that petitioner’s Constitutional Right to Due
for Morales and against Kukan, Inc., disposing as follows: This too was denied by the trial court in an Order dated Process was not violated by the public respondent in
May 24, 2005.10 rendering the Orders dated March 12, 2007 and June 7,
WHEREFORE, consistent with Section 5, Rule 18 of the 1997 2007 and in declaring petitioner to be liable for the
Rules of Civil Procedure, and by preponderance of Morales then sought the inhibition of the presiding judge, judgment obligations of the corporation "Kukan, Inc." to
evidence, judgment is hereby rendered in favor of the Eduardo B. Peralta, Jr., who eventually granted the motion. private respondent – as petitioner is a stranger to the
plaintiff, ordering Kukan, Inc.: The case was re-raffled to Branch 21, presided by public case and was never made a party in the case before the
1. to pay the sum of ONE MILLION TWO HUNDRED ONE respondent Judge Amor Reyes. trial court nor was it ever served a summons and a copy
THOUSAND SEVEN HUNDRED TWENTY FOUR PESOS of the complaint.
(P1,201,724.00) with legal interest at 12% per annum Before the Manila RTC, Branch 21, Morales filed a Motion to 2. There is no legal basis for the [CA] to resolve and
from February 17, 1999 until full payment; Pierce the Veil of Corporate Fiction to declare KIC as having declare that the Orders dated March 12, 2007 and June
2. to pay the sum of FIFTY THOUSAND PESOS no existence separate from Kukan, Inc. This time around, 7, 2007 rendered by public respondent declaring the
(P50,000.00) as moral damages; the RTC, by Order dated March 12, 2007, granted the petitioner liable to the judgment obligations of the
3. to pay the sum of TWENTY THOUSAND PESOS, motion, the dispositive portion of which reads: corporation "Kukan, Inc." to private respondent are
(P20,000.00) as reasonable attorney’s fees; and WHEREFORE, premises considered, the motion is hereby valid as said orders of the public respondent modify
4. to pay the sum of SEVEN THOUSAND NINE HUNDRED GRANTED. The Court hereby declares as follows: and/or amend the trial court’s final and executory
SIXTY PESOS and SIX CENTAVOS (P7,960.06) as litigation decision rendered on November 28, 2002.
expenses. 1. defendant Kukan, Inc. and newly created Kukan 3. There is no legal basis for the [CA] to resolve and
International Corp. as one and the same corporation; declare that the Orders dated March 12, 2007 and June
For lack of factual foundation, the counterclaim is 2. the levy made on the properties of Kukan 7, 2007 rendered by public respondent declaring the
DISMISSED. International Corp. is hereby valid; petitioner [KIC] and the corporation "Kukan, Inc." as
IT IS SO ORDERED.7 3. Kukan International Corp. and Michael Chan are one and the same, and, therefore, the Veil of Corporate
jointly and severally liable to pay the amount awarded Fiction between them be pierced – as the procedure
After the above decision became final and executory, to plaintiff pursuant to the decision of November [28], undertaken by public respondent which the [CA] upheld
Morales moved for and secured a writ of execution8 against 2002 which has long been final and executory. is not sanctioned by the Rules of Court and/or
Kukan, Inc. The sheriff then levied upon various personal established jurisprudence enunciated by this Honorable
properties found at what was supposed to be Kukan, Inc.’s SO ORDERED. Supreme Court.12
office at Unit 2205, 88 Corporate Center, Salcedo Village,
Makati City. Alleging that it owned the properties thus From the above order, KIC moved but was denied In gist, the issues to be resolved boil down to the question
levied and that it was a different corporation from Kukan, reconsideration in another Order dated June 7, 2007. of, first, whether the trial court can, after the judgment
Inc., Kukan International Corporation (KIC) filed an against Kukan, Inc. has attained finality, execute it against
Affidavit of Third-Party Claim. Notably, KIC was KIC went to the CA on a petition for certiorari to nullify the the property of KIC; second, whether the trial court
incorporated in August 2000, or shortly after Kukan, Inc. aforesaid March 12 and June 7, 2007 RTC Orders. acquired jurisdiction over KIC; and third, whether the trial
had stopped participating in Civil Case No. 99-93173. and appellate courts correctly applied, under the premises,
On January 23, 2008, the CA rendered the assailed the principle of piercing the veil of corporate fiction.
In reaction to the third party claim, Morales interposed an decision, the dispositive portion of which states:
Omnibus Motion dated April 30, 2003. In it, Morales prayed, The Ruling of the Court
applying the principle of piercing the veil of corporate WHEREFORE, premises considered, the petition is hereby
fiction, that an order be issued for the satisfaction of the DENIED and the assailed Orders dated March 12, 2007 and The petition is meritorious.
judgment debt of Kukan, Inc. with the properties under the June 7, 2007 of the court a quo are both AFFIRMED. No
name or in the possession of KIC, it being alleged that both costs. First Issue: Against Whom Can a Final and Executory
corporations are but one and the same entity. KIC opposed Judgment Be Executed
SO ORDERED.11

Page 151 of 160


CORPORATION LAW CASES MAYORDO, M.A
LLB 3

The preliminary question that must be answered is whether Republic v. Tango16 expounded on the same principle and its As may be noted, the above decision, in unequivocal terms,
or not the trial court can, after adjudging Kukan, Inc. liable exceptions: directed Kukan, Inc. to pay the aforementioned awards to
for a sum of money in a final and executory judgment, Morales. Thus, making KIC, thru the medium of a writ of
execute such judgment debt against the property of KIC. Deeply ingrained in our jurisprudence is the principle that a execution, answerable for the above judgment liability is a
decision that has acquired finality becomes immutable clear case of altering a decision, an instance of granting
The poser must be answered in the negative. and unalterable. As such, it may no longer be modified in relief not contemplated in the decision sought to be
any respect even if the modification is meant to correct executed. And the change does not fall under any of the
In Carpio v. Doroja,13 the Court ruled that the deciding erroneous conclusions of fact or law and whether it will be recognized exceptions to the doctrine of finality and
court has supervisory control over the execution of its made by the court that rendered it or by the highest court immutability of judgment. It is a settled rule that a writ of
judgment: of the land. x x x execution must conform to the fallo of the judgment; as an
inevitable corollary, a writ beyond the terms of the
A case in which an execution has been issued is regarded as The doctrine of finality of judgment is grounded on the judgment is a nullity.17
still pending so that all proceedings on the execution are fundamental principle of public policy and sound practice
proceedings in the suit. There is no question that the court that, at the risk of occasional error, the judgment of courts Thus, on this ground alone, the instant petition can already
which rendered the judgment has a general supervisory and the award of quasi-judicial agencies must become final be granted. Nonetheless, an examination of the other
control over its process of execution, and this power carries on some definite date fixed by law. The only exceptions to issues raised by KIC would be proper.
with it the right to determine every question of fact and the general rule are the correction of clerical errors, the
law which may be involved in the execution. so-called nunc pro tunc entries which cause no prejudice to Second Issue: Propriety of the RTC Assuming Jurisdiction
any party, void judgments, and whenever circumstances over KIC
We reiterated the above holding in Javier v. Court of transpire after the finality of the decision which render its
Appeals14 in this wise: "The said branch has a general execution unjust and inequitable. None of the exceptions The next issue turns on the validity of the execution the
supervisory control over its processes in the execution of its obtains here to merit the review sought. (Emphasis added.) trial court authorized against KIC and its property, given
judgment with a right to determine every question of fact that it was neither made a party nor impleaded in Civil
and law which may be involved in the execution." So, did the RTC, in breach of the doctrine of immutability Case No. 99-93173, let alone served with summons. In other
and inalterability of judgment, order the execution of its words, did the trial court acquire jurisdiction over KIC?
The court’s supervisory control does not, however, extend final decision in a manner as would amount to its
as to authorize the alteration or amendment of a final and prohibited alteration or modification? In the assailed decision, the appellate court deemed KIC to
executory decision, save for certain recognized exceptions, have voluntarily submitted itself to the jurisdiction of the
among which is the correction of clerical errors. Else, the We repair to the dispositive portion of the final and trial court owing to its filing of four (4) pleadings adverted
court violates the principle of finality of judgment and its executory RTC decision. Pertinently, it provides: to earlier, namely: (a) the Affidavit of Third-Party Claim;18
immutability, concepts which the Court, in Tan v. Timbal,15 (b) the Comment and Opposition to Plaintiff’s Omnibus
defined: WHEREFORE, consistent with Section 5, Rule 18 of the 1997 Motion;19 (c) the Motion for Reconsideration of the RTC
As we held in Industrial Management International Rules of Civil Procedure, and by preponderance of Order dated March 12, 2007;20 and (d) the Motion for Leave
Development Corporation vs. NLRC: evidence, judgment is hereby rendered in favor of the to Admit Reply.21 The CA, citing Section 20, Rule 14 of the
plaintiff, ordering Kukan, Inc.: Rules of Court, stated that "the procedural rule on service
It is an elementary principle of procedure that the of summons can be waived by voluntary submission to the
resolution of the court in a given issue as embodied in the 1. to pay the sum of ONE MILLION TWO HUNDRED ONE court’s jurisdiction through any form of appearance by the
dispositive part of a decision or order is the controlling THOUSAND SEVEN HUNDRED TWENTY FOUR PESOS party or its counsel."22
factor as to settlement of rights of the parties. Once a (P1,201,724.00) with legal interest at 12% per annum
decision or order becomes final and executory, it is from February 17, 1999 until full payment; We cannot give imprimatur to the appellate court’s
removed from the power or jurisdiction of the court which 2. to pay the sum of FIFTY THOUSAND PESOS appreciation of the thrust of Sec. 20, Rule 14 of the Rules
rendered it to further alter or amend it. It thereby (P50,000.00) as moral damages; in concluding that the trial court acquired jurisdiction over
becomes immutable and unalterable and any amendment 3. to pay the sum of TWENTY THOUSAND PESOS KIC.
or alteration which substantially affects a final and (P20,000.00) as reasonable attorney’s fees; and
executory judgment is null and void for lack of jurisdiction, 4. to pay the sum of SEVEN THOUSAND NINE HUNDRED Orion Security Corporation v. Kalfam Enterprises, Inc.23
including the entire proceedings held for that purpose. An SIXTY PESOS and SIX CENTAVOS (P7,960.06) as litigation explains how courts acquire jurisdiction over the parties in
order of execution which varies the tenor of the judgment expenses. a civil case:
or exceeds the terms thereof is a nullity. (Emphasis Courts acquire jurisdiction over the plaintiffs upon the
supplied.) x x x x (Emphasis supplied.) filing of the complaint. On the other hand, jurisdiction over
the defendants in a civil case is acquired either through the
service of summons upon them or through their voluntary

Page 152 of 160


CORPORATION LAW CASES MAYORDO, M.A
LLB 3

appearance in court and their submission to its authority. KIC could not file before the RTC a motion to dismiss and hesitate to disregard the corporate veil when it is misused
(Emphasis supplied.) its attachments in Civil Case No. 99-93173, precisely or when necessary in the interest of justice. x x x
because KIC was neither impleaded nor served with (Emphasis supplied.)
In the fairly recent Palma v. Galvez,24 the Court reiterated summons. Consequently, KIC could only assert and claim
its holding in Orion Security Corporation, stating: "[I]n civil through its affidavits, comments, and motions filed by The same principle was the subject and discussed in Rivera
cases, the trial court acquires jurisdiction over the person special appearance before the RTC that it is separate and v. United Laboratories, Inc.:
of the defendant either by the service of summons or by distinct from Kukan, Inc.
the latter’s voluntary appearance and submission to the While a corporation may exist for any lawful purpose, the
authority of the former." Following La Naval Drug Corporation,30 KIC cannot be law will regard it as an association of persons or, in case of
The court’s jurisdiction over a party-defendant resulting deemed to have waived its objection to the court’s lack of two corporations, merge them into one, when its corporate
from his voluntary submission to its authority is provided jurisdiction over its person. It would defy logic to say that legal entity is used as a cloak for fraud or illegality. This is
under Sec. 20, Rule 14 of the Rules, which states: KIC unequivocally submitted itself to the jurisdiction of the the doctrine of piercing the veil of corporate fiction. The
RTC when it strongly asserted that it and Kukan, Inc. are doctrine applies only when such corporate fiction is used to
Section 20. Voluntary appearance. – The defendant’s different entities. In the scheme of things obtaining, KIC defeat public convenience, justify wrong, protect fraud, or
voluntary appearance in the actions shall be equivalent to had no other option but to insist on its separate identity defend crime, or when it is made as a shield to confuse the
service of summons. The inclusion in a motion to dismiss of and plead for relief consistent with that position. legitimate issues, or where a corporation is the mere alter
other grounds aside from lack of jurisdiction over the ego or business conduit of a person, or where the
person of the defendant shall not be deemed a voluntary Third Issue: Piercing the Veil of Corporate Fiction corporation is so organized and controlled and its affairs
appearance. are so conducted as to make it merely an instrumentality,
The third and main issue in this case is whether or not the agency, conduit or adjunct of another corporation.
To be sure, the CA’s ruling that any form of appearance by trial and appellate courts correctly applied the principle of
the party or its counsel is deemed as voluntary appearance piercing the veil of corporate entity––called also as To disregard the separate juridical personality of a
finds support in the kindred Republic v. Ker & Co., Ltd.25 disregarding the fiction of a separate juridical personality corporation, the wrongdoing must be established clearly
and De Midgely v. Ferandos.26 of a corporation––to support a conclusion that Kukan, Inc. and convincingly. It cannot be presumed.33 (Emphasis
and KIC are but one and the same corporation with respect supplied.)
Republic and De Midgely, however, have already been to the contract award referred to at the outset. This
modified if not altogether superseded27 by La Naval Drug principle finds its context on the postulate that a Now, as before the appellate court, petitioner KIC
Corporation v. Court of Appeals,28 wherein the Court corporation is an artificial being invested with a personality maintains that the RTC violated its right to due process
essentially ruled and elucidated on the current view in our separate and distinct from those of the stockholders and when, in the execution of its November 28, 2002 Decision,
jurisdiction, to wit: "[A] special appearance before the from other corporations to which it may be connected or the court authorized the issuance of the writ against KIC for
court––challenging its jurisdiction over the person through a related.31 Kukan, Inc.’s judgment debt, albeit KIC has never been a
motion to dismiss even if the movant invokes other In Pantranco Employees Association (PEA-PTGWO) v. party to the underlying suit. As a counterpoint, Morales
grounds––is not tantamount to estoppel or a waiver by the National Labor Relations Commission,32 the Court revisited argues that KIC’s specific concern on due process and on
movant of his objection to jurisdiction over his person; and the subject principle of piercing the veil of corporate the validity of the writ to execute the RTC’s November 28,
such is not constitutive of a voluntary submission to the fiction and wrote: 2002 Decision would be mooted if it were established that
jurisdiction of the court."29 Under the doctrine of "piercing the veil of corporate KIC and Kukan, Inc. are indeed one and the same
fiction," the court looks at the corporation as a mere corporation.
In the instant case, KIC was not made a party-defendant in collection of individuals or an aggregation of persons
Civil Case No. 99-93173. Even if it is conceded that it raised undertaking business as a group, disregarding the separate Morales’ contention is untenable.
affirmative defenses through its aforementioned pleadings, juridical personality of the corporation unifying the group.
KIC never abandoned its challenge, however implicit, to the Another formulation of this doctrine is that when two The principle of piercing the veil of corporate fiction, and
RTC’s jurisdiction over its person. The challenge was business enterprises are owned, conducted and controlled the resulting treatment of two related corporations as one
subsumed in KIC’s primary assertion that it was not the by the same parties, both law and equity will, when and the same juridical person with respect to a given
same entity as Kukan, Inc. Pertinently, in its Comment and necessary to protect the rights of third parties, disregard transaction, is basically applied only to determine
Opposition to Plaintiff’s Omnibus Motion dated May 20, the legal fiction that two corporations are distinct entities established liability;34 it is not available to confer on the
2003, KIC entered its "special but not voluntary and treat them as identical or as one and the same. court a jurisdiction it has not acquired, in the first place,
appearance" alleging therein that it was a different entity over a party not impleaded in a case. Elsewise put, a
and has a separate legal personality from Kukan, Inc. And Whether the separate personality of the corporation corporation not impleaded in a suit cannot be subject to
KIC would consistently reiterate this assertion in all its should be pierced hinges on obtaining facts appropriately the court’s process of piercing the veil of its corporate
pleadings, thus effectively resisting all along the RTC’s pleaded or proved. However, any piercing of the corporate fiction. In that situation, the court has not acquired
jurisdiction of its person. It cannot be overemphasized that veil has to be done with caution, albeit the Court will not jurisdiction over the corporation and, hence, any

Page 153 of 160


CORPORATION LAW CASES MAYORDO, M.A
LLB 3

proceedings taken against that corporation and its property A motion generally relates to procedure and is often A corporation is an artificial being created by operation of
would infringe on its right to due process. Aguedo Agbayani, resorted to in order to correct errors which have crept in law. x x x It has a personality separate and distinct from
a recognized authority on Commercial Law, stated as much: along the line of the principal action’s progress. Generally, the persons composing it, as well as from any other legal
where there is a procedural defect in a proceeding and no entity to which it may be related. This is basic.
23. Piercing the veil of corporate entity applies to method under statute or rule of court by which it may be
determination of liability not of jurisdiction. x x x called to the attention of the court, a motion is an Equally well-settled is the principle that the corporate
appropriate remedy. In many jurisdictions, the motion has mask may be removed or the corporate veil pierced when
This is so because the doctrine of piercing the veil of replaced the common-law pleas testing the sufficiency of the corporation is just an alter ego of a person or of
corporate fiction comes to play only during the trial of the the pleadings, and various common-law writs, such as writ another corporation. For reasons of public policy and in the
case after the court has already acquired jurisdiction over of error coram nobis and audita querela. In some cases, a interest of justice, the corporate veil will justifiably be
the corporation. Hence, before this doctrine can be motion may be one of several remedies available. For impaled only when it becomes a shield for fraud, illegality
applied, based on the evidence presented, it is imperative example, in some jurisdictions, a motion to vacate an order or inequity committed against third persons.
that the court must first have jurisdiction over the is a remedy alternative to an appeal therefrom.
corporation.35 x x x (Emphasis supplied.) Hence, any application of the doctrine of piercing the
The implication of the above comment is twofold: (1) the Statutes governing motions are given a liberal construction. corporate veil should be done with caution. A court should
court must first acquire jurisdiction over the corporation or 36 (Emphasis supplied.) be mindful of the milieu where it is to be applied. It must
corporations involved before its or their separate be certain that the corporate fiction was misused to such
personalities are disregarded; and (2) the doctrine of The bottom line issue of whether Morales can proceed an extent that injustice, fraud, or crime was committed
piercing the veil of corporate entity can only be raised against KIC for the judgment debt of Kukan, Inc.––assuming against another, in disregard of its rights. The wrongdoing
during a full-blown trial over a cause of action duly hypothetically that he can, applying the piercing the must be clearly and convincingly established; it cannot be
commenced involving parties duly brought under the corporate veil principle––resolves itself into the question of presumed. Otherwise, an injustice that was never
authority of the court by way of service of summons or whether a mere motion is the appropriate vehicle for such unintended may result from an erroneous application.
what passes as such service. purpose. This Court has pierced the corporate veil to ward off a
judgment credit, to avoid inclusion of corporate assets as
The issue of jurisdiction or the lack of it over KIC has Verily, Morales espouses the application of the principle of part of the estate of the decedent, to escape liability
already been discussed. Anent the matter of the time and piercing the corporate veil to hold KIC liable on theory that arising from a debt, or to perpetuate fraud and/or confuse
manner of raising the principle in question, it is undisputed Kukan, Inc. was out to defraud him through the use of the legitimate issues either to promote or to shield unfair
that no full-blown trial involving KIC was had when the RTC separate and distinct personality of another corporation, objectives or to cover up an otherwise blatant violation of
disregarded the corporate veil of KIC. The reason for this KIC. In net effect, Morales’ adverted motion to pierce the the prohibition against forum-shopping. Only in these and
actuality is simple and undisputed: KIC was not impleaded veil of corporate fiction dated January 3, 2007 stated a similar instances may the veil be pierced and
in Civil Case No. 99-93173 and that the RTC did not acquire new cause of action, i.e., for the liability of judgment disregarded. (Emphasis supplied.)
jurisdiction over it. It was dragged to the case after it debtor Kukan, Inc. to be borne by KIC on the alleged
reacted to the improper execution of its properties and identity of the two corporations. This new cause of action In fine, to justify the piercing of the veil of corporate
veritably hauled to court, not thru the usual process of should be properly ventilated in another complaint and fiction, it must be shown by clear and convincing proof that
service of summons, but by mere motion of a party with subsequent trial where the doctrine of piercing the the separate and distinct personality of the corporation was
whom it has no privity of contract and after the decision in corporate veil can, if appropriate, be applied, based on the purposefully employed to evade a legitimate and binding
the main case had already become final and executory. As evidence adduced. Establishing the claim of Morales and commitment and perpetuate a fraud or like wrongdoings.
to the propriety of a plea for the application of the the corresponding liability of KIC for Kukan Inc.’s To be sure, the Court has, on numerous occasions,38 applied
principle by mere motion, the following excerpts are indebtedness could hardly be the subject, under the the principle where a corporation is dissolved and its assets
instructive: premises, of a mere motion interposed after the principal are transferred to another to avoid a financial liability of
action against Kukan, Inc. alone had peremptorily been the first corporation with the result that the second
Generally, a motion is appropriate only in the absence of terminated. After all, a complaint is one where the plaintiff corporation should be considered a continuation and
remedies by regular pleadings, and is not available to settle alleges causes of action. successor of the first entity.
important questions of law, or to dispose of the merits of
the case. A motion is usually a proceeding incidental to an In any event, the principle of piercing the veil of corporate In those instances when the Court pierced the veil of
action, but it may be a wholly distinct or independent fiction finds no application to the instant case. corporate fiction of two corporations, there was a
proceeding. A motion in this sense is not within this confluence of the following factors:
discussion even though the relief demanded is denominated As a general rule, courts should be wary of lifting the
an "order." corporate veil between corporations, however related. 1. A first corporation is dissolved;
Philippine National Bank v. Andrada Electric Engineering
Company37 explains why:

Page 154 of 160


CORPORATION LAW CASES MAYORDO, M.A
LLB 3

2. The assets of the first corporation is transferred to a practically doing the same kind of business as that of The two companies have a related business purpose.
second corporation to avoid a financial liability of the Kukan, Inc.39 (Emphasis supplied.) Considering these circumstances, the obvious conclusion is
first corporation; and that the creation of Kukan International Corporation served
3. Both corporations are owned and controlled by the As is apparent from its disquisition, the RTC brushed aside as a device to evade the obligation incurred by Kukan, Inc.
same persons such that the second corporation should the separate corporate existence of Kukan, Inc. and KIC on and yet profit from the goodwill attained by the name
be considered as a continuation and successor of the the main argument that Michael Chan owns 40% of the "Kukan" by continuing to engage in the same line of
first corporation. common shares of both corporations, obviously oblivious business with the same list of clients.42 (Emphasis
that overlapping stock ownership is a common business supplied.)
In the instant case, however, the second and third factors phenomenon. It must be remembered, however, that KIC’s
are conspicuously absent. There is, therefore, no properties were the ones seized upon levy on execution and Evidently, the CA found the meager paid-up capitalization
compelling justification for disregarding the fiction of not that of Kukan, Inc. or of Michael Chan for that matter. of Kukan, Inc. and the similarity of the business activities in
corporate entity separating Kukan, Inc. from KIC. In Mere ownership by a single stockholder or by another which both corporations are engaged as a jumping board to
applying the principle, both the RTC and the CA miserably corporation of a substantial block of shares of a corporation its conclusion that the creation of KIC "served as a device
failed to identify the presence of the abovementioned does not, standing alone, provide sufficient justification for to evade the obligation incurred by Kukan, Inc." The
factors. Consider: disregarding the separate corporate personality.40 For this appellate court, however, left a gaping hole by failing to
ground to hold sway in this case, there must be proof that demonstrate that Kukan, Inc. and its stockholders
The RTC disregarded the separate corporate personalities Chan had control or complete dominion of Kukan and KIC’s defrauded Morales. In fine, there is no showing that the
of Kukan, Inc. and KIC based on the following premises and finances, policies, and business practices; he used such incorporation, and the separate and distinct personality, of
arguments: control to commit fraud; and the control was the proximate KIC was used to defeat Morales’ right to recover from
cause of the financial loss complained of by Morales. The Kukan, Inc. Judging from the records, no serious attempt
While it is true that a corporation has a separate and absence of any of the elements prevents the piercing of the was made to levy on the properties of Kukan, Inc. Morales
distinct personality from its stockholder, director and corporate veil.41 And indeed, the records do not show the could not, thus, validly argue that Kukan, Inc. tried to
officers, the law expressly provides for an exception. When presence of these elements. avoid liability or had no property against which to proceed.
Michael Chan, the Managing Director of defendant Kukan, On the other hand, the CA held:
Inc. (majority stockholder of the newly formed corporation Morales further contends that Kukan, Inc.’s closure is
[KIC]) confirmed the award to plaintiff to supply and install In the present case, the facts disclose that Kukan, Inc. evidenced by its failure to file its 2001 General Information
interior signages in the Enterprise Center he (Michael Chan, entered into a contractual obligation x x x worth more than Sheet (GIS) with the Securities and Exchange Commission.
Managing Director of defendant Kukan, Inc.) knew that three million pesos although it had only Php5,000.00 paid- However, such fact does not necessarily mean that Kukan,
there was no sufficient corporate funds to pay its up capital; [KIC] was incorporated shortly before Kukan, Inc. had altogether ceased operations, as Morales would
obligation/account, thus implying bad faith on his part and Inc. suddenly ceased to appear and participate in the trial; have this Court believe, for it is stated on the face of the
fraud in contracting the obligation. Michael Chan neither [KIC’s] purpose is related and somewhat akin to that of GIS that it is only upon a failure to file the corporate GIS
returned the interior signages nor tendered payment to the Kukan, Inc.; and in [KIC] Michael Chan, a.k.a., Chan Kai Kit, for five (5) consecutive years that non-operation shall be
plaintiff. This circumstance may warrant the piercing of the holds forty percent of the outstanding stocks, while he presumed.
veil of corporation fiction. Having been guilty of bad faith formerly held the same amount of stocks in Kukan Inc.
in the management of corporate matters the corporate These would lead to the inescapable conclusion that Kukan, The fact that Kukan, Inc. entered into a PhP 3.3 million
trustee, director or officer may be held personally liable. x Inc. committed fraudulent representation by awarding to contract when it only had a paid-up capital of PhP 5,000 is
xx the private respondent the contract with full knowledge not an indication of the intent on the part of its
that it was not in a position to comply with the obligation it management to defraud creditors. Paid-up capital is merely
Since fraud is a state of mind, it need not be proved by had assumed because of inadequate paid-up capital. It seed money to start a corporation or a business entity. As in
direct evidence but may be inferred from the bears stressing that shareholders should in good faith put at this case, it merely represented the capitalization upon
circumstances of the case. x x x [A]nd the circumstances the risk of the business, unencumbered capital reasonably incorporation in 1997 of Kukan, Inc. Paid-up capitalization
are: the signature of Michael Chan, Managing Director of adequate for its prospective liabilities. The capital should of PhP 5,000 is not and should not be taken as a reflection
Kukan, Inc. appearing in the confirmation of the award sent not be illusory or trifling compared with the business to be of the firm’s capacity to meet its recurrent and long-term
to the plaintiff; signature of Chan Kai Kit, a British National done and the risk of loss. obligations. It must be borne in mind that the equity
appearing in the Articles of Incorporation and signature of portion cannot be equated to the viability of a business
Michael Chan also a British National appearing in the Further, it is clear that [KIC] is a continuation and successor concern, for the best test is the working capital which
Articles of Incorporation [of] Kukan International Corp. give of Kukan, Inc. Michael Chan, a.k.a. Chan Kai Kit has the consists of the liquid assets of a given business relating to
the impression that they are one and the same person, that largest block of shares in both business enterprises. The the nature of the business concern
Michael Chan and Chan Kai Kit are both majority emergence of the former was cleverly timed with the hasty
stockholders of Kukan International Corp. and Kukan, Inc. withdrawal of the latter during the trial to avoid the Neither should the level of paid-up capital of Kukan, Inc.
holding 40% of the stocks; that Kukan International Corp. is financial liability that was eventually suffered by the latter. upon its incorporation be viewed as a badge of fraud, for it

Page 155 of 160


CORPORATION LAW CASES MAYORDO, M.A
LLB 3

is in compliance with Sec. 13 of the Corporation Code,43 WHEREFORE, the petition is hereby GRANTED. The CA’s and 43% of the shares of NMIC, respectively, except for five
which only requires a minimum paid-up capital of PhP January 23, 2008 Decision and April 16, 2008 Resolution in qualifying shares.8 As of September 1984, the members of
5,000. CA-G.R. SP No. 100152 are hereby REVERSED and SET ASIDE. the Board of Directors of NMIC, namely, Jose Tengco, Jr.,
The levy placed upon the personal properties of Kukan Rolando Zosa, Ruben Ancheta, Geraldo Agulto, and Faustino
The suggestion that KIC is but a continuation and successor International Corporation is hereby ordered lifted and the Agbada, were either from DBP or PNB.9
of Kukan, Inc., owned and controlled as they are by the personal properties ordered returned to Kukan
same stockholders, stands without factual basis. It is true International Corporation. The RTC of Manila, Branch 21 is Subsequently, NMIC engaged the services of Hercon, Inc.,
that Michael Chan, a.k.a. Chan Kai Kit, owns 40% of the hereby directed to execute the RTC Decision dated for NMIC's Mine Stripping and Road Construction Program in
outstanding capital stock of both corporations. But such November 28, 2002 against Kukan, Inc. with reasonable 1985 for a total contract price of P35,770,120. After
circumstance, standing alone, is insufficient to establish dispatch. computing the payments already made by NMIC under the
identity. There must be at least a substantial identity of program and crediting the NMIC's receivables from
stockholders for both corporations in order to consider this No costs. SO ORDERED.
factor to be constitutive of corporate identity. Hercon, Inc., the latter found that NMIC still has an unpaid
It would not avail Morales any to rely44 on General Credit G.R. NO. 167530 : March 13, 2013 balance of P8,370,934.74.10 Hercon, Inc. made several
Corporation v. Alsons Development and Investment PHILIPPINE NATIONAL BANK, Petitioner, v. HYDRO demands on NMIC, including a letter of final demand dated
Corporation.45 General Credit Corporation is factually not RESOURCES CONTRACTORS CORPORATION, Respondent. August 12, 1986, and when these were not heeded, a
on all fours with the instant case. There, the common complaint for sum of money was filed in the RTC of Makati,
stockholders of the corporations represented 90% of the G.R. NO. 167561 Branch 136 seeking to hold petitioners NMIC, DBP, and PNB
outstanding capital stock of the companies, unlike here ASSET PRIVATIZATION TRUST, Petitioner, v. HYDRO solidarily liable for the amount owing Hercon, Inc.11 The
where Michael Chan merely represents 40% of the RESOURCES CONTRACTORS CORPORATION, Respondent. case was docketed as Civil Case No. 15375.
outstanding capital stock of both KIC and Kukan, Inc., not
even a majority of it. In that case, moreover, evidence was G.R. NO. 167603 Subsequent to the filing of the complaint, Hercon, Inc. was
adduced to support the finding that the funds of the second DEVELOPMENT BANK OF THE PHILIPPINES, Petitioner, v. acquired by HRCC in a merger. This prompted the
corporation came from the first. Finally, there was proof in HYDRO RESOURCES CONTRACTORS CORPORATION, amendment of the complaint to substitute HRCC for
General Credit Corporation of complete control, such that Respondent. Hercon, Inc.12
one corporation was a mere dummy or alter ego of the
other, which is absent in the instant case. DECISION Thereafter, on December 8, 1986, then President Corazon
LEONARDO-DE CASTRO, J.: C. Aquino issued Proclamation No. 50 creating the APT for
Evidently, the aforementioned case relied upon by Morales the expeditious disposition and privatization of certain
cannot justify the application of the principle of piercing These petitions for review on certiorari1 assail the government corporations and/or the assets thereof.
the veil of corporate fiction to the instant case. As shown Decision2 dated November 30, 2004 and the Resolution3 Pursuant to the said Proclamation, on February 27, 1987,
by the records, the name Michael Chan, the similarity of dated March 22, 2005 of the Court of Appeals in CA-G.R. CV DBP and PNB executed their respective deeds of transfer in
business activities engaged in, and incidentally the word No. 57553. The said Decision affirmed the Decision4 dated favor of the National Government assigning, transferring
"Kukan" appearing in the corporate names provide the November 6, 1995 of the Regional Trial Court (RTC) of and conveying certain assets and liabilities, including their
nexus between Kukan, Inc. and KIC. As illustrated, these Makati City, Branch 62, granting a judgment award of respective stakes in NMIC.13 In turn and on even date, the
circumstances are insufficient to establish the identity of P8,370,934.74, plus legal interest, in favor of respondent National Government transferred the said assets and
KIC as the alter ego or successor of Kukan, Inc. Hydro Resources Contractors Corporation (HRCC) with the liabilities to the APT as trustee under a Trust Agreement.14
modification that the Privatization and Management Office Thus, the complaint was amended for the second time to
It bears reiterating that piercing the veil of corporate (PMO), successor of petitioner Asset Privatization Trust implead and include the APT as a defendant.
fiction is frowned upon. Accordingly, those who seek to (APT),5 has been held solidarily liable with Nonoc Mining
pierce the veil must clearly establish that the separate and and Industrial Corporation (NMIC) 6 and petitioners In its answer,15 NMIC claimed that HRCC had no cause of
distinct personalities of the corporations are set up to Philippine National Bank (PNB) and Development Bank of action. It also asserted that its contract with HRCC was
justify a wrong, protect fraud, or perpetrate a deception. the Philippines (DBP), while the Resolution denied entered into by its then President without any authority.
In the concrete and on the assumption that the RTC has reconsideration separately prayed for by PNB, DBP, and APT. Moreover, the said contract allegedly failed to comply with
validly acquired jurisdiction over the party concerned, laws, rules and regulations concerning government
Morales ought to have proved by convincing evidence that Sometime in 1984, petitioners DBP and PNB foreclosed on contracts. NMIC further claimed that the contract amount
Kukan, Inc. was collapsed and thereafter KIC purposely certain mortgages made on the properties of Marinduque was manifestly excessive and grossly disadvantageous to
formed and operated to defraud him. Morales has not to us Mining and Industrial Corporation (MMIC). As a result of the the government. NMIC made counterclaims for the amounts
discharged his burden. foreclosure, DBP and PNB acquired substantially all the already paid to Hercon, Inc. and attorney's fees, as well as
assets of MMIC and resumed the business operations of the payment for equipment rental for four trucks, replacement
defunct MMIC by organizing NMIC.7 DBP and PNB owned 57%

Page 156 of 160


CORPORATION LAW CASES MAYORDO, M.A
LLB 3

of parts and other services, and damage to some of NMIC's controlled by the same parties, both law and equity will,
properties.16 when necessary to protect the rights of third persons, We find it therefore correct for the lower court to have
disregard legal fiction that two (2) corporations are distinct ruled that:
For its part, DBP's answer17 raised the defense that HRCC entities, and treat them as identical." (Phil. Veterans
had no cause of action against it because DBP was not privy Investment Development Corp. vs. CA, 181 SCRA 669). "From all indications, it appears that NMIC is a mere
to HRCC's contract with NMIC. Moreover, NMIC's juridical adjunct, business conduit or alter ego of both DBP and PNB.
personality is separate from that of DBP. DBP further From all indications, it appears that NMIC is a mere Thus, the DBP and PNB are jointly and severally liable with
interposed a counterclaim for attorney's fees.18 adjunct, business conduit or alter ego of both DBP and PNB. NMIC for the latter's unpaid obligation to plaintiff."26
Thus, the DBP and PNB are jointly and severally liable with (Citation omitted.)
PNB's answer19 also invoked lack of cause of action against NMIC for the latter's unpaid obligations to plaintiff.23
it. It also raised estoppel on HRCC's part and laches as The Court of Appeals then concluded that, "in keeping with
defenses, claiming that the inclusion of PNB in the Having found DBP and PNB solidarily liable with NMIC, the the concept of justice and fair play," the corporate veil of
complaint was the first time a demand for payment was dispositive portion of the Decision of the trial court reads: NMIC should be pierced, ratiocinating:
made on it by HRCC. PNB also invoked the separate
juridical personality of NMIC and made counterclaims for For to treat NMIC as a separate legal entity from DBP and
moral damages and attorney's fees. WHEREFORE, in view of the foregoing, judgment is hereby PNB for the purpose of securing beneficial contracts, and
rendered in favor of the plaintiff HYDRO RESOURCES then using such separate entity to evade the payment of a
APT set up the following defenses in its answer21: lack of CONTRACTORS CORPORATION and against the defendants just debt, would be the height of injustice and iniquity.
cause of action against it, lack of privity between Hercon, N O N O C M I N I N G A N D I N D U S T R I A L C O R P O R AT I O N , Surely that could not have been the intendment of the law
Inc. and APT, and the National Government's preferred lien DEVELOPMENT BANK OF THE PHILIPPINES and PHILIPPINE with respect to corporations. x x x.27
over the assets of NMIC.22 NATIONAL BANK, ordering the aforenamed defendants, to
pay the plaintiff jointly and severally, the sum of The dispositive portion of the Decision of the Court of
After trial, the RTC of Makati rendered a Decision dated P8,370,934.74 plus legal interest thereon from date of Appeals reads:
November 6, 1995 in favor of HRCC. It pierced the demand, and attorney's fees equivalent to 25% of the
corporate veil of NMIC and held DBP and PNB solidarily judgment award. WHEREFORE, premises considered, the Decision appealed
liable with NMIC: from is hereby MODIFIED. The judgment in favor of
The complaint against APT is hereby dismissed. However, appellee Hydro Resources Contractors Corporation in the
On the issue of whether or not there is sufficient ground to APT, as trustee of NONOC MINING AND INDUSTRIAL amount of P8,370,934.74 with legal interest from date of
pierce the veil of corporate fiction, this Court likewise CORPORATION is directed to ensure compliance with this demand is hereby AFFIRMED, but the dismissal of the case
finds for the plaintiff. Decision.24 as against Assets Privatization Trust is REVERSED, and its
successor the Privatization and Management Office is
From the documentary evidence adduced by the plaintiff, DBP and PNB filed their respective appeals in the Court of INCLUDED as one of those jointly and severally liable for
some of which were even adopted by defendants and DBP Appeals. Both insisted that it was wrong for the RTC to such indebtedness. The award of attorney's fees is
and PNB as their own evidence (Exhibits "I", "I-1", "I-2", pierce the veil of NMIC's corporate personality and hold DBP DELETED.
"I-3", "I-4", "I-5", "I5-A", "I-5-B", "I-5-C", "I-5-D" and and PNB solidarily liable with NMIC.25
submarkings, inclusive), it had been established that All other claims and counter-claims are hereby DISMISSED.
except for five (5) qualifying shares, NMIC is owned by The Court of Appeals rendered the Decision dated Costs against appellants.28
defendants DBP and PNB, with the former owning 57% November 30, 2004, affirmed the piercing of the veil of the
thereof, and the latter 43%. As of September 24, 1984, all corporate personality of NMIC and held DBP, PNB, and APT The respective motions for reconsideration of DBP, PNB,
the members of NMIC's Board of Directors, namely, Messrs. solidarily liable with NMIC. In particular, the Court of and APT were denied.29
Jose Tengco, Jr., Rolando M. Zosa, Ruben Ancheta, Geraldo Appeals made the following findings:
Agulto, and Faustino Agbada are either from DBP or PNB Hence, these consolidated petitions.
(Exhibits "I-5", "I-5-C", "I-5-D"). In the case before Us, it is indubitable that [NMIC] was 30chanroblesvirtualawlibrary

owned by appellants DBP and PNB to the extent of 57% and All three petitioners assert that NMIC is a corporate entity
The business of NMIC was then also being conducted and 43% respectively; that said two (2) appellants are the only with a juridical personality separate and distinct from both
controlled by both DBP and PNB. In fact, it was Rolando M. stockholders, with the qualifying stockholders of five (5) PNB and DBP. They insist that the majority ownership by
Zosa, then Governor of DBP, who was signing and entering consisting of its own officers and included in its charter DBP and PNB of NMIC is not a sufficient ground for
into contracts with third persons, on behalf of NMIC. merely to comply with the requirement of the law as to disregarding the separate corporate personality of NMIC
number of incorporators; and that the directorates of DBP, because NMIC was not a mere adjunct, business conduit or
In this jurisdiction, it is well-settled that "where it appears PNB and [NMIC] are interlocked. alter ego of DBP and PNB. According to them, the
that the business enterprises are owned, conducted and xxx application of the doctrine of piercing the corporate veil is

Page 157 of 160


CORPORATION LAW CASES MAYORDO, M.A
LLB 3

unwarranted as nothing in the records would show that the of the respective deeds of transfer executed by DBP and must be clearly and convincingly established; it cannot be
ownership and control of the shareholdings of NMIC by DBP PNB in favor of the Republic: presumed. x x x.45 (Emphases supplied; citations omitted.)
and PNB were used to commit fraud, illegality or injustice. Sarona v. National Labor Relations Commission46 has defined
In the absence of evidence that the stock control by DBP SECTION 2. TRANSFER OF BANK'S LIABILITIES the scope of application of the doctrine of piercing the
and PNB over NMIC was used to commit some fraud or a xxx corporate veil:
wrong and that said control was the proximate cause of the 2.02 With respect to the Bank's liabilities which are
injury sustained by HRCC, resort to the doctrine of contingent and those liabilities where the Bank's creditors The doctrine of piercing the corporate veil applies only in
"piercing the veil of corporate entity" is misplaced.31 consent to the transfer thereof is not obtained, said three (3) basic areas, namely: 1) defeat of public
liabilities shall remain in the books of the BANK with the convenience as when the corporate fiction is used as a
DBP and PNB further argue that, assuming they may be held GOVERNMENT funding the payment thereof.36 vehicle for the evasion of an existing obligation; 2) fraud
solidarily liable with NMIC to pay NMIC's exclusive and cases or when the corporate entity is used to justify a
separate corporate indebtedness to HRCC, such liability of After a careful review of the case, this Court finds the wrong, protect fraud, or defend a crime; or 3) alter ego
the two banks was transferred to and assumed by the petitions impressed with merit. cases, where a corporation is merely a farce since it is a
National Government through the APT, now the PMO, under mere alter ego or business conduit of a person, or where
the respective deeds of transfer both dated February 27, A corporation is an artificial entity created by operation of the corporation is so organized and controlled and its
1997 executed by DBP and PNB pursuant to Proclamation law. It possesses the right of succession and such powers, affairs are so conducted as to make it merely an
No. 50 dated December 8, 1986 and Administrative Order attributes, and properties expressly authorized by law or instrumentality, agency, conduit or adjunct of another
No. 14 dated February 3, 1987.32 incident to its existence.37 It has a personality separate and corporation. (Citation omitted.)
distinct from that of its stockholders and from that of other
For its part, the APT contends that, in the absence of an corporations to which it may be connected.38 As a Here, HRCC has alleged from the inception of this case that
unqualified assumption by the National Government of all consequence of its status as a distinct legal entity and as a DBP and PNB (and the APT as assignee of DBP and PNB)
liabilities incurred by NMIC, the National Government result of a conscious policy decision to promote capital should be held solidarily liable for using NMIC as alter ego.47
through the APT could not be held liable for NMIC's formation,39 a corporation incurs its own liabilities and is The RTC sustained the allegation of HRCC and pierced the
contractual liability. The APT asserts that HRCC had not legally responsible for payment of its obligations.40 In other corporate veil of NMIC pursuant to the alter ego theory
sufficiently shown that the APT is the successor-in-interest words, by virtue of the separate juridical personality of a when it concluded that NMIC "is a mere adjunct, business
of all the liabilities of NMIC, or of DBP and PNB as corporation, the corporate debt or credit is not the debt or conduit or alter ego of both DBP and PNB."48 The Court of
transferors, and that the adjudged liability is included credit of the stockholder.41 This protection from liability for Appeals upheld such conclusion of the trial court.49 In other
among the liabilities assigned and transferred by DBP and shareholders is the principle of limited liability. words, both the trial and appellate courts relied on the
PNB in favor of the National Government.33 42chanroblesvirtualawlibrary alter ego theory when they disregarded the separate
Equally well-settled is the principle that the corporate corporate personality of NMIC.
HRCC counters that both the RTC and the CA correctly mask may be removed or the corporate veil pierced when
applied the doctrine of "piercing the veil of corporate the corporation is just an alter ego of a person or of In this connection, case law lays down a three-pronged test
fiction." It claims that NMIC was the alter ego of DBP and another corporation. For reasons of public policy and in the to determine the application of the alter ego theory, which
PNB which owned, conducted and controlled the business interest of justice, the corporate veil will justifiably be is also known as the instrumentality theory, namely:
of NMIC as shown by the following circumstances: NMIC was impaled only when it becomes a shield for fraud, illegality
owned by DBP and PNB, the officers of DBP and PNB were or inequity committed against third persons.43 (1) Control, not mere majority or complete stock control,
also the officers of NMIC, and DBP and PNB financed the but complete domination, not only of finances but of policy
operations of NMIC. HRCC further argues that a parent However, the rule is that a court should be careful in and business practice in respect to the transaction attacked
corporation may be held liable for the contracts or assessing the milieu where the doctrine of the corporate so that the corporate entity as to this transaction had at
obligations of its subsidiary corporation where the latter is veil may be applied. Otherwise an injustice, although the time no separate mind, will or existence of its own;
a mere agency, instrumentality or adjunct of the parent unintended, may result from its erroneous application.44
corporation.34 Thus, cutting through the corporate cover requires an (2) Such control must have been used by the defendant to
approach characterized by due care and caution: commit fraud or wrong, to perpetuate the violation of a
Moreover, HRCC asserts that the APT was properly held statutory or other positive legal duty, or dishonest and
solidarily liable with DBP, PNB, and NMIC because the APT Hence, any application of the doctrine of piercing the unjust act in contravention of plaintiff's legal right; and
assumed the obligations of DBP and PNB as the successor- corporate veil should be done with caution. A court should
in-interest of the said banks with respect to the assets and be mindful of the milieu where it is to be applied. It must (3) The aforesaid control and breach of duty must have
liabilities of NMIC.35 As trustee of the Republic of the be certain that the corporate fiction was misused to such proximately caused the injury or unjust loss complained of.
Philippines, the APT also assumed the responsibility of the an extent that injustice, fraud, or crime was committed 50 (Emphases omitted.)

Republic pursuant to the following provision of Section 2.02 against another, in disregard of its rights. The wrongdoing

Page 158 of 160


CORPORATION LAW CASES MAYORDO, M.A
LLB 3

The first prong is the "instrumentality" or "control" test. operation.62 With respect to the control element, it refers whether the corporate fiction was invoked for fraudulent or
This test requires that the subsidiary be completely under not to paper or formal control by majority or even malevolent ends, if the findings in the assailed decision are
the control and domination of the parent.51 It examines the complete stock control but actual control which amounts to either not supported by the evidence on record or based on
parent corporation's relationship with the subsidiary.52 It "such domination of finances, policies and practices that a misapprehension of facts.
inquires whether a subsidiary corporation is so organized the controlled corporation has, so to speak, no separate
and controlled and its affairs are so conducted as to make mind, will or existence of its own, and is but a conduit for In this case, nothing in the records shows that the
it a mere instrumentality or agent of the parent its principal."63 In addition, the control must be shown to corporate finances, policies and practices of NMIC were
corporation such that its separate existence as a distinct have been exercised at the time the acts complained of dominated by DBP and PNB in such a way that NMIC could
corporate entity will be ignored.53 It seeks to establish took place.64 be considered to have no separate mind, will or existence
whether the subsidiary corporation has no autonomy and of its own but a mere conduit for DBP and PNB. On the
the parent corporation, though acting through the Both the RTC and the Court of Appeals applied the alter ego contrary, the evidence establishes that HRCC knew and
subsidiary in form and appearance, "is operating the theory and penetrated the corporate cover of NMIC based acted on the knowledge that it was dealing with NMIC, not
business directly for itself."54 on two factors: (1) the ownership by DBP and PNB of with NMIC's stockholders. The letter proposal of Hercon,
effectively all the stocks of NMIC, and (2) the alleged Inc., HRCC's predecessor-in-interest, regarding the contract
The second prong is the "fraud" test. This test requires that interlocking directorates of DBP, PNB and NMIC. 65 for NMIC's mine stripping and road construction program
the parent corporation's conduct in using the subsidiary Unfortunately, the conclusion of the trial and appellate was addressed to and accepted by NMIC.71 The various
corporation be unjust, fraudulent or wrongful. 55 It courts that the DBP and PNB fit the alter ego theory with billing reports, progress reports, statements of accounts
examines the relationship of the plaintiff to the respect to NMIC's transaction with HRCC on the premise of and communications of Hercon, Inc./HRCC regarding NMIC's
corporation.56 It recognizes that piercing is appropriate complete stock ownership and interlocking directorates mine stripping and road construction program in 1985
only if the parent corporation uses the subsidiary in a way involved a quantum leap in logic and law exposing a gap in concerned NMIC and NMIC's officers, without any indication
that harms the plaintiff creditor.57 As such, it requires a reason and fact. of or reference to the control exercised by DBP and/or PNB
showing of "an element of injustice or fundamental While ownership by one corporation of all or a great over NMIC's affairs, policies and practices.72
unfairness."58 majority of stocks of another corporation and their
interlocking directorates may serve as indicia of control, by HRCC has presented nothing to show that DBP and PNB had
The third prong is the "harm" test. This test requires the t h e m s e l v e s a n d w i t h o u t m o r e , h o w e v e r, t h e s e a hand in the act complained of, the alleged undue
plaintiff to show that the defendant's control, exerted in a circumstances are insufficient to establish an alter ego disregard by NMIC of the demands of HRCC to satisfy the
fraudulent, illegal or otherwise unfair manner toward it, relationship or connection between DBP and PNB on the unpaid claims for services rendered by HRCC in connection
caused the harm suffered.59 A causal connection between one hand and NMIC on the other hand, that will justify the with NMIC's mine stripping and road construction program
the fraudulent conduct committed through the puncturing of the latter's corporate cover. This Court has in 1985. On the contrary, the overall picture painted by the
instrumentality of the subsidiary and the injury suffered or declared that "mere ownership by a single stockholder or by evidence offered by HRCC is one where HRCC was dealing
the damage incurred by the plaintiff should be established. another corporation of all or nearly all of the capital stock with NMIC as a distinct juridical person acting through its
The plaintiff must prove that, unless the corporate veil is of a corporation is not of itself sufficient ground for own corporate officers.73
pierced, it will have been treated unjustly by the disregarding the separate corporate personality."66 This
defendant's exercise of control and improper use of the Court has likewise ruled that the "existence of interlocking Moreover, the finding that the respective boards of
c o r p o r a t e f o r m a n d , t h e r e b y, s u f f e r d a m a g e s . directors, corporate officers and shareholders is not enough directors of NMIC, DBP, and PNB were interlocking has no
60chanroblesvirtualawlibrary justification to pierce the veil of corporate fiction in the basis. HRCC's Exhibit "I-5,"74 the initial General Information
To summarize, piercing the corporate veil based on the absence of fraud or other public policy considerations."67 Sheet submitted by NMIC to the Securities and Exchange
alter ego theory requires the concurrence of three Commission, relied upon by the trial court and the Court of
elements: control of the corporation by the stockholder or True, the findings of fact of the Court of Appeals are Appeals may have proven that DBP and PNB owned the
parent corporation, fraud or fundamental unfairness conclusive and cannot be reviewed on appeal to this Court, stocks of NMIC to the extent of 57% and 43%, respectively.
imposed on the plaintiff, and harm or damage caused to provided they are borne out of the record or are based on However, nothing in it supports a finding that NMIC, DBP,
the plaintiff by the fraudulent or unfair act of the substantial evidence.68 It is equally true that the question and PNB had interlocking directors as it only indicates that,
corporation. The absence of any of these elements prevents of whether one corporation is merely an alter ego of of the five members of NMIC's board of directors, four were
piercing the corporate veil.61 another is purely one of fact. So is the question of whether nominees of either DBP or PNB and only one was a nominee
a corporation is a paper company, a sham or subterfuge or of both DBP and PNB.75 Only two members of the board of
This Court finds that none of the tests has been whether the requisite quantum of evidence has been directors of NMIC, Jose Tengco, Jr. and Rolando Zosa, were
satisfactorily met in this case. adduced warranting the piercing of the veil of corporate established to be members of the board of governors of DBP
personality.69 Nevertheless, it has been held in Sarona v. and none was proved to be a member of the board of
In applying the alter ego doctrine, the courts are concerned National Labor Relations Commission70 that this Court has directors of PNB.76 No director of NMIC was shown to be
with reality and not form, with how the corporation the power to resolve a question of fact, such as whether a also sitting simultaneously in the board of governors/
operated and the individual defendant's relationship to that corporation is a mere alter ego of another entity or directors of both DBP and PNB.

Page 159 of 160


CORPORATION LAW CASES MAYORDO, M.A
LLB 3

There being a total absence of evidence pointing to a the amount of P8,370,934.74, with legal interest thereon
In reaching its conclusion of an alter ego relationship fraudulent, illegal or unfair act committed against HRCC by from date of demand.
between DBP and PNB on the one hand and NMIC on the DBP and PNB under the guise of NMIC, there is no basis to
other hand, the Court of Appeals invoked Sibagat Timber hold that NMIC was a mere alter ego of DBP and PNB. As As trustee of the. assets of NMIC, however, the APT should
Corporation v. Garcia,77 which it described as "a case under this Court ruled in Ramoso v. Court of Appeals82: ensure compliance by NMIC of the judgment against it. The
a similar factual milieu."78 However, in Sibagat Timber APT itself acknowledges this.84chanroblesvirtualawlibrary
Corporation, this Court took care to enumerate the As a general rule, a corporation will be looked upon as a
circumstances which led to the piercing of the corporate legal entity, unless and until sufficient reason to the WHEREFORE, the petitions are hereby GRANTED.
veil of Sibagat Timber Corporation for being the alter ego contrary appears. When the notion of legal entity is used to
of Del Rosario & Sons Logging Enterprises, Inc. Those defeat public convenience, justify wrong, protect fraud, or The complaint as against Development Bank of the
circumstances were as follows: holding office in the same defend crime, the law will regard the corporation as an Philippines, the Philippine National Bank, and the Asset
building, practical identity of the officers and directors of association of persons. Also, the corporate entity may be Privatization Trust, now the Privatization and Management
the two corporations and assumption of management and disregarded in the interest of justice in such cases as fraud Office, is DISMISSED for lack of merit. The Asset
control of Sibagat Timber Corporation by the directors/ that may work inequities among members of the Privatization Trust, now the Privatization and Management
officers of Del Rosario & Sons Logging Enterprises, Inc. corporation internally, involving no rights of the public or Office, as trustee of Nonoc Mining and Industrial
third persons. In both instances, there must have been Corporation, now the Philnico Processing Corporation, is
Here, DBP and PNB maintain an address different from that fraud, and proof of it. For the separate juridical personality DIRECTED to ensure compliance by the Nonoc Mining and
of NMIC.79 As already discussed, there was insufficient proof of a corporation to be disregarded, the wrongdoing must be Industrial Corporation, now the Philnico Processing
of interlocking directorates. There was not even an clearly and convincingly established. It cannot be Corporation, with this Decision.
allegation of similarity of corporate officers. Instead of presumed.
evidence that DBP and PNB assumed and controlled the SO ORDERED.
management of NMIC, HRCC's evidence shows that NMIC As regards the third element, in the absence of both
operated as a distinct entity endowed with its own legal control by DBP and PNB of NMIC and fraud or fundamental
personality. Thus, what obtains in this case is a factual unfairness perpetuated by DBP and PNB through the
backdrop different from, not similar to, Sibagat Timber corporate cover of NMIC, no harm could be said to have
Corporation. been proximately caused by DBP and PNB on HRCC for
which HRCC could hold DBP and PNB solidarily liable with
In relation to the second element, to disregard the NMIC.
separate juridical personality of a corporation, the
wrongdoing or unjust act in contravention of a plaintiff's Considering that, under the deeds of transfer executed by
legal rights must be clearly and convincingly established; it DBP and PNB, the liability of the APT as transferee of the
cannot be presumed. Without a demonstration that any of rights, titles and interests of DBP and PNB in NMIC will
the evils sought to be prevented by the doctrine is present, attach only if DBP and PNB are held liable, the APT incurs
it does not apply.80chanroblesvirtualawlibrary no liability for the judgment indebtedness of NMIC. Even
In this case, the Court of Appeals declared: HRCC recognizes that "as assignee of DBP and PNB 's loan
receivables," the APT simply "stepped into the shoes of DBP
We are not saying that PNB and DBP are guilty of fraud in and PNB with respect to the latter's rights and obligations"
forming NMIC, nor are we implying that NMIC was used to in NMIC.83 As such assignee, therefore, the APT incurs no
conceal fraud. x x x.81 liability with respect to NMIC other than whatever
liabilities may be imputable to its assignors, DBP and PNB.
Such a declaration clearly negates the possibility that DBP
and PNB exercised control over NMIC which DBP and PNB Even under Section 2.02 of the respective deeds of transfer
used "to commit fraud or wrong, to perpetuate the executed by DBP and PNB which HRCC invokes, the APT
violation of a statutory or other positive legal duty, or cannot be held liable. The contingent liability for which the
dishonest and unjust act in contravention of plaintiff's legal National Government, through the APT, may be held liable
rights." It is a recognition that, even assuming that DBP and under the said provision refers to contingent liabilities of
PNB exercised control over NMIC, there is no evidence that DBP and PNB. Since DBP and PNB may not be held solidarily
the juridical personality of NMIC was used by DBP and PNB liable with NMIC, no contingent liability may be imputed to
to commit a fraud or to do a wrong against HRCC. the APT as well. Only NMIC as a distinct and separate legal
entity is liable to pay its corporate obligation to HRCC in

Page 160 of 160

Potrebbero piacerti anche